Land Sachsen

„Wikipedia:Auskunft“ – Versionsunterschied

Inhalt gelöscht Inhalt hinzugefügt
SignaturBot (Diskussion | Beiträge)
K Bot: Signaturnachtrag für Beitrag von Elrond: " →‎Essen und Atmen: nenene"
Format
Markierung: Zurückgesetzt
Zeile 1: Zeile 1:
{{/Intro}}<!-- Bitte beachten, dass Anzahl Tage („Alter=“) mit „Wikipedia:Auskunft/Intro“ übereinstimmen soll. -->
{{/Intro}}<!-- Bitte beachten, dass Anzahl Tage („Alter=“) mit „Wikipedia:Auskunft/Intro“ übereinstimmen soll. -->
{{Autoarchiv|Alter=3|Ziel='((Lemma))/Archiv/((Jahr))/Woche ((Woche:##))'|Mindestbeiträge=1|Zeigen=Nein}}
{{Autoarchiv|Alter=3|Ziel='((Lemma))/Archiv/((Jahr))/Woche ((Woche:##))'|Mindestbeiträge=1|Zeigen=Nein}}
{{Autoarchiv|Alter=1|Ziel='((Lemma))/Archiv/((Jahr))/Woche ((Woche:##))'|Mindestbeiträge=1|Modus=Alter, Erledigt|Zeigen=Nein}}
{{Autoarchiv|Alter=1|Ziel='((Lemma))/Archiv/((Jahr))/Woche ((Woche:##))'|Mindestbeiträge=1|Modus=Alter, Erledigt|Zeigen=Nein}}
{{Autoarchiv-Erledigt|Alter=1|Ziel='((Lemma))/Archiv/((Jahr))/Woche ((Woche:##))'|Zeitbeschränkung=3|Zeigen=Nein}}
{{Autoarchiv-Erledigt|Alter=1|Ziel='((Lemma))/Archiv/((Jahr))/Woche ((Woche:##))'|Zeitbeschränkung=3|Zeigen=Nein}}


= 18. September =
= 17. Februar 2017 =


== Wer war evtl. der 50000. Schütze? ==
== „eine Viertelstunde zu Pferd“ ==


Laut Kicker gibt es [http://www.kicker.de/news/fussball/bundesliga/startseite/670775/artikel_bellarabi-gefeiert_das-mysterium-um-das-50-000-tor.html vier Varianten zur Zählung der Gesamt-Bundesligatore.]
In älteren Publikationen finde ich gelegentlich unpräzise Angaben wie "der nächste Nachbar war eine Viertelstunde zu Pferd entfernt". Welche Wegstreckenlänge kann man sich dabei ungefähr vorstellen? --[[Benutzer:Emeritus|Emeritus]] ([[Benutzer Diskussion:Emeritus|Diskussion]]) 16:39, 18. Sep. 2020 (CEST)
Variante 1 liefert "erst" 49950, Variante 2 49991, Variante 3 nun 50003 Tore. Bei der letzteren Variante heißt der Schütze des 50000. Bundesligatores [[Karim Bellarabi]].
:Unter [[:en:Horse gait]] findest Du ungefähre Geschwindigkeiten für die einzelnen [[Pferdegangart]]en. --[[user:Rotkaeppchen68|R<span style="color:red">ô</span>tkæppchen₆₈]] 16:44, 18. Sep. 2020 (CEST)
:::Wobei man für einen gewöhnlichen Nachbarschaftsbesuch eher selten im Galopp geritten sein dürfte. So oder so ist das auf dem Land sehr nah: Eine Strecke die man heute zu Fuß oder auf dem Rad in einer Viertelstunde bewältigen könnte. --[[Benutzer:Geoz|Geoz]] ([[Benutzer Diskussion:Geoz|Diskussion]]) 17:13, 18. Sep. 2020 (CEST)
Variante 4 - mit den Toren von abgebrochenen Spielen - weist 50017(?) Tore aus. Wer war denn hier der Schütze des 50000. Bundesligatores (es muss am letzten Spieltag gefallen sein)? --[[Benutzer:ObersterGenosse|ObersterGenosse]] ([[Benutzer Diskussion:ObersterGenosse|Diskussion]]) 22:05, 17. Feb. 2017 (CET)
Eine Gangart, die für Pferd und Reiter angenehm ist und die längere Zeit durchgehalten werden kann ist der Kanter, ein leichter Galopp. Damit kommt man so rund 20 km/ h vorran, Deine oben genannte Distanz wäre also grob 5 km. Wenn die Angaben noch älter sind, meint man wahrscheinlich einen Zelter, ein Pferd, das die Gangart Tölt beherrscht und grobe so schnell wie Kantern ist, also 20 km/h. Diese Gangart ist für den Reiter besonders angenehm, weil sehr ruhig für ihn. --[[Benutzer:Elrond|Elrond]] ([[Benutzer Diskussion:Elrond|Diskussion]]) 17:17, 18. Sep. 2020 (CEST)


: Guck dir die halt Spielberichte vom letzten (20.) Spieltag an (findest du allesamt bei kicker.de) und zähl es selbst aus. Nach der 4. Zählvariante hat Bellarabi ja das <s>50013.</s> 50014. Tor erzielt. Das 50000ste fiel dann wahrscheinlich letzte Woche Samstag, denn am Sonntag fielen nur 6 Treffer, am Samstag hingegen <s>15</s> 14. Guck dir die Spiele an, und erstelle eine Tabelle, in der du auflistest, wer in welcher Spielminute ein Tor erzielt hat. Allerdings musst du dabei voraussetzen, dass bei jedem der Spiele die Spielzeit simultan lief, was spätestens ab der 2. Halbzeit wegen unterschiedlicher Nachspielzeiten am Ende der 1. Halbzeit unrealistisch ist. Fiel der rein von der Spielminute in Frage kommende Treffer also in der 2. Halbzeit und liegt er dort eng mit anderen Treffern beieinander (<&nbsp;5&nbsp;min), müsste man sich anschauen, um welche ''Uhrzeit'' die jeweils tatsächlich gefallen sind... --[[Benutzer:Gretarsson|Gretarsson]] ([[Benutzer Diskussion:Gretarsson|Diskussion]]) 14:01, 19. Feb. 2017 (CET)<small>; zuletzt. geänd. 11:25, 22. Feb. 2017 (CET)</small>
::::Für diese Wegangabe rechnet man eine Gangart, die das Pferd den ganzen Tag beibehalten kann, auch mit Wagen dran oder ein Kaltblüter, auch bei schlechtem Wetter, also im Schritt. Jeder weiß, dass man kurzfristig mit dem Pferd schneller sein kann und jeder weiß auch dass nicht alle Pferde gleich schnell sind. Im Schritt ist ein Pferd nicht viel schneller als bei einem strammen Fußmarsch. Man kann also sagen, dass der Nachbar kaum mehr als einen Kilometer weg ist. Diese Angabe eine "Viertelstunde Wegs Entfernung" fand ich einmal für die Positionsangabe einer Mühle vom Ort und nach Google entspricht das einer gemessenen Strecke von 915 Metern auf dem heutigen Wegverlauf, der alte Weg verlief auf der selben Trasse, war aber wahrscheinlich etwas gewundener und länger, also sagen wir mal 960 Meter. Diese Angabe rechnet dabei ein, dass der Weg nicht geradlinig verläuft und Steigungen und Gefälle hat, sowie den Zustand des Wegs, der ein schnelleres oder langsameres Fortkommen bedingt und dass man eventuell Tore oder Gatter öffnen und schließen muss. Bei längeren Angaben sind die nötigen Pausen mit eingerechnet und eine "Tagesreise" besagt nicht die Strecke, die man in 24 Stunden maximal zurücklegen kann, sondern die Strecke, die man auf einer längeren Reise im Durchschnitt pro Tag schafft, dabei ist die Übernachtung und die Erholungszeit inbegriffen. Je nach Gegebenheit, Jahreszeit und Wetter ist die Tagesreise dann länger oder kürzer. In Deutschland ist die Verteilung von Ortschaften nach der Weglänge implizit: Für Arbeiten auf dem Feld wird ein Weg von mehr als einer Stunde ineffektiv: von 8 Arbeitsstunden bist du dann zwei pro Tag unterwegs vom Bauernhof bis zum Arbeitsplatz, also zwölf Stunden pro Woche bei sechs Arbeitstagen. Somit fangen die Leute damit an in einer Stunde Entfernung eine Schutzhütte oder Scheune zu bauen, die erst temporär während der Ernte und dann später permanent besiedelt wird, somit wird das dann ein Gehöft und ein Weiler und unter guten Bedingungen ein Ort. Diese Verteilung von einer Wegstunde Abstand findet man quer über ganz Deutschland und die Abstände werden um so größer, je flacher das Land ist. In Kilometern ausgedrückt befinden sich die Orte in Gebirgstälern zwei Kilometer auseinander und im Flachland 5 Kilometer.--[[Benutzer:Giftzwerg 88|Giftzwerg 88]] ([[Benutzer Diskussion:Giftzwerg 88|Diskussion]]) 18:16, 18. Sep. 2020 (CEST)
:::::Danke Euch erstmal, mit 2 km bis 5 km kommt das schon hin, die Wege waren noch Ende des 19. Jahrhunderts Schneisen durch den Urwald über "Berg und Tal", nicht jeder hatte dort unten (ich beziehe mich auf Espírito Santo bei meinem Beispiel) ein Reitpferd, oft gab es nur Mulis im Hochland. --[[Benutzer:Emeritus|Emeritus]] ([[Benutzer Diskussion:Emeritus|Diskussion]]) 18:48, 18. Sep. 2020 (CEST)
::Wobei die Methode nicht ganz unproblematisch ist, wenn z.B. bei einem Spiel mit Beginnzeit 15:30 ein Tor in der 73. Minute fällt und bei einem anderen Spiel am selben Tag mit der selben Ankickzeit in der 74. Minute, muss das Tof in der 73. Minute nicht unbedingt vor dem in def 74 Minute gefallen sein, da die Uhrheit vom Ankick vor Allem bei der zweitdn Halbzeit einige Minuten auseinanderliegen kann. --[[Benutzer:MrBurns|MrBurns]] ([[Benutzer Diskussion:MrBurns|Diskussion]]) 17:29, 19. Feb. 2017 (CET)
::: <small>Hab ich das oben nicht geschrieben? ;-) --[[Benutzer:Gretarsson|Gretarsson]] ([[Benutzer Diskussion:Gretarsson|Diskussion]]) 18:29, 19. Feb. 2017 (CET)</small>
: Folgefrage: Wenn nicht jeder in der Gegend ein Pferd hatte, und nicht jedes Pferd unter allen Umständen gleich schnell ist, und man bei der Zu-Pferd-Angabe davon ausgehen kann, dass sie mit einer Zu-Fuss-Angabe gleichzusetzen wäre, und die Strecke auch nicht so lang ist, dass man sich die Fussarbeit trotzdem sparen möchte: Warum werden überhaupt solche Angaben gemacht? Wäre "der nächste Nachbar war eine Viertelstunde zu Fuss entfernt" oder noch einfacher "der nächste Nachbar war eine Viertelstunde entfernt" nicht eindeutiger und allgemeinverständlicher? Welchen Vorteil, das Pferd da überhaupt mit reinzunehmen? --[[Spezial:Beiträge/2A02:110:0:3002:0:0:0:1067|2A02:110:0:3002:0:0:0:1067]] 12:55, 21. Sep. 2020 (CEST)
::: <small>P.S. Am Sonntag und beim 18.30-Uhr-Spiel am Samstag fielen insgesamt nur 8 Treffer, das fragliche Tor muss also eines der <s>13</s> 12 sein, die während der fünf Nachmittagspartien fielen (davon war ich oben sowieso ausgegangen) und zwar tatsächlich eines der 6 <s>7</s>, die während der 2. Halbzeit fielen (damit muss man dann also nur noch 4 Spielberichte angucken, denn bei Bremen-M’Gladbach war der Halbzeitstand auch der Endstand)... --[[Benutzer:Gretarsson|Gretarsson]] ([[Benutzer Diskussion:Gretarsson|Diskussion]]) 19:03, 19. Feb. 2017 (CET)<small>; zuletzt. geänd. 11:25, 22. Feb. 2017 (CET)</small></small>
::Der Zusatz spricht eher dafür, dass es zu Pferd schneller ging als zu Fuß, d.h. die Strecke war eher 2-3 km als 1 km. Und wahrscheinlich hatte der Autor des Textes ein Pferd und ist die Strecke mal geritten. Und wahrscheinlich richtete sich der Text ursprünglich an Leute, die mit dem Hinweis "zu Pferd" etwas anfangen konnten. --[[Benutzer:Optimum|Optimum]] ([[Benutzer Diskussion:Optimum|Diskussion]]) 13:26, 21. Sep. 2020 (CEST)


::: Es scheint dich ja brennend interessiert zu haben, [[Benutzer:ObersterGenosse|ObersterGenosse]], wer denn nun der Rekordtorschütze nach inoffizieller Zählung war, wenn du hier zwar die Frage einstellst, dich dann aber anscheinend nicht weiter drum scherst, wie die Antwort lautet. Ist dir die Recherche selbst dann noch zu anstrengend, wenn dir das Ergebnis schon fast auf dem Silbertablett präsentiert wird?
:::Falsch. Ein Weg kann mit Pferd auch länger dauern, wenn man auf Wegen mit Steilstrecken, Felsen und umgestürzten Bäumen, Sumpf, Furten und sonstwas konfrontiert ist und Wagen und Gepäck dabei hat. Unter Umständen muss man da sogar eine Rast für die Pferde einlegen. Und natürlich kann man auch eine Strecke schneller schaffen, wenn man das Pferd entsprechend antreibt. Ortsnamen wie Via mala, Schinderbuckel, Ochsenwald und Teufelsloch sprechen eine eigene Sprache. Die alten Straßen waren selten so gut, dass man da gefahrlos galoppieren konnte. Da gabs Schlaglöcher, im guten Fall Kopfsteinpflaster (auch im Zustand feucht und schlammbedeckt anzutreffen), Fahrrinnen, Gräben, Schlammstellen, niederhängende Zweige, querende Bachläufe und sonstwas. Die Wegangaben waren immer "pessimistische" Angaben, das heißt mit Gepäck und sind immer nur grobe Annäherungen. Die meisten Reiter wollten damals nicht möglichst schnell irgendwohin kommen, sondern möglichst sicher, also so dass sich weder das Pferd ein Bein bricht, noch der Reiter den Hals, noch der Wagen umfällt oder ein Rad bricht. Fällt er in den Graben, fressen ihn die Raben.--[[Benutzer:Giftzwerg 88|Giftzwerg 88]] ([[Benutzer Diskussion:Giftzwerg 88|Diskussion]]) 15:14, 23. Sep. 2020 (CEST)
::: Da es mich letztlich selbst interessiert hat, und für evtl. interessierte Mitlesende hier die Auflösung:
::: Die Partien Leipzig–HSV und Ingolstadt–Bayern scheiden aus, weil dort die in der 2. Halbzeit erzielten Tore (Numero 50003 bis 50005) alle in der 90. min oder später fielen. Das entscheidende Tor fiel also entweder bei Darmstadt–Dortmund oder L’kusen–Frankfurt, und, was soll ich sagen, auch nach dieser Zählung war der Rekordtorschütze ein „Pillendreher“: Es war '''Chicharito''', der in der 63. Spielminute gegen Frankfurt das 2:0 für sein Team erzielte. Der 2:1 Siegtreffer der Darmstädter durch Colak fiel zwar in der 67. Minute und damit relativ dicht dran, doch den Uhrzeitangeben in den jeweiligen Livetickern ist zu entnehmen, dass Chicharito um [http://www.kicker.de/news/fussball/bundesliga/spieltag/1-bundesliga/2016-17/20/3317425/spielverlauf_bayer-leverkusen-9_eintracht-frankfurt-32.html 16:50 Uhr] traf, Colak hingegen erst [http://www.kicker.de/news/fussball/bundesliga/spieltag/1-bundesliga/2016-17/20/3317431/spielverlauf_sv-darmstadt-98-98_borussia-dortmund-17.html 16:54], die 2. Halbzeit bei beiden Spielen also ungefähr gleichzeitig angepfiffen wurde. Colak erzielte demnach also nur Tor Nummer 50001. Das 50002., der Vollständigkeit halber, schoss Kevin Volland (78.), der damit den 3:0-Endstand gegen die Eintracht herstellte. --[[Benutzer:Gretarsson|Gretarsson]] ([[Benutzer Diskussion:Gretarsson|Diskussion]]) 11:14, 22. Feb. 2017 (CET)


Danke für die Antworten! Hier hatte sich einige Tage lang niemand gemeldet, also dachte ich, die Frage sei unbeantwortet ins Archiv gewandert oder sowas. Tschuldigung an alle!
::"Bei uns Reitern ist "Zu Pferd" eigentlich immer mit Trab oder Gallop die Geschwindigkeit gemeint. Bei längeren Ritten, wird nach ca. 45min abgesessen und bei schnellem Fußmarsch im Schritt weitergegangen. Der Weg eines bewegten Pferdes sollte nicht länger an 2 Stunden sein. Mit anschließend 2 Stunden Pause. War früher so - ist auch heute noch immer so! Deshalb wurde früher auch immer ein zweites Pferd am Strick mitgeführt. [[Benutzer:Schnittwerk|Schnittwerk]] ([[Benutzer Diskussion:Schnittwerk|Diskussion]]) 12:48, 26. Sep. 2020 (CEST)
::::<small><small>[[wikt:Galopp]] --[[File:Lemon with white background.jpg|15px|Quetsch mich aus, ... ]] [[Benutzer:Itu|itu]] ([[Benutzer Diskussion:Itu|Disk]]) 20:50, 26. Sep. 2020 (CEST) </small></small>


Und danke an [[Benutzer:Gretarsson|Gretarsson]] für die Recherche! Beide Jubiläumstore gehen also auf das Konto von Vizekusen!
:::Den letzten Satz verstehe ich nicht. Das Pferd, das am Strick mitgeführt wird, muss doch auch mitlaufen. --[[Benutzer:Digamma|Digamma]] ([[Benutzer Diskussion:Digamma|Diskussion]]) 14:43, 26. Sep. 2020 (CEST)
Und wenn sie jetzt noch die beiden anderen (mehr oder weniger) offiziellen Jubiläumstore schießen... wäre doch cool, oder? --[[Benutzer:ObersterGenosse|ObersterGenosse]] ([[Benutzer Diskussion:ObersterGenosse|Diskussion]]) 14:08, 22. Feb. 2017 (CET)


: OK, dann nichts für ungut. Ja, wäre schon ein großer Zufall, wenn tatsächlich „alle“ 50.000sten Tore ein Leverkusener schießen täte. Ob das cool wäre, weiß ich nicht. Davon kaufen könnten sie sich nichts. Viele fänden das wahrscheinlich gar nicht so cool, denn m.W. hat Leverkusen jetzt nicht sooo wahnsinnig viele Anhänger und der finanzielle Rückhalt, den sie durch die Bayer AG haben, wird ihnen auch gerne geneidet, wobei ich persönlich das schlicht als Standortvorteil werten würde... --[[Benutzer:Gretarsson|Gretarsson]] ([[Benutzer Diskussion:Gretarsson|Diskussion]]) 14:37, 22. Feb. 2017 (CET)
::::Das zweite Pferd muss keinen Reiter tragen. Der Weg alleine ist nicht das, was das Pferd müde macht und auch der Reiter wird müde und braucht mal ne Pause. Man könnte also nach 45 Minuten das Pferd wechseln und die letzte halbe Stunde im Schritt zurücklegen für die maximale Wegstrecke in zwei Stunden. Die Pausen werden mitgezählt.--[[Benutzer:Giftzwerg 88|Giftzwerg 88]] ([[Benutzer Diskussion:Giftzwerg 88|Diskussion]]) 15:54, 26. Sep. 2020 (CEST)
:::::Die Formulierung "zu Pferd" schließt m. E. das Mitführen eines Wagens aus, und dass man ständig ein zweites Pferd mitschleift, das ja dann jedesmal auch gesattelt und aufgezäumt werden müsste etc., halte ich auch für unwahrscheinlich. In den Romanen Fontanes, in denen die Leute noch viel zu Pferd unterwegs sind, und in denen freilich auch viel geplaudert wird (auch beim Reiten), ist man "zu Pferd" in aller Regel im Schritt unterwegs. Wenn man sich in Trab setzt, wird eigentlich jedesmal extra betont, dass man gerade einen besonders günstigen Reitweg erwischt hat, auf dem man bequem traben kann, oder dass man in besonderer Eile ist. --[[Benutzer:Xocolatl|Xocolatl]] ([[Benutzer Diskussion:Xocolatl|Diskussion]]) 16:32, 29. Sep. 2020 (CEST)


: Nachtrag: Das 50.000ste nach Variante 2 fiel übrigens auch noch am vergangenen Spieltag und zwar wieder am Samstag und wieder in der 2. Halbzeit (damit ist klar, dass kein Bayer-Spieler „alle vier“ Jubiläumstreffer erzielen wird). Ausgehend von 49.991 Toren nach dem Spiel L’kusen–Augsburg fielen am Samstag Nachmittag in der 1. Halbzeit die Treffer 49.992 bis 49.996. Das Jubiläumstor nach dieser Zählung war demnach, wenn ich richtig gezählt hab, das 2:0 der Ingolstädter im Spiel gegen Frankfurt. Torschütze war '''[[Pascal Groß]]'''. Das Tor fiel in der 69. Minute, laut Kicker-Liveticker um 16:57, d.h. 7 min nach dem Treffer des Hoffenheimers Kramaric zum 1:0 gegen Darmstadt (64.) und deutlich vor den beiden letzten Treffern des Nachmittages, die beide erst in der Nachspielzeit erzielt wurden. Das letzte der vier Jubiläumstore dürfte dann am übernächsten Spieltag fallen (Stand derzeit, wenn ich richtig gezählt hab, 49.970). --[[Benutzer:Gretarsson|Gretarsson]] ([[Benutzer Diskussion:Gretarsson|Diskussion]]) 21:29, 22. Feb. 2017 (CET)
::::::Ich bleibe dabei: Wer "...eine Viertelstunde zu Pferd entfernt" schreibt, legt diese Strecke gewöhnlich mit dem Pferd zurück. Von mir aus dann im Schritt. Wie man leicht recherchieren kann :), bewegen sich Pferde im Schritt mit einer Geschwindigkeit von 4 bis 8 km/h vorwärts (wohl je nach Pferd und Zustand des Pferdes), Menschen dagegen mit 3 bis 5,5 km/h. Sicherlich wird man zum Nachbarn nicht im militärischen Marschtempo gehen und dann ganz verschwitzt ankommen. Eher könnte ich mir vorstellen, dass man das Pferd etwas anspornt, wenn der Nachbarhof in Sichtweite kommt.
::::::Ich halte daher 4 km/h für den durchschnittlichen Fußgänger und 6 km/h für dem Reiter für realistisch. Die Entfernung zum Nachbarn wäre dann mindestens 1500 m. --Optimum (Diskussion) 20:32, 30. Sep. 2020 (CEST) {{unsigniert|Optimum|20:36, 30. Sep. 2020 (CEST)}}


Wir haben jetzt also schon:
:::::::Und wieder hast du den Zustand des Wegs außer Acht gelassen. Es gibt zu der Zeit keine "Durchschnittsgeschwindigkeit", weil es keinen "durchschnittlichen Weg" gibt. Was früher die Landstraße war, würde man heute vielleicht gerade noch als Feldweg durchgehen lassen. Und keiner von uns weiß, ob es überhaupt einen markierten Weg zum Nachbarn gibt, oder ob es nicht vielmehr über sumpfige Wiesen mit Bächen und über Kuhweiden mit Trampelpfad ging. Befestigte Wege gab es bestenfalls zwischen Ortschaften. Nachbarn in mehr als 1,5 km Entfernung gibts nur in sehr dünn besiedelten Gegenden, also dort, wo die Wege besonders schlecht sind. --[[Benutzer:Giftzwerg 88|Giftzwerg 88]] ([[Benutzer Diskussion:Giftzwerg 88|Diskussion]]) 21:47, 1. Okt. 2020 (CEST)
[[Karim Bellarabi]]
[[Chicharito]]
[[Pascal Groß]]


und werden bei einem torreichen Spieltag am nächsten, sonst am übernächsten Spieltag wissen, wer das "vierte" Jubiläumstor geschossen hat. Daher:
::::::::Wenn man allerdings überhaupt nichts über den Weg sagen kann und niemand weiß, ob es ein sumpfiges Moor oder eine alte Römerstraße war, dann ist die Angabe "der Nachbar wohnt eine Viertelstunde zu Pferd entfernt" unbestimmbar und damit völlig sinnlos. Der Autor wird aber doch sicherlich keine Nonsensinformationen von sich geben, sondern von seinen Zuhörern erwarten, dass sie sich darunter eine relativ genaue Entfernung vorstellen können. Ohne weitere Zusatzinformationen muss man also von einem normalen Ritt ohne besondere Vorkommnisse ausgehen. Der Nachbar wohnt also eher nicht in einem hundert Meter entfernten Haus, zu dem man durch den Sumpf aber eine Viertelstunde bräuchte. Und er wohnt auch nicht am gegenüberliegenden Flußufer, zu dem der Weg bis zur Furt und zurück eine Viertelstunde dauert. "Der nächste Nachbar..." weist eher auf eine Information über die Bevölkerungsdichte hin als auf eine besonders unpassierbare Umgebung. --[[Benutzer:Optimum|Optimum]] ([[Benutzer Diskussion:Optimum|Diskussion]]) 23:23, 1. Okt. 2020 (CEST)


{{Nicht archivieren|Wir warten noch auf das vierte 50000te Tor!|--[[Benutzer:ObersterGenosse|ObersterGenosse]] ([[Benutzer Diskussion:ObersterGenosse|Diskussion]]) 01:12, 23. Feb. 2017 (CET)}}
= 25. September =


So, wenn 49.970 Tore bis zum letzten Sonntag gefallen sind - so wie es Gretarsson vermutet hat - und da ja gestern und heute schon 25 Tore fielen (es war also ein "torreicher Spieltag"), fehlen nur noch fünf. Dafür hätten wir noch Ingolstadt - Gladbach und Schalke - Hoffenheim. Also könnten durchaus morgen fünf Tore fallen und wir hätten den vierten 5000er schon morgen. Ansonsten am nächsten Spieltag... --[[Benutzer:ObersterGenosse|ObersterGenosse]] ([[Benutzer Diskussion:ObersterGenosse|Diskussion]]) 20:51, 25. Feb. 2017 (CET)
== Morellóbilder mit und ohne Bahngleise harren der Lokalisierung ==


: Heute fielen nochmal vier Tore, d.h., der Schütze zum 1:0 am nächsten Freitag beim Spiel Augsburg-Leipzig wird der Schütze des 50.000sten Bundesligatores nach Zählvariante 1. --[[Benutzer:Gretarsson|Gretarsson]] ([[Benutzer Diskussion:Gretarsson|Diskussion]]) 00:45, 27. Feb. 2017 (CET)
<gallery>
Datei:Estació de tren Salbertrand.jpg|Nr. 1: Die Quelle siedelt's in der Schweiz an, ein Diskutant in Italien.
Datei:Tren passant per la riba d'un llac i un home en primer terme.jpeg|Nr. 2: Laut Quelle im Kanton Luzern aufgenommen...
Datei:Casa enmig d'uns jardins a Lucerna.jpeg|Nr. 3: Ein leider ziemlich unscharfes Gebäude mit Palmen, angeblich im Kanton Luzern
</gallery>
Die Ortsangaben aus der Quelle dürften wie immer mit Vorsicht zu genießen sein. Bei Nr. 1 ist die Diskussionsseite zu beachten; Nr. 2 wurde von Isidre blanc als Aufnahme vom Vierwaldstättersee kategorisiert, aber ohne nähere Angabe. --[[Benutzer:Xocolatl|Xocolatl]] ([[Benutzer Diskussion:Xocolatl|Diskussion]]) 18:41, 25. Sep. 2020 (CEST)
:Zum ersten Bild: Im Abschnitt [[Bahnstrom#Verwendung von extern erzeugtem Drehstrom]] sind die in Frage kommenden Strecken und Netze aufgeführt. Dabei kommt die sowohl die Schweiz, als auch Norditalien in Frage. In den französischen Pyrenäen und in Brasilien gibt es zwei weitere Drehstromstrecken, die aber beide Zahnradbahnen sind im Gegensatz zum obigen Bild. --[[user:Rotkaeppchen68|R<span style="color:red">ô</span>tkæppchen₆₈]] 19:18, 25. Sep. 2020 (CEST) Die beiden Schweizer Drehstromstrecken sind ebenfalls Zahnradbahnen, sodass tatsächlich nur Norditalien in Frage kommt. --[[user:Rotkaeppchen68|R<span style="color:red">ô</span>tkæppchen₆₈]] 19:20, 25. Sep. 2020 (CEST) [[:en:List_of_railway_electrification_systems#Two_wires]] kennt noch mehr Strecken. [https://www.ferrovieinrete.com/trifase-varie.htm Hier] eine Karte der in Frage kommenden italienischen Strecken. --[[user:Rotkaeppchen68|R<span style="color:red">ô</span>tkæppchen₆₈]] 19:46, 25. Sep. 2020 (CEST)
::Hmm, die Italien-Bilderreihe, die wir von Morelló haben, ist wahrscheinlich recht unvollständig... Novara, Verona, Pavia, Milano, Turin, Florenz, Tivoli, Rom, Venedig, Pompeji... da passt manches. Was ist das da rechts, ein Kraftwerk? --[[Benutzer:Xocolatl|Xocolatl]] ([[Benutzer Diskussion:Xocolatl|Diskussion]]) 20:28, 25. Sep. 2020 (CEST)
:::Ich war ja wegen der Form der Isolatoren zuerst bei der Simplonbahn, werde aber nicht richtig fündig. Die Burgdorf-Thun-Bahn ist es nicht, da dort wegen der geringeren Betriebsspannung kleinere Isolatoren verwendet wurden. Porrettana-Bahn, Brennerbahn und die Strecke Rom–Sulmona kommen nicht in Frage, da diese erst nach dem Tod Morellós elektrifiziert oder in Betrieb genommen wurden. Es bleiben Simplon oder Veltlin. Ich halte das Gebäude für ein Unterwerk bzw eine Trafostation. --[[user:Rotkaeppchen68|R<span style="color:red">ô</span>tkæppchen₆₈]] 20:41, 25. Sep. 2020 (CEST)
:::(BK)Also das Bahnkraftwerk/Schaltposten wäre schon typische für italienische Drehstrombahnen, vom Bautypus her jedenfalls kein Widerspruch. Und die Norditalienische 15/16 2/3 Hz Netze brauchten auch Umformer, da war nichts mit „direkt über Trafo ab Ortsnetz“. Ergo braucht es da wie bei den Gleichstrombahnen auch ordentlich grosse Gebäude zur Stromversorgung. Das ist nur bei >40 Hz realistisch. Und es sieht mir doch verdächtig nach Normalspur und Vollbahn aus (wäre auch Italien). Isolatoren hingegen wurde in der Form auch am Simplon verwendet, allerdings in Brig ohne der gebogenen Halter (Auf der Südseite aber durchaus italienische Fahrleitungsmasten möglich). BTB würde ich jetzt Verneinen, 1. Isolatoren passen nicht (das waren einfach Isolatoren, ohne die hier sichtbaren kleinen "Querträger") 2. die haben in Bahnhöfen abgespannt, soll heissen da hingen die Isolatoren nicht an Tragarmen/Joche, sondern Querseilen. Grundsätzlich sagt mein Bauch Drehstromnetz-Oberitalien. --[[Benutzer:Bobo11|Bobo11]] ([[Benutzer Diskussion:Bobo11|Diskussion]]) 20:55, 25. Sep. 2020 (CEST)
::::Meine Googelei ergab, dass die [[Bahnstrecke Modane–Turin]] (Ferrovia del Frejus) zu Drehstromzeiten solchen Isolatoren verwendete. Das muss aber nicht heißen, dass sie nicht auch anderswo verwendet wurden. --[[user:Rotkaeppchen68|R<span style="color:red">ô</span>tkæppchen₆₈]] 21:18, 25. Sep. 2020 (CEST)
::::::(BK Einschieb)Die Art Isolatoren wurden auch am Simplon verwendet. Würde mich nicht erstaunen, wenn die [[Rete Adriatica]] auch auf diesen Typ Isolator gesetzt hätte. Denn es war ja dieselbe Spannung (3000 Volt) und ähnliche Frequenz (15,6 vs. 16 Hz) am Simplon wie in Oberitalien. Denn sonst hätte das mit Lokomotiven ausleihen, nicht funktioniert. Die beiden [[Drehstrombetrieb_Brig–Iselle#Fb_3/5_(später_Ae_3/5)_364–365|Fb 3/5]] waren ja abgezweigte FS Lokomotiven, und dazu kamen noch die drei Mietloks [[RA 361–363]] im Einsatz. Mit diesen fünf Lok wurde der elektrische Betrieb im Simplontunnel aufgenommen. --[[Benutzer:Bobo11|Bobo11]] ([[Benutzer Diskussion:Bobo11|Diskussion]]) 21:41, 25. Sep. 2020 (CEST)
:::::Dann nehmen wir doch an der Strecke [https://www.google.de/maps/@45.0736847,6.8884695,3a,75y,66.37h,78.6t/data=!3m6!1e1!3m4!1sIhSzN1rS0umIpJk5Q4MKYQ!2e0!7i13312!8i6656 Salbertrand]. Das [https://www.google.de/maps/@45.0736434,6.8890483,3a,75y,40.41h,89.48t/data=!3m6!1e1!3m4!1snMr2X_Y4jjTd-bhWOpTS6w!2e0!7i13312!8i6656 Gebäude] steht heute noch.--[[Benutzer:Mhunk|Mhunk]] ([[Benutzer Diskussion:Mhunk|Diskussion]]) 21:39, 25. Sep. 2020 (CEST)
::::::Scheint zu passen. --[[Benutzer:Bobo11|Bobo11]] ([[Benutzer Diskussion:Bobo11|Diskussion]]) 21:42, 25. Sep. 2020 (CEST)
:::::::[https://www.google.de/maps/@45.0749864,6.8875371,3a,75y,70.89h,90.88t/data=!3m6!1e1!3m4!1sLEexfWm-rgZ70_LjydfWUw!2e0!7i13312!8i6656 Hier] noch ein besserer Blick auf den Hintergrund.--[[Benutzer:Mhunk|Mhunk]] ([[Benutzer Diskussion:Mhunk|Diskussion]]) 21:48, 25. Sep. 2020 (CEST)
Bild 2:Vierwaldstättersee bei der Tellskapelle [https://www.google.de/maps/@46.938533,8.6106226,3a,75y,126.3h,91.47t/data=!3m8!1e1!3m6!1sAF1QipOooiTWlUfZm_bspsv0CqPUhOFwYW-gNDMo4P5s!2e10!3e11!6shttps:%2F%2Flh5.googleusercontent.com%2Fp%2FAF1QipOooiTWlUfZm_bspsv0CqPUhOFwYW-gNDMo4P5s%3Dw203-h100-k-no-pi-0-ya291.26453-ro0-fo100!7i8704!8i4352].--[[Benutzer:Mhunk|Mhunk]] ([[Benutzer Diskussion:Mhunk|Diskussion]]) 22:19, 25. Sep. 2020 (CEST)
:Ja, passt, die [[Tellskapelle]] ist erkennbar (auch hier mal wieder mit mehr Auflösung wäre es einfacher). Aber eben nicht auch da, das ist nicht Kanton Luzern, sondern Uri. --[[Benutzer:Bobo11|Bobo11]] ([[Benutzer Diskussion:Bobo11|Diskussion]]) 22:39, 25. Sep. 2020 (CEST)
::Es leben die Isolatorenkenner! Danke! - Mit dem dritten Bild ist's aber noch nicht weitergegangen, helft mir mal sehen: Wir haben hohen Baumbestand und viele geschwungene und sehr schlanke Laternenmasten (?), die innerhalb der Einfriedung im Rasen stehen, an denen aber entweder gar keine oder sehr unauffällige Lampen zu hängen scheinen. Dann das Blumenarrangement mit den breitblättrigen Palmen o. ä. mittendrin. Ein relativ kleines Fachwerkgebäude, an das sich aber ein Wandelgang o. ä. anschließt und auf dem ein Dachreiter mit Glocke (?) zu sehen ist. Ganz links glaube ich Menschen sitzen zu sehen... aufgereiht als Zuschauer oder an Cafétischen? Und was ist das Weiße links vor dem Haus? Eine Statue? Für mich sieht's wie die Gestalt einer überlebensgroßen Schlafwandlerin aus, und das ist es ganz bestimmt nicht;-) Rechts, vielleicht als Gegenstück dazu, eventuell eine große Glaskugel... --[[Benutzer:Xocolatl|Xocolatl]] ([[Benutzer Diskussion:Xocolatl|Diskussion]]) 16:15, 26. Sep. 2020 (CEST)
::: Es sind zwei Dachreiter, der zweite ist fast vollständig hinter dem Stamm verborgen. Ein botanischer Garten? [[Spezial:Beiträge/91.54.33.246|91.54.33.246]] 18:02, 26. Sep. 2020 (CEST)
::::Wenn's nicht überhaupt Türmchen/Kamine o. ä. dahinter sind. Hmmm, ein Gärtner- oder Heizungshaus, wenn botanischer Garten o. ä.? Und der "Wandelgang" könnte auch aus Volieren/Käfigen bestehen. Aber gefunden habe ich bislang nichts, was auch nur ansatzweise passt. --[[Benutzer:Xocolatl|Xocolatl]] ([[Benutzer Diskussion:Xocolatl|Diskussion]]) 20:28, 26. Sep. 2020 (CEST)
:::::Wenn Luzern dann wäre Dietschiberg naheliegend. Die Dreilindenhöhe ist es schon mal nicht, stilistisch zwar ähnlich aber eben nicht passend [https://docplayer.org/71431951-Stadt-luzern-liegenschaft-dreilinden-ausschreibung-ueber-die-vermietung-der-drei-gebaeude-haupthaus-oekonomiegebaeude-pfoertnerhaus.html]. Allerdings im INSA fand ich da nichts, aber solche kleinen Gebäude haben da in der Regel auch kein Foto. Und so was kann fast überall stehen. Und ja Xocolatl, ich tendiere auch auf ein Nebengebäude. --[[Benutzer:Bobo11|Bobo11]] ([[Benutzer Diskussion:Bobo11|Diskussion]]) 01:25, 27. Sep. 2020 (CEST)
::::::Wie gesagt, die Ortsangaben kann man bei dieser Datenbank meist den Hasen geben. Entweder wurde da schon beim Erstellen phantasiert oder es sind später Zeilen verrutscht oder so... Wahrscheinlich muss man sich eher an die Architektur und die Pflanzen und die Skulptur (?) halten, wenn man das lokalisieren will. Aber schauen wir mal noch ein paar Morelló-Stereobilder an, die mit dem Bild in Zusammenhang stehen könnten, falls Luzern doch stimmt: --[[Benutzer:Xocolatl|Xocolatl]] ([[Benutzer Diskussion:Xocolatl|Diskussion]]) 09:26, 27. Sep. 2020 (CEST)


= 23. Februar 2017 =
<gallery>
Datei:Alguns edificis de Lucerna des d'un balcó.jpeg|Nr. 4 soll auch im Kanton Luzern entstanden sein, ist aber auch noch nicht lokalisiert.
Datei:Edifici de l'oficina de correus a Lucerna.jpg|Nr. 5 stellt die Hauptpost in Luzern dar, er hat also mit Sicherheit in dieser Stadt Stereoskopbilder gemacht. Auf der Kappellbrücke war er ja übrigens, wie wir schon wissen, ebenfalls mit dieser Technik unterwegs.
Datei:Part de la ciutat de Lucerna amb vegetació en primer terme.jpeg|Nr. 6 wird schon in der Quelle als ein Teil der Stadt Luzern bezeichnet, ließe sich aber vielleicht auch präzisieren.
</gallery>


== Trappist-1 - wie funktioniert das mit der Gravitation? ==
Was mich immer noch irritiert, sind diese "Bischofsstäbe" innerhalb der eigentlichen Einfriedung. Ob man da Rankhilfen einhängen konnte? Oder temporär einen Käfig errichten? Ein Bewässerungssystem? Für irgendwas, was mit Beleuchtung zu tun hat, stehen sie ja eigentlich zu eng und an einem seltsamen Ort. --[[Benutzer:Xocolatl|Xocolatl]] ([[Benutzer Diskussion:Xocolatl|Diskussion]]) 09:43, 27. Sep. 2020 (CEST)
:Zu Bild 6: Hypothese: Vom Aussichtsturm oberhalb des [[Gletschergarten Luzern]]. Man sieht, wenn richtig, einen Teil der [[Museggmauer]].--[[Benutzer:Mhunk|Mhunk]] ([[Benutzer Diskussion:Mhunk|Diskussion]]) 13:03, 27. Sep. 2020 (CEST)
::Anmerkung, der SUVA-Hauptsitz, der heute die Sichtachse dominiert, wurde erst 1914/1915 gebaut.--[[Benutzer:Mhunk|Mhunk]] ([[Benutzer Diskussion:Mhunk|Diskussion]]) 13:13, 27. Sep. 2020 (CEST)
::: Noch eigenem Suchen vermute ich bei Bild 6 auch eine Aufnahme etwa vom Gletschergarten und möglicherweise mit dem Spitzdach eines Turms der Stadtmauer. Das ist das einzige Tal, das ich gefunden habe und passen könnte. Bild 4 könnte auch etwa vom Gletschergarten in Richtung West-Nordwest aufgenommen sein. Wegen der großen Veränderung in der Bebauung fehlen mir alte Bilder zur Identifikation. [[Spezial:Beiträge/91.54.33.246|91.54.33.246]] 13:35, 27. Sep. 2020 (CEST)
::::zu Bild 4: von der [https://map.geo.admin.ch/?lang=de&topic=ech&bgLayer=ch.swisstopo.pixelkarte-farbe&layers=ch.swisstopo.zeitreihen,ch.bfs.gebaeude_wohnungs_register,ch.bav.haltestellen-oev,ch.swisstopo.swisstlm3d-wanderwege,ch.swisstopo.lubis-luftbilder_schwarzweiss&layers_opacity=1,1,1,0.8,1&layers_visibility=true,false,false,false,false&layers_timestamp=19031231,,,,99991231&E=2665939.35&N=1212380.64&zoom=10 Bebauung] her könnte es hinkommen.--[[Benutzer:Mhunk|Mhunk]] ([[Benutzer Diskussion:Mhunk|Diskussion]]) 13:52, 27. Sep. 2020 (CEST)
::::: Ein [[:Datei:ETH-BIB-Luzern-LBS H1-015180.tif|Luftbild von 1953]] zu Bild 4, leider von Süden aufgenommen. [[Spezial:Beiträge/91.54.33.246|91.54.33.246]] 14:13, 27. Sep. 2020 (CEST)
::::: Nochmal Bild 4: In aktuellen Luftbildern (z.B. map.geo.admin.ch) kann man die Dächer einzelner Häuser vergleichen. Zürichstr. 65: das Walmdach mit den 4 kleinen Gauben (eine ist heute größer); Zürichstr. 69: ein breiteres Haus mit kürzerem Dachfirst; Zürichstr. 71: Dachterrasse. Fluhmattstr. 44: Satteldach mit Seitengiebel, Fluhmattstr. 48: Krüppelwalmdach mit zwei Dachgauben und Seitengiebel (ebenfalls Krüppelwalmdach); zwischen beiden Häusern ist eine Terrasse zu erkennen, im Bild 4 ein eingeschossiger Bau mit Flachdach. Es paßt also: Bild 4 ist etwa vom Gletschergarten in Richtung West-Nordwest aufgenommen. [[Spezial:Beiträge/91.54.33.246|91.54.33.246]] 14:30, 27. Sep. 2020 (CEST)
::::::Ich tendiere aber dazu, dass das Bild 4 von einem Gebäude oberhalb des [[Aussichtsturm Gletschergarten]] aufgenommen wurde. Der Turm liegt zu niedrig und hat auch keine entsprechenden Balkons.--[[Benutzer:Mhunk|Mhunk]] ([[Benutzer Diskussion:Mhunk|Diskussion]]) 14:40, 27. Sep. 2020 (CEST) Wenn ich auf der Karte die Sichtachse abschätze, komme ich beim Kapuzinerkloster raus. Dieses hätte auch die passende Höhenlage. Da das Kloster keine Balkons hat, wohl eher eines der heute nicht mehr existenten Gebäude davor. Vielleicht war eines davon ein Gasthof.--[[Benutzer:Mhunk|Mhunk]] ([[Benutzer Diskussion:Mhunk|Diskussion]]) 14:53, 27. Sep. 2020 (CEST)
::::::: Meiner Meinung nach paßt die Richtung von Aussichtsturm Gletschergarten ziemlich gut. Auf diesem [[:Datei:Gletschergartenturm 360 Panorama.jpg|Panoramabild]] sind Häuser an der Zürichstraße leider durch neuere Gebäude verdeckt. Und die hinteren Häuser durch den Ast (etwa in der Bildmitte), der bis in den Himmel ragt. Aber links von diesem Ast meine ich in dem roten Haus das Haus Fluhmattstr. 44 und rechts vom Ast das Haus Fluhmattstr. 48 zu erkennen. Die Perspektive ist ähnlich wie in Bild 4. [[Spezial:Beiträge/91.54.33.246|91.54.33.246]] 15:17, 27. Sep. 2020 (CEST)
::::::: Ergänzung: Die Position in [[Kloster Wesemlin]] war nicht der Aufnahmestandort. Von dieser Position können die Häuser an der Zürichstraße nur von Osten/Nordosten aufgemommen werden. In Bild 4 sind aber auch die Südseiten sichtbar. [[Spezial:Beiträge/91.54.33.246|91.54.33.246]] 16:53, 27. Sep. 2020 (CEST)
::::::::Schade, bezüglich der eigentlich gesuchten Location bringt uns das leider nicht weiter... aber immerhin gut, dass man jetzt weiß, dass Nr. 4 und Nr. 6 wirklich zu Luzern gehören! --[[Benutzer:Xocolatl|Xocolatl]] ([[Benutzer Diskussion:Xocolatl|Diskussion]]) 16:22, 29. Sep. 2020 (CEST)
zurück zu Bild 3: Ich habe mich gefragt, ob es sich nicht um ein Taubenhaus o.ä. handeln könnte. Diese waren um 1900 teilweise ähnlich aufwändig gebaut. Das würde die komischen Proportionen erklären. Und ich habe mich immer gefragt, was diese "Öffnungen" im 1. Stock sind. Rechts davon könnte ein Entenhaus sein. Auf jeden Fall scheint dahinter noch ein Gebäude zu sein. Es ist links ansatzweise erkennbar.--[[Benutzer:Mhunk|Mhunk]] ([[Benutzer Diskussion:Mhunk|Diskussion]]) 07:54, 1. Okt. 2020 (CEST)
:Mhm... mit "Taubenschlag" hatte ich es schon probiert, aber das ist natürlich ein sehr blöder Suchbegriff. "Taubenhaus" hat bislang leider auch noch nicht wirklich weitergeholfen, "Voliere" und "Fasanerie" auch nicht, wobei für eine Fasanerie die Baulichkeiten auch nicht wirklich passen... ich vermute aber schon so etwas Ähnliches wie einen (privaten?) Zoo. In "normalen" Parks sind ja die Rasenflächen meist nicht so martialisch eingezäunt. Man kann sein Glück mal mit Ortschaften probieren, in denen Morelló sonst noch Stereobilder gemacht hat. [[Thun]] hatte einst einen Kursaal, der vom Stil her nicht ganz unpassend wäre, aber der Park haut wohl nicht hin... --[[Benutzer:Xocolatl|Xocolatl]] ([[Benutzer Diskussion:Xocolatl|Diskussion]]) 17:55, 1. Okt. 2020 (CEST)
:: Zum Taubenhaus passen auch die Bretter unter den Fenstern, die die runden Schatten auf die Wand werfen. Und sind auf dem Dachfirst auch Tauben? Meine Suche in Luzern brachte mich zum [https://www.hirschmatt-neustadt.ch/de/neustadtplan/voegeligaertli/ Vögeligärtli]. Die Voliere hatte zwar von 1901 bis 1908 einen Vorgängerbau, von dem ich kein Bild gefunden habe. Aber ist Bild 3 mitten in der Stadt aufgenommen worden? [[Spezial:Beiträge/91.54.33.246|91.54.33.246]] 18:33, 1. Okt. 2020 (CEST)
:::Wenn das auf dem Dach auch Tauben sind, ist das Häuschen wirklich winzig. Ob wir es hier mit einem Märchengarten mit Aschenputtelfigur oder sowas zu tun haben? Oder sind da nur ein bis zwei Passantinnen in komischer Haltung unterwegs? Ich frage mich auch immer noch, was man in diese Dinger in dem eingezäunten Garten eingehängt hat. Lampions? - Beim Vögeligärtli war ich neulich auch mal gelandet, konnte da aber keine Ähnlichkeiten entdecken. Ob diese Ornithologische Gesellschaft irgendwas Aufschlussreiches hinterlassen hat? --[[Benutzer:Xocolatl|Xocolatl]] ([[Benutzer Diskussion:Xocolatl|Diskussion]]) 19:56, 1. Okt. 2020 (CEST)


Möglicherweise ganz einfach, aber meine 3-D-Vorstellungskraft ist gerade überfordert: Wie schaffen es die sieben Planeten ähnlicher Größe auf so nahen Orbits, dass sie nicht kollidieren, bzw. wie muss ich mir ihre Bewegungen zueinander vorstellen? [[Spezial:Beiträge/89.12.251.193|89.12.251.193]] 16:36, 23. Feb. 2017 (CET)
= 27. September =


<s>[[File:Umlaufbahnen_Planeten.svg|mini|Planetenbahnen]]
== Bohemian Rhapsody ==
:Man muss sich gar nicht viel 3D vorstellen, da die Bewegung der Planeten im Wesentlichen in einer Ebene abläuft. Und im Wesentlichen laufen sie auf konzentrischen Kreisen (im Zentrum ist die Sonne; in Wirklichkeit sind es Ellipsen, aber das macht keinen großen Unterschied, siehe Grafik rechts). Sie können sich also nicht in die Quere kommen. Die Orbits sind auch auch nicht nah. Auf vielen Darstellungen sind die Abstände im Vergleich zu den Größen viel zu klein dargestellt (bzw. die Größen zu groß), da man sonst die Größen oder die Abstände nicht wirklich erkennen könnte. Die Erde hat z.B. einen Durchmesser von ca. 13 000 km; der Abstand zu den Nachbarplaneten beträgt aber mindestens 30 Mio. km. Die Planeten sind von der Erde alle soweit entfernt, dass sie mit bloßem Auge nur als Punkt zu sehen sind. Im Gegensatz etwa zum Mond, der kleiner als alle Planeten ist, aber deutlich größer zu sehen ist, weil wesentlich näher (ca. 400 000 km), was immer noch 60mal soviel ist wie der Erdradius. --[[Benutzer:Digamma|Digamma]] ([[Benutzer Diskussion:Digamma|Diskussion]]) 16:46, 23. Feb. 2017 (CET)


::Ergänzung: [[Planetenweg]]e helfen, sich die Größenverhältnisse vorzustellen. Im häufig verwendeten Maßstab von 1 : 1 Milliarde hat die Erde einen Durchmesser von 1,3 cm. Der Abstand zur Venus beträgt mindestens 38 m, der zum Mars mindestens 55 m. --[[Benutzer:Digamma|Digamma]] ([[Benutzer Diskussion:Digamma|Diskussion]]) 17:04, 23. Feb. 2017 (CET)</s>
Gibt es einen Ersatzlink zu http://einestages.spiegel.de/static/entry/clip_clip_hurra/89736/queen_bohemian_rhapsody.html (tot) aus https://de.wikipedia.org/wiki/Bohemian_Rhapsody#cite_ref-spiegel_20-0 ? --[[Benutzer:Dr Lol|Dr Lol]] ([[Benutzer Diskussion:Dr Lol|Diskussion]]) 11:11, 27. Sep. 2020 (CEST)
:::Vielen Dank; allerdings ging es mir um das Trappist-1-System, und dort sollen die sieben Planeten alle innerhalb der Merkur-Bahn passen. Aber wahrscheinlich ist trotzdem mehr Platz, als ich mir vorgestellt hatte, auch wenn es heißt, man könnte dort die Nachbarplaneten in Mondgröße am Himmel bewundern. [[Spezial:Beiträge/89.12.251.193|89.12.251.193]] 17:09, 23. Feb. 2017 (CET)
:[http://web.archive.org/web/20131111082454/http://einestages.spiegel.de/static/entry/clip_clip_hurra/89736/queen_bohemian_rhapsody.html?o=position-ASCENDING&s=0&r=48&a=22713&of=4&c=1] --[[Spezial:Beiträge/134.3.35.36|134.3.35.36]] 14:50, 28. Sep. 2020 (CEST)
::::Die kleinen Bahnen ergeben sich, weil die Zentralsonne ein [[Roter Zwerg]] ist, der weit weniger Masse hat als unsere Sonne. Laut [[TRAPPIST-1|Wiki-Artikel]] rund 8 %. Die Oberflächentemperatur ist weit niedriger und die Strahlungsdichte ebenfalls, daher ist die habitable Zone weit näher am Stern. --[[Benutzer:Elrond|Elrond]] ([[Benutzer Diskussion:Elrond|Diskussion]]) 18:24, 23. Feb. 2017 (CET)


:schau mal unter https://www.spiegel.de/geschichte/30-jahre-mtv-a-947214.html Grüße --[[Benutzer:H-stt|h-stt]] [[Benutzer Diskussion:H-stt|<small>!?</small>]] 21:20, 30. Sep. 2020 (CEST)
:<s>Missverständnis. (Schreib doch ganze Sätze.) --[[Benutzer:Digamma|Digamma]] ([[Benutzer Diskussion:Digamma|Diskussion]]) 17:12, 23. Feb. 2017 (CET)</s>
::Linkservice: [[TRAPPIST-1]] --[[Benutzer:Neitram|<span style="color:#008800">Ne</span><span style="color:#005555">it</span><span style="color:#005588">ra</span><span style="color:#0000FF">m</span>]]&nbsp;[[Benutzer_Diskussion:Neitram|<span style="font-size:large">✉</span>]] 17:28, 23. Feb. 2017 (CET)


:::Sorry, das "Trappist 1" in der Überschrift habe ich überlesen. Die Sache ist vermutlich wirklich komplizierter, denn der gegenseitige Gravitationseinfluss der Planeten untereinander ist hier wahrscheinlich nicht zu vernachlässigen. Im Artikel zu [[:en:Trappist 1]] in der englischen Wikipedia steht, dass die Umlaufdauern der Planeten in Resonanz stehen. Wenn ich das richtig verstehe, bedeutet dies u.a. dass die Bahnstörungen durch die Gravitation der andern Planeten sich rausmittelt und die Bahnen nicht instabil macht. --[[Benutzer:Digamma|Digamma]] ([[Benutzer Diskussion:Digamma|Diskussion]]) 17:30, 23. Feb. 2017 (CET)
== Welche Seitenlänge hat ein Würfel ... ==
::::Vielleicht hilft dem Fragesteller ja auch schon der Satz aus dem Artikel: "Der Aufbau dieses Planetensystems ähnelt somit eher dem System der [[Liste der Jupitermonde|Jupitermonde]] als unserem Sonnensystem"? --[[Benutzer:Neitram|<span style="color:#008800">Ne</span><span style="color:#005555">it</span><span style="color:#005588">ra</span><span style="color:#0000FF">m</span>]]&nbsp;[[Benutzer_Diskussion:Neitram|<span style="font-size:large">✉</span>]] 17:32, 23. Feb. 2017 (CET)
[[Datei:Zollstock&Winkel2.JPG|mini|Exakt nachgemessen!]]
[[Datei:Blanc.png|mini|1 mol gasförmiger Sauerstoff]]
...festen Sauerstoffs am absoluten Nullpunkt, der bei 0°C ein Raumvolumen von 1 Kubikmeter einnimmt/ausfüllt? Wie berechnet man das?
--[[Spezial:Beiträge/91.141.0.192|91.141.0.192]] 11:14, 27. Sep. 2020 (CEST)
:Sauerstoff hat eine kritische Temperatur von −118,4 °C. Bei 0 °C gibt es Sauerstoff nur als Gas. Um Deine Frage beantworten zu können fehlt daher der Druck, den der Sauerstoff bei 0 °C haben soll. Ansonsten: 0 °C liegt so weit über der kritischen Temperatur, dass Du Stoffmenge und Masse ganz gut über die allgemeine Gasgleichung und die molare Masse von Sauerstoff ausrechnen kannst. Dann musst Du nur noch die Dichte von festem Sauerstoff herausfinden. Daraus berechnest Du dann das Volumen bei 0 K und mit ein bisschen Geometrie rechnest Du die Kantenlänge des volumengleichen Würfels aus. --[[user:Rotkaeppchen68|R<span style="color:red">ô</span>tkæppchen₆₈]] 11:33, 27. Sep. 2020 (CEST) Erg: Im Artikel [[:en:Solid oxygen]] steht doch tatsächlich die Dichte festen Sauerstoffs: 21 bis 23,5 cm³/mol. --[[user:Rotkaeppchen68|R<span style="color:red">ô</span>tkæppchen₆₈]] 11:42, 27. Sep. 2020 (CEST)


::::: Wie das mit der Gravitation funktioniert, weiß ich auch nicht. Aber wenn der Merkur 0,3 bis 0,5 Astronomische Einheiten weit von der Sonne entfernt ist, und die sieben Planeten zwischen 0,01 und 0,06 Astronomischen Einheiten um ihren Stern kreisen, dann ist dazu wohl genug Platz. Jedenfalls kann ich mir das so grad noch vorstellen. fz [[Benutzer Diskussion:Jahn Henne|<small><small>J</small></small>a<small><small>H</small></small>n]] 00:48, 24. Feb. 2017 (CET)
Bei Normalbedingungen (0 °C, 1013 mbar) verteilen sich 1,429 kg Sauerstoff auf einen Kubikmeter. Bei einer molaren Masse von <s>15,9994 g/mol sind das 89,3 mol</s> 31,9988 g/mol bei O<sub>2</sub> sind das 44,66 mol. Laut [[:en:solid oxygen]] hat 1 mol Sauerstoff bei 0 K ein Volumen von 21 Kubikzentimeter. Ziehe die Kubikwurzel von 44,66 * 21. --[[Spezial:Beiträge/188.61.132.244|188.61.132.244]] 11:59, 27. Sep. 2020 (CEST)
:<small>Beitrag wurde später angepasst, [https://de.wikipedia.org/w/index.php?title=Wikipedia:Auskunft&diff=204029514&oldid=204029501 diff] [[Benutzer:Victor_Schmidt|Victor Schmidt]] <small>[[Benutzer_Diskussion:Victor_Schmidt|Was auf dem Herzen?]]</small> 16:17, 27. Sep. 2020 (CEST)</small>
:Wer den Fehler findet, darf ihn behalten. --[[user:Rotkaeppchen68|R<span style="color:red">ô</span>tkæppchen₆₈]] 12:03, 27. Sep. 2020 (CEST)
::Das stimmt so. Weil 1. mach ich nie Fehler und 2. habe ich es nachgemessen. --[[Spezial:Beiträge/188.61.132.244|188.61.132.244]] 12:14, 27. Sep. 2020 (CEST) ...und 3. habe ich den Fehler doch längst korrigiert :) --[[Spezial:Beiträge/188.61.132.244|188.61.132.244]] 15:08, 27. Sep. 2020 (CEST)
:::Aha...für Dich ist Sauerstoff also ein Edelgas? --[[Benutzer:Blutgretchen|Blutgretchen]] ([[Benutzer Diskussion:Blutgretchen|Diskussion]]) 12:24, 27. Sep. 2020 (CEST)
:::: Ich frage mich eher, wo denn die 1,429 kg vom Himmel gefallen sind. Das kommt zwar hin, weil Sauerstoff ein bißchen schwerer als Luft mit 1,2 kg ist, aber eigentlich kommt bei den Gasgesetzen bzw. Avocado oder wie der heißt doch direkt das Molvolumen raus. Und die Doppelbindung: wie hat man sich das eigentlich im Kristall vorzustellen? Da gibt es doch keine Zweiatomigkeit mehr. (Wobei das für die Fragestellung auch egal ist, aber was ist "1 mol Sauerstoff_sol"?) --[[Spezial:Beiträge/95.116.16.179|95.116.16.179]] 12:48, 27. Sep. 2020 (CEST)
:::::Die 1,429 kg stehen in unserem [[Sauerstoff]]-Artikel, rechts in der Box unter den physikalischen Eigenschaften. Warum gibt es im Kristall keine Zweiatomigkeit? Auch ein Wassermolekül ist im Kristall ein Wassermolekül mit 3 Atomen. --[[Benutzer:Blutgretchen|Blutgretchen]] ([[Benutzer Diskussion:Blutgretchen|Diskussion]]) 13:04, 27. Sep. 2020 (CEST)
::::::Wasser besteht aber aus Atomen verschiedener Sorten, es gibt da zwei verschiedene Bindungsmechanismen, die sich unterscheiden lassen: einmal die im Moleküle (also H-O-H, die es ja schon bei Gas/flüssig gibt, exakt sind das Kovalenze Bindungen) und dann noch die zwischen den Molekülen (Molekül-Molekül, bei fest und kurzzeitig/schwach bei flüssig, exakt sind das Wasserstoffbrücken-Bindungen). Bei Festkörpern aus nur einer Atomsorte (O) können dagegen keine Unterscheidung getroffen werden, ob ein Atom jetzt mit einem Nachbaratom innerhalb des eigenen (O-O) oder mit einem des nächsten "Moleküls" (O-O) verbindet, lässt sich nicht sagen, da es der selbe Bindungsmechanismus (z.B. nur Kovalenze Bindung) ist.--[[Benutzer:Naronnas|Naronnas]] ([[Benutzer Diskussion:Naronnas|Diskussion]]) 15:03, 27. Sep. 2020 (CEST)
:::::::Die Bindung, die die Sauerstoffmoleküle im Kristall zusammenhält, ist ganz sicher keine kovalente Bindung. Sonst läge der Schmelzpunkt wesentlich höher. --[[Benutzer:Digamma|Digamma]] ([[Benutzer Diskussion:Digamma|Diskussion]]) 19:47, 27. Sep. 2020 (CEST)
:::::::: Irgendwie überzeugt mich das nicht. Ich glaube, daß es in Eis keine Wasser- und in festem Sauerstoff keine Sauerstoffmoleküle gibt. Wenn die Substanzen schmelzen, gibt es eine auch energetische Umgruppierung: Beim Sauerstoff gehen je zwei O-Atome eine feste kovalente Bindung ein, die dabei freiwerdende Bindungsenergie wird benutzt, um die feste kovalente Bindung im Kristallverbund aufzubrechen, deshalb ist makroskopisch die Schmelzwärme und damit die Schmelztemperatur niedrig. Ähnliches passiert beim Schmelzen von Wassereis: Die Wassermoleküle entstehen erst beim Schmelzen, im Eiskristall existieren sie nicht. Wie sehen denn die Kristallgitter aus, wo sollen denn da die Molekülbindungen sein? --[[Spezial:Beiträge/95.112.231.60|95.112.231.60]] 12:54, 28. Sep. 2020 (CEST)
::::::::::Wenn Du das sicher belegen kannst, hast Du mit einiger Sicherheit recht schnell einen Termin mit dem schwedischen König! Deinen Glauben kann ich DIr nicht nehmen, aber u.a. Röntgenbeugung und ähnliches Zeugs hat ziemlich klar gezeigt, dass in Molekülkristallen selbige als solche vorliegen. Und nur zur Information, lies mal den Artikel [[London-Kraft]]. --[[Benutzer:Elrond|Elrond]] ([[Benutzer Diskussion:Elrond|Diskussion]]) 13:03, 28. Sep. 2020 (CEST)
::::::::::: Wie wollte ihre bei Wasser einzelne Moleküle voneinander abgrenzen? Ich habe gelernt, daß Wasser letztlich ein Cluster aus H und O im Verhältnis 2:1 ist, bei dem die Bindungselektronen den O näher sind als den H. --[[Spezial:Beiträge/94.219.186.65|94.219.186.65]] 00:43, 30. Sep. 2020 (CEST)
::::::::::::Die Sauerstoff- und Wassserstoffatome sind durch [[Atombindung]]en/[[kovalente Bindung]]en miteinander verbunden. Die einzelnen Wassermoleküle in den [[Wassercluster]]n sind durch [[Wasserstoffbrückenbindung]]en miteinander verbunden. Diese zwei Bindungsarten lassen sich problemlos voneinander abgrenzen. --[[user:Rotkaeppchen68|R<span style="color:red">ô</span>tkæppchen₆₈]] 09:54, 30. Sep. 2020 (CEST)
::::::::::::: Du denkst in die falsche Richtung. In dem Glas Wasser vor mir sind H und O (2:1), aber die sind einander jeweils nicht fest zugeordnet, sondern das ist schlimmer als im Swingerclub, weil da nicht nacheinander, sondern gleichzeitig jeder mit jedem, der neben dran ist, rummacht. Wie ich bereits schrob, sind die Bindungselektronen währenddessen näher am O, deshalb sind das kovalente Bindungen, aber dem O ist es völlig schnuppe, welches der umher wandernden Protonen Ladungsausgleich spielt, denn das ist bereits mit einem H als Spielpartner sehr zufrieden, kann locker auch drei Protonen zur gleichen Zeit mitführen und ist dabei letzlich immer zufrieden. Stell Dir zur Veranschaulichung [https://www.u-helmich.de/che/lexikon/W/bilderW/wassermolekuel1.jpg diese Grafik] in 3D x-fach dicht an dicht vor und erkenne, wie leicht es für ein Proton ist, eine Bindung zu verlassen und es mit einem anderen O zu machen. --[[Spezial:Beiträge/178.4.185.67|178.4.185.67]] 13:20, 1. Okt. 2020 (CEST)
::::::::::::::Trotzdem gibt es die starken Bindungkräfte (kovalente und ionische Bindungen) die mittelstarken (Wasserstoffbrücken) und schwachen (u.a. die oben genannten Londonkräfte). Dass sich in Wasser die Protonen so schön schnell austauschen liegt an der [[Autoprotolyse]] des Wassers. --[[Benutzer:Elrond|Elrond]] ([[Benutzer Diskussion:Elrond|Diskussion]]) 12:16, 2. Okt. 2020 (CEST)


:::: (Gedächtnisfragment) Es gibt auch in userem Sonnensystem ein Mondpaar deren Bahnen so nah beieinander sind das sie regelmäßig kollidieren müssten. Irgendwie wird das dadurch "gelöst" das der Impuls des von hinten aufholenden auf den vorderen übertragen worauf der vordere schneller wird und irgendwann selber von hinten kommt. Weis jemand welche Monde/Planet das ist? --[[Benutzer:Dreifachaxel|Dreifachaxel]] ([[Benutzer Diskussion:Dreifachaxel|Diskussion]]) 13:31, 24. Feb. 2017 (CET)


::::: Das sind die beiden Saturnmonde [[Janus (Mond)|Janus]] und [[Epimetheus (Mond)|Epimetheus]]. --[[Benutzer:Gretarsson|Gretarsson]] ([[Benutzer Diskussion:Gretarsson|Diskussion]]) 14:09, 24. Feb. 2017 (CET)
:::::(BK)Das ist der Unterschied zwischen 0 °C und 20 °C. Bei 0 °C hat Sauerstoff 1,42895 kg/m³. Bei 20 °C sind es laut GESTIS-Stoffdatenbank 1,33 kg/m³ <small>(Ich halte das „mg“ dort für einen Schreibfehler)</small>. Luft hat bei 20 °C 1,204 kg/m³, bei 0 °C sind es 1,293 kg/m³. Alle Werte bei 101325 Pa. Du bringst also die Bezugstemperaturen 0 °C und 20 °C durcheinander. --[[user:Rotkaeppchen68|R<span style="color:red">ô</span>tkæppchen₆₈]] 13:09, 27. Sep. 2020 (CEST)


::::: Es kann höchstens der hintere schneller werden. Abstoßende Impulsübertragung ohne Berührung wie bei elastischem Stoß oder wie bei gleich gepolten elektrischen oder magnetischen Feldern gibt es nicht und die Gravitation, die Ursache des Ganzen, wirkt immer nur anziehend. --[[Benutzer:Gerold Broser|Geri]],[[BD:Gerold Broser|&nbsp;✉&nbsp;]] 18:37, 25. Feb. 2017 (CET)
:Der Fehler liegt in der Annahme, dass Sauerstoff einatomig vorliegt. Die IP hat sich bei der Berechnung der Molzahl pro Normalkubikmeter um den Faktor 2 verrechnet (es sind abgerundet 44,6 mol pro Kubikmeter). --[[Benutzer:Blutgretchen|Blutgretchen]] ([[Benutzer Diskussion:Blutgretchen|Diskussion]]) 14:42, 27. Sep. 2020 (CEST)
:::::: <small>[[Hufeisenumlaufbahn]], [[Janus (Mond)#Bahnverhalten von Janus und Epimetheus]]. --[[Benutzer:Neitram|<span style="color:#008800">Ne</span><span style="color:#005555">it</span><span style="color:#005588">ra</span><span style="color:#0000FF">m</span>]]&nbsp;[[Benutzer_Diskussion:Neitram|<span style="font-size:large">✉</span>]] 10:56, 28. Feb. 2017 (CET)</small>


:::::: Es handelt sich um eine gravitative Übertragung von Bahnernergie und -impuls, ohne dass die Monde sich berühren. Der Hintere wird bei Annäherung an den Vorderen beschleunigt und steigt daher auf eine etwas höhere Umlaufbahn. Der vordere wird hingegen abgebremst und sinkt auf eine etwas tiefere Umlaufbahn. Da die Bahngeschwindigkeit auf der tieferen ULB größer ist als auf der höheren, entfernt sich der Vordere wieder und wird irgendwann selbst zum Hinteren, weil er den nun auf der höheren, langsameren ULB befindlichen Mond einholt. Dann beginnt das Spiel von neuem. --[[Benutzer:Gretarsson|Gretarsson]] ([[Benutzer Diskussion:Gretarsson|Diskussion]]) 21:12, 1. Mär. 2017 (CET)
Aber die allgemeine Gasgleichung gilt doch für ein ideales Gas und deswegen ist der [[Absoluter Nullpunkt|absolute Nullpunkt]] definiert wie er definiert ist und bei -273,15°C, weil das Volumen dann Null ist?!?
--[[Spezial:Beiträge/178.115.130.43|178.115.130.43]] 19:05, 27. Sep. 2020 (CEST)
:[[Sauerstoff]] ist aber bei tiefen Temperaturen kein ideales Gas (Schmelzpunkt 54,8 K, Siedepunkt 90,15 K, K-Punkt 154,7 K/50,4 bar). Da macht sich das Eigenvolumen der Moleküle deutlich bemerkbar. --[[user:Rotkaeppchen68|R<span style="color:red">ô</span>tkæppchen₆₈]] 19:44, 27. Sep. 2020 (CEST)
::Das ist er schon bei Standardbedingungen nicht, Schon da braucht es für genaue Ergebnisse die [[Van-der-Waals-Gleichung]]. --[[Benutzer:Elrond|Elrond]] ([[Benutzer Diskussion:Elrond|Diskussion]]) 21:25, 27. Sep. 2020 (CEST)
:Ich hab noch einen Ansatz für die Lösung des Problems per Kopfrechnen. Das [[Molares Volumen|molare Volumen]] von Sauerstoff ist bei 0 °C und 101325 Pa ca. 22,4 Liter. Das molare Volumen festen Sauerstoffs ist ca. 21 bis 23,5 cm³. Das ist grob ein Tausendstel davon. Demzufolge hat ein Kubikmeter Sauerstoff nach dem Einfrieren nur noch ca. ein Tausendstel des ursprünglichen Volumens. Der Würfel hat dann nur noch ein Zehntel der ursprünglichen Kantenlänge, also ca. 10 cm. Nach obiger Rechnung sind es genau 9,79 bis 10,2 cm. --[[user:Rotkaeppchen68|R<span style="color:red">ô</span>tkæppchen₆₈]] 01:19, 28. Sep. 2020 (CEST)


= 25. Februar 2017 =
== Frage der Atommüllentsorgung ==


== Wieso überschätzen wir seltene Ereignisse (Terroranschläge, Flugzeugabstürze) dermaßen? ==
Als 1962 das erste Kernkraftwerk in Deutschland ans Netz ging, hat sich da niemand Gedanken darüber gemacht, wie man den zwangsläufig entstehenden Atommüll entsorgen will? Mir scheint es, als jemand der etliche Jahrzehnte später geboren ist, als wären die Verantwortlichen damals völlig naiv gewesen. --[[Spezial:Beiträge/95.175.104.31|95.175.104.31]] 23:30, 27. Sep. 2020 (CEST)
:Wieviele von den Entscheidern für Billigstrom leben heute noch? --[[Spezial:Beiträge/91.1.215.49|91.1.215.49]] 23:35, 27. Sep. 2020 (CEST)
Es war wohl eine Mischung aus Gier, Leichtgläubigkeit, geschicktem Lobbyismus, Unwissenheit und anderen negativen Komponenten. Dass wir das jetzt auszubaden haben und in anderen Ländern das Bad noch strammer gefüllt ist und sogar noch weiter gefüllt wird zeigt, dass oben genannte Beweggründe immer noch wirken. --[[Benutzer:Elrond|Elrond]] ([[Benutzer Diskussion:Elrond|Diskussion]]) 23:41, 27. Sep. 2020 (CEST)
:Ist das heutzutage anders? Beispiel China: Baut seit 1985 Kernreaktoren zur Energiegewinnung, hat derzeit über vierzig in Betrieb und plant ein dutzend weitere zu bauen. --[[Spezial:Beiträge/84.190.199.213|84.190.199.213]] 23:49, 27. Sep. 2020 (CEST)
:Man hat sofort begonnen, Möglichkeiten der Endlagerung zu untersuchen, siehe [[Schachtanlage Asse#Einlagerungsphase 1965 bis 1978]]. --[[Benutzer:BlackEyedLion|BlackEyedLion]] ([[Benutzer Diskussion:BlackEyedLion|Diskussion]]) 00:08, 28. Sep. 2020 (CEST)
::Wenn ich eine Anlage zur Produktion von chemischen Produkten beantragen will, muss ich '''vor''' einer Inbetriebnahme im Rahmen des Genehmigungsverfahrens bis ins Detail nachweisen, was mit den Abfallstoffen passiert und das seit mindestens 30 Jahren. Nicht nur in D, ich habe solche Verfahren in mehreren Ländern begleitet, in der EU, den USA und Südostasien. In letzterer Ecke kommt es darauf an, in Südkorea ist es kaum anders, eher strenger als in D. Nur bei den wirklich gefährlichen Abfällen der Kernspaltung wird nahezu überall davon abgesehen, ein Beispiel von hervorragender Lobbyarbeit (oder was auch sonst immer nützlich ist). --[[Benutzer:Elrond|Elrond]] ([[Benutzer Diskussion:Elrond|Diskussion]]) 00:36, 28. Sep. 2020 (CEST)


Derzeit diskutiert die ganze Welt ja mal wieder über das Thema Terrorismus. Den US-Präsidenten hat das sogar dazu veranlasst, ganze Nationen von der Einreise in die USA auszuschliessen. Bei genauem Hinsehen zeigt sich, dass es in Deutschland 4000 mal wahrscheinlicher ist an einer Grippe zu sterben und immerhin 1,13 mal wahrscheinlicher an einem Blitzschlag zu sterben als an einem Terroranschlag ([http://www.suedkurier.de/nachrichten/panorama/Warum-vieles-wahrscheinlicher-ist-als-Opfer-eines-Terroranschlags-zu-werden;art409965,8657606 s. Berechnungen]). Auch Flugangst ist so ein Fänomen: Wenn immer ich einen Flug antrete, ist meine Freundin sehr besorgt. Dabei steht dies in gar keiner Relation mit der im Vergleich dazu deutlich höheren Wahrscheinlichkeit bei einer Autofahrt zu sterben. Was aber führt zu solchen Fehleinschätzungen? Ist es der Medien-Hype der das Terrorthema gerne aufgreift? Ich erinnere mich an beinahe wochenlange Medienberichterstattungen nach dem Absturz des Germanwings-Flugzeuges. Problematisch ist dies ja, weil dadurch knappe Gelder auf die falschen Prioritäten verteilt werden: Ganz rational betrachtet müssten die Zeitungen ja voll von Nachrichten über Grippetote sein und die Politik müsste nichts anderes mehr tun als Gelder auf Grippeforschung zu verteilen, um das Risiko daran zu sterben zu reduzieren. Auch der Aufwind von Rechtspopulisten lässt sich wohl auf diese Fehleinschätzung zurückführen. Trump erfindet ja sogar Terroranschläge in Schweden, weil die extreme Friedfertigkeit in Schweden trotz der Aufnahme mehrerer 100.000 Flüchtlinge einfach so gar nicht in sein Vorurteil des kriminellen Zuwanderers passen will. Gibt es wissenschaftliche Arbeiten, die die Ursache für diese Fehleinschätzungen bei der Risikobewertung untersucht haben? Ich könnte mir vorstellen, dass die Kognitionsforschung sich doch bestimmt einmal mit Risikowahrnehmung befasst hat. Wo finde ich mehr darüber? Ich bin durchaus an wissenschaftlichen Artikeln darüber interessiert. [[Spezial:Beiträge/62.44.135.249|62.44.135.249]] 07:38, 25. Feb. 2017 (CET)
:Man darf nicht übersehen, dass Atomenergie in den ersten zwei bis drei Jahrzehnten ihrer technischen Entwicklung '''die Rettung der Erde überhaupt''' war: ''kein'' Kohlebergbau, der die Landschaft zerstört, ''kein'' Ölverbrauch, durch den die Industrieländer von einer endlichen Ressouce abhängig werden, ''kein'' Schadstoffausstoß, ''kein'' Smog, dafür sogar noch billiger, und den Reaktor muss man sogar nur alle paar Monate einmal auffüllen, nachhaltiger kann es doch gar nicht gehen!--[[Spezial:Beiträge/91.221.58.20|91.221.58.20]] 08:24, 28. Sep. 2020 (CEST)
:Ich würde mal mit Wikipedia anfangen: [[Risikowahrnehmung]]. Dieser Artikel stützt sich allerdings wesentlich auf nur eine Arbeit, die zwar einmal (Fußnote 1) als Quelle angegeben wird, aus der aber in Tat und Wahrheit das Meiste <s>abgeschrieben</s> übernommen ist. Umfassender und mit sehr viel mehr Literaturangaben versehen ist der englische Schwesterartikel [[:en:Risk perception]]. Grüße [[Benutzer:Dumbox|Dumbox]] ([[Benutzer Diskussion:Dumbox|Diskussion]]) 08:00, 25. Feb. 2017 (CET)
:: Das ist - Entschuldigung! - dummes Zeug. 'Atoms for Peace', also die "zivile" Nutzung der Kernenergie, war nie etwas anderes als die Verlagerung der Kosten der Plutoniumproduktion für Waffenzwecke auf die Stromkunden. (Und daß Reaktorplutonium nicht waffentauglich wäre, war schon immer eine Zwecklüge.) Rentiert haben sich Kernkraftwerke unsubventioniert nie, und umweltfreundlich waren sie auch nicht, dafür war bereits die Kernbrennstoffherstellung vom Gebirge bis zur Brennelementefabrik zu verheerend. Der radioaktive Abraum ist noch immer weltweit vielerorts zu bewundern, Abfälle aus bestrahlten Brennelementen (Spalt- und Aktivierungsprodukte) sind dabei noch gar nicht eingerechnet. --[[Spezial:Beiträge/95.112.231.60|95.112.231.60]] 13:08, 28. Sep. 2020 (CEST)
:(BK)Nach meiner Erinnerung war es bis in die 1970er hinein wissenschaftlicher Konsens und bis in die 1990er hinein herrschende wissenschaftliche Meinung, dass die Risiken beherrschbar und die Entsorgungsthemen lösbar sind. In den 1960ern war ein Atomgegner eher sowas wie ein Impfgegner heute und es war noch nicht absehbar, dass die Endlagerung irgendwann ein enormes politisches Problem werden würde - es scheitert heute ja offenbar nicht an der Technik, sondern an der Akzeptanz. --[[Benutzer:Rudolph Buch|Rudolph Buch]] ([[Benutzer Diskussion:Rudolph Buch|Diskussion]]) 08:25, 28. Sep. 2020 (CEST)
::Bis zu einem gewissen Punkt stimmt das, "es ist eine Frage der Akzeptanz, nicht der Technik". Gerade für die grosse Masse, also die schwach- und mittelradioktiven Stoffe gäbe es technische Lössungen. Bei den stark radioaktiven Abfälle, sprich die Brennstäbe und hier vorallem wegen dem langlebigen [[ Plutonium]], siht es bisschen anders aus. Da ist man sich auch in der Fachwelt nicht einig (oder besser nicht mehr einig), ob verbudeln und vergessen wirklich die richtige Lössung ist. Weil die spielen was Halbwertszeit anbetrifft einfach in einer anderen Liga.--[[Benutzer:Bobo11|Bobo11]] ([[Benutzer Diskussion:Bobo11|Diskussion]]) 09:23, 28. Sep. 2020 (CEST)
:::"''die schwach- und mittelradioktiven Stoffe gäbe es technische Lössungen''" hui, da bin ich aber gespannt, da weißt Du mehr als die mir bekannten Fachmenschen aus (z.B.) dem Forschungszentrum Jülich oder Karlsruhe. Mach mich schlau! --[[Benutzer:Elrond|Elrond]] ([[Benutzer Diskussion:Elrond|Diskussion]]) 12:30, 28. Sep. 2020 (CEST)
::::Das Hervorheben von "vorallem wegen dem langlebigen Plutonium" verstehe ich nicht. Plutonium-239 hat eine Halbwertszeit von gut 24 Tsd. Jahren, Uran-238 eine von über 4 Milliarden Jahren. --[[Spezial:Beiträge/188.61.132.244|188.61.132.244]] 20:18, 28. Sep. 2020 (CEST)


:Für eine Naturkatastrophe hätten wir Verständnis. Menschengemachtes halten wir zurecht für Vermeidbares, da es politisch ist, sei es auch nur Organisations- oder Firmenpolitik. --<span style="color:#00A000;">Hans Haase ([[BD:Hans Haase|有问题吗]])</span> 08:45, 25. Feb. 2017 (CET)
::''In den ersten zwei bis drei Jahrzehnten ihrer technischen Entwicklung'' lebte man noch in einem Traum, denn natürlich zerstört auch der Uranabbau die Umwelt. Abbau, Transport und Verarbeitung verbrauchen Öl und verursachen Schadstoffe, von der Endlagerung und dem komplizierten "Rückbau" der Atomkraftwerke mal ganz abgesehen. --[[Benutzer:Optimum|Optimum]] ([[Benutzer Diskussion:Optimum|Diskussion]]) 12:07, 28. Sep. 2020 (CEST)
::Es könnte auch andersherum sein, dass wir wahrscheinliche Risiken viel schneller ausblenden. Die Beeinflussbarkeit ist der andere Faktor der zu Verzerrungen führt. Die Wahrscheinlichkeit einen Autounfall zu vermeiden, meinen wir durch unser Verhalten beeinflussen zu können. Beim Flugzeug sind wir dem Piloten usw. ausgeliefert (wir haben nur die Wahl zwischen fliegen oder nicht fliegen). Diese beeinflusst und verzehrt die Wahrnehmung ungemein.--[[Benutzer:Bobo11|Bobo11]] ([[Benutzer Diskussion:Bobo11|Diskussion]]) 08:57, 25. Feb. 2017 (CET)
::: Durch andere Maßnahmen, z.B. Umweltzerstörung, Klimawandel, mache wir den Globus auf absehbare Zeit unbewohnbar. Wer wird sich dann für Halbwertzeiten interessieren? Ich halte die Lösung anderer Probleme für brisanter. Der Atommüll ist nicht optimal untergebracht, aber ist emissionsarm, kaum klimarelevant und nicht großflächig verkehrsbehindernt. Ich halte es durchaus für denkbar, das man die Strahlungsenergie irgendwann sinnvoll nutzen kann --- falls wir uns bis dahin nicht schon die Erde kaputt gemacht haben. --[[Benutzer:Gadacz|Klaus-Peter ''<small>(<sup>auf</sup><small>und</small><sub>davon</sub>)</small>'']] 13:22, 28. Sep. 2020 (CEST)
:::Ja. Das Schlüsselwort ist (oben) Risikowahrnehmung.
:::Hm. Auch ohne detaillierte technische Kenntnisse konnte und musste man wohl auch schon in den 1960er-Jahren davon ausgehen, dass mit der zivilen und militärischen atomaren Technologie Atommüll entsteht, der mehrere hunderttausend Jahre sicher gelagert werden muss und dass Mensch und Umwelt zuverlässig vor den Folgen einer Kontamination durch diesen Müll geschützt werden müssen. Blicke ich nun historisch in die andere Richtung und überschaue die letzten nur 6.000 Jahre dessen, was wir als bisherige menschliche Zivilisation begreifen, dann versteht auch ein Zehnjähriger, dass man für mehrere hunderttausend Jahre in die Zukunft gesehen keine ernstzunehmende Prognose abgeben kann. Ergo hat man auch schon damals verantwortungslos extrem problematischen Müll produziert und das Problem (und die anfallenden Kosten) den nächsten Generationen aufgebürdet. Das ist mit dem militärischen Müll der letzten beiden Weltkriege, der in den Meeren lauert oder dem (nicht nur als Mikroplastik problematischen) Plastikmüll ebenso. Es scheint also bei den Verantwortlichen die Kontinuität einer entsprechend widerlichen Grundhaltung zu geben, die andere Handlungsmaxime bevorzugt als ein vernünftiges, vorausschauendes und den Menschen gegenüber verantwortliches Planen, Entscheiden und Handeln. Detailliert wird das sichtbar, wenn man sich Biografien wie die des [[Gerald Hennenhöfer]] genauer anschaut. Die junge Generation, die langsam versteht, dass ihre Zukunft wesentlich mieser sein wird als ihr jetziges Leben, weil sie die Kosten für unsere Party am Hals hat, beginnt das wohl zu ahnen. Wenn man da nur von Naivität, Fortschrittsgläubigkeit oder in einem Traum leben spricht, verharmlost man meines Erachtens die Dinge. Es ist wohl erhellender, die Grundhaltungen, Wertmaßstäbe und ökonomischen und politischen Triebkräfte dieser Misere zu erforschen und zu analysieren. Der Ansatz, die Gewinne zu privatisieren, die Kosten für diese Gewinne aber zu sozialisieren durchzieht ja schon lange grundlegend unsere Wirtschaftsgeschichte. Als Antwort auf die Frage also: Nein, sie waren nicht völlig naiv, sie haben gewusst, was sie tun und anrichten und auch heute wissen sie, was sie tun und was sie anrichten. Dass Trump sich aktuell in einem Interview mal verplappert, was er alles weiß, ist dafür nur eines der unzähligen Beispiele. Und ich vermute, mehr zu dem unseligen Spirit der Atommafia könnte man beispielhaft sicher bei [[Klaus Traube]] nachlesen (s.a. [https://www.fr.de/politik/anti-atom-papst-11095494.html Nachruf in der Frankfurter Rundschau]). --[[Spezial:Beiträge/2.206.111.221|2.206.111.221]] 15:06, 28. Sep. 2020 (CEST)
:::[https://www.spds.uni-konstanz.de/publications/risk-perception Hier ein wissenschaftlicher Artikel von 2015] (den man da auch komplett herunterladen kann), der die heute gängigen Ansätze abarbeitet. Wichtig der Hinweis auf das soziokulturelle Umfeld der Risikowahrnehmung (das Beispiel unten mit den ''white males'' und der Verbreitung von Waffen!). Auch die Medien kriegen ihr Fett.
:::: Das ist ein Irrtum! 1960 ff. war Atomktaft das angesagte Wundermittel. Militärtisch akzeptiert, um den Feind abzuschrecken und Hiroschima war ja gut, brachte es doch Frieden. Atombombenpilze sah man öfter in der Wochenschau. Das war schon Klasse, wenn sie nur amerikanisch genug waren. Zivil war es die Wunderwaffe, mit der alles möglich erschien. Krebs-Bestrahlungen in der Medizin, eadioaktive Pillen, um von Innen aufzuräumen. Kraftwerke waren ohnehin unsrer Stolz, je größer um so besser. Ja, meine Junghans-Wanduhr heißt '''Ato-m'''at und läuft immer noch, allerdings stets mit Batterie. Frauen wie die Loren oder Bardot lockten mit Atombusen und trugen Bikini, weil es da so oft krachte. Mein Peilkompass leuchtet seit Jahrzehnten immer noch grünlich dank Uranzelle und der Geigerzähler freut sich knatternd darüber. Autos sollten damit ewig laufen und es gab, vorzugsweise in der UdSSR, handliche Nuklear-Stromgeneratoren, aus Massenproduktion, selbst für entlegene Haushalte, nicht nur Schiffe. Da hat sich echt kein normaler Mensch Gedanken über den Müll gemacht, denn in der Tonne staubte Brikettasche und die stank. Atome sind herrlich geruchlos uns winzig klein, also eher harmlos. Klar, die Wissenschaftler mussten es wissen, aber die sägten sich nicht den Ast ab, auf dem sie saßen. Irgendwie wird man den Müll schon los und ins Meer schmeißen wurde als optimal betrachtet und reichlich praktiziert, damals sogar legal.. So tief taucht keiner und Wasser schirmt ja auch ab. Erst als so nach den 68ern die jungen wach und kritisch wurden und nicht mehr so brav und folgsam waren, wurde die Problematik ins Bewusstsein gedrängt. Mit Umwelt und Klima war es genau so, da wurde demonstrativ der Rauch abgeblasen, man erinnere sich an alte Fabrikfotos, die fleißige Schlote zeigten oder Autos mit sportlichen Wölkchen hinten raus. Wasser gab es viel und konnte jeglichen Müll verdünnen. Also immer rein mit der Chemie! Das ist alles noch nicht so lange her, ja das geht fröhlich so weiter, nur anders ‚verpackt‘! --[[Benutzer:Gadacz|Klaus-Peter ''<small>(<sup>auf</sup><small>und</small><sub>davon</sub>)</small>'']] 15:52, 28. Sep. 2020 (CEST)
:::Ich persönlich würde eine Unterscheidung machen zwischen
:::::Hm. Ergänzend ein Zitat aus einem sehr lesenswerten Artikel: ''„In der Bundesrepublik hebt die Diskussion in den fünfziger Jahren an, noch bevor 1957 in Garching bei München der erste Forschungsreaktor in Betrieb geht. Anfangs besteht Hoffnung, das Problem löse sich von selbst: Wenn man die Brennelemente aus den USA importiert, können die abgebrannten Überreste dorthin zurückgebracht werden, damit die Amerikaner den heißen Bombenstoff Plutonium aus dem Atommüll rausholen. Die Verschiffung der Abfälle in die USA ist nur einer von vielen Wunschträumen. In den Folgejahren bleibt die Lage unklar. Die "unschädliche Abführung radioaktiver Abfallstoffe" müsse jedenfalls gelöst werden, bevor der erste Reaktor in der dicht besiedelten Bundesrepublik gebaut werde, heißt es unmissverständlich in einem Vermerk des Wirtschaftsministeriums vom 15. Februar 1955.''
:::* "Was sehe ich in meinem täglichen Leben als Risiko" [Ich habe beispielsweise die gefährliche Angewohnheit, dass ich mir morgens das T-Shirt anziehe, ''während ich die Treppe hinunter gehe''...] und
:::::''Zwei Jahre später indes bläst Atomminister Siegfried Balke (CSU) nach einem USA-Besuch Entwarnung. In den Staaten würden "die "Abfallprodukte zunächst zehn oder zwölf Jahre in unterirdischen Tankanlagen gesammelt, sodass nichts an die Außenwelt kommt", erklärt er. Nach fünf weiteren Jahren müsse man erst mal nachprüfen: "Ist dann überhaupt noch Aktivität da?" Dass der Müll nicht 10 oder 15, sondern 50.000 Jahre und länger strahlt, war dem Minister für Atomfragen offenbar fremd.'' [Anm.: Tatsächlich? Kaum zu glauben...]
:::* "Was verklickern mir die Medien VON ALLEN SEITEN als Risiko".
:::::''Für das Versenken im Meer kann sich das Ministerium Ende der fünfziger Jahre nicht so richtig begeistern, die Verantwortungslosigkeit ist zu offensichtlich. Dafür beginnt eine bizarr anmutende Diskussion über das "Einschmelzen" der Atomabfälle in Polargebieten. (...)“'' ([https://www.zeit.de/2012/38/Atommuell-Endlager-Geschichte/komplettansicht Manfred Kriener: ''Weg! Weg!! Weg!!!'' In: Die Zeit Nr. 38/2012, 13. September 2012]) --[[Spezial:Beiträge/2.206.111.221|2.206.111.221]] 16:27, 28. Sep. 2020 (CEST)
::::Da sieht alles schon mal gaaanz anders aus. <small style="color:grey"><b>'''GEEZER'''</b></small><sup>[[BD:Grey Geezer|<span style="color:grey"> … nil nisi bene</span>]]</sup> 10:31, 25. Feb. 2017 (CET)
::::: Das ist Thema der [[Kognitionspsychologie]], insbesondere [[Wahrnehmungsfehler]]. Auch, wenn es eigentlich nicht zu dem Themengebiet gehört, findest du unter [[Prospect Theory]] eine ganze Reihe von Wahrnehmungsfehlern, die zur Erklärung von Fehleinschätzungen herangezogen werden. [[Benutzer:Yotwen|Yotwen]] ([[Benutzer Diskussion:Yotwen|Diskussion]]) 11:13, 25. Feb. 2017 (CET)
::::::Lies auch mal alles über den [[Thalamus]]. Das ist das Tor zum Bewusstsein im Hirn, hier wird entschieden, was beachtenswert oder nicht ist. Das gleiche Bild an der Wand wird mehr oder minder ausgeblendet, es wird keine Info an das Gehirn geschickt. Erst wenn ein anderes da hängt, bekommt das Bewusstsein eine Meldung. Damit wird eine Überlastung des Hirns durch zu viele Eindrücke unterbunden. So ist es auch mit den "Unglücken", alles was da auf die Schiene des gewohnten/täglichen geschoben wird, wird weniger beachtet, erst wenn was außerordentliches passiert, schiebt es der Thalamus ins Bewusstsein.--[[Spezial:Beiträge/2003:75:AF0C:6900:D5AB:114F:5900:6B6E|2003:75:AF0C:6900:D5AB:114F:5900:6B6E]] 15:10, 25. Feb. 2017 (CET)


Ich glaube man orientiert sich zu sehr an Medien. Früher hat man als Bub mitbekommen, dass der Karl von der Leiter gefallen ist und sich das Genick gebrochen hat und 20 Jahre später ist ähnliches passiert und daraus hat man sich eine Gefahr errechnet. Heute steht in der Zeitung, dass in Spanien 2 Leute an der Todesgrütze gestorben sind und nächste Woche sind es 3 in Österreich und dann glauben die Leute schon, bald sind sie dran. Ausserdem ist man in D ziemlich neurotisch geworden und glaubt, mann müsse jedes Risiko in den Griff bekommen. Bei uns hat ein ganzes Freibad seinen Betrieb eingestellt weil an verschiedenen Stellen die Geländer nicht mehr den aktuellen Sicherheitsanforderungen genügt haben. In Russland oder Vietnam würde man die Deutschen dafür auslachen. Das gleiche auch z.B. mit Rauchmeldern. Das ist auf einmal total wichtig und relevant. --[[Spezial:Beiträge/2.246.78.200|2.246.78.200]] 15:16, 25. Feb. 2017 (CET)
: Die Logik war damals dieselbe wie heute. Unternehmer erklären das wie folgt: "Wir machen jetzt etwas, von dem wir wissen, daß es schlecht für die Umwelt ist, aber das macht gar nichts, weil wir damit viel Geld einnehmen, von dem Forschung finanziert wird, die neue Technologien entwickelt, mit denen die Umweltschäden dann repariert werden." Der Glaube an die Funktionalität dieses Systems ist ähnlich unumstößlich wie der Glaube an die [[unsichtbare Hand]]. --[[Spezial:Beiträge/94.219.186.65|94.219.186.65]] 00:58, 30. Sep. 2020 (CEST)
::Hm. Das sehe ich etwas anders. Die zivile Atomtechnologie entsteht nicht aus kommerziellen Interessen sondern sie ist dezidiert ein Folgeprodukt der Erforschung der militärischen Atomtechnik. Urananreicherungsanlagen und die ersten Atomreaktoren wurden aus der Notwendigkeit entwickelt, Plutonium und angereichertes Uran zum Atomwaffenbau herzustellen. Dazu kommt die Entwicklung von Reaktoren für U-Boote. Auf dieser Grundlage konnte man dann auch zu zivilen Produkten der [[Atomindustrie]] switchen und damit die Akzeptanz dieser Technologie erhöhen (''„This new power, which has proved itself to be such a terrifying weapon of destruction, is harnessed for the first time for the common good of our community.“'' – Die Queen am 17. Oktober 1956 bei der Einweihung des Atomreaktors Calder Hall [http://news.bbc.co.uk/onthisday/hi/dates/stories/october/17/newsid_3147000/3147145.stm]). Wirtschaftlich gesehen war bei kritischer und genauer Betrachtung diese Technologie niemals profitabel, es sei denn man nimmt die Steuerzahler aus und sozialisiert Forschungskosten, Risiken und – was für ein Euphemismus! – die „Entsorgung“. Das hat ja dann auch prima geklappt.
::Historisch haben wir zunächst eine internationale Forschung, die bis 1938 die Basis der nötigen theoretischen Grundlagen für eine [[Atombombe]] liefert, dann einen Rüstungswettlauf der Alliierten, weil sie unter anderem davon ausgehen, dass auch die Nazis Atomwaffen entwickeln (vgl. [[Uranprojekt]], Albert Einsteins Brief an Präsident Roosevelt vom 2. August 1939, [[Frisch-Peierls-Memorandum]] vom März 1940) und weil sie sich nach dem Sieg über die Nazis im Kalten Krieg befinden. Zur Erinnerung über die Verschränkungen und Verzahnungen des Abfallprodukts zivile Atomtechnologie eine kleine Übersicht der Kerndaten dieser Entwicklung (etwas Wortwitz muss auch hier sein), der die Rolle der zivilen Atomtechnologie transparenter macht:
::* Dezember 1941, Eintritt der Vereinigten Staaten in den Zweiten Weltkrieg.
::* Juni 1942, [[Robert Oppenheimer]] veranstaltet an der University of California in Berkeley einen „Forschungssommer“. Die Konferenzen (''The Los Alamos Primer'' (LA-1) liefern die theoretische Grundlage zum Bau der Atombombe.
::* Juni 1942, Präsident Roosevelt entscheidet, das seit 28. Juni 1941 bestehende wissenschaftliche Projekt eines Atomenergie-Entwicklungsprogramms ([[OSRD]]) in ein militärisches Projekt zur Entwicklung von Atomwaffen umzuwandeln.
::* 16. September 1942, Brigadegeneral Leslie R. Groves wird mit der hauptverantwortlichen „militärischen“ Leitung dieses Waffen-Projektes beauftragt. Der Standort des Hauptquartiers in New York City wird als ''Manhattan Engineer District (MED)'' bezeichnet, woraus sich später die Bezeichnung [[Manhattan-Projekt]] entwickelt.
::* 28. September 1942, mit der Direktive Nr. 2352 startet auch die Sowjetunion die Forschung für ein Atombombenprojekt.
::* November 1942 bis Januar 1943, Vorbereitung des [[Los Alamos National Laboratory|Los Alamos Laboratory]].
::* 2. Dezember 1942, Chicago, erste Demonstration des unter Leitung von [[Enrico Fermi]] entwickelten Versuchreaktors CP-1 (vgl. [[Chicago Pile]]).
::* Ab 1943 kooperiert das britisch-kanadische Atomwaffenprojekt mit dem Codenamen ''Tube Alloys'' mit den USA.
::* 10. März 1943, Moskau, Gründung von ''Laboratorium Nummer 2'' unter der Leitung von [[Igor Kurtschatow]] (das spätere [[Kurtschatow-Institut]]).
::* 20. April 1943, Kooperationsvertrag zwischen der University of California und dem United States Army Corps of Engineers über das Geheimlabor ''The Los Alamos Laboratory'' (auch: ''Site Y''). Das Labor hat als Hauptaufgabe die Erforschung und den Bau einer Atombombe.
::* 16. Juli 1945, White Sands Proving Grounds, New Mexico, USA, erster Atomwaffentest mit einer Atomwaffenexplosion ([[Trinity-Test]]).
::* 6. und 9. August 1945, [[Atombombenabwürfe auf Hiroshima und Nagasaki|Atombombenabwürfe der USA auf Hiroshima und Nagasaki]], Tötung von mehr als 200.000 Zivilisten des japanischen Kriegsgegners.
::* 19. Dezember 1945, Obninsk, Gründung des [[IPPE|Staatlichen Wissenschaftlichen Zentrums der Russischen Föderation – Physikalisch-Energetisches-Institut A. I. Leipunski]] (IPPE). Das Forschungsinstitut wurde auch als Labor „V“ (Лаборатория „В“) bezeichnet.
::* 30. Juni 1946, Bikini-Atoll (Marshallinseln, südwestlich von Hawaii), erster oberirdischer Atomtest nach dem Weltkrieg im Rahmen der [[Operation Crossroads]].
::* 1947, nahe Idaho Falls, Idaho, USA, das Gelände des [[Idaho National Laboratory]] (INL) wird als Außenstelle des [[Argonne National Laboratory]] unter dem Namen ''National Reactor Testing Station'' eingerichtet.
::* 1947, Windscale, Cumberland (England), "Windscale Works", Bau von Reaktoren zur Produktion von atomwaffenfähigem Plutonium (vgl. [https://en.wikipedia.org/wiki/Windscale_Piles Windscale Piles]).
::* 19. Juni 1948 bis 16. April 1949, [[Kerntechnische Anlage Majak]], nahe Osjorsk (Oblast Tscheljabinsk), Produktion von Plutonium für die erste sowjetische Atomwaffe.
::* 1949, nahe Idaho Falls, Idaho, USA, Bau der National Reactor Testing Station (NRTS).
::* 29. August 1949, [[Atomwaffentestgelände Semipalatinsk]], Zündung der ersten sowjetischen Atombombe (vgl. [[RDS-1]]).
::* Oktober 1950, Windscale, Cumberland (England), Inbetriebnahme von ''Windscale Pile No. 1'', gefolgt im Juni 1951 von ''Pile No. 2''. Diese luftgekühlten graphitmoderierten Reaktoren sind die erste britische Produktionsanlage zur Herstellung von waffenfähigem Plutonium-239.
::* 12. Juni 1951, das IPPE wird angewiesen, das erste experimentelle sowjetische Atomkraftwerk zu bauen.
::* Dezember 1951, nahe Idaho Falls, Idaho, USA, der Versuchsreaktor [[Experimental Breeder Reactor I]] (EBR-I) erzeugt erstmals elektrischen Strom aus Atomenergie.
::* Juni 1952, Connecticut, USA, öffentliche Kiellegung des erstmals mit einem atomaren Antrieb ausgestatteten U-Bootes [[USS Nautilus (SSN-571)]].
::* 12. September 1952, Regierungsdekret von Stalin „Über das Design und den Bau von Objekt 627“. Projekt 627 „Кит“ (russisch „kit“ bedeutet Wal) ist ein U-Boot-Typ.
::* 1953, die britische Regierung beschließt den Bau eines Atomkraftwerks.
::* 1. Juni bis 27. Juli 1954, das [[Kernkraftwerk Obninsk|Atomkraftwerk Obninsk]] nimmt den Betrieb auf.
::* 30. September 1954, Groton, Connecticut, USA, Indienststellung des erstmals mit einem atomaren Antrieb ausgestatteten U-Bootes [[USS Nautilus (SSN-571)]].
::* 1955, Calder Hall nahe Workington, Cumberland (England), Bau des [[Kernkraftwerk Calder Hall|Atomkraftwerks Calder Hall]] auf dem Gelände des Nuklearkomplexes Windscale bzw. nach dem Störfällen von 1957 mit dem Namen Nuklearkomplex Sellafield.
::* 17. Oktober 1956, Inbetriebnahme des [[Kernkraftwerk Calder Hall|Atomkraftwerks Calder Hall]].
::* 9. August 1957, Molotowsk (später: Sewerodwinsk), nahe Archangelsk, UdSSR, Inbetriebnahme des atomar angetriebenen sowjetischen U-Bootes [[K-3 Leninski Komsomol]] (Start des Reaktors am 19. Mai 1958).
::* Ostern 1958, erster [[Ostermarsch]] der [[Campaign for Nuclear Disarmament]] von London zum Atomforschungszentrum Aldermaston, Berkshire, England.
::--[[Spezial:Beiträge/2.206.111.221|2.206.111.221]] 18:46, 30. Sep. 2020 (CEST)


:Naja, [http://www1.wdr.de/nachrichten/ruhrgebiet/rauchmelder-114~_format-mobile_variant-html5.nimex Rauchmelder sind durchaus sinnvoll]. Es hat sich gezeigt, dass pro Tag im Schnitt etwa ein Mensch durch ein Feuer stirb (95% dabei aufgrund der dabei entstehenden giftigen Dämpfe). Vor der Einführung von Rauchmeldern waren es noch doppelt so viele. Für mich ist das eher ein Beispiel dafür wie ein ''unterschätztes'' Risiko mit einer einfachen Massnahme verringert werden kann. Das ist also so ziemlich das Gegenteil von den reisserischen Themen wie Terrorismus oder Flugzeugabstürzen. [[Spezial:Beiträge/130.226.41.19|130.226.41.19]] 15:58, 25. Feb. 2017 (CET)
::Die "Unternehmer", speziell die Energieversorgungsunternehmen, hatten gar kein Interesse an der Atomenergie. Warum sollte man sich mit einer unerprobten neuen Technologie herumärgern, wenn doch Kohle und Wasserkraft so schön übersichtlich liefen? Der Druck kam von Technologieunternehmen wie Siemens (die Millarden in die Forschung investierten und nun natürlich diese neue Kraftwerkstechnik verkaufen wollten) und vor allem von der Politik (die ebenfalls Milliarden als Subventionen ausgegeben hatten und den Anschluss an diese neue Zukunftstechnologie nicht verpassen wollten). Ob bei F.J. Strauß irgendwo im Hinterkopf die Idee flackerte, sich später mal eigenes Plutonium zu erbrüten, läßt sich wohl nicht belegen, aber immerhin war er in den 1950ern zuerst Atomminister und danach Verteidigungsminister. --[[Benutzer:Optimum|Optimum]] ([[Benutzer Diskussion:Optimum|Diskussion]]) 20:07, 30. Sep. 2020 (CEST)
:: Der niederländische Soziologe [[Geert Hofstede]] hat eine Kulturbeschreibung auf der Basis einer empirischen Befragung von ca. 140000 Leuten aufgestellt. Demnach gibt es eine Dimension kulturellen Verhaltens, inwieweit eine Gesellschaft Risiken vermeidet oder nicht. Ein Beispiel für Gesellschaften mit niedriger Risikovermeidung ist Indien. Hohe Werte findet man in den sogenannten reichen Ländern, Deutschland, USA usw.
:: Das Modell ist statisch, aber es gibt keinen Grund anzunehmen, dass sich Kulturen nicht verändern. Es kann also durchaus sein, dass die Risikovermeidung mit der Zeit zunimmt. Ich fände es normal, dass in einer hysterischen Gesellschaft (und das sind wir spätestens seit die modernen Massenmedien auf uns eintrommeln) Veränderungen stattfinden. Was früher nur etwas für Paranoiker war, ist heute schon Mainstream. [[Benutzer:Yotwen|Yotwen]] ([[Benutzer Diskussion:Yotwen|Diskussion]]) 17:34, 25. Feb. 2017 (CET)
:::"eine Dimension kulturellen Verhaltens" Na absolut! In einem Land, wo Gottgewolltes passiert, ist es halt so. Punkt. In einem Land, in dem jede Sendung unterbrochen wird, weil ein Idiot mit einer Waffe(l) 'rumweldelt, werden Leute permanent auf den hormonellen Zehenspitzen gehalten.
:::Die "Gnade des Nichtwissens" (oder man entwickelt halt Mechanismens, direkte Realität und indirekte Realität einordnen zu können). <small style="color:grey"><b>'''GEEZER'''</b></small><sup>[[BD:Grey Geezer|<span style="color:grey"> … nil nisi bene</span>]]</sup> 14:29, 26. Feb. 2017 (CET)
:Das hat sicherlich auch was mit den Medien zu tun. Wenn etwa, sagen wir mal, ein japanischer Kernreaktor nach einem Erdbeben und einer Flutwelle havariiert, dann schalten manche Nationen auf Basis eines Moratorium alle Kernkraftwerke ab. Dabei ist die Wahrscheinlich, dass es zu einer Reaktorkatastrophe in diesen Nationen kommt, von den Ereignissen in Japan doch völlig unbeeinflusst. Sprich: Die ''rationalen Gründe'' für oder gegen Atomkraft haben sich nicht geändert, nur die (Risiko-)Wahrnehmung. <small>(''nicht [[Hilfe:Signatur|signierter]] Beitrag von'' [[Spezial:Beiträge/188.101.73.75|188.101.73.75]] ([[Benutzer Diskussion:188.101.73.75|Diskussion]])<nowiki/> 23:20, 26. Feb. 2017 (CET))</small>
:: Der Unfall in Japan zeigt ziemlich eindrucksvoll, dass die Abschätzungen der Techniker und Ingenieure ziemlich gut sind. Die haben nämlich vorausgesagt, dass es alle ca. 20000 Jahre zu einer Kernschmelze kommt. Bei ca. 500 Kernkraftwerken weltweit kommen wir also auf 20000/500=40; d.h. so alle ca. 40 Jahre müssen wir mit einer Kernschmelze rechnen. Und wenn wir uns dann die Geschichte ansehen, dann haben wir mit [[Kernkraftwerk Three Mile Island|Three Mile Island]] (1979); [[Kernkraftwerk Tschernobyl|Tschernobyl]] (1986); [[Kernkraftwerk Fukushima Daiichi|Fukushima Daiichi]] (2011) drei Kernschmelzen in den rund 80 Jahren Kernkraftgeschichte. Das ist so dicht an der Vorhersage der Techies, dass ich dazu tendiere, die Risikoabschätzung für realistisch zu halten.
:: Irreal sind die Folgerungen, die daraus gezogen wurden. Denn das Risiko verändert sich nicht erheblich, wenn ein Kernkraftwerk "abgeschaltet" wird. Wir sparen, so scheint es mir, ca. 20-30 Jahre von einer Million Jahre, also 0.03 %. Unsere Nachfahren werden uns auf ewig dankbar sein. [[Benutzer:Yotwen|Yotwen]] ([[Benutzer Diskussion:Yotwen|Diskussion]]) 09:12, 27. Feb. 2017 (CET)


In [https://books.google.de/books/about/Das_Risikoparadox.html?id=GsaBnAEACAAJ&redir_esc=y diesem Buch] wird das ganz eindrücklich als das „Problem des großen Nenners“ beschrieben. Wenn man ein Risiko als [[Bruchrechnung|Bruch]] betrachtet, dann bildet die Schwere des Ereignisses den Zähler des Bruches, während die Eintrittswahrscheinlichkeit im Nenner ist. Und leider neigen Menschen dazu, denn Nenner zu übersehen und den Zähler extrem zu überschätzen.
::: Du hast ja gerade selber erklärt, daß und wie die Politik den Unternehmern die Atomenergie schmackhaft gemacht hat. (Bis heute gilt das [https://rsw.beck.de/aktuell/daily/meldung/detail/atomkonzerne-und-staat-unterzeichnen-vertrag-zur-finanzierung-der-entsorgung-von-atommuell unver][https://rsw.beck.de/aktuell/daily/meldung/detail/atomkonzerne-und-staat-unterzeichnen-vertrag-zur-finanzierung-der-entsorgung-von-atommuell ändert.]) Ich habe auch nie behauptet, daß es immer und ausschließlich Unternehmer sind, die so denken und argumentieren. Ich gehe allerdings davon aus, daß alle, die bei sowas mitmachen, so oder ähnlich denken, weil sie den Inhalt dieses Fortschrittsglaubens als Rechtfertigung für ihr unmittelbar schädliches Handeln benötigen, um nicht [[Kognitive Dissonanz#Festingers Theorie|in Gewissenskonflikte zu geraten]].
::: Daß Politiker selten derart geredet haben in der Öffentlichkeit, dürfte daran liegen, daß sie für den Erhalt ihrer Macht - ganz im Unterschied zu Unternehmern - von einer öffentlichen Zustimmung abhängig sind. Die [[Anti-Atomkraft-Bewegung|Anti-Atomkraft Bewegung]] wäre sicherlich um ein Vielfaches größer/heftiger ausgefallen <small>(und Loriot hätte sich niemals [https://www.youtube.com/watch?v=q0hIsLCbDq0 für Werbezwecke einspannen lassen];)</small>, hätte die Politik die Folgen für Mensch und Umwelt im Kontext mit dem durchaus beklagenswerten Umstand erwähnt, daß man bislang keinerlei Lösungsstrategien für diese Probleme hat und sich blind auf den zukünftigen technischen Fortschritt verläßt. Heutzutage können Politiker sowas offen aussprechen, weil der Fortschrittsglaube religiöse Ausmaße angenommen hat und nur noch von ketzerischen Mießmachern in Zweifel gezogen wird. Ich denke dabei u.a. an ein Interview mit Scaramucci vor nicht allzu langer Zeit, in dem dieser exakt das ausgesagt hat, was ich zuvor beschrieben habe. Der Fortschritt wird kommen, wie eine Art Heiland, und uns von all unserem Leid und all unseren Sünden aus der Vergangenheit befreien <small>… und das wüßten diese irrgläubigen FFF-Kinder auch, würden die nicht ständig die Schule schwänzen!</small> --[[Spezial:Beiträge/178.4.185.67|178.4.185.67]] 15:31, 1. Okt. 2020 (CEST)


Wenn man z.B. die Gefahren von verschiedenen Verkehrsmitteln vergleicht, dann ist der Zähler (also die Schwere des potentiellen Ereignisses) beim Flugzeugabsturz deutlich höher (viele Tote) als bei einem Autonunfall (relativ wenige Tote, höhere Übelebenschance). Beim Nenner (also der Eintrittswahrscheinlichkeit) verhält es sich aber mehr als andersherum: Hier ist es um ein vielfaches wahrscheinlicher in einem normalen Verkehrsunfall verwickelt zu werden, als jemals von einem Flugzeugabsturz betroffen zu sein. Zusammengenommen ist das Risiko bei einem Flugzeugabsturz zu sterben deutlich geringer, als das, bei einem Autounfall ums Leben zu kommen. Trotzdem fürchten sich die meisten Menschen vor ersterem mehr (eben weil sie den Zähler zu Ungunsten des Nenners überbewerten). // [[Benutzer:Martin Kraft|Martin K.]] ([[Benutzer Diskussion:Martin Kraft|Diskussion]]) 09:30, 27. Feb. 2017 (CET)
= 28. September =
:Martin das ist der eine Punkt. Der andere ist eben der, dass man sich einredet das man mit vorsichtiger Fahrweise ja die Unwahrscheinlichkeit senken kann. Was ja Statistisch auch stimmt, leider reicht das aber nicht um unter die Wahrscheinlichkeit eines Flugzeugabsturzes zu kommen. Der Punkt, kann aber unser Verständnis ganz gut ausblenden. Wir werden unbeeinflussbare Risiken immer höher bewerten als die Risiken die wir beeinflussen, weil wir ja das Risiko um 50% senken können. Dazu kommt eben die Menge der Tote auf einmal. Ein Unfall mit einem Toten bleibt nun mal nicht so hängen wie einer mit 100 Toten. Wenn jetzt statistisch beide Unfallarten wirklich vergleichbar wären weil sie genau gleich sind. Wir werden das Risiko das alle 100 Tage ein Unfall mit 100 Toten gibt höher gewichten, als der täglich Tote. Obwohl es bei beiden Risiken so ist, dass es in 100 Tagen 100 Tote gibt. Interessant ist ja auch der Bericht [http://www.srf.ch/risiko/ueberschaetzte-risiken hier], da wird erklärt warum die Leute bei den Schätzungen der Toten pro Jahr und Unfallart so daneben liegen können (geschätzt jährliche 36 Tote durch Blitzschlag in der Schweiz, effektiv wären es aber 0,5 Tote pro Jahr).---[[Benutzer:Bobo11|Bobo11]] ([[Benutzer Diskussion:Bobo11|Diskussion]]) 23:20, 27. Feb. 2017 (CET)
:Noch ein Stichwort: [[Verfügbarkeitsheuristik]] / [[:en:availability heuristic|availability heuristic]]. Auch (Nicht-Not)Ärzte leiden darunter - werden sie gefragt, ob Verkehrsunfälle oder Herzinfarkte häufiger sind, schätzen sie die Häufigkeit von Unfällen höher ein, obwohl es tatsächlich das andere ist. Weil Berichte über Unfälle im Bewusstsein schlicht präsenter sind als Berichte über Herzinfarkte. Zum Beispiel kennen viele Leute jemanden, der nach Paris gereist ist und dort die massiven Sicherheitsmassnahmen gesehen hat - aber kaum jemand kennt einen, der einen Blitzschlag überlebt hatte (oder daran gestorben ist).--[[Benutzer:Keimzelle|Keimzelle]] <small>[[Benutzer_Diskussion:Keimzelle| talk ]]</small> 09:53, 2. Mär. 2017 (CET)


== Streaming über WLAN ==
= 26. Februar 2017 =


== Lernstrategien im Studium: Was führt zum Erfolg? ==
Ich habe folgende Idee: Vielleicht kommen noch ein paar schöne Tage dieses Jahr. Dann könnte ich meinen Geburtstag (mit ca. 20..30 Leuten) nachfeiern – covidmäßig natürlich draußen und mit Abstand. MMn. gehört zu einer Feier auch Musik. Um die Nachbarschaft nicht unnötig aufzuscheuchen, erinnerte ich mich an die vor einigen Jahren als "Silent Party" bekannten Funkkopfhörer-Events. So kam mir die Idee, ein WLAN (mit einem handelsüblichen Heimrouter oder vielleicht per Software vom Notebook aus) aufzuspannen und dort einen Multicast-/Broadcast-Stream einzurichten. Die Gäste bringen ein Smartphone mit, gehen damit in das WLAN und empfangen den Stream.


Gibt es Studien oder sogar Metastudien darüber, welche Lernstrategien man im Studium idealerweise anwenden sollte, um erfolgreich zu sein? Sollte man beispielsweise eher alleine lernen oder mit Kommilitonen? Sollte man Texte lieber vor oder nach der Vorlesung lesen, damit sie hängen bleiben? Solche oder ähnlich Fragen interessieren mich. Am liebsten wären mir Studien aus hochrangigen Fachzeitschriften. [[Spezial:Beiträge/62.44.135.249|62.44.135.249]] 17:00, 26. Feb. 2017 (CET)
* Mit welcher Software könnte sowas klappen? Vielleicht ffmpeg oder VLC zum Senden (Daten vor der Soundkartenausgabe abgegriffen)? (Idealerweise auf einer Linuxkiste.)
:Es interessiert DICH als Einzelperson? Dann bringen dir Statistiken über den Erfolg von Strategien nichts. Du folgst einfach deinem Bauch/Kopf ... und was funktioniert, machst du weiter.
* Welches Protokoll sollte man nehmen? Z.B. RTP? Gibt es etwas, dessen Empfang heutige Browser üblicherweise out of the box beherrschen? (So dass man einfach sagen kann, buche dich ins Netz "SSID_XY" ein und surfe dann auf "rtp://192.168.x.y", und wenn das Handy nicht gerade Jahrhunderte alt ist, funktioniert das. (Was bei rtp: bei den von mir getesteten Handys nicht der Fall war.))
:Nur einen Hinweis zu "Fachzeitschriften": Ich habe am Anfang des Studiums aus den drei besten Lehrbüchern gelernt, die es damals gab. Die Klausurfragen kamen aber aus Lehrbuch O. Ich habe 3 nicht so gute Freitagsklausuren gebraucht, bis ich das Aha!-Erlebnis hatte und dann im Durchschnitt bei 95 % der möglichen Punktzahl lag. Hehehe! <small style="color:grey"><b>'''GEEZER'''</b></small><sup>[[BD:Grey Geezer|<span style="color:grey"> … nil nisi bene</span>]]</sup> 17:31, 26. Feb. 2017 (CET)
* Bei wievielen Teilnehmern ist da Ende der Fahnenstange? (Hängt vermutlich vom Router ab.) Können bei einem derartigen Szenario Broadcast-Pakete über die Luft gehen? Oder geht jedes Handy mit einer individuellen Luftverbindung ins WLAN, auch wenn auf LAN-Ebene keine Punkt-zu-Punkt-Verbindung, sondern Broadcast benutzt wird?
* Oder warum ist das ein blöde Idee?


::Nein, es interessiert mich nicht als Einzelperson. Ich bin Dozent und möchte meinen "Schäfchen" am Anfang des Semesters Lerntipps mit auf den Weg geben und da ich "research-based teaching" betreibe, d.h. jede PPT-Folie mit einer Quelle versehe, möchte ich nicht einfach behaupten "lernt in Gruppen!", wie das damals meine Dozenten gemacht hatten, sondern wirklich abgesichertes Wissen weitergeben. Das erscheint mir nicht nur im AfD-/Trump-Zeitalter besonders angebracht, sondern generell. [[Spezial:Beiträge/62.44.135.249|62.44.135.249]] 17:42, 26. Feb. 2017 (CET)
Danke für wissende Hinweise oder Hinweise auf Wissen. --[[Spezial:Beiträge/213.196.213.65|213.196.213.65]] 00:40, 28. Sep. 2020 (CEST)
:::PPT-Folien waren für mich persönlich immer ein triftiger Grund, ein Seminar nicht weiter zu besuchen, denn verblöden kann ich mich auch selbst. Aber auch da sind die Geschmäcker verschieden...--[[Benutzer:Edith Wahr|Edith Wahr]] ([[Benutzer Diskussion:Edith Wahr|Diskussion]]) 17:47, 26. Feb. 2017 (CET)
:Linux? [https://itsfoss.com/best-linux-media-server/] plus Router (-software), der als DHCP-Server dienen kann. Dann aber mal die Reichtweite checken, die wird nicht so dolle sein, wie bei einem "richtigen" WLAN-AP.
::::Für mich auch. Aber darum mache ich es jetzt ja besser. Ich habe sehr lange an einem Konzept für "gute" PPT-Folien gearbeitet. Geholfen hat mir auch, dass ich parallel zum Studium als Webdesigner gearbeitet habe, was beim guten Foliendesign sehr hilft. Ausserdem, das ist glaube ich in Deutschland noch immer nicht so, wird man hier in Dänemark didaktisch sehr gefördert, auch parallel zur Arbeit als Professor. Ich kriege in jeder Lehrevaluation gesagt, dass dies auch so ist. Liebe Edith, das musst Du mir jetzt glauben, aber meine Vorlesung würdest Du zumindest nicht wegen der Folien verlassen. [[Spezial:Beiträge/62.44.135.249|62.44.135.249]] 19:17, 26. Feb. 2017 (CET)
:Gibt es bereits einen Raspi bei Dir? Raspi als WLAN-accesspoint: [https://www.elektronik-kompendium.de/sites/raspberry-pi/2002171.htm] und als Media-Server: [https://www.raspberrypi.org/documentation/usage/kodi/] Ich vermtue, Du darfst gar nicht genügend Gäste bei Dir z hause bewireten, bevor es techn. Probleme gibt. Die Musik wird ja als Multicast (einer sendet, viele hören) verteilt = Anzahl der Clients relativ egal.
:Interesse/Eigenmotivation fördern, von Anfang! an Lernziele (Leistungsnachweisaufgaben) deutlich darstellen, (mehrere) Wege zeigen, individuelle Lernweisen ermöglichen.--[[Benutzer:Wikiseidank|Wikiseidank]] ([[Benutzer Diskussion:Wikiseidank|Diskussion]]) 18:32, 26. Feb. 2017 (CET)
:Ansonsten: DLNA-Server unter Windows einrichten [https://www.giga.de/downloads/windows-10/tipps/windows-10-dlna-server-einrichten-so-gehts/]
::Kapiert!
:Bei DLNA müsstest Du je nachdem aber noch Deine Gäste für eine Client-App begeistern: [https://android.izzysoft.de/applists/category/named/multimedia_dlna?lang=de]
::Lerntechniken: [http://www2.le.ac.uk/offices/ld/resources/study/learning-revision-techniques Hier], [http://www.csc.edu/learningcenter/study/studymethods.csc Hier]. Aber ich bleibe dabei: Wie und wann man am besten lernt, ist sehr individuell (z.B. Morgenmenschen, Abendmenschen, mag andere Menschen, lernt lieber allein...)! Ein Student sollte in der Lage sein, seine ''eigene Strategie'' zu entwickeln, sonst hat er eigentlich an der Uni nichts verloren. Darauf hinzuweisen, würde dir Zeit sparen. <small style="color:grey"><b>'''GEEZER'''</b></small><sup>[[BD:Grey Geezer|<span style="color:grey"> … nil nisi bene</span>]]</sup> 18:44, 26. Feb. 2017 (CET)
:Generell scheint mir RTP eher ein Unicast-Protokoll zu sein, jeder Client macht also "extra Arbeit" für den Server. Sieht bestimmt witzig aus, wenn die Leute anfangen zu tanzen und jeder an einer anderen Stelle im Lied ist. Was Du brauchst ist Multicast, alle hören das Gleiche zu gleichen Zeit, was laut unserem Artikel wohl auch mit RTP gehen soll. Also evtl.einfach ein WLAN aufspannen mit Router(-Software) und eine Software zum selber Webradio streamen: [https://wiki.ubuntuusers.de/Internetradio/].
:::+1, jeder muss das selbst wissen. Ich hatte folgende Strategie und bin gut damit gefahren. Ich habe alles versucht zu verstehen, was der da vorne erzählte. Was ich nicht verstanden habe, habe ich mir notiert und möglichst noch am gleichen Tag nachgelesen bis ich es verstanden hatte. So war ich vor der nächsten Vorlesung auf dem Laufenden. Ich musste vor Klausuren relativ wenig nachbüffeln.--[[Spezial:Beiträge/2003:75:AF07:FF00:4913:10E3:C308:CB52|2003:75:AF07:FF00:4913:10E3:C308:CB52]] 19:28, 26. Feb. 2017 (CET)
:Evtl. geht es auch, dass einfach alle ihre BT-Kopfhörer auf einem zentralen Server anmelden, dann kann das Handy aus bleiben. Wieviel BT-Devices pro Gerät gehen, weiß ich allerdings gar nicht. [[Benutzer:Flossenträger|Flossenträger]] 10:15, 28. Sep. 2020 (CEST)
::Bitte auch [[:en:IP multicast#Wireless considerations]] beachten. Was am ehesten funktioniert wäre eine Anzahl baugleicher analoger Funkkopfhörer im 862-MHz-Band. Die werden alle auf den gleichen Kanal geschaltet und eine einzige Basisstation versorgt die Kopfhörer dann. --[[user:Rotkaeppchen68|R<span style="color:red">ô</span>tkæppchen₆₈]] 10:51, 28. Sep. 2020 (CEST)
:[https://support.spotify.com/li/article/group-session/ Spotify Gruppen-Session] funktioniert gut --[[Benutzer:Patchall|Patchall]] ([[Benutzer Diskussion:Patchall|Diskussion]]) 13:25, 28. Sep. 2020 (CEST)
:::Ich denke mal, die Anschaffung von zwei Dutzend baugleicher Kopfhörer steht jetzt nicht gerade auf seiner Agenda. Er sprach ja von "Die Gäste bringen ein Smartphone mit...", also dann wohl auch die Kopfhörer. [[Benutzer:Flossenträger|Flossenträger]] 11:56, 29. Sep. 2020 (CEST)
::::Mittlerweile habe ich entdeckt, dass man die "Silent Party"-Hardware mieten kann. Ist sogar durchaus leistbar. Allerdings sprengt das den von mir angedachten Rahmen. Danke aber für alle Hinweise! --[[Spezial:Beiträge/213.196.213.151|213.196.213.151]] 23:56, 30. Sep. 2020 (CEST)


::Ich haue auch noch mal in Wikiseidanks Kerbe: Das Fundament sind Interesse und Motivation! Methoden mögen zwar besser oder schlechter funktionieren, aber das spielt dem gegenüber eine weit geringere Rolle. Wo dieses Fundament nicht mitgebracht wird (was es bereichsweise überall gibt), macht es einen guten Lehrer aus, das so gut wie irgend möglich zu vermitteln.
== Also, wie ist das jetzt nochmal mit dem ''Hochachtungsvoll''? ==
::Was dann Lernmethoden angeht: Da sollte man alternative Möglichkeiten vorstellen und zum Ausprobieren anregen. Je nach Aufgabe und Persönlichkeit kann individuell mal dies, mal das besser funktionieren.
::[[Benutzer:Rainer Zenz|Rainer Z]] [[Benutzer Diskussion:Rainer Zenz| ...]] 19:47, 26. Feb. 2017 (CET)


https://www.kikidan.com/news/repraesentationssystem-visuell-auditiv-kinaesthetisch.html --[[Spezial:Beiträge/2.246.98.233|2.246.98.233]] 23:26, 26. Feb. 2017 (CET)
Heißt es jetzt "Sie Arschloch" oder drückt es wirklich Hochachtung in der Schlussformel eines Briefes aus? Im Archiv, sieht man, dass die Frage zwar schon diskutiert wurde, aber die Jahre ziehen ins Land und da gibt es immer Veränderungen in der Wahrnehmung. Persönlich glaube ich Folgendes: Früher stand dies unter jedem behördlichen Schreiben. Dann wollte man freundlicher klingen und schreibt seitdem "''mit freundlichen Grüßen''". Die Schreiben, die dann übrig blieben, sind eben die Schreiben, bei denen man sich die Freundlichen Grüße bewusst sparen möchte. [[Benutzer:Le Duc de Deux-Ponts|Le Duc de Deux-Ponts]] ([[Benutzer Diskussion:Le Duc de Deux-Ponts|Diskussion]]) 11:58, 28. Sep. 2020 (CEST)
:Sagen wir es so: Es ist eine respektvolle Schlussformel, die aber (vielleicht mit Ausnahme von Österreich, wo sich viele Dinge länger halten) heutzutage nur noch selten ernsthaft verwendet wird und bei denen der Anteil der ironisch-sarkastischen Verwendungen über die Zeit hinweg deutliche angewachsen ist. -- [[Spezial:Beiträge/79.91.113.116|79.91.113.116]] 12:10, 28. Sep. 2020 (CEST)
:: Ihr glaubt ernsthaft, dass eine Behörde oder ein Anwalt (oder von wem auch immer die Formel noch benutzt wird) ein "Hochachtungsvoll" benutzt, um dem Adressaten auf Umwegen klar zu machen, dass er ein Arschloch ist? Und das wollt ihr in der Auskunft diskutieren? ---> Café bitte. Gruß --[[Benutzer:Apraphul|Apraphul]] <small><sup> [[Benutzer Diskussion:Apraphul|Disk]] </sup> <sub> [[Benutzer:Apraphul/WP:SNZ|WP:SNZ]]</sub></small> 12:17, 28. Sep. 2020 (CEST)
:::Lies bitte nochmal: '' '''Früher''' stand dies unter jedem behördlichen Schreiben. '' Also nicht auf den heutigen Gebrauch bezogen. -- [[Spezial:Beiträge/79.91.113.116|79.91.113.116]] 12:22, 28. Sep. 2020 (CEST)
:::: Lies Du bitte nochmal. Es geht sehr wohl um die heutige Nutzung, Bedeutung und Wahrnehmung; und das gibt es ja schließlich auch heute noch - und zwar in wortwörtlichem Gebrauch. Hier aber soll unterstellt werden, es würde wenn, dann nur noch als "Sie Arschloch" (oder ähnliches) benutzt. Das ist einer WP:Auskunft nicht wirklich würdig ... Gruß --[[Benutzer:Apraphul|Apraphul]] <small><sup> [[Benutzer Diskussion:Apraphul|Disk]] </sup> <sub> [[Benutzer:Apraphul/WP:SNZ|WP:SNZ]]</sub></small> 12:34, 28. Sep. 2020 (CEST)
:::::Unter Anwälten wird heute in D "Mit freundlichen kollegialen Grüßen" geschrieben. Wird das kollegialen weggelassen, ist das Verhältnis gespannt, da der Gegenseite die Kollegialität abgesprochen wird. "Hochachtungsvoll" ist da dann die Steigerung - es gibt keine Freundlichkeit und Höflichkeit mehr, zwei Jahrhunderte früher hätte man sich duellieren müssen. Im allgemeinen Sprachgebrauch dürfte Hochachtungsvoll ausgestorben sein, im amtlichen und betrieblichen Sprachgebrauch sind die Grußformeln meist irgendwo per Dienst- oder Arbeitgeberanweisung vorgeschrieben. -- [[Benutzer:Chuonradus|Chuonradus]] ([[Benutzer Diskussion:Chuonradus|Diskussion]]) 12:43, 28. Sep. 2020 (CEST)
: Ernstzunehmende Anschreiben beginnen mit "''Hochzuverehrender Herr Bundespräsident!''", und wie sie enden, habe ich vergessen und kann ich auch nicht nachschlagen, weil ich meinen ''Graudenz'' verlegt habe, und brauche ich auch nicht zu wissen, weil ich solche Schreiben nicht verfasse. --[[Spezial:Beiträge/95.112.231.60|95.112.231.60]] 13:23, 28. Sep. 2020 (CEST)
::Hängt wohl stark vom Kontext ab, s. [[Grußformel (Korrespondenz)]]. Gegenüber Anwälten eher vernichtend, bei Ärzten wohl üblich, gegenüber hohen Amtsträgern verpflichtend. Und manchmal vielleicht auch einfach gedankenlos.--[[Benutzer:Mhunk|Mhunk]] ([[Benutzer Diskussion:Mhunk|Diskussion]]) 13:28, 28. Sep. 2020 (CEST)
::: Ich unterschreibe vorzugsweise „Mit (noch) freundlichen Grüßen ...“--[[Benutzer:Gadacz|Klaus-Peter ''<small>(<sup>auf</sup><small>und</small><sub>davon</sub>)</small>'']] 15:56, 28. Sep. 2020 (CEST)
Okay, also als Quintessenz: Da man nicht weiß, ob das Gegenüber, also der Briefempfänger, von diesen Vermutungen gehört hat, oder ob er 'Hochachtungsvoll' ganz altmodisch als Ehrenbezeugung wahrnimmt, und auch unabhängig davon ob 10 oder 90 Prozent der Bevölkerung davon gehört haben, dass es als Ersatzbeleidigung dienen kann, empfiehlt es sich, im Falle der ernst gemeinten Höflichkeit auf 'Hochachtungsvoll' zu verzichten. Will man dem Empfänger aber eins reinwürgen, dann schreibt es eben. Eine Alternative wie man in der Grußformel wirklich (moralisch-legal) beleidigend werden könnte, sehe ich nicht. [[Benutzer:Le Duc de Deux-Ponts|Le Duc de Deux-Ponts]] ([[Benutzer Diskussion:Le Duc de Deux-Ponts|Diskussion]]) 10:42, 29. Sep. 2020 (CEST)
: Du suchst eine "Alternative", wie man in einer ''„Grußformel wirklich [..] beleidigend werden könnte“''?? Warum?? Wir sind hier die (hoffentlich enzyklopädische) WP:Auskunft. Wie könnte hier eine solche Schlussfolgerung, wie Du sie gerade getroffen hast, von wegen dass "hochachtungsvoll" als Ersatzbeleidigung dienen kann, unwidersprochen stehenbleiben? Der oben verlinkte Artikel [[Grußformel (Korrespondenz)]] ist möglicherweise der schlechteste - weil nahezu belegfrei -, den man hier anführen dürfte. Irgendwelche Behauptungen über Anwaltsjargon bzgl. des "hochachtungsvoll" oder bzgl. des "kollegialen" sind erstmal nur so dahergesagt und auch nur POV oder gibt es dafür irgendwo reputable Belege? Der Artikel ''Grußformel (Korrespondenz)'' behauptet übrigens für das Weglassen von "kollegial" unter Anwälten das genaue Gegenteil von dem, was hier schon erzählt wurde. Alles also nur Hören-Sagen! Wo ist der amtlich und gesellschaftlich anerkannte Knigge für Grußformeln, den man zitieren könnte?? Hat den einer? Gut, her damit. Bis dahin bitte keinen weiteren POV und etwas mehr Seriösität hier in WP:Auskunft ... oder aber ab ins Café. Sorry, aber manchmal ... Gruß --[[Benutzer:Apraphul|Apraphul]] <small><sup> [[Benutzer Diskussion:Apraphul|Disk]] </sup> <sub> [[Benutzer:Apraphul/WP:SNZ|WP:SNZ]]</sub></small> 11:03, 29. Sep. 2020 (CEST)
:: Lieber [[Benutzer:Apraphul|Apraphul]], die Anfänge des Artikels [[Grußformel (Korrespondenz)]] führen uns in die Zeiten zurück, als es noch keine Belegpflicht gab. Es ist schade, dass da noch keine gründliche Überarbeitung stattgefunden hat, aber die Kapitel Einzelnachweise, Literatur und Weblinks gibt es immerhin. Schon alles durchgelesen? Ich halte die Beschäftigung mit diesem Artikel in der Auskunft für durchaus nutzbringend, auch wenn ich durch das regelmäßige Studium der VM die Überzeugung gewonnen habe, dass der Begriff Höflichkeit die meisten der dort genannten Personen kaum zu interessieren scheint. Aber deshalb solche Fragen einschließlich der Befassung mit dem genannten Artikels gleich ins Café verschieben zu wollen, geht mir dann doch zu weit. Das passt schon in die Auskunft, auch wenn die Eingangsfrage provokant formuliert ist, aber ohne diese würden wohl alle nur gähnen. MfG --[[Benutzer:Regiomontanus|Regio]] <small>([[Benutzer Diskussion:Regiomontanus|Fragen und Antworten]])</small> 11:39, 29. Sep. 2020 (CEST)
::: Hi Regio, Themen, die ohne reißerische Aufmachung nur Gähnen erzeugen würden, sind vielleicht auch einfach nur zum Gähnen. ;-) Eine Themenrelevanz wird durch Gähnen aber weder erreicht noch gemindert. Zu den Referenzen im Artikel: Ich habe sie zugegebenerweise maximal nur überflogen. Magst Du mir helfen und mir aufzeigen, wo "hochachtungsvoll" als negativ besetzte Grußformel beschrieben wird? Danke im Voraus. Allerdings befindest Du Dich m. E. grundsätzlich im Irrtum, wenn du denkst, dass dieser Anschnitt sich mit dem Artikel beschäftigt. Dieser Abschnitt beschäftigt sich vielmehr nur mit der Frage, ob "hochachtungsvoll" heutzutage negativ besetzt ist. Was der Artikel dazu beitragen kann, ist mir persönlich nicht nur unklar, sondern ich hatte es ja sogar bestritten. Also kurz: Um dem Artikel geht es gar nicht. Dann weiter: Um die grundsätzliche Höflichkeit der Menschen untereinander geht es auch nicht - und schon gar nicht um die wenige Höflichkeit in Wiki-VMn. Dieser Abschnitt postuliert oder hinterfragt schlicht und ergreifend nur eine unterstellte und nicht reputabel belegte Bedeutung von "hochachtungsvoll". Und in der bisher erfolgten Formulierung der Frage sowie der meisten Antworten gehört das maximal ins Café. Das ändert sich, sobald jemand endlich die heutige Bedeutung von "hochachtungsvoll" beschreiben und belegen kann, sofern sie von dem ursprünglichen Wortsinn abweicht. WP:Auskunft ist kein Forum zur Ergründung und Festigung neuer Weisheiten, sondern sie dient der Feststellung ''vorhandener'' Weisheiten, wenn nach ihnen gefragt wird. Gruß --[[Benutzer:Apraphul|Apraphul]] <small><sup> [[Benutzer Diskussion:Apraphul|Disk]] </sup> <sub> [[Benutzer:Apraphul/WP:SNZ|WP:SNZ]]</sub></small> 12:02, 29. Sep. 2020 (CEST)
::::Bitte keinen Streit! Sicherlich habe ich provokant gefragt. Vielleicht sogar ein wenig zu extrem. Tatsächlich, hat meine Frage aber mit meiner täglichen Arbeit zu tun. Jedes Schreiben unserer Stadt, wird eigentlich immer mit ''mit freundlichen Grüßen'' beendet. "Böse" Schreiben, wie Androhungen von Ersatzvornahmen unterzeichne ich aber seit Jahren mit ''Hochachtungsvoll''. Ein älterer Kollege meinte genau das Gegenteil: Wenn er sich an den OB wendet, hält er ''mit freundlichen Grüßen'' für unangemessen und unterschreib mit ''Hochachtungsvoll''. Ich habe Sorge um meinen Kollegen, dass der um einiges jüngere OB, dies wiederum genau falsch verstehen könnte. [[Benutzer:Le Duc de Deux-Ponts|Le Duc de Deux-Ponts]] ([[Benutzer Diskussion:Le Duc de Deux-Ponts|Diskussion]]) 07:33, 30. Sep. 2020 (CEST)
::::: @[[Benutzer:Le Duc de Deux-Ponts|Le Duc de Deux-Ponts]], dann lies [https://gfds.de/grussformel-bei-briefen-mit-unangenehmem-inhalt/ das hier mal (klick)]. Ist zwar auch schon wieder sieben Jahre alt, aber die GfdS ist vielleicht zusammen mit dem Duden die würdigste Quelle zur Schärfung seiner Grußgewohnheiten. :-) Gruß --[[Benutzer:Apraphul|Apraphul]] <small><sup> [[Benutzer Diskussion:Apraphul|Disk]] </sup> <sub> [[Benutzer:Apraphul/WP:SNZ|WP:SNZ]]</sub></small> 08:17, 30. Sep. 2020 (CEST)


Es gibt nicht '''die''' Lernstrategie, sondern höchstens '''Deine''' Lernstrategie. Welche das ist, musst Du selber rauskriegen. Es gibt Gruppenlerner, die am besten mir zweidrei anderen lernen können, aber die anderen muss man erst mal finden. Dann gibt es die Einzelkämpfer, die nur alleine oder bevorzugt alleine lernen können/wollen. Dann gibt es die Permanentnachbereiter, die stets in Häppchen die Veranstaltungen rekapitulieren, auf der anderen Seite die Frontallerner, die binnen kürzester Zeit den Stoff von mehreren Semestern in die Birne knallen können (meist aber genauso schnell wieder vergessen) und vieles mehr. Versuch diverse Methoden aus, wenn Du nicht jetzt schon eine Ahnung hast, was Dir wahrscheinlich am besten liegt. Reine Fakten kann man auch mit der [[Lernkartei|Karteikastenmethode]] erarbeiten. Viele Hochschulen bieten Seminare über Lerntechniken an, wo stliche Methosen vorgestellt werden, da solltest Du ggf. hingehen. --[[Benutzer:Elrond|Elrond]] ([[Benutzer Diskussion:Elrond|Diskussion]]) 23:33, 26. Feb. 2017 (CET)
== Studio Ghibli gibt 400 Bilder aus seinen Filmen für alle frei ==
:Lies den Thread einfach mal genauer. Der Fragesteller ''ist'' Dozent und sucht fundierte Lerntipps für seine Studenten. [[Benutzer:Rainer Zenz|Rainer Z]] [[Benutzer Diskussion:Rainer Zenz| ...]] 23:52, 26. Feb. 2017 (CET)
::Warum sollte für den Herrn Dozenten etwas anderes gelten. Es gibt nun mal nicht den Königsweg, sondern jeder Studierende muss seinen Weg finden. Was nutzt eine ach so erfolgreiche, mit zig Studien validierte Methode, wenn der einzelne Studi nicht damit klar kommt! Ich arbeite selber an einer Hochschule als Lehrender und habe nun mal diese Erfahrung gemacht. In meinem Studium dereinst aber auch schon. Wenn der Frager als Dozent die diversen Hochschuldidaktischen Zentren nicht kennt, oder nutzt, wird er hier sicher auch nichts besseres erfahren. --[[Benutzer:Elrond|Elrond]] ([[Benutzer Diskussion:Elrond|Diskussion]]) 00:04, 27. Feb. 2017 (CET)


:Hm. Ich bin etwas verwirrt (und das mag auch an meinem Alter liegen...). Als ich – vor langer Zeit – mein Abitur bekam hatten wir am Gymnasium bereits weitgehend gelernt, wie man lernt. Neu waren die vielfältige Infrastruktur der Universität und der Fachbereiche, die (über die UB hinaus) zum Abrufen von Informationen zur Verfügung stand. Dafür gab es Einführungskurse und Führungen für die Erstsemester. Die meisten Vorlesungen und Seminare kamen auch ohne medialen Schnickschnack klar. Eine Tafel reichte zur Not aus. (Das mag in den naturwissenschaftlichen Fächern anders gewesen sein als in den Sozialwissenschaften.) Wir waren – wenn der kleine Scherz erlaubt ist – Hörerinnen und Hörer und nicht Seherinnen und Seher. Meines Erachtens gehört die Fähigkeit, selbstorganisiert zu lernen, zu dem Fundament einer „Hochschulreife“. Warum ist man davon abgekommen? Was machen die Kids dann eigentlich auf dem Gymnasium, wenn nicht (auch) die Vorbereitung auf ein (mögliches) Studium absolvieren? Damit es nicht mißverstanden wird: Zweifellos hat sich die Didaktik fortentwickelt und auch Medien können hilfreich sein. Das Anliegen des Fragestellers ist also durchaus ehrenwert. Aber das sind doch die Sahnehäubchen. ''Erfolgreich'' im Lernen kann man auch ohne das sein. Die Voraussetzungen für einen Erfolg liegen imho überwiegend anderswo und nicht (provokant zugespitzt) im mundgerechten, die eigene Konzentrationsleistung und Denkanstrengung auf ein Minimum reduzierenden Spektakel. --[[Spezial:Beiträge/2003:46:A0C:6E00:E22A:82FF:FEA0:3113|2003:46:A0C:6E00:E22A:82FF:FEA0:3113]] 05:41, 27. Feb. 2017 (CET)
Kann jemand Japanisch, und hier mal schauen, ob die Bilder auch für Wikipedia in einer passenden Lizenz verfügbar sind. Nachricht: [https://stadt-bremerhaven.de/studio-ghibli-gibt-400-bilder-aus-seinen-filmen-fuer-alle-frei]. Danke. -- [[Benutzer:Stefan Kühn|sk]] ([[Benutzer Diskussion:Stefan Kühn|Diskussion]]) 12:54, 28. Sep. 2020 (CEST)


:: Ich habe "Studium" und "Powerpoint" gelesen. Jedem, der so etwas tut, möchte ich [[Yiannis Gabriel]] empfehlen und den folgenden Link [http://www.yiannisgabriel.com/2012/04/against-tyranny-of-powerpoint.html Against the Tyrrany of Powerpoint].
:Da steht auch nicht viel mehr als in der deutschen Übersetzung von stadt-bremerhaven.de, nämlich "Bitte zögern Sie nicht, die Fotos im Rahmen des gesunden Menschenverstandes zu verwenden.", ich bezweifele, dass das eine gültige Commons-Lizenz ist. Wäre ggf. auch eher was für [[Wikipedia:Urheberrechtsfragen]]. -- [[Benutzer:Jonathan Haas|Jonathan]] 13:58, 28. Sep. 2020 (CEST)
:: Demnach verarmt Powerpoint Studenten und Dozenten. Es gibt sicher gute Gründe es zu verwenden, aber es gibt auch gute Gründe, hin und wieder darauf zu verzichten. [[Benutzer:Yotwen|Yotwen]] ([[Benutzer Diskussion:Yotwen|Diskussion]]) 09:23, 27. Feb. 2017 (CET)


:@Hass at Powerpoint: Ich (aus dem Naturwissenschaften-Bereich: harte Fakten und Zahlen) will euch was zeigen. Wie soll ich das machen? Mein Zeuch mit Kreide an die Tafel malen? Eh ich fertig bin, ist der Zuhoerer schon "verbloeded" (Zitat von oben). Ich muesste also mein Diagramm irgendwie schnell hinkratzen und die wichtigen Dinge einkringeln, um es dann 5 Minuten spaeter wieder abzuwischen (weil die anderen 3 Tafel-Abschnitte auch schon mit schiefen Diagrammen voll sind). Ich koennte aber auch Overhead-Projektor-Folien nehmen und die ueber den Projektor schmeissen. Und weil ich faul bin, werden die alten Folien (vom letzten Jahr) nicht voellig neu gemalt, sondern ich korrigier / male neues Zeug dort dazwischen. Na wenn euch das lieber ist... ;) Oder ich nehm Powerpoint, kann direkt die Grafiken aussm Lehrbuch zeigen, und einfach neues Zeugs einfuegen und altes anpassen. Meine Folien haben nie mehr als 15 Woerter auf einer Folie (Titel, Beschriftungen, alles mitgezaehlt; selbst auf der Zusammenfassungsfolie am Schluss max. 15). Oder isses euch am liebsten, wenn ich garnix zeige und einfach nur "vorlese", wies oben auch schon vorgeschlagen wurde ("Hoerer statt seher"): "Also, wat is en Dampfmaschin? Da stelle mehr uns janz dumm. Und da sage mer so: En Dampfmaschin, dat is en grosset schwarzet Loch..." Nee, wem das so lieber ist, der ist bei mir verkehrt!--[[Benutzer:Nurmalschnell|Nurmalschnell]] ([[Benutzer Diskussion:Nurmalschnell|Diskussion]]) 12:04, 27. Feb. 2017 (CET)
::Japanisch kann ich nicht, aber ich sehe bei Ghibli auch nichts anderes als [[Benutzer:Jonathan Haas|Jonathan]], nämlich den schwammigen Hinweis "※画像は常識の範囲でご自由にお使いください。", den Google als "Bitte zögern Sie nicht, die Bilder im Rahmen des gesunden Menschenverstandes zu verwenden" übersetzt. Eine echte "Freigabe" bzw. freie Lizenz in unserem Sinne ist nirgends zu finden. Insbesondere würde ich vermuten, dass das Studio Ghibli die kommerzielle Weiternutzung durch jedermann, die hier auch erlaubt sein muss (man dürfte also z.B. seine eigenen abgeleiteten Werke auf Basis dieser Bilder schaffen und verkaufen), nicht vom "Rahmen des gesunden Menschenverstands" abgedeckt sehen wird. Im Prinzip handelt es sich wohl einfach um eine Art Pressebilder, wie wir sie hier nicht verwenden können. [[Benutzer:Gestumblindi|Gestumblindi]] 21:37, 29. Sep. 2020 (CEST)


:: Gabriel sagt nirgends, dass man vollständig auf PP verzichten soll. Er sagt, dass überall da, wo man es einsetzt, weniger Fragen gestellt werden, weniger hinterfragt wird und weniger Auslassungen erkannt werden. Kurz: mit Powerpoint scheinen Studenten zu verdummen. Dem kannst du nur entgegenwirken, indem du ihnen hin und wieder Powerpoint entziehst. Das mag durchaus mehr in den humanistischen Fächern erforderlich sein, aber auch in ganz nicht-humanistischen Fächern wie Ingenieurstudiengängen muss man irgendwann über Qualität, Sicherheit und Recht und Gesetz sprechen. Und da kann man vielleicht gut auf PP verzichten. [[Benutzer:Yotwen|Yotwen]] ([[Benutzer Diskussion:Yotwen|Diskussion]]) 12:15, 27. Feb. 2017 (CET)
:::Also wenn ich die deutsche Phrase ''„im Rahmen des gesunden Menschenverstandes“'' nehme, dann kann kommerzielle Weiternutzung schwerlich ausgeschlossen sein und so würde ich die durchaus auch verwenden. Die Interpretationshoheit kann ja nicht beim Lizenzgeber liegen, wenn es mit rechten Dingen zugeht.
:::Meiner Erfahrung nach eignet sich PP zwar recht gut für Vorträge, aber schlecht um damit zu lernen. Die wenigen Worte pro Folie eignen sich gut als Gedankenstütze für den Vortragenden und als Merkhilfe für die Zuhöhrer, aber es fehlt häufig an Erklärungen und Zusammenhängen für das spätere Lernen, die typischerweise mündlich dargeboten werden. Und Mitschreiben fällt schwerer, weil das Vortragstempo höher ist als wenn der Dozent nur an der Tafel arbeitet und selber schreibt.
:::Wie sich die Phrase aber mit einer Freien Lizenz verträgt ist .. zumindest eine gute Frage. --[[File:Lemon with white background.jpg|15px|Quetsch mich aus, ... ]] [[Benutzer:Itu|itu]] ([[Benutzer Diskussion:Itu|Disk]]) 22:49, 29. Sep. 2020 (CEST)
:::Wie sinnvoll PP ist scheint mir durchaus vom Fach abhängig zu sein. In Mathe und Physik sind wohl Vorträge mit Tafel / ohne PP üblich, was sicher Gründe hat. In den ingenieurwissenschaftlichen Fächern die ich hatte, waren PP-Vorträge üblich, aber man möchte man ja schließlich ein Bild von der Maschine sehen um die es geht und nicht nur eine verbale Beschreibung oder Skizze an der Tafel. Die Vorlesungen die ich zu juristischen Themen hatte, kamen fast ganz ohne PP oder Tafel aus: Rein mündliche Vorträge und die Aufmerksamkeit war deutlich höher als sonst. --[[Benutzer:Der-Wir-Ing|DWI]] ([[Benutzer Diskussion:Der-Wir-Ing|Diskussion]]) 15:14, 27. Feb. 2017 (CET)


::::Folienpräsentationen sind vor allem ermüdend. Kennt man von Diavorträgen. Der Raum wird abgedunkelt und auf wundersame Weise entschlummert das Publikum. Heutzutage dank lichtstarker Projektoren nicht mehr ganz so ein Problem. Weiterhin neigen die Vortragenden dazu, viel zu viele Folien zu machen. Wurden an der Tafel pro Vorlesung zuvor vielleicht drei Tafelbilder (zu 2-4 Tafeln) gezeigt, sind es in der Folienform gleich 20 Folien oder gar mehr. Ich weiß nicht, wer den Lehrenden diesen Quatsch beigebracht hat, aber es gibt kaum etwas entnervenderes als wenn man als "Seher" durch zu schnelles Umblättern abgehängt wird. Da hilft es dann auch nicht, wenn nur ein einzelner Punkt pro Folie behandelt wird "den dann ja jeder verstanden haben müsste". Der Zusammenhang kann im Kopf nicht so schnell gebildet werden, wie der Vortragende ihn abspult. Man ist gefangen in einem Actionfilm, in dem man nicht zur Besinnung kommt. -- [[Benutzer:Janka|Janka]] ([[Benutzer Diskussion:Janka|Diskussion]]) 15:49, 28. Feb. 2017 (CET)
::::So oder so ist "im Rahmen des gesunden Menschenverstands" keine für die Wikipedia oder für Commons geeignete Lizenz, solange wir keine Lizenzvorlage {{CVL|Gesunder Menschenverstand}} haben, die diesen genau definiert ;-) [[Benutzer:Gestumblindi|Gestumblindi]] 22:51, 29. Sep. 2020 (CEST)
:::::Die iss schnell gemacht! <small>/[[Heinz_Becker_(Bühnenfigur)|Heinz Becker]]</small> --[[File:Lemon with white background.jpg|15px|Quetsch mich aus, ... ]] [[Benutzer:Itu|itu]] ([[Benutzer Diskussion:Itu|Disk]]) 23:37, 29. Sep. 2020 (CEST)


== angebliches Kafka-Zitat über Edgar Allan Poe ? ==


In einem Buch habe ich diese englische Übersetzung (?) gefunden, die man auch online finden kann (allerdings nur 4 Treffer, davon eine Wikipedia-Kopie und das Buch, in dem ich es gefunden habe):


Auch wenn das heutzutage so gesehen wird: Das eigentliche Ziel der Uni ist nicht der "Job", sondern die Ausbildung zum Spezialisten. Darum gibt es "die" Strategie nicht. Lernen muss man, aber "lernen" ist auch nicht das einzige Ziel des Studiums. Du bist hier nicht in einer Fachhochschule. Es geht nicht so sehr darum, dass Du "gute" Noten hast (Was wichtig ist) als vielmehr darum, dass Du ein Wissensfeld und Kenntnisfeld abdeckst. Da musst Du Dich dann wirklich gut auskennen, bundesweit oder international. Geistiges Arbeiten erfordert nicht nur lesen (das meiste wirst Du exzerpieren), v.a. aber die Fähigkeit Standpunkte zu überdenken, zu recherchieren und intensiv geistig zu arbeiten (wie ein Mönch). --[[Spezial:Beiträge/217.238.148.36|217.238.148.36]] 12:38, 27. Feb. 2017 (CET)
{{Zitat-en|He was a poor devil who had no defenses against the world. So he fled into drunkenness. Imagination served him only as a crutch. He wrote tales of mystery to make himself at home in the world. That's perfectly natural. Imagination has fewer pitfalls than reality ... I know his way of escape and his dreamer's face.}}


: Hier fehlt etwas: Der Überblick über das gesamte Fachgebiet und die Verzahnung ihrer Teile! Ich habe Chemie studiert und erst bei den Vorbereitungen zum Diplom so richtig "gelernt", wie alles zusammenhängt. Das erwartet z. B. ein auf Forschung ausgerichteter Arbeitgeber oder Doktorvater nach Abschluss des Studiums.
Ich kann nirgends das deutsche Original finden. Stammt der Text überhaupt von Kafka (und wenn ‚ja‘, bezog er sich tatsächlich auf Poe)?


: Außerdem "Was man von einem Studierten erwartet (oder verlangt), was aber kein 'Lehrstoff' irgendeiner Uni ist": Lösung einer Aufgabe, die gar nicht zum studierten Fach gehört. Mich hat mal eine Germanistik-Studentin in ihren 5. Semester Folgendes gefragt: "Wir machen ein Praktikum, das besteht darin, dass wir die Kloster-Dalheimer Musikfestspiele organisieren - ich bin für die Sicherheit verantwortlich und habe keine Ahnung, was ich dafür tun muss". Meine Auskunft: "Wenn ein Arbeitgeber einen Akadamiker mit so etwas konfrontiert, so erwartet er nach ein paar Tagen ein Konzept mit "Was muss erfüllt werden, was ist schon erfüllt, woran fehlt es, wer könnte das machen, was kostet das und wo nehmen wir das Geld dafür her". Am Anfang gibt es also nur ein leeres, weißes Papier und am Ende eine vortragsreife Antwort. Und dieser Vortrag sollte auch dann stotterfrei sein, wenn es 1000 Zuhörer gibt oder nur 3: Den Oberbürgermeister, den Kultusminister und den Hochschulrektor.
Zweifelnd, [[Benutzer:Brunswyk|Brunswyk]] ([[Benutzer Diskussion:Brunswyk|Diskussion]]) 14:37, 28. Sep. 2020 (CEST)


: Lasst Euch nach einem langen Berufsleben sagen: Wer das am besten kann, der prägt sich seinem Abteilungsleiter als sein Nachfolger ein, denn als Vorgesetzer hat man jeden Tag so ein weißes Papier und das - nach wie vor wichtige - Fachwissen tritt immer mehr in den Hintergrund! Gruß -- [[Benutzer:Dr.cueppers|Dr.cueppers]] - [[Benutzer Diskussion:Dr.cueppers|Disk.]] 14:18, 27. Feb. 2017 (CET)
:Janouch, Gustav. Conversations with Kafka. Trans. Goronwy Rees. New York: A New Directions Book, 1971. Möglicherweise im Orginal in der deutschen Ausgabe von 1968.--[[Benutzer:Mhunk|Mhunk]] ([[Benutzer Diskussion:Mhunk|Diskussion]]) 16:01, 28. Sep. 2020 (CEST)
Als Dozent wird der Fragesteller nie allen Bedürfnissen und Lernertypen gleichzeitig gerecht werden. Variiere also die Didaktisierung deiner Lehrveranstaltungen; mach aber nichts, das du für Unfug hältst. Halte deine Studenten dazu an, verschiedene Strategien und Techniken auszuprobieren; wenn sie das eh nicht schon bis zum Abwinken in der Schule gemacht und ihr Ding gefunden haben. -- [[Benutzer:MonsieurRoi|MonsieurRoi]] ([[Benutzer Diskussion:MonsieurRoi|Diskussion]]) 14:50, 27. Feb. 2017 (CET)
::Wir haben den Artikel [[Gustav Janouch]]. Kurz: "Gespräche mit Kafka" - Authenzität zweifelhaft, könnte also auch von Janouch selbst stammen.--[[Benutzer:IP-Los|IP-Los]] ([[Benutzer Diskussion:IP-Los|Diskussion]]) 17:24, 28. Sep. 2020 (CEST)
:::In der deutschen Ausgabe von Janouchs „Gespräche mit Kafka“ (S. Fischer 1961) fällt Poes Name nur ein einziges Mal als Auslöser eines kurzen Gesprächs über ''Die Verwandlung''. Das angebliche Zitat steht dort nirgendwo. --[[Benutzer:Jossi2|Jossi]] ([[Benutzer Diskussion:Jossi2|Diskussion]]) 19:33, 28. Sep. 2020 (CEST) ''Edit:'' Ich sehe gerade, die Ausgabe von 1968 (die ich nicht habe) ist „erweitert“. Dann könnte es dort natürlich irgendwo stehen. --[[Benutzer:Jossi2|Jossi]] ([[Benutzer Diskussion:Jossi2|Diskussion]]) 19:40, 28. Sep. 2020 (CEST)


Puh, oben wurde geschrieben: "Das eigentliche Ziel der Uni ist nicht der "Job", sondern die Ausbildung zum Spezialisten." Da muss ich aber heftig widersprechen. Das Ziel der Universität war noch nie Ausbildung, sondern stehts Bildung. Zum Glück haben wir im Deutschen dafür zwei Wörter. Der Philosoph Nida-Rümelin hat das einmal die Unterscheidung zwischen Humboldt und McKinsey genannt. Wilhelm von Humboldt schrieb selbst mal in einem Brief an den preussischen König: „Es gibt schlechterdings gewisse Kenntnisse, die allgemein sein müssen, und noch mehr eine gewisse Bildung der Gesinnungen und des Charakters, die keinem fehlen darf. Jeder ist offenbar nur dann ein guter Handwerker, Kaufmann, Soldat und Geschäftsmann, wenn er an sich und ohne Hinsicht auf seinen besonderen Beruf ein guter, anständiger, seinem Stande nach aufgeklärter Mensch und Bürger ist. Gibt ihm der Schulunterricht, was hierfür erforderlich ist, so erwirbt er die besondere Fähigkeit seines Berufs nachher so leicht und behält immer die Freiheit, wie im Leben so oft geschieht, von einem zum andern überzugehen.“ Ich glaube dies macht es sehr deutlich. Ähnlich äusserte sich John Stuart Mill: „Universitäten sind nicht da, um ein Wissen zu lehren, welches erforderlich ist, um zu einer bestimmten Art des Broderwerbs zu befähigen. Ihre Aufgabe ist es nicht, geschickte Rechtsgelehrte oder Aerzte oder Ingenieure zu bilden, sondern tüchtige und veredelte menschliche Wesen.“ Das bringt das Ziel der Universität sehr schön auf den Punkt. Für die Jobausbildung sind Universitäten jedenfalls nicht da. Das ist ein fundamentales Missverständnis und übersieht die Anstrengungen der aufgeklärten Bildung des Humanismus. [[Spezial:Beiträge/62.44.135.249|62.44.135.249]] 17:30, 27. Feb. 2017 (CET)
::::Ob es da steht, ist dann aber wohl auch relativ egal, weil es sich damit definitiv nicht um ein "Kafka-Zitat" handeln würde, sondern um ein Zitat aus dem Buch von Janouch, das auf erst später niedergeschriebenen Erinnerungen an Unterhaltungen, die er mit Kafka geführt haben will, basiert - damit bleibt höchstens die Möglichkeit, dass Kafka vielleicht so etwas Ähnliches zu Janouch gesagt hat, vielleicht aber auch nicht. [[Benutzer:Gestumblindi|Gestumblindi]] 20:40, 28. Sep. 2020 (CEST)
:::::Stimmt. Hinzu kommt die bekannte Unzuverlässigkeit bzw. Unglaubwürdigkeit Janouchs. In summa also: unbelegt und nicht verwendbar. --[[Benutzer:Jossi2|Jossi]] ([[Benutzer Diskussion:Jossi2|Diskussion]]) 22:48, 29. Sep. 2020 (CEST)


:Ich kenne den Namen des Dünkels nicht, den du da beschreibst, aber ich kann erkennen, dass es einer ist. Auch in einer Universität gibt es wirtschaftliche Zwänge. Die Forschung ist die Kirsche auf dem Sahnehäubchen der Bildung und die findet ihre Grundlage auf dem weichen Fundament einer möglichst breiten, nicht unmittelbar zweckbestimmten Ausbildung. Jede Universität muss diese Ausbildung anbieten, denn sonst kommt sie gar nicht auf den erforderlichen Umsatz, um die eigene Existenz zu rechtfertigen. Zudem kann keine Universität es sich wirtschaftlich leisten, allen Studenten die dazu befähigt wären die Weiterbeschäftigung im Wissenschaftsbetrieb zu ermöglichen. Das wären einfach zu viele, gesetzt, dass die Ausbildung an dieser Einrichtung sehr gut ist. Und umgekehrt kann die Masse der Studenten es sich auch nicht leisten, auf lange Sicht auf Einkommen zu verzichten. Über ein den eigenen Fähigkeiten entsprechendes Einkommen rede ich dabei noch nicht einmal.
== Wie schließe ich eine SAS-HD an ein Board mit SATA an? ==
:Der wirklich schlimme Zustand ist ja eigentlich der, dass die mühsam erworbene Ausbildung (oder gar Bildung) inhaltlich später gar nicht abgerufen wird. Der spätere Arbeitgeber verknüpft mit einem bestimmten Abschluss die Fähigkeit, eigenständig Probleme zu lösen. Schön, gut und wichtig. Hier stellt sich mir aber die Frage, ob derjenige, der statt einer universitären Ausbildung eine zum Hochstapler genossen hat, nicht wesentlich besser fährt. Die Problemlösung sieht dann anders aus, dass das lange genug gut geht, haben wir in jüngster Vergangenheit immer wieder erfahren. -- [[Benutzer:Janka|Janka]] ([[Benutzer Diskussion:Janka|Diskussion]]) 16:16, 28. Feb. 2017 (CET)
::Zum "eigentliche[n] Ziel der Uni" haben viele Leute eine eigene Meinung. Im Artikel [[Universität]] werden diese definiert als Hochschulen "die der Pflege und Entwicklung der Wissenschaften durch Forschung, Lehre und Studium dienen". Es ist also eine Mischung aus Forschungsinstitut und (Aus-)Bildungsstätte. Ob man am Ende des Studiums eher Spezialist oder Generalist ist, hängt vom Studiengang und den gewählten Schwerpunkten ab.
::[[Benutzer:Janka|Janka]], den "den Namen des Dünkels" angeht: [[Humboldtsches Bildungsideal]]. In der Praxis legen Leute die es vertreten werte auf (Schul-)fächer wie Deutsch, Fremdsprachen, Geschichte, Kunst und Musik wie man im [[Humanistisches Gymnasium|humanistischen Gymnasium]] sieht, das von diesem Bildungsideal geprägt wurde. Ich seh das aber eher wie du. --[[Benutzer:Der-Wir-Ing|DWI]] ([[Benutzer Diskussion:Der-Wir-Ing|Diskussion]]) 20:07, 28. Feb. 2017 (CET)


Zurück zur Eingangsfrage: Was funktioniert, ist das Vor- und Nachbereiten von Veranstaltungen. Dabei geht es um die Optimierung der Wissensaufnahme und Speicherung. Optimale Wissensaufnahme ist bspw. gegeben, wenn der Student ~ 70 % des vorgestellten Inhaltes bereits kennt und ~ 30 % neue Informationen präsentiert bekommt. Zu viele neue Infos überfordern die Aufnahmekapazitäten, zu wenige führen zu abschweifenden Gedanken. Die Studenten sollten sich also grob ins Thema einlesen, um optimal von Studienveranstaltungen profitieren zu können. Damit sie das neu aufgenommene auch behalten, ist die Nachbereitung notwendig, sonst können sie später nicht auf das gespeicherte zugreifen. Ziel der Nachbereitung ist, Verständnislücken zu schließen und das neu aufgenommene möglichst intensiv mit dem schon bekannten zu verknüpfen. Hilfreich dabei ist, das neu aufgenommene zu hinterfragen, denn so kann man sich eine kritische Meinung bilden, was automatisch die selektive Bahnung im Gehirn fördert. Komplett abraten würde ich von auswendig lernen, denn das ist Lebenszeitverschwendung. Inhalte, die man auswendig lernen kann, kann man auch bei Bedarf nachschlagen, das ist wertlos. Im Zentrum des Lernens sollte immer das Verständnis, das gedankliche durchdringen der Materie stehen - zumindest an Hochschulen. Wem das zuviel ist, der sollte lieber eine Ausbildung machen. P.S.: Studien zu Deiner Frage findest Du, wenn Du nach der Neurobiologie des Lernens suchst. Das ist übrigens auch so ein Ding: Selber recherchieren macht schlau. Oder Du machst zu dem Thema ein Seminar und läßt das Deine Studenten als Hausaufgabe erledigen ...;) --[[Spezial:Beiträge/88.69.253.77|88.69.253.77]] 01:28, 28. Feb. 2017 (CET)
Muss eine SAS-HD umpartitionieren und kann das nicht in-System machen, sondern muss einen externen PC bemühen. Wie bekomme ich die HD an den PC am besten (=am günstigsten) angeschlossen? Braucht es eine PCIe-SAS-Controller Karte oder kann man das irgendwie auch an einen normalen SATA-6 Anschluß per Adapter bewerkstelligen? [[Benutzer Diskussion:morty|Benutzerkennung: '''43067''']] 14:43, 28. Sep. 2020 (CEST)


== Internetseiten möglichst schnell aufbauen und Videos abspielen ==
:SAS-Platte an SATA-Mainboard-Stecker geht in der Regel nicht, bzw. nur bei manchen Platten mit SAS-Anschluss, die zusätzlich oder nur SATA (im Kompatibilitätsmodus) sprechen. Umgekehrt SATA-Platte an SAS-Mainboard-Anschluss würde gehen. Ich sehe ohne SAS-Controller-Karte keine sinnvolle Möglichkeit. -- [[Benutzer:Jonathan Haas|Jonathan]] 15:33, 28. Sep. 2020 (CEST)
:: Danke, ich hatte es schon befürchtet. Kannst Du zufällig einen nicht allzu teuren Controller empfehlen? [[Benutzer Diskussion:morty|Benutzerkennung: '''43067''']] 17:01, 28. Sep. 2020 (CEST)
::: Nicht wirklich. Am effektivsten wird wohl sein, auf eBay oder einen gebrauchten Markencontroller (vermutlich aus alten Servern) zu kaufen, die scheint es da relativ günstig zu geben. -- [[Benutzer:Jonathan Haas|Jonathan]] 22:15, 28. Sep. 2020 (CEST)
:Es soll SAS-Platten mit SATA-Kompatibilitätsmodus geben. Da sind die Adapter nicht so teuer.[https://smile.amazon.de/DeLock-62469-Adapter-SAS-SATA/dp/B00PIH44G6/ref=sr_1_9] Da würde ich als allererstes mal das Datenblatt der Platte befragen. --[[user:Rotkaeppchen68|R<span style="color:red">ô</span>tkæppchen₆₈]] 17:12, 28. Sep. 2020 (CEST)
:Mit [https://www.itsco.de/dell-poweredge-raid-controller-perc-h310-sata-sas.html so einem Controller] für den PCI-X-Anschluss deines Mainboards sollte das gehen, dann wird noch ein entsprechendes Kabel benötigt. --[[Benutzer:Joschi71|Joschi<sub>71</sub>]] <small>([[Benutzer Diskussion:Joschi71|Diskussion]])</small> 16:09, 29. Sep. 2020 (CEST)
::Aus Text und Foto geht hervor, dass das eine ganz normale PCIe-Steckkarte ist und keinesfalls eine PCI-X-Steckkarte. Diese Erweiterung des parallelen PCI-Busses konnte sich auch nicht wirklich durchsetzen, in etwa wie mitte der 1990er-Jahre der VESA-Local-Bus als Erweiterung des ISA-Busses. --[[user:Rotkaeppchen68|R<span style="color:red">ô</span>tkæppchen₆₈]] 21:08, 29. Sep. 2020 (CEST)


Hallo, so ganz habe ich es noch nicht durchschaut. Ich möchte ein Notebook, welches zusammen mit einem angeschlossenen Monitor, Internetseiten möglichst schnell aufbaut und Videos im Browser möglichst schnell und gut anzeigt, auch bei mehreren Tabs. Auf was kommt es da hardwareseitig vor allem an? Ein schneller Prozessor? Viel Arbeitsspeicher? Welchen Einfluss hat eine Grafikkarte darauf? --[[Benutzer:Eibbloff|Eibbloff]] ([[Benutzer Diskussion:Eibbloff|Diskussion]]) 19:26, 26. Feb. 2017 (CET)
== Artikel bestätigen ==
:.....es kommt auf ein schnelles Internet an.--[[Spezial:Beiträge/2003:75:AF07:FF00:4913:10E3:C308:CB52|2003:75:AF07:FF00:4913:10E3:C308:CB52]] 19:38, 26. Feb. 2017 (CET)
::Es kommt immer auf das schwächste Glied in der Kette an. Dies kann auch die Internetanbindung sein. --[[Benutzer:FriedhelmW|FriedhelmW]] ([[Benutzer Diskussion:FriedhelmW|Diskussion]]) 19:41, 26. Feb. 2017 (CET)
:::Serverseitig: Schau Dir [[Akamai#Technologie|diese Links an]]. --<span style="color:#00A000;">Hans Haase ([[BD:Hans Haase|有问题吗]])</span> 19:52, 26. Feb. 2017 (CET)
:Ram ist hier sehr wichtig. Browser schlucken pro Tab erstaunliche viel davon. Wenn du was neues kaufst fang nicht unter 8 GB an. Vier sind jetzt schon knapp, bei fortschreitender Entwicklung werden Sie in zwei Jahren zu knapp sein. --[[Spezial:Beiträge/2003:76:4E5B:795A:B115:CDF5:8A85:7198|2003:76:4E5B:795A:B115:CDF5:8A85:7198]] 13:01, 27. Feb. 2017 (CET)


:Zu Notebooks und kompakten Computern: Der einzige, der im Moment [[4K2K|4K-Videos]] in Basisausstattung abspielen kann, ist die intel-[[System-on-a-Chip]]-CPU J3455 und ihr großer Bruder J4205. Diese wurden klammheimlich auf den Markt geworfen und sind im Moment tendenziell im Preis am steigen. Denselben Trend machen Arbeitsspeicher derzeit mit. Von der CPU sind die nichts besonderes, außer, dass sie wenig strom benötigen und passive Kühlung üblich sein sollte. 8 GB RAM und eine SSD dazu machen den mechanikfreien Computer. --<span style="color:#00A000;">Hans Haase ([[BD:Hans Haase|有问题吗]])</span> 15:11, 27. Feb. 2017 (CET)
<small>Umzug nach [[Wikipedia:Fragen von Neulingen#Artikel bestätigen]]. --[[Spezial:Beiträge/94.219.186.65|94.219.186.65]] 01:06, 30. Sep. 2020 (CEST)</small>
::So ziemlich jeder neue Laptop mit diskreter [[Grafikkarte]] sollte 4K-Videos flüssig abspielen können. Das ist auch zukunftssicherer als jede [[Accelerated Processing Unit|APU]]- oder [[System-on-a-Chip]]-basierende Lösung, es steht ja z.B. schon 5K vor der Tür. Zwar kann kaum ein Laptop-Display 4K anzeigen (allerdings ist mindestens eines der wenigen Modelle mit 4K und dedizierter Grafikkarte [http://geizhals.eu/acer-aspire-v15-nitro-be-vn7-592g-74fp-nx-g6kev-001-a1346140.html?hloc=at&hloc=de&hloc=pl&hloc=uk&hloc=eu gar nicht mal so extrem teuer]), jedoch schauen 4K-videos meist auch runterskaliert besser aus, weil sie auch eine höhere [[Datenrate]] haben und die durch Kompression erzeugten Blöcke auch kleiner sind.
::Was den Aufbau von Webseiten betrifft: am wichtigsten ist natürlich die Internetverbindung, aber die CPU-Geschwindiglkeit kann durchaus einen unterschied machen, vor Allem wenn man mehrere [[Tabbed Browsing|Browser-Tabs]] geöffnet hat, besonders schwachbrüstige CPUs wie z.B. alles, was auf [[Intel Atom]] basiert, machen sich aber bei entsprechend komplexen Webseiten schon bei einem einzigen Tab bemerkbar.
::Arbeitsspeichermäßig reichen jedenfalls 8 GB, falls nur der Browser läuft, sonst sollte man sicherheitshalber 16GB nehmen, ev. per [[Upgrade]] (Arbeitsspeicher ist heute ja nicht mehr teuer). --[[Benutzer:MrBurns|MrBurns]] ([[Benutzer Diskussion:MrBurns|Diskussion]]) 14:11, 1. Mär. 2017 (CET)


== Nobilitierung ==
== Unangenehme Animationen in Excel 2016 ==


Wie ist es korrekter, oder üblicher? Wenn ein Mensch, z.B. ein Wissenschaftler, im fortgeschrittenen Alter für seine Verdienste nobilitiert wurde, und man in Textstellen über Ereignisse auf ihn Bezug nimmt, die zeitlich (noch lange) vor seiner Nobilitierung liegen, sollte man dann besser "Max von Mustermann" schreiben (d.h. sein Name, wie er auf dem Grabstein steht), oder besser "Max Mustermann", weil er zum betreffenden Zeitpunkt eben so hieß und noch nicht nobilitiert war? (Es geht nicht um Wikipedia, sondern um einen wissenschaftlichen Text, allerdings keine Biografie über Mustermann, sondern einen beliebigen Textabschnitt, der Mustermann erwähnt.) Ich finde Argumente für beide Varianten und suche eine Entscheidungshilfe, welche ich verwenden soll. --[[Benutzer:Neitram|<span style="color:#008800">Ne</span><span style="color:#005555">it</span><span style="color:#005588">ra</span><span style="color:#0000FF">m</span>]]&nbsp;[[Benutzer_Diskussion:Neitram|<span style="font-size:large">✉</span>]] 19:43, 26. Feb. 2017 (CET)
Beim Update von Excel 2010 nach 2016 fällt unangenehm auf, dass nun eine, ich nenne es mal, "Zellenmarkierungsanimation" erfolgt. Beim Klick von einer Zelle auf eine andere, wird die Markierung kleiner und fliegt dann quasi zur nächsten Zelle, wo sie wieder größer wird. Die Entwickler sind wahrscheinlich stolz auf sich, was sie da fabrieziert haben. Im Arbeitsalltag jedoch nervt das ungemein. Kann man das abschalten?
::Hm, ich würde behaupten wollen, dass der Werther von Goethe verfasst wurde, und nicht etwa nicht von von Goethe, aber Wikipedia sieht das anders...--[[Benutzer:Edith Wahr|Edith Wahr]] ([[Benutzer Diskussion:Edith Wahr|Diskussion]]) 20:48, 26. Feb. 2017 (CET)
:::ich glaube, dass in deiner Antwort ein 'nicht' zuviel ist! [[Spezial:Beiträge/79.224.220.217|79.224.220.217]] 21:43, 26. Feb. 2017 (CET)
::::WP vertritt an vielen Stellen den "Was er/sie am Ende war, wird genommen."-Ansatz (Heute: Der schwedische Komponist und Musikkritiker Wilhelm Peterson-Berger kommt zur Welt. ... und fängt gleich an zu komponieren?). Auch bei später verheirateten Frauen machen sie ihr Abitur als Verheiratete. Ich fände Differenzierung besser (in [[Marie Curie]] ganz gut durchexerziert), aber der Grabstein-Ansatz scheint für die meisten einfacher zu sein (man muss weniger aufpassen...). <small style="color:grey"><b>'''GEEZER'''</b></small><sup>[[BD:Grey Geezer|<span style="color:grey"> … nil nisi bene</span>]]</sup> 08:13, 27. Feb. 2017 (CET)
:::::Ist durchaus auch Kontext abhängig. Beispiel im Artikel von X.Y. Dann ist sinnvoll zu schrieben. „Maria Müller war die Jugendfreundin von X.Y.“. auch wenn sie später geheiratet hat und als Maria Bäcker begraben wurde (gerade wenn die Verbindung zu X.Y. mit der Heirat gekappt wurde). Ist Maria selber relevant, ist die Frage eher unter welchem Namen man sie kennt. Wenn sie weiter hin unter ihrem Jugendnahmen Bücher verfasst hat, dann Maria Müller. Schrieb sie aber ihre Bücher nach der Heirat unter Maria Bäcker, dann Maria Bäcker selbst dann wenn ihr Meisterwerk unter Maria Müller veröffentlicht wurde. Das jedenfals meine Meinung dazu. --[[Benutzer:Bobo11|Bobo11]] ([[Benutzer Diskussion:Bobo11|Diskussion]]) 08:24, 27. Feb. 2017 (CET)
::::::Guter Punkt, genau die gleiche Frage habe ich nämlich ebenfalls bei Frauen und Ereignissen, die vor ihrer Ehe (und der damit verbundenen Namensänderung) liegen. Es widerstrebt mir ehrlich gesagt innerlich, den Ehenamen der Frau zu verwenden, wenn es um ein Ereignis in ihrer Kindheit geht, oder z.B. um ihre Verlobung. An sich ist es ganz analog mit Nobilitierungen. --[[Benutzer:Neitram|<span style="color:#008800">Ne</span><span style="color:#005555">it</span><span style="color:#005588">ra</span><span style="color:#0000FF">m</span>]]&nbsp;[[Benutzer_Diskussion:Neitram|<span style="font-size:large">✉</span>]] 15:39, 27. Feb. 2017 (CET)
Das Problem haben wir ständig: Gebäude, die umbenannt werden, Stadien inzwischen z.B. gefühlt jährlich, Menschen, die ihr Geschlecht ändern, Städte, die auf einmal zu anderen Ländern gehören und sich anders schreiben. Eine wirklich stringente Linie haben wir aber in keinem der Fälle. --[[Benutzer:Studmult|Studmult]] ([[Benutzer Diskussion:Studmult|Diskussion]]) 12:31, 27. Feb. 2017 (CET)
:Bei Orten und Straßen schreibe ich in meinem Text gerne Klammerzusätze, etwa "Er wurde am 1. Mai 1844 in Oberdietzbach geboren (der Ort wurde 1935 nach München eingemeindet) oder "Er zog in die Gartenstraße 20 (heute Schlossallee 44)". Bei Frauen verwende ich, wo immer ich es im Kontext korrekt empfinde, ihren Mädchennamen, und bei später Nobilitierten würde ich es eigentlich vorzugsweise auch so wie bei den Frauen halten wollen. --[[Benutzer:Neitram|<span style="color:#008800">Ne</span><span style="color:#005555">it</span><span style="color:#005588">ra</span><span style="color:#0000FF">m</span>]]&nbsp;[[Benutzer_Diskussion:Neitram|<span style="font-size:large">✉</span>]] 17:07, 27. Feb. 2017 (CET)

Nobilitierung? Ist in D eigentlich nicht üblich. Nobilitierung erinnert schwer an den Lateinuntericht. ... :-) Und sie fällt auf! Und zwar negativ. --[[Spezial:Beiträge/80.187.119.44|80.187.119.44]] 21:59, 27. Feb. 2017 (CET)
::Jaja - wer seinen Artikel selbst erstellt oder dahintersteckt: Ich schrieb schon, sie fällt auf, nicht positiv und eben eigentlich nur bei der "Zielgruppe" :-) (Hier macht aber eh jeder was er will) --[[Spezial:Beiträge/80.187.119.44|80.187.119.44]] 22:01, 27. Feb. 2017 (CET)
:::Hier ist die WP:Auskunft, bitte weitergehen zum [[WP:Café]]. --[[Benutzer:Neitram|<span style="color:#008800">Ne</span><span style="color:#005555">it</span><span style="color:#005588">ra</span><span style="color:#0000FF">m</span>]]&nbsp;[[Benutzer_Diskussion:Neitram|<span style="font-size:large">✉</span>]] 10:59, 28. Feb. 2017 (CET)
Ich tendiere auch dazu, den Nobilitierungszeitpunkt zu beachten. Meine „gefühlte“ Beobachtung: Je „berühmter“ jemand ist, umso eher wird auf das „von“ oder den „Freiherrn“ verzichtet. Beispiel: Müller wurde kurz vor seinem Tode geadelt, also heißt die Straße später nach ihm todsicher „Von-Müller-Straße“; gibt es auch eine „Von-Schiller“-oder „Von-Goethe“-Straße? --[[Benutzer:Dioskorides|Dioskorides]] ([[Benutzer Diskussion:Dioskorides|Diskussion]]) 14:52, 28. Feb. 2017 (CET)

== E-Mail-Testsystem ohne echte Funktion ==

Ich bin auf der Suche nach einem quasi E-Mail-Testsystem zum Erlernen des Umgangs mit E-Mail. Quasi mit Oberfläche zum E-Mails senden und E-Mails empfangen, wobei das aber nur pseudomäßig erfolgen soll und E-Mails quasi nur intern als Nachricht weitergeleitet werden soll. Da ich hierfür weder Webserver noch Domain zur Verfügung habe, sollte das also irgendwie lokal im eigenen Netz erfolgen. Hat da jemand eine Idee? <small>(''nicht [[Hilfe:Signatur|signierter]] Beitrag von'' [[Benutzer:Quedel|Quedel]] ([[Benutzer Diskussion:Quedel|Diskussion]]&nbsp;&#124;&nbsp;[[Spezial:Beiträge/Quedel|Beiträge]])<nowiki/> 21:13, 26. Feb. 2017 (CET))</small>
:In einem [[Linux]]-System kannst du dir deinen eigenen Mail-Server einrichten. --[[Benutzer:FriedhelmW|FriedhelmW]] ([[Benutzer Diskussion:FriedhelmW|Diskussion]]) 21:22, 26. Feb. 2017 (CET)
::Du kannst Dir dazu einfach ein paar Adressen bei einem Kostenlosen (werbefinanzierten) Anbieter einrichten. oder Dein Provider erlaubt dir die Einrichtung mehrerer E-Mail-Adressen (bei mir sind 25 Adressen in 5 verschiedenen Postfächern möglich). --[[Spezial:Beiträge/212.183.52.150|212.183.52.150]] 21:55, 26. Feb. 2017 (CET)
:::[[Postfix (Mail Transfer Agent)|Postfix]] oder [[Sendmail]] --<span style="color:#00A000;">Hans Haase ([[BD:Hans Haase|有问题吗]])</span> 22:03, 26. Feb. 2017 (CET)
::In einem Windows-System geht das genauso gut, beispielsweise mit [[Hamster (Software)|Hamster]] oder [[Jana-Server]]. --[[Benutzer:Kreuzschnabel|Kreuz]][[Benutzer Diskussion:Kreuzschnabel|schnabel]] 10:49, 1. Mär. 2017 (CET)

:Warum alles so kompliziert? Ein kostenloser Account bei GMX und ein weiterer kostenloser Account bei web.de und bei Bedarf ein kostenloses Thunderbird auf dem eigenen Computer einrichten und ab geht die Post. --[[Spezial:Beiträge/80.130.225.92|80.130.225.92]] 06:21, 27. Feb. 2017 (CET)
::Da es zum Austesten ist, sollte das System nicht wirklich aktiv sein, außerdem sprechen datenschutzrechtliche Gründe gegen Postfächer bei Web.de/GMX u.ä. Das System soll ja durch mehrere Kinder aktiv genutzt werden, halt aber nur lokal bzw. auf eigenem Webspace. --[[Benutzer:Quedel|Quedel]] <sup>[[Benutzer Diskussion:Quedel|Disk]]</sup> 10:51, 27. Feb. 2017 (CET)
:::Unter [[:en:Comparison of mail servers]] findest Du eine Vergleichstabelle verschiedener Mailserver, auch Opensource. Vielleicht ist da etwas passendes dabei. Es gibt einen Windows IIS SMTP-Server, der aber nur bei den Serverversionen von Windows dabei ist, nicht beim abgespeckten IIS von Windows Professional/Pro/Ultimate. --[[user:Rotkaeppchen68|R<span style="color:red">ô</span>tkæppchen₆₈]] 11:11, 27. Feb. 2017 (CET)
:<small>Die Frage war: ''„ohne echte Funktion“'', also ein Mailer im Intranet, der nicht nach außen austauscht. Das bedeutet aufs reale Leben bezogen: funktionsfähige Briefkästen bei streikenden Zustellern. Es geht wohl um das testen von Funktionen, die einen Mailserver bedienen. --<span style="color:#00A000;">Hans Haase ([[BD:Hans Haase|有问题吗]])</span> 15:03, 27. Feb. 2017 (CET)</small>
:[http://www.minimehl.de/ minimehl.de] --[[user:Rotkaeppchen68|R<span style="color:red">ô</span>tkæppchen₆₈]] 00:18, 28. Feb. 2017 (CET)

= 27. Februar 2017 =

== Alternativprogramm in der Provence ==
Wir sind gerade in Saint Vincent les Forts und morgen geht es micht zum vol libre. Gibt es hier Schlechtwetterprogramm wie Schwimmbäder etc. im Umkreis? --[[Spezial:Beiträge/109.40.1.42|109.40.1.42]] 17:11, 27. Feb. 2017 (CET)
:[http://www.blanche-serre-poncon.com/ Voi­là]. --[[Benutzer:Komischn|Komischn]] ([[Benutzer Diskussion:Komischn|Diskussion]]) 18:08, 27. Feb. 2017 (CET)
::Im Umkreis von dort wo du jetzt gerade bist? Sicher, schau unter https://www.dortmund-tourismus.de/home.html --[[Benutzer:King Rk|King Rk]] ([[Benutzer Diskussion:King Rk|Diskussion]]) 18:44, 27. Feb. 2017 (CET)
::Wow, bist du cool. Was weiß ich, wo meine Mobil IP hinroamt? Gruß aus mittlerweile Saint André les Alps. [[Spezial:Beiträge/109.40.3.25|109.40.3.25]] 00:56, 28. Feb. 2017 (CET)
:::Ok, sorry, von IP-Roaming wusste ich bisher noch nichts. Ich hätte auch nett fragen können. --[[Benutzer:King Rk|King Rk]] ([[Benutzer Diskussion:King Rk|Diskussion]]) 05:48, 28. Feb. 2017 (CET)
::::Es gibt in der Provence an einigen Stellen Höhlen, die man besichtigen kann, die auch bei Regenwetter schön sind. --[[Benutzer:Sr. F|Sr. F]] ([[Benutzer Diskussion:Sr. F|Diskussion]]) 16:59, 28. Feb. 2017 (CET)

== Was ist Bossing ? ==

Wenn angeblich alle Mitarbeiter leiden, ist das dann Bossing und was ist, wenn der auch leidet, vielleicht unter den Mitarbeitern oder anderen Vorgesetzten. Kann man da was machen ausser dicke Haut zulegen ?Urlaub nehmen, doppelt so viel arbeiten, die Kollegen zu einem Eis einladen, trösten finde ich zu persönlich (oft sind die Leute ja selbst beteiligt ), krank melden, an was anderes denken ?


<!-- Lass die nachfolgende Zeile am ENDE deiner Frage stehen. Sie wird in deine Signatur umgewandelt. -->
<!-- Lass die nachfolgende Zeile am ENDE deiner Frage stehen. Sie wird in deine Signatur umgewandelt. -->
--[[Benutzer:Nina Eger|Nina Eger]] ([[Benutzer Diskussion:Nina Eger|Diskussion]]) 18:48, 27. Feb. 2017 (CET)
--[[Spezial:Beiträge/188.111.103.32|188.111.103.32]] 15:23, 28. Sep. 2020 (CEST)
:Der Duden bezeichnet Bossing als Schikanieren (nur) einzelner Mitarbeiter zum Zweck aus der Firma zu ekeln. Was du da umschreibst ist eher (übermäßiger) Leistungsdruck in der ganzen Firma. Ich nehme mal an, es handelt sich um einen Kleinbetrieb, sonst wäre der Betriebsrat die richtige Anlaufstelle. Miteinander darüber reden ist immer gut, wenn aber der Druck "von oben" kommt, muss mit diesem oben geredet werden. Alles andere, wie dickes Fell, krank melden ist ungeeignet--[[Spezial:Beiträge/2003:75:AF0D:2800:22:1927:5543:1FCD|2003:75:AF0D:2800:22:1927:5543:1FCD]] 19:23, 27. Feb. 2017 (CET)
:[[Google:excel+2016+animationen+ausschalten]] --[[user:Rotkaeppchen68|R<span style="color:red">ô</span>tkæppchen₆₈]] 16:08, 28. Sep. 2020 (CEST)


:Vereinfacht gesagt: Der [[Mobbing|mobbende]] Chef/Vorgesetzte. Bossing wird benutzt, um Mitarbeiter zur Kündigung zu bewegen. Der häufigste Grund ist das Vermeiden von Abfindungen, die ab einer gewissen Unternehmenszugehörigkeit fällig werden. --<span style="color:#00A000;">Hans Haase ([[BD:Hans Haase|有问题吗]])</span> 20:09, 27. Feb. 2017 (CET)
Libre Office löst viele Probleme. --[[Spezial:Beiträge/188.61.132.244|188.61.132.244]] 20:24, 28. Sep. 2020 (CEST)
::+1. Mobbing = Quälerei auf der Horizontalen (durch Kollegen = Konkurrenten), Bossing = Quälerei in der Vertikalen (von oben nach unten); jeweils mit Schwerpunkt gegen eine einzelne Person, die weggeekelt werden soll. Dazu die alte Weisheit: Der wahre Charakter eines Mensch zeigt sich daran, wie er/sie Untergebene behandelt. -- [[Benutzer:Zerolevel|Zerolevel]] ([[Benutzer Diskussion:Zerolevel|Diskussion]]) 16:03, 1. Mär. 2017 (CET)
:…bedeutet unter Windows aber [[Extremzeitraubing]], da es dort keine automatischen Updates gibt wie bei LibO für Linux oder µ$ Office für Windows. --[[user:Rotkaeppchen68|R<span style="color:red">ô</span>tkæppchen₆₈]] 21:13, 28. Sep. 2020 (CEST)
::Ein guter Grund damit gleich in [[Linux|artgerechte Haltung]] zu gehen. --[[File:Lemon with white background.jpg|15px|Quetsch mich aus, ... ]] [[Benutzer:Itu|itu]] ([[Benutzer Diskussion:Itu|Disk]]) 20:21, 29. Sep. 2020 (CEST)


== Krieg zwischen Demokratien ==
Danke, der Google-Tipp hat funktioniert, ein zusätzlicher Eintrag in der Windows-Registrierung hat dem Spuk ein Ende bereitet. --[[Spezial:Beiträge/188.111.103.32|188.111.103.32]] 08:18, 29. Sep. 2020 (CEST)
:Bei mir ging das ohne Registry-Bastelei. --[[user:Rotkaeppchen68|R<span style="color:red">ô</span>tkæppchen₆₈]] 10:16, 29. Sep. 2020 (CEST)


Ziemlich oft stößt man auf die Behauptung, zwei demokratische Staaten hätten noch niemals die Waffen gegeneinander aufgenommen (so ich gestern in einem Philosophie-Kalender). Ohne geringste Verzögerung ist mir sofort der [[Amerikanischer Bürgerkrieg|Krieg zwischen den demokratischen Republiken USA und CSA]] eingefallen – aber dann wurde es schon dünner. Südamerika rund um 1900 drängt sich irgendwie rein, aber mir fällt es schwer, sofort die Demokratien von den Nicht-ganz-so-lupenrein-Demokratien zu unterscheiden … Wer kann die Liste erweitern? Kann man sie vielleicht sogar vervollständigen, kann man alle zwischen demokratischen Staaten geführten Kriege listen?
== google passwortcheck auslesen ==
--[[Spezial:Beiträge/85.178.126.152|85.178.126.152]] 19:07, 27. Feb. 2017 (CET)
: Krieg der USA gegen Mexiko, Irakkrieg, Koreakrieg, Vietnamkrieg, 1. und 2. WK. - kommt auf den eigenen politischen Standpunkt an, da ja alle diese kriegsbeteiligten je nach Standpunkt als "demokratisch" oder "undemokratisch" bezeichnet werden. Selbst bei den USA ist diese Frage letztlich offen, da letztich nicht der "demos" die Macht "krateia" hat, sondern eine Oligarchie vorliegt. [[Benutzer:Andy king50|andy_king50]] ([[Benutzer Diskussion:Andy king50|Diskussion]]) 19:12, 27. Feb. 2017 (CET)
::[https://books.google.de/books/about/Die_lateinischen_Demokratien_Amerikas.html Die "lateinischen Demokratien Amerikas"] "um 1900" waren doch keine Demokratien! Republiken ja, einige hatten sogar schon allgemeines Wahlrecht, aber es herrschte stets eine kleine aristokratische Elite. Auch bezweifle ich das die CSA als "Demokratie" betrachtet werden können, allein schon wegen ihrer Sklaverei, und inwieweit das Wahlrecht in Südstaaten schon demokratisiert war müsste ich mal recherchieren. Also ich bin noch auf keinen Fall gekommen... Manche sehen die Grenzzusammenstöße zwischen Peru und Ecuador um 1980 oder die ersten indisch-pakistanischen Krieg als Gegenbeispiele. Peru, Ecuador und Pakistan würde ich aber jeweils den Status als Demokratie nicht zusprechen. Wirklich schon länger demokratische Staaten gibt es nur wenige, da fällt eine Einzelbetrachtung nicht zu schwer.--[[Benutzer:Antemister|Antemister]] ([[Benutzer Diskussion:Antemister|Diskussion]]) 21:01, 27. Feb. 2017 (CET)
:: genau das ist der Knackpunkt. "Demokratie" ist ein sehr vager Begriff. Die [[Attische Demokratie]], vermeintlich das plakative "Vorbild" für diese, würde heute allenfalls als Oligarchie/Plutokratie durchgehen, da das "demos" eine geringe Zahl wohlhabender Männer war (Fremde, Sklaven, Frauen, Habenichtse, nicht wehrfähige und so ein Gesox: nö danke = nix demos = nix Mitsprache) - [[Benutzer:Andy king50|andy_king50]] ([[Benutzer Diskussion:Andy king50|Diskussion]]) 22:22, 27. Feb. 2017 (CET)
Lese mal hier [[http://www.dailymail.co.uk/news/article-2039453/How-America-planned-destroy-BRITAIN-1930-bombing-raids-chemical-weapons.html?ITO=1490]] und sehe beim ZDF hier über YT [[https://www.youtube.com/watch?v=--b1vX_DV8E]]. Zu dieser Planung (Operation oder Planung RED) gabs noch in diesem Jahr Einträge in en und dt Wiki. Heute finde ich sie nicht mehr. :-) --[[Spezial:Beiträge/80.187.104.249|80.187.104.249]] 21:28, 27. Feb. 2017 (CET)
:Die Vorstellung geht auf die Schrift [[Zum ewigen Frieden]] von [[Immanuel Kant]] zurück. In der Tat fällt mir kein (tatsächlich militärisch geführter) Krieg zwischen zwei Staaten ein, die sich als „demokratisch“ im Sinne des Ideals der philosophischen [[Aufklärung]] verstanden haben. Der Sezessionskrieg („civil war“!) fällt sicherlich nicht hierunter, ebensowenig Kriege zwischen demokratischen Staaten in diesem Sinne und Staaten mit anderen Regierungskonzepten (Volksdemokratien) oder Wirtschaftskriege (die es reichlich auch zwischen „westlichen“ Demokratien gibt − und geben wird). --[[Benutzer:Melekeok|Melekeok]] ([[Benutzer Diskussion:Melekeok|Diskussion]]) 23:33, 27. Feb. 2017 (CET)
::Das Demokratien aggressiv Kriege gegen Diktaturen führen ist als Nachsatz zu obiger Behauptung Allgemeingut. Wirtschaftskriege sind keine Kriege i. eig. Sinn. Die sog. [[Kabeljaukriege]] fallen in dieses Muster.--[[Benutzer:Antemister|Antemister]] ([[Benutzer Diskussion:Antemister|Diskussion]]) 23:40, 27. Feb. 2017 (CET)
Die englische Wikipedia hat sogar einen Artikel dazu: https://en.wikipedia.org/wiki/List_of_wars_between_democracies .--[[Benutzer:Alexmagnus|Alexmagnus]] [[Benutzer Diskussion:Alexmagnus|<small>Fragen?</small>]] 00:44, 28. Feb. 2017 (CET)


:Man kann es natürlich immer so hindrehen, dass einer der Kontrahenten keine Demokratie ist. Sind die ganzen Monarchien in Europa wirklich Demokratien? Im Vereinigten Königreich z.B. könnte der Monarch das Parlament einfach auflösen und nach Hause schicken. Macht er nur nicht, weil das Volk ziemlich sicher dagegen ist und das im Endeffekt zur Abschaffung der Monarchie führen würde (und weil er keinen Grund dazu hat). In jeder Monarchie kann das Staatsoberhaupt nicht vom Volke bestimmt werden, nichtmal indirekt. Das wird immer der Zausel, der altersmäßig gerade dran ist.
Ich habe ein Google-Konto. Nun bekam ich die Nachricht, dass ich gefährdete Paßwörter von anderen Webseiten habe (u.a. gleich Passwörter für verschiedene Anbieter). Da war ich sehr baff: Wie kann Google denn meine Paßwörter auslesen (ich habe keine Google Tastatur am Handy)? Ganz schön unheimlich<!--, so wie Facebook sämtliche Cookie-Setzungen von anderen besuchten Websites registriert.--> Undenkbar, was passiert, wenn diese Daten in falsche Hände geraten. --[[Spezial:Beiträge/77.9.72.207|77.9.72.207]] 15:38, 28. Sep. 2020 (CEST)
:Die südamerikanischen Staaten waren zeitweise immer mal wieder Millitärdiktaturen. Ebenso die afrikanischen Staaten. Die osteuropäischen Staaten sind ja erst seit kurzer Zeit Demokratien. Es gibt also nur relativ wenige Demokratien, sie bestehen meistens erst seit kurzer Zeit, die Chance, das zwei davon Krieg führen ist also relativ gering. --[[Benutzer:Optimum|Optimum]] ([[Benutzer Diskussion:Optimum|Diskussion]]) 02:02, 28. Feb. 2017 (CET)


: <small>Gott! Trump ist erst seit fünf Wochen im Amt. Gebt ihm doch ein bisschen Zeit. Er wird die Theorie auch noch kippen. [[Benutzer:Yotwen|Yotwen]] ([[Benutzer Diskussion:Yotwen|Diskussion]]) 07:29, 28. Feb. 2017 (CET)</small>
:Wie bekamst du denn die Nachricht? --[[Spezial:Beiträge/2003:ED:AF1A:FC97:F6DC:128B:F2A1:83A1|2003:ED:AF1A:FC97:F6DC:128B:F2A1:83A1]] 16:02, 28. Sep. 2020 (CEST)
::Woher kommt denn die Behauptung aus der Ausgangsfrage?--[[Benutzer:Wikiseidank|Wikiseidank]] ([[Benutzer Diskussion:Wikiseidank|Diskussion]]) 10:15, 28. Feb. 2017 (CET)
:::[[Dean Babst]], 1963, siehe [[Demokratischer Frieden]]. --[[Benutzer:Neitram|<span style="color:#008800">Ne</span><span style="color:#005555">it</span><span style="color:#005588">ra</span><span style="color:#0000FF">m</span>]]&nbsp;[[Benutzer_Diskussion:Neitram|<span style="font-size:large">✉</span>]] 11:13, 28. Feb. 2017 (CET)
::::Der englische Artikel legt den Begriff dabei aber sehr weit aus, in dem er alle Länder mit gewählter Regierung (sind heute die meisten, aber das ist noch nicht lange so) als "Demokratien" betrachtet. Dem würden viele nicht zustimmen. Interessant finde ich in Liste aber: Die [[Burenkriege]]!--[[Benutzer:Antemister|Antemister]] ([[Benutzer Diskussion:Antemister|Diskussion]]) 18:58, 28. Feb. 2017 (CET)
:::::Dem könnte ich folgen, solange es eine wirksame Regelung (=|= Putsch / Revolte etc) zur Ablösung der jeweils abgelaufenen Regierung gibt. Manche Quasi-Diktatoren argumentieren: Einmal demokratisch gewählt ist mir genug Demokratie bis an mein Lebensende. -- [[Benutzer:Zerolevel|Zerolevel]] ([[Benutzer Diskussion:Zerolevel|Diskussion]]) 15:58, 1. Mär. 2017 (CET)


== Begriff „Flüchtlingskrise“ ==
::Genauer: Von wem? Und wozu wurdest du aufgefordert? Wenn’s von elitehackors@gotohell.ru kam und dir angeboten hat, nach einmaligem Eingeben deines Passworts für Sicherheit zu sorgen, hast du’s hoffentlich nicht gemacht --[[Benutzer:Kreuzschnabel|Kreuz]][[Benutzer Diskussion:Kreuzschnabel|schnabel]] 16:09, 28. Sep. 2020 (CEST)


''Flüchtlingskrise'' ist das Anfangswort mehrerer einschlägiger Lemmata; jedoch halte ich diesen Begriff, auch wenn er in aller Munde ist, für nicht unproblematisch, da er impliziert, dass die Flüchtlinge selbst, d. h. als Personen, eine Krise darstellen. Auf den Diskussionsseiten der Artikel ''[[Flüchtlingskrise in Europa ab 2015]]'' und ''[[Flüchtlingskrise in Deutschland ab 2015]]'' habe ich ebenfalls bereits darauf hingewiesen, allerdings bislang nur eine (ablehnende) Rückmeldung zu meinem dort geäußerten Vorschlag erhalten, die Lemmata jeweils mit der abstrakteren Bezeichnung ''Migrationskrise'' statt ''Flüchtlingskrise'' beginnen zu lassen und die alten Lemmata als Weiterleitungen beizubehalten. Zur Begründung: Man denke zum Vergleich bspw. an einen analogen Ausdruck wie ''Studentenkrise'', mit das Problem der Überlastung der Universitäten bezeichnet werden könnte. Dabei sind ja nicht die Studierenden selbst daran schuld, sondern in Wahrheit ein Versagen der Gesellschaft, da nicht-akademische Berufe aufgrund vielfältiger Faktoren offenbar immer mehr an Attraktivität verlieren. Ich hoffe, mein Anliegen ist durch diesen zugegeben nur behelfsmäßigen Vergleich womöglich etwas verständlicher geworden. Wie seht ihr das?--[[Benutzer:Hubon|Hubon]] ([[Benutzer Diskussion:Hubon|Diskussion]]) 19:08, 27. Feb. 2017 (CET)
::Speicherst Du Passwörter im Browser? Das ist bei mir die Quelle für diese Nachrichten. --[[Benutzer:Windharp|Windharp]] ([[Benutzer Diskussion:Windharp|Diskussion]]) 10:03, 29. Sep. 2020 (CEST)
: willkommen in der [[Euphemismus-Tretmühle]] - eine euphemisierende Namensänderung bei fortbestehendem Problem hat dieses noch nie gelöst. - [[Benutzer:Andy king50|andy_king50]] ([[Benutzer Diskussion:Andy king50|Diskussion]]) 19:18, 27. Feb. 2017 (CET)
:::Die Nachricht kam von Google selbst (E-Mail). Das Abspeichern der Passwörter im Firefox (was ich tue) ist wohl keine so gute Idee, wenn Dritte die geheimen Informationen einfach abgreifen kann. --[[Spezial:Beiträge/77.7.35.116|77.7.35.116]] 10:08, 29. Sep. 2020 (CEST)
::Hubon hat durchaus recht, dass »Flüchtlingskrise« kein neutraler Begriff ist. Allerdings hat er sich etabliert. So etwas kommt häufiger vor und färbt natürlich die Wahrnehmung der Situation. Wer erfolgreich einen Begriff prägt, setzt auch die Agenda. [[Benutzer:Rainer Zenz|Rainer Z]] [[Benutzer Diskussion:Rainer Zenz| ...]] 19:28, 27. Feb. 2017 (CET)
::::Firefox sendet die Passwörter garantiert nicht an Google. Aber was steht denn genau in der E-Mail, was ist der Betreff und der Absender? --[[Benutzer:FGodard|FGodard]]|[[Benutzer_Diskussion:FGodard|✉]]|[[Benutzer:FGodard/Bewertung|±]] 12:48, 29. Sep. 2020 (CEST)
:::Dankeschön! @[[Benutzer:Andy king50|andy_king50]]: Es geht mir keineswegs, wie von dir unterstellt, um Euphemismus. Und ich bin auch gewiss nicht der Meinung, dass durch ein anderes Wort ein politisches Problem gelöst werden kann! Vielmehr geht es mir einfach darum, dem Kind einen inhaltlich halbwegs passenden Namen zu geben. Und ich finde übrigens nicht unbedingt, dass wir da gezwungenermaßen einfach das nachplappern müssen, was Politik und Medien aufgrund ihrer systembedingt ohnehin leider allzu oft verkürzten Sprache kommunizieren – für eine Enzyklopädie, so glaube ich, sollten da doch etwas höhere Standards gelten, auch was eine möglichst objektive bzw. politisch neutrale Begriffswahl anbelangt... Kollegial--[[Benutzer:Hubon|Hubon]] ([[Benutzer Diskussion:Hubon|Diskussion]]) 21:05, 27. Feb. 2017 (CET)
:::::Die E-Mail habe ich schon gelöscht, aber ich kann mich erinnern: Ich solle gleichlautende Passwörter bei verschiedenen Portalen abänder, sodass sie einmalig sind. Dann wurde mir der Link zur Passwordcheck-Seite von Google gegeben und dort wurden mir die Portale genannt, wo ich gleiche Passwörter hinterlegt hatte. --[[Spezial:Beiträge/77.2.72.105|77.2.72.105]] 14:58, 29. Sep. 2020 (CEST)
:::: genau das habe ich gemeint: egal welche "Umbenennungsversuche" aus der eigenen offensichtlich einseitiger polischer Sicht aus vermeintlich "einzige neutraler" Sicht erfolgen, sie sind vergeblich. sie werdeb stets aufs Neue vom Sprachbrauch verworfen. Das was da abläuft ist allenfalls eine brutale,geradezu kriminelle Ausnutzung der Schwäche des deutschen politischen Sytems gegen alle Deutschen, Europäer und im Übrigen auch der integrierten Türken, Italiener etc. Da kommt eine Masse illegal und rechtswidrig ins Land und soll dann noch Rechte genießen ???? [[Benutzer:Andy king50|andy_king50]] ([[Benutzer Diskussion:Andy king50|Diskussion]]) 22:10, 27. Feb. 2017 (CET)


Hallo! Also es ist nicht hilfreich, wenn mit einer bereits feststehenden Meinung an ein Problem heranzugehen. Denn Deine Logik ist schlicht Unfug Hubon! Denn genausowenig ist der Speisenwert an der [[Nahrungsmittelpreiskrise 2007–2008]] Schuld, oder das Öl an der [[Ölkrise]]/[[Ölpreiskrise]]. Es sind immer Qualifikatoren, welche umgangsprachlich wie fachsprachlich am besten ein Problem bezeichnen. Und ohne das ich AFD-Anhänger bin, stimmt es eben nicht, daß die Flüchtlinge nicht die Krise darstellen. Deren mehrfachen illegale Grenzübertritte haben doch erst eine Sitution herbeigeführt, welche zu den Merkmalen der Krise führte, welche eben nicht aus deren Zahl weltweit oder den Gründen für die Flucht beruhte, sondern der unregulierten Einreise ohne staatliche Kontrolle, die das reine Versorgungsproblem mit einem Verwaltungs und Justizproblem verbanden. Darum ist der Begriff doch für die meisten Deutschsprachigen sehr wohl zutreffend, wenn es um die Ereignisse 2015/16 geht. Und es ist eben keine "Migration" gewesen, weder illegal noch legal! Zur Migration wird es erst durch die staatlichen Genehmigung des Aufenthalts, wobei manche davon auch strikt die Einwanderung zum Daueraufenthalt unterscheiden wollen. Wenn man vermeintlich neutrale Begriffe will, sollte man zumindest die juristische Fachsprache als Grundlage akzeptieren. Und da ist Migration in der Deutschen Sprache wie Amtssprache unbestimmter als Flüchtling.[[Benutzer:Oliver S.Y.|Oliver S.Y.]] ([[Benutzer Diskussion:Oliver S.Y.|Diskussion]]) 22:20, 27. Feb. 2017 (CET)
== Glycol in die Regentonne ==
::Die Frage hatten "wir" hier genau so, als das Lemma 2015 immer öfter hochkam (Archiv), auch im Bezug zur Bankenkrise und den zuvor genannten "Krisen".--[[Benutzer:Wikiseidank|Wikiseidank]] ([[Benutzer Diskussion:Wikiseidank|Diskussion]]) 10:14, 28. Feb. 2017 (CET)
Nach WK II gab es in Deutschland Flüchlinge und Vertriebene, sowohl gemeinsprachlich als auch amtlich definiert. Gemäß dermn individuellem Schicksal passte das auch nicht immer: Manche „Vertriebene“ waren eigentlich geflohen und für die andere Kombination gibt's wahrscheinlich auch passende Beispiele. --[[Benutzer:Dioskorides|Dioskorides]] ([[Benutzer Diskussion:Dioskorides|Diskussion]]) 14:46, 28. Feb. 2017 (CET)
:<small>Es gibt auch einen prominenten Fall, wo sich eine von einer Dienstreise in die damaligen deutsch besetzten Gebiete heimgekehrte Person als vertrieben bezeichnet. --[[user:Rotkaeppchen68|R<span style="color:red">ô</span>tkæppchen₆₈]] 16:05, 28. Feb. 2017 (CET)</small>


== Kletterrosen ==
Eine Frage an biologisch-chemisch Gebildete: Ich habe eine 330 l Regentonne aus Plaste, die ich eigentlich ungern für den Winter leeren würde, aber sie soll natürlich auch nicht kaputtfrieren. Jetzt hatte ich die Idee ein wenig Glykol rein zu tun. Weiß jemand, ob Glykol für die Pflanzen, die ich ja dann mit dem Wasser gießen möchte, giftig oder irgendwie ungünstig ist? Wohne in einer Gegend, in der es nicht so riesig kalt wird... müsste als nicht hoch dosieren. --[[Benutzerin:Mirkur|Mirkur]] ([[Benutzerin Diskussion:Mirkur|Diskussion]]) 16:00, 28. Sep. 2020 (CEST)
:Für/gegen Gras hab ich [https://de.ezgardentips.com/effects-of-antifreeze-on-grass-8437 diesen] maschinenübersetzten Artikel gefunden. --[[user:Rotkaeppchen68|R<span style="color:red">ô</span>tkæppchen₆₈]] 16:17, 28. Sep. 2020 (CEST)
Um einen merklichen Effekt (Grfrierpunkterniedrigung) zu erzielen, wirst Du bummelig 5 % Ethylenglycol zugeben müssen, das sind genauso bummelig gute 15 kg. Abgesehen vom Preis ist das tatsächlich eine Umweltsauerei. Das Wasser wirst Du sowieso entsorgen müssen (entsorgen, nicht in den Kanal fließen lassen!) was weitere Kosten mit sich bringt. Das Wasser ist dann auch futsch. Was also spricht für die Billigvariante, das Wasser nach der Gartensaison in den Garten fließen zu lassen? --[[Benutzer:Elrond|Elrond]] ([[Benutzer Diskussion:Elrond|Diskussion]]) 16:34, 28. Sep. 2020 (CEST)


Meine Kletterrosen am Rosenbogen sind die ersten zwei Jahre normal gewachsen und geblüht. Plötzlich haben sie nur mehr kleine weiße Blüten bekommen und nach dem Abblühen rote so ovale Beeren bekommen. Was muss ich tun das sie wieder blühen wie am Anfang ? <small>(''nicht [[Hilfe:Signatur|signierter]] Beitrag von'' [[Spezial:Beiträge/62.93.107.245|62.93.107.245]] ([[Benutzer Diskussion:62.93.107.245|Diskussion]])<nowiki/> 19:23, 27. Feb. 2017 (CET))</small>
:(Ethylen-)Glykolvergiftungen sind im Winter gerne bei allen Haustieren genommene Tierarzttermine. Ett schmeckt so jut wie der [[Glykolwein-Skandal|beste Wein aus Österreich]] und wird ratzeputz weggeleckt. Und schon braucht man neue Katzen, Hunde, Vögel. Н[[Benutzer:Syrcro/Automatik|а]]ктаффэ 16:37, 28. Sep. 2020 (CEST)
::Trinken Katzen echt Glykol-Wasser ? Ich dachte die haben einen sehr feinen Gescmackssinn.... --[[Spezial:Beiträge/87.135.10.144|87.135.10.144]] 19:52, 28. Sep. 2020 (CEST)
:::Marder und Ratten fressen auch gerne Elektrokabel an, weil die im Isoliermaterial (weichgemachtes PVC) Substanzen drin sind (besagte Weichmacher), die auch süß schmecken. --[[Benutzer:Elrond|Elrond]] ([[Benutzer Diskussion:Elrond|Diskussion]]) 20:13, 28. Sep. 2020 (CEST)


:Die roten Beeren sind wohl [[Hagebutte]]n. Wegen der Blüten - Hast du die Rosen beschnitten? ev. sogar sehr stark? Dann kann es passiert sein, dass Du die veredelte Rose abgeschnitten hast und dann die Unterlage ausgetrieben und geblüht hat. --[[Benutzer:Elrond|Elrond]] ([[Benutzer Diskussion:Elrond|Diskussion]]) 19:43, 27. Feb. 2017 (CET)
Bei uns hat man früher in eingegrabene Blechregentonnen einfach einen oder mehrere Holzpfähle reingestellt. Wenn's bei dir nicht so kalt wird, sollte das reichen. Und wenn doch mal eine Plastiktonne kaputtfriert, kannst du sie als sortenreinen hochwertigen Kunststoff zum Recyclinghof bringen. Vermutlich macht's auch beim Glycol [[Paracelsus#Über_die_Ursachen_der_Krankheiten|die Dosis]]: Wenn du's reintust und dein Fass trotzdem kaputt geht, war's zu wenig. Wenn deine Pflanzen dahinscheiden und die Kinder im Nachbargarten seltsam euphorisch werden, war's zu viel.--[[Benutzer:ペーター|Katakana-Peter]] ([[Benutzer Diskussion:ペーター|Diskussion]]) 16:39, 28. Sep. 2020 (CEST)
:Die Frage ist: Warum willst du sie nicht ausleeren?
:Im Winter brauchst du sowieso kein Giesswasser und im Frühjahr füllt sie sich bei einigen Regenfällen schnell wieder auf.
:Ich entleere meine (300 L, rechteckig) nach den ersten 4 aufeinanderfolgenden Tagen Frost (reingucken) kostenfrei per Schlauch. [[Benutzer:Odore di spirito adolescenziale|Odore di spirito adolescenziale]] ([[Benutzer Diskussion:Odore di spirito adolescenziale|Diskussion]]) 17:09, 28. Sep. 2020 (CEST)
::''und im Frühjahr füllt sie sich bei einigen Regenfällen schnell wieder auf'' – ja damals, als es im Frühjahr noch regnete, aber daran können sich ja nur die älteren Semester hier noch schwammig erinnern :D --[[Benutzer:Kreuzschnabel|Kreuz]][[Benutzer Diskussion:Kreuzschnabel|schnabel]] 17:34, 28. Sep. 2020 (CEST)
::::<small>Dieses Jahr hat's allerdings im Februar noch ordentlich geregnet und länger dauert der "Winter" sowieso nicht mehr. --[[Benutzer:J budissin|j.budissin]]<sup>[[Benutzer Diskussion:J budissin|+/-]]</sup> 00:27, 29. Sep. 2020 (CEST)</small>
:::Egal was [[Benutzerin:Mirkur]] als Frostschutzmittel nimmt: Im Frühjahr muss das überwinterte Wasser dann mit frischem Regenwasser verdünnt werden, damit das Frostschutzmittel den Pflanze nicht mehr schadet. [[Benutzerin:Mirkur]] könnte einen leicht wasserlöslichen Mineraldünger nehmen, z.B. Harnstoff, Pottasche o.ä. Da weiß man, dass sie der Pflanze nicht schaden, wenn man es mit der Konzentration nicht übertreibt. Aus molarer Masse des Frostschutzmittels (bei Salzen durch die Zahl der Ionen teilen), der [[Kryoskopische Konstante|kryoskopischen Konstante]] von Wasser −1,853 K·kg/mol und der gewünschten Frostbeständigkeit lässt sich dann die Massenkonzentration des Frostschutzmittels berechnen. Um beispielsweise mit Harnstoff einen Frostschutz bis –10 °C zu erreichen, braucht man 324 Gramm Harnstoff pro Kilogramm Wasser. Bei einem 330-Liter-Tank sind das ca. 107 kg Harnstoff. Bei MEG (Autofrostschutz) wären es bei Frostbeständigkeit bis −10 °C ca. 76,5 Liter Monoethylenglykol auf 253,5 Liter Wasser. --[[user:Rotkaeppchen68|R<span style="color:red">ô</span>tkæppchen₆₈]] 18:13, 28. Sep. 2020 (CEST)
::::Die Kryostatische Konstante gilt, wie ideale Gase nur bei niedrigem Druck, nur in sehr verdünnten Lösungen, so einfach und schnell mal was ausrechnen geht nicht. Zumal die kryoskopischen Konstante und der Schmelzpunkt der anderen Substanz dann auch eine Rolle spielen würde. Auf die Schnelle habe ich nur was dänisches gefunden, aber die Zahlen sind ja verständlich https://lcglad.dk/glycol/ Trotzdem ist das eine Schnapsidee. --[[Benutzer:Elrond|Elrond]] ([[Benutzer Diskussion:Elrond|Diskussion]]) 19:13, 28. Sep. 2020 (CEST)
:::::107 kg Harnstoff wären ca. 25 kg reiner Stickstoff, was für mindestens 9 Ar Garten reicht. Für einen durchschnittlichen Hausgarten sollte das mehrere Jahre reichen. Damit hätte man das Stickstoffdüngeproblem zulasten des Bewässerungsproblems gelöst. --[[user:Rotkaeppchen68|R<span style="color:red">ô</span>tkæppchen₆₈]] 22:21, 28. Sep. 2020 (CEST)
Ich danke allen für die schönen Ideen und heiteren Berechnungen {{smilie}} und werde wohl das mit den Holzpfählen probieren bzw. bei größerer Kälte dann doch wie gewohnt das Wasser ablassen. Aber so weiß ich jetzt wenigstens, was eine kryoskopische Konstante ist --[[Benutzerin:Mirkur|Mirkur]] ([[Benutzerin Diskussion:Mirkur|Diskussion]]) 23:13, 28. Sep. 2020 (CEST)
:Vielleicht könntest Du die Zisterne einfach tieferlegen, siehe [[Wikipedia:Auskunft/Archiv/2020/Woche 36#Gartenteich 80 cm tief]]. Dazu eine formschöne Schwengelpumpe und das ganze sieht auch noch nostalgisch-dekorativ aus. --[[user:Rotkaeppchen68|R<span style="color:red">ô</span>tkæppchen₆₈]] 00:53, 29. Sep. 2020 (CEST)


: Frage: nur kleinere Blüten aber sonst wie vorher gestaltet, oder aber einfache, ungefüllte wie bei Wildrosen? Ggf hast Du beim Schnitt die "gesunden, wüchsigen Triebe" = Wildtriebe belassen und weniger attraktiven Triebe der Veredelung rausgeschnitten ? - [[Benutzer:Andy king50|andy_king50]] ([[Benutzer Diskussion:Andy king50|Diskussion]]) 19:51, 27. Feb. 2017 (CET)
: Es ist tatsächlich am sinnvollsten, das Kaputtfrieren zu verhindern - ob diese Gefahr überhaupt besteht und ob Holzpfähle ein zuverlässiger Schutz dagegen sind, könnte man auch hinterfragen. Zunächst einmal: Warum gehen Behälter beim Einfrieren eigentlich kaputt? Es liegt am Mechanismus des Gefrierens von Wasser und ganz wesentlich an der Behälterform: Wassereis hat ein deutlich höheres Volumen als das flüssige Wasser, aus dem es entstanden ist. Zweitens besitzt Wasser eine Dichteanomalie: es hat bei ca. 4° C seine höchste Dichte. Das führt dazu, daß ein Wasserbehälter, der vorwiegend von oben gekühlt wird, etwa, weil die Wasseroberfläche dort guten thermischen Kontakt zur kalten Luft hat, an der Oberfläche eine feste Eisschicht ausbildet, deren Dicke mit fortschreitendem Einfrieren langsam wächst und die sich dabei nach unten ausdehnt. Für sich genommen wäre das noch kein Problem, wird es aber dann, wenn sich der Behälter nach oben hin verengt. Dann versucht das Wasser unter dem Eis beim weiteren Einfrieren nämlich, sich auszudehnen, wird daran aber von dem Eispfropf in der Verengung gehindert, baut Druck auf und kann den Behälter sprengen. Das passiert z. B. nicht, wenn der Behälter konisch wie ein auf der Spitze stehender Kegel ist: Das sich ausdehnende Wasser hebt dann einfach die darüberliegende Eisschicht an. Es würde als Schutz im Prinzip also schon ausreichen, ein relativ dickwandiges Rohr senkrecht in die Mitte der Tonne zu stellen. Das friert dann oben zwar auch zu, aber der Propf ist nicht formschlüssig blockiert, sondern kann vom Wasser nach oben herausgeschoben werden. Wahrscheinlich wird das Wasser im Rohr zunächst auch nicht gefrieren, wenn es sich um ein Rohr aus einem relativ gut wärmeleitenden Metall handelt, weil es mit der Wärme des flüssigen Wassers unten in der Tonne das Wasser oben beheizt und langsamer gefrieren läßt als außen herum. Die Holzpfähle funktionieren vermutlich so, daß Wasser unten aus der Tonne durch die Kapillaren des Holzes nach oben entweichen kann und sich deswegen im Wasser kein Druck aufbaut, aber wie zuverlässig das ist, weiß ich nicht. Noch eine Methode wäre, die Tonne mit einer relativ gut wärmeisolierenden Kappe abzudecken. Dann erfolgt die Eisbildung sehr viel langsamer, falls überhaupt, und vorwiegend unten in der Tonne an der Wand, und die Eiskristalle haften dort an oder schwimmen auf und bilden jedenfalls keine feste Deckschicht, das Ganze bleibt vielmehr ein Wasser-Eis-Gemisch, das sich nur langsam verfestigt und dabei auch keinen Druck aufbaut. --[[Spezial:Beiträge/95.112.134.2|95.112.134.2]] 06:16, 29. Sep. 2020 (CEST)


Das wäre ''eigentlich'' dann ganz gut, denn diese Form von Rosen wäre "natürlicher". Oder? --[[Benutzer:ObersterGenosse|ObersterGenosse]] ([[Benutzer Diskussion:ObersterGenosse|Diskussion]]) 22:27, 27. Feb. 2017 (CET)
::Den ganzen Chemiewahnsinn kann und sollte man wohl definitiv abhaken.
:Falsch beschnitten, die Rose ist verwildert. --[[Benutzer:Ralf Roletschek|M@rcela]] [[Bild:Miniauge2.gif|27px]]05:52, 28. Feb. 2017 (CET)
::Die Rohrtheorie des Vorredners schätze ich als untauglich ein.
::Was theoretisch funktionieren müsste: eine dichte Folie ~als Tasche eintauchen, durch füllen mit Sand,etc unten hinreichend beschwert und darüber mit etwas Schaumstoff gefüllt, evt geht auch Styropor, Knüllpapier ... Ohne dichte Folie könnte vielleicht auch Styropor allein gehen.
::Sollte unabhängig vom og. Propfeneffekt helfen. Bei Nichtfunktionieren Geld zurück.
::Ansonsten sollte auch tägliches Kontrollieren und ggf. Intervenieren helfen, weil der Gefriervorgang bestimmt länger als eine Nacht dauert, zumindest wenn das Fass an einer Wand steht. --[[File:Lemon with white background.jpg|15px|Quetsch mich aus, ... ]] [[Benutzer:Itu|itu]] ([[Benutzer Diskussion:Itu|Disk]]) 19:07, 29. Sep. 2020 (CEST)
:::Unser Pool (aus Blech und zur Hälfte eingegraben, daher durfte er nicht komplett geleert werden, weil er sonst durch den Druck von Außen eingebrochen wäre) wurde immer mit Styroporplatten abgedeckt. Dadurch entstand nie eine vollflächige Eisschicht, sodass genug Spielraum für das abkühlende Wasser vorhanden war. (PS: @itu: "Bei Nichtfunktionieren Geld zurück" - ist das nur für den Originalposter gültig oder kann sich da jeder anschließen?) --[[Benutzer Diskussion:TheRunnerUp|TheRunnerUp]] 23:06, 29. Sep. 2020 (CEST)
Ich wollte nicht versäumen, mich nochmals bei allen zu bedanken. Die physikalisch-mechanischen Ideen, Gedanken und Theorien überzeugen mich sehr und ich habe mich zu einer an meine Bedingungen hier angepasste Mischung entschlossen (ohne zu graben ;-) ) und werde jedenfalls auf alle Chemie verzichten!! --[[Benutzerin:Mirkur|Mirkur]] ([[Benutzerin Diskussion:Mirkur|Diskussion]]) 19:48, 30. Sep. 2020 (CEST)


:Die Frage ist doch: "Was muss ich tun das sie wieder blühen wie am Anfang ?". Das ist einfach: Finde alle Triebe die noch so blühen wie gewünscht und schneide alle anderen weg. Falls keine mehr zu finden sind: Neue Rose kaufen (oder sich an die aktuelle Variante gewöhnen). Ggf. noch eine Anleitung zum Schnitt von Kletterrosen besorgen und verstehen. Grüße --[[Benutzer:RalfDA|RalfDA]] ([[Benutzer Diskussion:RalfDA|Diskussion]]) 18:52, 28. Feb. 2017 (CET)
== CO2-Ampel - Kaufberatung ==
::<u>Sehr stark vereinfacht</u>: Triebe, die aus dem Boden kommen, sind wild. Sie haben meist 7 Blätter, die Triebe der Zuchtrose 5 Blätter. Allerdings ist das nicht dogmatisch, es gibt seit einigen Jahrzehnten auch Sorten mit 7 oder mehr Blättern. Wilde Triebe wachsen sehr schnell, sind oft gerade und damit sehr lang. --[[Benutzer:Ralf Roletschek|M@rcela]] [[Bild:Miniauge2.gif|27px]] 18:58, 28. Feb. 2017 (CET)


Edelrosen werden meist auf [[Hundsrosen]] [[Okulation|okuliert]]. Sehen die Blüten so oder so ähnlich aus? Oder [https://www.google.de/search?hl=de&site=imghp&tbm=isch&source=hp&biw=1600&bih=793&q=geiltriebe+rosen&oq=geiltriebe+rosen&gs_l=img.3...2871.13085.0.13539.16.10.0.6.6.0.90.834.10.10.0....0...1ac.1.64.img..0.11.831...0j0i10i24k1j0i24k1.4TUwztCGPu4#hl=de&tbm=isch&q=hundsrose&* so]? --[[Benutzer:Elrond|Elrond]] ([[Benutzer Diskussion:Elrond|Diskussion]]) 19:42, 28. Feb. 2017 (CET)
Hallo, für die kühlere Jahreszeit heißt es, solle man wegen Corona regelmäßig in Innenräumen lüften. Da ist immer die Rede von einer [[CO2-Ampel]]. Wenn ich mir jetzt so etwas gerne besorgen möchte, auf was habe ich da zu achten, so das es auch wirklich funktioniert? Gibt es da seriöse Hersteller die man empfehlen kann? Und wenn es dann immer heißt, dass diese nicht teuer sind, was ist denn ''nicht teuer'' für ein gutes Gerät? Gibt es auch gute, günstige Geräte nicht nur als Ampel sondern auch mit der Anzeige eines Wertes (und wie genau sind diese Werte in der Regel?)? --[[Benutzer:Paonka|Paonka]] ([[Benutzer Diskussion:Paonka|Diskussion]]) 16:50, 28. Sep. 2020 (CEST)
:Da gibt es auch Fake-Geräte wie das Technoline WL 1000, das CO<sub>2</sub>-„Äquivalente“ anzeigt, aber gar keinen echten CO<sub>2</sub>-Sensor hat. Richtige CO<sub>2</sub>-Messgeräte kosten mindestens das dreifache derartiger Fake-Geräte. --[[user:Rotkaeppchen68|R<span style="color:red">ô</span>tkæppchen₆₈]] 18:53, 28. Sep. 2020 (CEST)
:So als Anhaltspunkt, da ich mich gerade mit dem Selbstbau beschäftige: Die Einzelteile kosten so um die 100€ (siehe [https://www.umwelt-campus.de/forschung/projekte/iot-werkstatt/ideen-zur-corona-krise-1 hier]), davon der Sensor alleine etwa 50€. In der Region würde ich also schauen. --[[Benutzer:FGodard|FGodard]]|[[Benutzer_Diskussion:FGodard|✉]]|[[Benutzer:FGodard/Bewertung|±]] 23:31, 28. Sep. 2020 (CEST)
::Erste Hinweise liefert unser Artikel [[Luftgütesensor]]. --[[Spezial:Beiträge/2003:CB:6708:BE01:95F1:F850:C13C:A04A|2003:CB:6708:BE01:95F1:F850:C13C:A04A]] 14:58, 29. Sep. 2020 (CEST)


== Ist die Teilnahme an [[Russisch Roulette]] legal? ==
== Monitor muckt ==


Frage bezieht sich auf Deutschland, aber Antorten für andere Länder sind auch interessant. Zur Sicherheit: Nein, ich habe nichts dergleichen vor :) --[[Spezial:Beiträge/89.246.169.72|89.246.169.72]] 20:03, 27. Feb. 2017 (CET)
Ich habe einen betagten Mac mit DVI-Anschluss für den Monitor. Und ich habe einen neuen Monitor mit HDMI. Den habe ich mit Adapter an den Mac angeschlossen, doch der Bildschirm bleibt schwarz (Mac hat die Startmelodie gespielt). Woran kanns liegen, was kann ich tun?
Verschärfend kommt hinzu, dass jetzt auch der alte Monitor kein Signal mehr bekommt. Au weia! [[Benutzer:Rainer Zenz|Rainer Z]] [[Benutzer Diskussion:Rainer Zenz| ...]] 16:50, 28. Sep. 2020 (CEST)


{{Rechtsauskunft}}
:Gibt denn der betagte Mac (davon gehe aber aus) auch ein digitales Signal über DVI aus? Und ist das Kabel voll beschaltet, also vor allem auch der digitale Teil? Kann man beim Mac einstellen, was ausgegeben werden soll? --[[Benutzer:Tsungam|Magnus]] [[BD:Tsungam|(Diskussion)]] 16:54, 28. Sep. 2020 (CEST)
:Hast Du mal eine genaue Bezeichnung für den alten Monitor? Es kann nämlich sein, dass das alte Teil „nur“ DVI-A konnte. Ich hab keinen Mac, aber dafür drei Bildschirme mit DVI-Anschluss und das sind alles drei Samsung-Monitore mit DVI-D und HDCP. Da ist ein Adapter HDMI→DVI-D kein Problem. --[[user:Rotkaeppchen68|R<span style="color:red">ô</span>tkæppchen₆₈]] 17:02, 28. Sep. 2020 (CEST) Die andere Möglichkeit ist, dass der Mac kein HDCP kann oder es abgeschaltet ist, der neue Bildschirm aber zwingend HDCP verlangt. Für DVI-D ist HDCP optional, für HDMI ist HDCP Bestandteil der Spezifikation, ansonsten sind HDMI 1.4 und DVI-D elektrisch identisch. --[[user:Rotkaeppchen68|R<span style="color:red">ô</span>tkæppchen₆₈]] 17:16, 28. Sep. 2020 (CEST)


:Eigentlich eine interessante Frage. Nur: Falls nicht legal. Mit welcher Strafandrohung willst Du dies sanktionieren, bei jemand, der bereit ist, dieses Spiel zu spielen? :-) Du liest: Die Frage fällt in sich selbst zusammen. --[[Spezial:Beiträge/80.187.118.11|80.187.118.11]] 20:45, 27. Feb. 2017 (CET)
::Hm. Es ist ein Mac aus der "Käsereiben"-Generation, davon Mittelklasse. Mehr kann ich gerade nicht sagen, es geht jetzt ja gar kein Monitor mehr. [[Benutzer:Rainer Zenz|Rainer Z]] [[Benutzer Diskussion:Rainer Zenz| ...]] 17:23, 28. Sep. 2020 (CEST)
:::Das hilft schon mal ein bisschen weiter. Kräftige Googelei ergibt, dass es sich um einen Power Mac G5 handelt, der mit einer Nvidia Geforce 7800 GT oder schlechter ausgeliefert wurde. Diese Graka kann DVI-A und DVI-D, aber kein HDCP. Wenn der neue Monitor eine VGA-Buchse hat, wäre das die letzte Rettung mit einem Adapter von DVI-A auf VGA, der bei vielen Grakas damals zum Lieferumfang gehört hat, so auch beim Power Mac G5. Vielleicht schaffst Du es ja auch irgendwie, beim neuen Monitor HDCP zu deaktivieren. --[[user:Rotkaeppchen68|R<span style="color:red">ô</span>tkæppchen₆₈]] 18:36, 28. Sep. 2020 (CEST)
::::Ergänzung: Einen solchen Adapter gips für [https://www.reichelt.de/dvi-adapter-vga-buchse-an-dvi-stecker-gc-vga-dvi-p202006.html?&trstct=pos_5&nbc=1 1,56 bei Angelika] oder wahlweise fürs [https://www.mediamarkt.de/de/product/_hama-dvi-stecker-vga-kupplung-3853838.html 5½-fache beim Blödmarkt]. --[[Benutzer:Kreuzschnabel|Kreuz]][[Benutzer Diskussion:Kreuzschnabel|schnabel]] 22:04, 28. Sep. 2020 (CEST)


::Regelt das [[Waffengesetz (Deutschland)]] Kurz: Nur mit Wasserpistole oder auf dem Schießstand. Stichwort "Führen einer Waffe" [[Benutzer:Graf Umarov|Graf Umarov]] ([[Benutzer Diskussion:Graf Umarov|Diskussion]]) 20:59, 27. Feb. 2017 (CET)
===Anschlussfrage: Warum bleibt auch der alte Monitor schwarz?===
Der Rechner startet und spielt die Startmelodie. Die Diode des Monitors leuchtet und er blendet ein Fenster "Kein Signal" ein. Aber danach bleibt es jetzt duster. Wie kreise ich das Problem ein? Der Wechsel zwischen den beiden DVI-Anschlüssen des Rechners hat auch nix gebracht. Was kann da kaputt sein? [[Benutzer:Rainer Zenz|Rainer Z]] [[Benutzer Diskussion:Rainer Zenz| ...]] 13:55, 30. Sep. 2020 (CEST)
:Da kann dann auch die Grafikkarte oder das Kabel hinüber sein. Versuche mal, die Bildschirme mit einer anderen DVI-D-Quelle oder HDMI mit deaktiviertem HDCP zur Funktion zu bringen. Bei neueren Nvidia-Grafikkarten überprüft der Treiber, ob der Bildschirm HDCP kann. Wenn nicht, dann könnern bestimmte Inhalte nicht angezeigt werden. Du könntest also auch einen Windows- oder Linux-PC mit DVI-D-Ausgang zum Testen nehmen. Bei manchen Digitalfernsehboxen lässt sich HDCP ebenfalls deaktivieren. Dann können nicht mehr alle Programme geschaut werden, aber die Box läuft dann auch mit älteren PC-Bildschirmen. --[[user:Rotkaeppchen68|R<span style="color:red">ô</span>tkæppchen₆₈]] 01:20, 1. Okt. 2020 (CEST)
::Kabel wäre natürlich ein harmloses Problem. Grafikkarte würde mich wundern. Es geht ja erst seit dem Umstöpseln nix mehr, das wäre ein großer Zufall. [[Benutzer:Rainer Zenz|Rainer Z]] [[Benutzer Diskussion:Rainer Zenz| ...]] 14:21, 1. Okt. 2020 (CEST)
:::Das könnte ein Konfigurationsproblem sein. Von Mac-Konfiguration hab ich leider keine Ahnung. Bei PCs hilft mitunter Festplatte und Stomversorgung abstöpseln, Knopfzelle entfernen und dann die beiden Kontakte für den CMOS-Reset einen Moment kurzschließen. --[[user:Rotkaeppchen68|R<span style="color:red">ô</span>tkæppchen₆₈]] 14:36, 1. Okt. 2020 (CEST)
::::Aber ich habe doch gar nichts an der Konfiguration geändert. Nur einen anderen Monitor kurz angeschlossen und wieder zurück. [[Benutzer:Rainer Zenz|Rainer Z]] [[Benutzer Diskussion:Rainer Zenz| ...]] 12:20, 2. Okt. 2020 (CEST)


:::Hm. Es war nicht danach gefragt, ob und wo die Waffe rechtmäßig geführt wird. Das ist also Unsinn. Mal sehn: Gegen Tote wird ja ohnehin nicht ermittelt. Es geht also nur um den oder die Gewinner und/oder um die mögliche Strafbarkeit des Versuchs. Und ob da irgendwo ein Delikt vorliegt. Offenbar ist Russisches Roulette – bei allen hier angebrachten Vorbehalten (keine Rechtsberatung!) – nicht strafbar. Ich lese in: [https://books.google.de/books?id=od9tDAAAQBAJ&pg=PP183&lpg=PP183&dq=russisch+roulette+strafbar&source=bl&ots=3906oQB5OU&sig=xt2qM4Naf9LufvNXJ7U_Ingrn6U&hl=de&sa=X&ved=0ahUKEwiYwOHZ_rDSAhXBBSwKHdG6CwQ4ChDoAQhHMAg#v=onepage&q=russisch%20roulette%20strafbar&f=false Robert Esser, Volker Krey: ''Deutsches Strafrecht, Allgemeiner Teil'', Kohlhammer Verlag, Stuttgart, 6, Aufl. 2016], ISBN 978317029882 unter: ''„§ 11 Objektiver Tatbestand, IV. Lehre von der objektiven Zurechnung, 4. Handeln des Opfers auf eigene Gefahr “ als Spezialfall des Ausschlusses der objektiven Zurechnung, a) Teilnahme an einer freiverantwortlichen Selbstgefährdung des Opfers, (2) Straflosigkeit der Beteiligung an der freiverantwortlichen Selbstgefährdung eines anderen, Fall 56 – Russisches Roulette Nr.1 –“'' ein Zitat aus einem BGH-Urteil, das ich auf die Schnelle nicht recherchieren kann (und vielleicht angesichts der seriösen Quelle auch nicht muss): ''„Eigenverantwortlich gewollte und verwirklichte Selbstgefährdung unterfallen nicht dem Tatbestand eines Körperverletzungs- oder Tötungsdelikts, wenn das mit der Gefährdung bewusst eingegangene Risiko sich realisiert. Wer lediglich eine solche Selbstgefährdung veranlasst, ermöglicht oder fördert, macht sich nicht wegen eines Körperverletzungs- oder Tötungsdelikts strafbar.“'' --[[Spezial:Beiträge/80.130.225.92|80.130.225.92]] 21:22, 27. Feb. 2017 (CET)
== Suche nach einem krassen Lösungsmittel ==


: in den U.S.A wurde 1946 jemand verurteilt, weil er bei einer Variante („Russisch Poker“) einen Freund tötete: [[:en:Russian roulette]]... --[[user:Homer Landskirty|Heimschützenzentrum]] ([[user talk:Homer Landskirty|?]]) 21:19, 27. Feb. 2017 (CET)
Ich möchte ein Auto umstreichen in Richtung Steampunk, wenn möglich rostig oder so. Aus Gründen des Aufwandes möchte ich nicht unbedingt den Lack abschleifen um die Farbe haftend zu bekomme. Welche Farbe auf Basis welches Lösungsmittels äzt den Lack an und vermittelt eine Haftung auf Autolack? (Aussehen egal) [[Spezial:Beiträge/188.103.248.114|188.103.248.114]] 21:21, 28. Sep. 2020 (CEST)
:Probier mal lösemittelbasierten [[Abbeizer]]. --[[user:Rotkaeppchen68|R<span style="color:red">ô</span>tkæppchen₆₈]] 21:44, 28. Sep. 2020 (CEST)
::Das gibts für Autos in Form von Haftvermittlern, die nach Abschleifen der oberen Schicht aufgetragen wird. Das möchte ich vermeiden. Ich will einfach nur streichen, auch aus Kostengründen und weil das Aussehen egal ist. [[Spezial:Beiträge/188.103.248.114|188.103.248.114]] 22:44, 28. Sep. 2020 (CEST)


Also entweder bist Du nach der Teilnahme tot oder du wirst so in die Mangel genommen, dass Du über aller juristischen Theoriefuxereien und (zumindest bei Nichtjuristen) allfälliger moralischer Probleme hinaus sowas sicher sowas nie wieder machst. Die Nachweislast, dass es sich nicht um einen Mord handelt, liegt dann wohl allein bei Dir. Viel Spaß beim juristisch wasserdichten Beweis. 22:16, 27. Feb. 2017 (CET) <small>(''ohne Name [[Hilfe:Signatur|signierter]] Beitrag von'' [[Benutzer:Andy king50|Andy king50]] ([[Benutzer Diskussion:Andy king50|Diskussion]]&nbsp;&#124;&nbsp;[[Spezial:Beiträge/Andy king50|Beiträge]]))</small>
Mit den üblichen Lösemitteln wirst Du wahrscheinlich wenig bis nichts reißen können, da stehen moderne Lacke drüber. Solche, die ev. was bewirken könnten, sind für Laien aus guten Gründen nicht so ohne weiteres zu kriegen. Gründe sind zum einen toxikologische und zum anderen ökologische. Zudem würden bei nicht sachgemäßer Anwendung sicher auch andere Sachen neben dem Lack geschädigt werden. --[[Benutzer:Elrond|Elrond]] ([[Benutzer Diskussion:Elrond|Diskussion]]) 23:14, 28. Sep. 2020 (CEST)
::::Hast Du vielleicht einen Namen von diesem umweltschädlichen Zeug? --[[Spezial:Beiträge/188.103.248.114|188.103.248.114]] 18:49, 30. Sep. 2020 (CEST)
:::Da gibt es wunderbare Folien in allen Farben, die man auf den unbehandelten Lack kleben kann. Je glatter je besser.--[[Spezial:Beiträge/2003:E8:3703:E000:7461:D06B:B123:3F60|2003:E8:3703:E000:7461:D06B:B123:3F60]] 00:27, 29. Sep. 2020 (CEST) Ansonsten wurde ich mit Aceton oder Nitro den Lack vor den Streichen von Fett reinigen. <small>(''nicht [[Hilfe:Signatur|signierter]] Beitrag von'' [[Spezial:Beiträge/2003:E8:3703:E000:7461:D06B:B123:3F60|2003:E8:3703:E000:7461:D06B:B123:3F60]] ([[Benutzer Diskussion:2003:E8:3703:E000:7461:D06B:B123:3F60|Diskussion]]) 00:31, 29. Sep. 2020 (CEST))</small>


Aus moralischen oder ethischen Gründen kann viel passieren. Aber der Staat muss euch erstmal beweisen, dass du ein Mörder bist! Siehe [[Unschuldsvermutung]] oder [[in dubio pro reo]]. Auch [[Eigenverantwortliche Selbstgefährdung]] (auf die im Rahmen eines AT-Lehrbuchs oben schon verwiesen wurde), aber auch [[Disponibilität]] oder [[Rechtsgut]] könnten, ebenso wie [[Tötung auf Verlangen]] interessant sein! --[[Benutzer:ObersterGenosse|ObersterGenosse]] ([[Benutzer Diskussion:ObersterGenosse|Diskussion]]) 22:30, 27. Feb. 2017 (CET)
:Vielleicht den Lack mit einem Sandstrahler aufrauen und dann überstreichen? --[[Spezial:Beiträge/94.219.186.65|94.219.186.65]] 01:15, 30. Sep. 2020 (CEST)
: dann wäre es besser, es wären immer alle Kammern geladen, und das Gesox würde uns damit solche theoretischen Eventualtitäten von vorn herein ersparen. - [[Benutzer:Andy king50|andy_king50]] ([[Benutzer Diskussion:Andy king50|Diskussion]]) 22:35, 27. Feb. 2017 (CET)


:@Andy king50, (Beitrag 22:16, 27. Feb. 2017): Das ist gequirlter Unsinn. [[Mord]] setzt Vorsatz und mindestens ein sogenanntes Mordmerkmal voraus. (§ 211 Abs. 2 StGB: ''„Mörder ist, wer aus Mordlust, zur Befriedigung des Geschlechtstriebs, aus Habgier oder sonst aus niedrigen Beweggründen, heimtückisch oder grausam oder mit gemeingefährlichen Mitteln oder um eine andere Straftat zu ermöglichen oder zu verdecken, einen Menschen tötet.“'') In der Regel wird man, wenn der Tatort nicht verändert wird, bei einem Opfer durch Russisches Roulette die freiverantwortlichen Selbstgefährdung problemlos nachweisen können. --[[Spezial:Beiträge/2003:46:A0C:6E00:E22A:82FF:FEA0:3113|2003:46:A0C:6E00:E22A:82FF:FEA0:3113]] 22:37, 27. Feb. 2017 (CET) Nachtrag zur der inkompetenten und irreführenden Bemerkung über die Nachweislast: Die [[Staatsanwaltschaft (Deutschland)]] hat [https://dejure.org/gesetze/StPO/160.html nach § 160 StPO] die ''„Pflicht zur Sachverhaltsaufklärung“''. Ich lese: ''„(2) Die Staatsanwaltschaft hat nicht nur die zur Belastung, sondern auch die zur Entlastung dienenden Umstände zu ermitteln und für die Erhebung der Beweise Sorge zu tragen, deren Verlust zu besorgen ist.“'' Bei einem Toten durch Gewalteinwirkung sind Spurensicherung, KTU und Obduktion obligatorisch. --[[Spezial:Beiträge/2003:46:A0C:6E00:E22A:82FF:FEA0:3113|2003:46:A0C:6E00:E22A:82FF:FEA0:3113]] 22:47, 27. Feb. 2017 (CET) (hier auch als 80.130.225.92 unterwegs)
= 29. September =
::: ich finde diese ständige lästige Beanspruchung der Auskunft durch virtuelle juristische Fragen stellende IP (offensichtlich immer die gleiche Person) definitiv zum Kotzen: Alle diese IP exekutieren = sperren. 23:11, 27. Feb. 2017 (CET) <small>(''ohne Name [[Hilfe:Signatur|signierter]] Beitrag von'' [[Benutzer:Andy king50|Andy king50]] ([[Benutzer Diskussion:Andy king50|Diskussion]]&nbsp;&#124;&nbsp;[[Spezial:Beiträge/Andy king50|Beiträge]]))</small>
::::Ich hab hier noch nie so eine Frage gestellt, reg dich mal ab. Danke für alle Antworten. --[[Spezial:Beiträge/89.246.169.72|89.246.169.72]] 23:28, 27. Feb. 2017 (CET)
::::Du namenloses Datum mit Uhrzeit, diese Frage ist immer wieder interessant und leicht zu beanworten: [[Selbstgefährdendes Verhalten]] inklusive [[Selbstverletzendes Verhalten]] ist nicht strafbar. --[[user:Rotkaeppchen68|R<span style="color:red">ô</span>tkæppchen₆₈]] 23:34, 27. Feb. 2017 (CET)
::::: Lasst einfach mal die Waffen weg und macht Bunjee-Jumping mit uralten Seilen. Solange jeder Springer sich das Seil nimmt, dass er erwürfelt... [[Benutzer:Yotwen|Yotwen]] ([[Benutzer Diskussion:Yotwen|Diskussion]]) 07:34, 28. Feb. 2017 (CET)
:Eine Verurteilung wegen Mord ist unwahrscheinlich (Anklage nicht!), eine Verurteilung [[Totschlag (Deutschland)|Totschlag]] kann aber durchaus drin sein. Setz aber eben voraus, dass ein Vorsatz nachgewiesen werden kann. Aber eben im Bereich der Selbsttötung gibt es noch einige andere Punkte die zu Verurteilung führen können. Einmal im Bereich des Waffenrechts, gerade wenn es nicht der Revolver des Verlierers war (Ohne Revolver, ist kein Russisch Roulette möglich). Dann eben im Bereich der Unterlassung und Anstiftung, gibt es durchaus Möglichkeiten die der Staatsanwalt ausprobieren könnte. Gerade wenn der Verlieren betrunken (oder sonst wie benebelt) war, andere Teilnehmer aber nicht. Wenn die Gefahr also für den Verliere nicht mehr klar ersichtlich war, für die umstehenden schon. Das kann eben durchaus so ausgelegt werden, dass man bewusst diese Situation herbeigeführt hat. Da der Verlieren nur in dem Zustand überhaupt beim russisch Roulette mit gemacht hat. Kurzum es würde darauf hinauslaufen zu beweisen, dass ein Vorsatz da war. Und dann -mit Vorsatz-, sind ziemlich deftige Strafmasse möglich. Wenn es nur auf ein Urteil wegen nur Unterlassung und Fahrlässigkeit, hinausläuft. Nun ja, dann könnte man Glück haben und mit einer bedingte Strafe davonkommen. Aber Ärger wird man so oder so kriegen, wenn man dabei war und nichts unternommen hat das russisch Roulette zu verhindern.--[[Benutzer:Bobo11|Bobo11]] ([[Benutzer Diskussion:Bobo11|Diskussion]]) 08:16, 28. Feb. 2017 (CET)
::<small>Wie meinen? Hast du die anderen Antworten gelesen? Und schreibst dennoch solchen Unsinn? Und wenn du schon abwandelst... Bring doch mal Aliens ins Spiel. Oder schwule Fußballer, die mal Akne hatten. Wäre sicher mal interessant zu erfahren, wie die Rechtslage dann ist... also nach deiner Einschätzung, die von der Realität ja allzu oft abweicht. <small>-- <span style="text-shadow:grey 0.1em -0.1em 0.1em;"> [[Benutzer:Ian Dury|Ian Dury]]<sup> [[Benutzer Diskussion:Ian Dury|Hit me]]</sup></span>&nbsp;</small> 09:58, 28. Feb. 2017 (CET) </small>
::: Keine Ahnung, wie das in der Schweiz so ist. Ich kann mir aber nicht vorstellen, dass es erheblich von der deutschen Regelung abweicht. Es ist eher liberaler als dort. Und in Deutschland...
:::: ''„Beihilfe“ zur Selbsttötung bedeutet die Selbsttötung mit Hilfe einer Person, die ein Mittel (meist ein Medikament) '''[oder Pistole]''' zur Selbsttötung bereitstellt. Eine Selbsttötung liegt aber nur dann vor, wenn der Suizident den letzten Schritt noch selbst beherrscht, also die sogenannte Tatherrschaft über das Geschehen hat. Sofern die andere Person die letzte todbringende Handlung vornimmt, ist kein Suizid mehr gegeben, sondern es liegt eine Tötungshandlung vor. Die Beihilfe zum Suizid, beispielsweise das Besorgen oder Bereitstellen tödlich wirkender Medikamente, ist in Deutschland mangels Vorliegens einer fremden, rechtswidrigen Haupttat nicht strafbar (Prinzip der limitierten Akzessorietät). Der Suizid richtet sich nicht gegen eine „andere“ Person und ist mithin kein Tötungsdelikt im Sinne der §§ 211 ff. StGB, sodass auch die Hilfe hierzu keine strafbare Tat darstellt. Dies bezieht sich jedoch auf die Beihilfe im juristischen Sinne und ist zu unterscheiden von einer aktiven Hilfestellung bei der Selbsttötung, die als Beteiligung nach Täterschaftsgrundsätzen strafbar sein kann.''
::: Viel klarer gehts nicht. [[Benutzer:Yotwen|Yotwen]] ([[Benutzer Diskussion:Yotwen|Diskussion]]) 12:18, 28. Feb. 2017 (CET)
Die Frage ist, ob die Teilnehmer tatsächlich noch eigenständige Entscheidungen treffen. Zumindest wenn der erste Teilnehmer seinen Versuch unbeschadet überstanden hat, dürfte eine Situation eintreten, die sich nicht mehr völlig rational voraussehen und daher auch nicht pauschal ex ante beurteilen lässt. Ganz abgesehen davon, dass solche Spiele nicht selten mit Erblindung, aber nicht mit dem Tod enden. --[[Benutzer_Diskussion:Zxmt|Zxmt]] <sup>[[Benutzer:Zxmt|Nutze Dein Stimmrecht!]]</sup> 13:03, 28. Feb. 2017 (CET)
:::: Es gibt halt Leute, die ihr Hirn deutlich grösser einschätzen, als es tatsächlich ist. Und wenn sie an solchen Happenings teilnehmen, dann wundert mich das nicht mal. [[Benutzer:Yotwen|Yotwen]] ([[Benutzer Diskussion:Yotwen|Diskussion]]) 13:40, 28. Feb. 2017 (CET)


:Nochmal deutlich: Es geht nicht um Selbsttötung. Die Teilnehmer an dem Spiel haben nicht das Ziel, sich auf diesem Weg das Leben zu nehmen. Sie spielen in der Hoffnung, zu gewinnen und zu überleben und die Teilnahme an dem Spiel ist Ausdruck einer freien Entscheidung. Es geht, wie oben bereits ausgeführt, um eine freiverantwortliche Selbstgefährdung und bei dem bzw. den Überlebenden um Teilnahme an einer freiverantwortlichen Selbstgefährdung des Opfers. Das oben verlinkte, bei Esser/Krey 2016 gefundene Zitat stammt aus einem Urteil des BGH. BGH ist hier nicht die Abkürzung für ''„bin gleich hier“'' sondern für [[Bundesgerichtshof]]. Der BGH entscheidet als einer der fünf obersten Gerichtshöfe der Bundesrepublik zwar letztinstanzlich, aber angesichts der geballten juristischen Kompetenz in diesem Thread wird man dort sicher nochmals in sich gehen und prüfen, ob man Russisches Roulette statt als freiverantwortliche Selbstgefährdung nicht doch lieber als [[Suizid]] und Ausdruck des Selbstbestimmungsrechts (straffrei) oder als Teilnahme an einem Suizid (ebenfalls straffrei) versteht. Es wäre ja nicht das erste Mal, dass auf der Wikipedia-Auskunft von anonymen Accounts einem Obersten Gericht kräftig der Marsch geblasen wurde, weil wir es einfach besser wissen. Die werden sich also noch ganz schön umgucken, wenn wir erstmal richtig loslegen... --[[Spezial:Beiträge/91.40.196.119|91.40.196.119]] 14:39, 28. Feb. 2017 (CET) (hier auch als 80.130.225.92 und 2003:46:A0C:6E00:E22A:82FF:FEA0:3113 unterwegs)
== Nebensatz ==
::Wie bereits richtig festgestellt, der BGH urteilt letztinstanzlich – aber eben mehrheitlich im Einzelfall und eher selten als Grundsatzentscheidung. Dementsprechend halte ich es für verwegen, Entscheidungen, die einen ganz anderen Sachverhalt betreffen einfach umzusetzen. Daher halte ich die Thesen über mir für zumindest nicht stichhaltig.
::Eine Teilnahme egal, ob an einem Suizid oder einer anderen Tat ist ausgeschlossen – hier hat der BGH mehrfach Klartext gesprochen: Da beide Beteiligten abwechselnd den Revolver betätigen, liegt auf jedem Fall für beide Tatherrschaft vor. Es kann also gar keinen Teilnehmer unter den Spielern geben (BGH 4 Str 328/08). Ein Suizid liegt ebenfalls nicht vor, da jeder Teilnehmer die Erwartung hegt, das Spiel zu gewinnen, also gerade nicht zu sterben. Aber auch eine eigenverantwortliche Selbstgefährdung dürfte vor dem BGH kaum Zuspruch ernten. Diese Einwilligung ist nämlich höchstrichterlich eingeschränkt: Sie „verliert ihre (insoweit) rechtfertigende Wirkung dort, wo die Grenze zur Sittenwidrigkeit überschritten ist, also bei konkreter Todesgefahr, unabhängig von der tatsächlich eingetretenen Rechtsgutverletzung“ (aus oben zitiertem Urteil, mit weiteren Nachweisen).
::Ohne mich zu weit aus dem Fenster lehnen zu wollen (wer hätte vor zwei Tagen daran zu denken gewagt, dass ein Gericht ein illegales Straßenrennen als Mord werten könnte), würde ich mich auf den Standpunkt stellen, dass die Rechtsprechung zu tödlich verlaufenen Straßenrennen hier anwendbar sein könnte, sich die überlebenden Teilnehmer damit zumindest der fahrlässigen Tötung schuldig machen, da sie als rechtstreue Bürger gar nicht erst ein Russisches Roulette hätten veranstalten oder daran hätten teilnehmen dürfen und sich somit nicht auf eigenverantwortliche Selbstgefährdung berufen dürfen. Auch derjenige, der den Revolver zur Verfügung stellt macht sich strafbar, die geht jedoch schon aus waffentechnischen Spezialgesetzen hervor, die einen Waffenträger einige besondere Sorgfaltspflichten aufbürden. Eine Verurteilung wegen Totschlags würde ich nur anhand der aktuellen („Berliner“) Rechtsprechung nicht vollständig ausschließen wollen – siehe aktuelle Berichterstattung, einen Mord könnte man unter dem Merkmal „niedriger Beweggrund“ an den Haaren herbeiziehen. Aber ich bevorzuge die „ältere“ Rechtsprechung hinsichtlich fahrlässiger Tötung.--[[Benutzer:OnlyMe|OnlyMe]] ([[Benutzer Diskussion:OnlyMe|Diskussion]]) 22:31, 28. Feb. 2017 (CET)


:::Ok, es wird mir jetzt langsam zu blöd und deshalb habe ich nochmal nach dem von Esser/Krey 2016 zitierten BGH-Urteil gesucht. Ich komme nicht ran, weil die Vorschau begrenzt ist und finde es leider auch nicht über Websuche. Aber so etwas landet ja nicht ohne Grund in einem Lehrbuch. Wer möchte kann sich auch in den Artikeln über [[Robert Esser]] und [[Volker Krey]] zu deren Vita informieren. Immerhin finde ich ein Urteil des 1. Strafsenates des Bundesgerichtshofs aus dem Jahr 1984: [https://www.jurion.de/urteile/bgh/1984-02-14/1-str-808_83 BGH, 14.02.1984 - 1 StR 808/83]. Ich lese dort: ''„Amtlicher Leitsatz: Eigenverantwortlich gewollte und verwirklichte Selbstgefährdungen unterfallen nicht dem Tatbestand eines Körperverletzungs- oder Tötungsdelikts, wenn das mit der Gefährdung bewußt eingegangene Risiko sich realisiert. Wer lediglich eine solche Selbstgefährdung veranlaßt, ermöglicht oder fördert, macht sich nicht wegen eines Körperverletzungs- oder Tötungsdelikts strafbar.“'' Der Fall (von Esser/Krey als ''„weiteres Beispiel“'' angeführt) behandelt eine erfolgreiche Sachbeschwerde gegen die Verurteilung wegen fahrlässiger Tötung. Ein Drogenkranker hatte zusammen mit seinem Freund Heroin gespritzt, der Freund war dadurch verstorben. Das verlinkte Urteil stellt sowohl die rechtliche Würdigung der Jugendkammer dar, die den Überlebenden verurteilte als auch die Erwägungen, die das BGH einen Erfolg der Beschwerde erkennen lassen (wegen einer Gesamtfreiheitsstrafe wird allerdings nicht freigesprochen sondern zurückverwiesen, damit eine neue Gesamtfreiheitsstrafe gebildet werden kann). Wen es interessiert mag die recht komplexen Überlegungen dort nachvollziehen. Sie sind grundlegender Natur und auf die Frage des Russischen Roulettes durchaus ebenfalls anwendbar. Es mag richtig sein, dass fallbezogen gedacht werden muss. Und dass der ''„Amtliche Leitsatz“'' dieses Urteils bei Esser/Krey 2016 auch auf einen Fall von Russisch Roulette mit Todesfolge angewandt wird liegt dann wohl sicherlich an der evidenten Inkompetenz der beiden Autoren. Zum Thema Fallbezogenheit daher auch: Im Kontext gehen Esser/Krey 2016 an gleicher Stelle nach dem ''„Fall 56 – Russisches Roulette Nr.1 –“'' auch auf gemeinschaftlichen Drogenkonsum und Autorennen mit Todesfolge ein. Das kann ja dann, wer will, mit ihrer Sicht auf das sogenannte [http://juris.bundesgerichtshof.de/cgi-bin/rechtsprechung/document.py?Gericht=bgh&Art=en&Datum=Aktuell&nr=45954&linked=pm ''„Konstanzer Urteil“''] (BGH 4 Str 328/08) abgleichen, das OnlyMe hier einbringt und dessen Sichtweise er übernimmt. Verrate ich zuviel, wenn ich die auf den ''„Fall 56 – Russisches Roulette Nr.1 –“'' bezogene Einordnung bei Esser/Krey 2016 dazu zitiere? Ich lese (fallbezogen...): ''„Anders dagegen lag es beim sog. »Konstanzer Wagenrennen«: Der Tod des Beifahrers war den Fahrern auch objektiv zurechenbar, weil sie im Gegensatz zum Beifahrer die Herrschaft über das Geschehen innehatten (Fremdgefährdung)“''. Das von OnlyMe angeführte Urteil BGH 4 Str 328/08 ist für die Fragestellung also untauglich. --[[Spezial:Beiträge/2003:46:A19:1200:E22A:82FF:FEA0:3113|2003:46:A19:1200:E22A:82FF:FEA0:3113]] 02:25, 1. Mär. 2017 (CET)
Im Artikel [[Elektrisches Bauelement]] kam dieser Text vor: "Aktive Bauelemente können … ein Signal mit höherer Leistung abgeben als die Quelle … bereitstellen kann …"


:::: In der deutschen Rechtstheorie weiss ich keinen Titel, aber in Großbritannien haben [[H.L.A. Hart]] und [[Tony Honoré]] mit ''Causation in Law'' einen sehr großen Einfluss auf was schuldhaft zurechenbar ist und was nicht. Und bei den Verbindungen von Honoré nach Freiburg kann ich mir nicht vorstellen, dass das auf die deutsche Rechtstheorie keinen Einfluss gehabt haben soll.
Ich möchte die Stilistik des Satzes jetzt nicht diskutieren. Aber ein Mitautor hat daraus "Aktive Bauelemente können … ein Signal mit höherer Leistung abgeben, als sie die Quelle … bereitstellen kann …" gemacht mit der Begründung "Vollständigen Nebensatz mit Subjekt und Prädikat … mit Komma abgegrenzt". Da ich es nicht verstehe, wollte ich fragen: Woran erkenne ich, dass es ein Hauptsatz mit Nebensatz ist? Was ist mit "sie" gemeint? Die Leistung? Empfinde nur ich die neue Formulierung als ungrammatisch? -- [[Benutzer:Pemu|Pemu]] ([[Benutzer Diskussion:Pemu|Diskussion]]) 01:03, 29. Sep. 2020 (CEST)
:::: Denn das ist die zugrundeliegende Frage: Ist ein Teilnehmer an dem "Spiel" kausal für die Handlungen der anderen (mit)verantwortlich? [[Benutzer:Yotwen|Yotwen]] ([[Benutzer Diskussion:Yotwen|Diskussion]]) 04:27, 1. Mär. 2017 (CET)
: "''als die Quelle … bereitstellen kann''" ist bereits ein vollständiger Nebensatz mit dem Subjekt "''die Quelle''" und dem Prädikat "''kann''" in Endstellung. "''sie''" ist ein Akkusativ-Pronomen, das sich offensichtlich auf "''die Leistung''" bezieht. Die neue Formulierung ist nicht ungrammatisch. --[[Spezial:Beiträge/95.112.134.2|95.112.134.2]] 05:19, 29. Sep. 2020 (CEST)
::Kurz gesagt: Die Änderung ist nicht falsch, war aber unnötig. --[[Benutzer:Joyborg|Joyborg]] 11:26, 29. Sep. 2020 (CEST)
:::Das halte ich für falsch. Sowohl das Komma als auch das Objekt im Nebensatz halte ich für erforderlich. --[[Benutzer:BlackEyedLion|BlackEyedLion]] ([[Benutzer Diskussion:BlackEyedLion|Diskussion]]) 11:43, 29. Sep. 2020 (CEST)
:::(BK) Ne, das ist nach den Regel der deutschen Sprache ein Muss-Komma: "Nebensätze grenzt man mit Komma ab; sind sie eingeschoben, so schließt man sie mit paarigem Komma ein." Н[[Benutzer:Syrcro/Automatik|а]]ктаффэ 11:45, 29. Sep. 2020 (CEST)
::::Dann sind wir wohl verschiedener Meinung. M.E. war der usrprüngliche Satz (auch) ohne Komma richtig. Sobald das "sie" im Nebensatz dazu kommt, muss auch das Komma dazu. Aber bitte, das ist reines "Sprachgefühl" und lässt sich gern eines Besseren belehren. --[[Benutzer:Joyborg|Joyborg]] 11:53, 29. Sep. 2020 (CEST)
:::::Der Duden schreibt dazu: „Für die vergleichenden Konjunktionen ''als'' und ''wie'' gilt: 1. Wenn diese Konjunktionen nur Wörter oder Wortgruppen (ohne Verb) einleiten, setzt man kein Komma. 2. Wenn diese Konjunktionen einen Nebensatz (mit Verb) einleiten, steht ein Komma.“ Als Beispiel wird u. a. gegeben: „Wir haben mehr Stühle, als nötig sind“. Es kommt also nicht auf das Akkusativobjekt „sie“ an, sondern auf das Verb „bereitstellen kann“. Das Komma muss in beiden Fällen stehen. --[[Benutzer:Jossi2|Jossi]] ([[Benutzer Diskussion:Jossi2|Diskussion]]) 12:22, 29. Sep. 2020 (CEST)
:::::: Das Interessante an dem Duden-Beispielsatz ist, daß der Nebensatz überhaupt kein Subjekt hat. --[[Spezial:Beiträge/95.112.134.2|95.112.134.2]] 12:53, 29. Sep. 2020 (CEST)
:Komma muss, das ist fraglos. Nicht sicher bin ich mir, ob die Leistung durch ein Pronomen wiederaufgenommen werden sollte. "Ich bekam ein Paket mit mehr Büchern, als ich ''sie'' ins Regal stellen konnte" - klingt für mich komisch, aber ich finde auf Anhieb keine Regel dazu. Grüße [[Benutzer:Dumbox|Dumbox]] ([[Benutzer Diskussion:Dumbox|Diskussion]]) 15:10, 29. Sep. 2020 (CEST)
::Diesen Satz empfinde ich auf die gleiche Art daneben wie den aus dem Ursprungsposting. <small>(Und auch hier hat mein Sprachempfinden absichtlich zwischen "daneben" und "wie" kein Komma gesetzt.)</small> Durch das Komma und das "sie" verstehe ich eher sowas wie "ich bekam noch mehr Bücher (per Paket) just in dem Moment, als ich sie [wen auch immer, je nach Kontext – z. B. die Vase] ins Regal stellen konnte." <small>(Und wieder kein Komma zwischen "sowas" und "wie" gesetzt.)</small> -- [[Benutzer:Pemu|Pemu]] ([[Benutzer Diskussion:Pemu|Diskussion]]) 23:52, 29. Sep. 2020 (CEST)
:::Dein Sprachempfinden ist völlig in Ordnung, denn da gehört auch in beiden Fällen kein Komma hin: „wie den aus dem Ursprungsposting“ ist eine Wortgruppe, kein Nebensatz; also kein Komma. Im zweiten Satz ist „wie“ keine Konjunktion, die einen Nebensatz einleitet, sondern ein Vergleichspartikel zwischen „sowas“ und dem Zitat; also auch kein Komma. Auch dein Verständnis des von Dumbox angeführten Satzes ist sprachlich richtig, sofern man „als“ nicht als vergleichende, sondern als temporale Konjunktion auffasst. Man brauchte dann zum Verständnis allerdings ein Bezugswort für das „sie“ im vorhergehenden Satz (also etwa „die Vase“) und würde zur Verdeutlichung auch eher so etwas schreiben wie „als ich sie gerade ins Regal stellen wollte“. Der von Dumbox angeführte Beispielsatz klingt, glaube ich, mit „als“ als vergleichender Konjunktion deshalb so falsch, weil man das „sie“ nicht auf „Bücher“, sondern auf „mehr Bücher“ bezieht. Im Satz des Ursprungsposters war „sie“ die höhere Leistung, in diesem Satz hier wäre „sie“ die mehr Bücher. Allerdings halte ich auch im Ursprungssatz die Variante ohne „sie“ für die eindeutig bessere. Da der Satz auch ohne „sie“ eindeutig, klar und verständlich ist, ist das „sie“ hier eine völlig überflüssige Doppelung, die eher stört, als dass sie zum Verständnis beitragen würde. Falls die Änderungsbegründung implizieren wollte, man brauche das „sie“ für einen vollständigen Nebensatz mit Subjekt und Prädikat, ist das Quatsch, wie schon in der ersten Antwort präzise dargelegt wurde. --[[Benutzer:Jossi2|Jossi]] ([[Benutzer Diskussion:Jossi2|Diskussion]]) 23:44, 30. Sep. 2020 (CEST)


:: 1. mir fällt da dann noch ein, dass die Einwilligung des Opfers manchmal nicht ernst genommen werden darf... etwa beim Arzt, wenn der Pat unbedingt n Alpenveilchen im Knie möchte (dann muss man erstmal ganz oft zum Psychiater...)... oder beim Meiwes (bei solchen Wünschen hilft auch kein Psychiater mehr... also die werden nie ernst genommen...)... oder beim Kneipenwirt, wenn der Gast sagt, dass er nun aber mit Vollgas nach Hause fahren muss, damit er ob seines Vollrausches durch mitgebrachte Spirituosen nich das ganze Auto vollspuckt... 2. das passt auch zu [[user:Bobo11]]'s Überlegungen mit betrunkenen und nüchternen Teilnehmern/Zuschauern... --[[user:Homer Landskirty|Heimschützenzentrum]] ([[user talk:Homer Landskirty|?]]) 05:37, 1. Mär. 2017 (CET)
== Frage zu Reisebeschränkung ==
:::Bobo11s 'Überlegungen' haben aber nichts mit der Fragestellung zu tun. Wer am RR teilnimmt und dabei zu Tode kommt ist übrigens allenfalls 'Opfer' seines Übermuts. <small>-- <span style="text-shadow:grey 0.1em -0.1em 0.1em;"> [[Benutzer:Ian Dury|Ian Dury]]<sup> [[Benutzer Diskussion:Ian Dury|Hit me]]</sup></span>&nbsp;</small> 11:26, 1. Mär. 2017 (CET)


::::Die Frage war ja "Ist die Teilnahme ''legal''?" Und damit sind die Implikationen in Zusammenhang mit dem Waffenrecht eben nicht trivial. Selbst mit Waffenschein darf ich nicht just for fun in der Gegend rumballern. Und ich darf nach §34 WaffG die Waffe niemandem überlassen, der nicht die nötige Berechtigung hat. Und ohne Rechte darf mein Gegenüber die Waffe auch gar nicht benutzen. Selbst, wenn der Waffenbesitzer nicht an einem "Körperverletzungs- oder Tötungsdelikt" schuldig ist, ist doch die Frage, ob ein sachgemäßer Umgang mit der Waffe vorliegt, wenn jemand damit zu Tode kommt. Legal wäre somit wahrscheinlich nur die merkwürdige Situation, in der zwei Besitzer von Waffenscheinen mit ihren eigenen Revolvern auf einem dafür geeigneten Schießplatz Russisches Roulette spielen. --[[Benutzer:Optimum|Optimum]] ([[Benutzer Diskussion:Optimum|Diskussion]]) 23:15, 1. Mär. 2017 (CET)
Wenn ich jetzt von Oberösterreich nach Deutschland einreise und von Deutschland wieder nach Tirol ausreise (keine Transitfahrt, habe dazwischen Aufenthalt in Deutschland), muss man dann in Quarantäne oder nicht?


:::: ehm? 1. aber der Gedanke an Alkohol-Rausch oder sonstige Geisteskrankheit oder sogar Zwangslage drängt sich einem bei Russisch Roulette doch gerade auf, denn: Warum sonst sollte man an sowas teilnehmen? normal isst man doch wohl möglichst sogar Bio-Gemüse... 2. zusehen und nicht helfen ist normal verboten... 3. Manipulationen an der Waffe sind auch ein beliebtes Szenario im Fernsehen... dann wird es aber doch auf jeden Fall kriminell? --[[user:Homer Landskirty|Heimschützenzentrum]] ([[user talk:Homer Landskirty|?]]) 05:47, 2. Mär. 2017 (CET)
<!-- Lass die nachfolgende Zeile am ENDE deiner Frage stehen. Sie wird in deine Signatur umgewandelt. -->
:::::Etwas muss nicht ausdrücklich verboten sein, wenn das Resultat strafbar ist, bzw. es nur mit Übertrettung von Gesetzen möglich ist. Beim russisch Roulette haben wird durchaus Punkte die nicht legal sind. Denn die von Optimum angesprochene Fremdgefährdung ist da durchaus so ein Punkt, wenn man in einem Raum eine Waffe abfeuert, selbst wenn das nur ein „vielleicht“ ist. Der Waffenbesitzer möchte ich jedenfalls nie sein, wenn die Waffe freiwillig ausgehändigt wird, dies im Wissen das damit russisch Roulette gespielt werden soll. Aber lassen wir das ganze mal ausser vor, wer in welcher Position welche Tatbestände erfüllen könnte. Denn Prinzipiell wird es für alle Beteiligte unangenehm wenn es zu einem Unfall mit Schusswaffen kommt. Weil spätestens dann wird sich die Polizei und die Staatsanwaltschaft der Sache annehmen und anfangen Fragen zu stellen. Die Fragen werden sicher mehr, wenn es sich herauskristallisiere, dass es um einen vermeidbaren Unfall handelt (es muss niemand russisch Roulette spielen, ergo vermeidbar). Richtig unangenehm werden die Fragen wenn der Staatsanwalt findet „vermeidbarer Unfall“ passt irgend wie nicht so recht. Und findet da habe wer vorsätzlich gehandelt damit es zu dem "Unfall" kam. Die Wahrscheinlichkeit das für die aktiven Teilnehmer auch Untersuchungshaft angeordnet wird, halte ich nicht für ausgeschlossen. --[[Benutzer:Bobo11|Bobo11]] ([[Benutzer Diskussion:Bobo11|Diskussion]]) 07:53, 2. Mär. 2017 (CET)
--[[Benutzer:Hans Kachelmann|Hans Kachelmann]] ([[Benutzer Diskussion:Hans Kachelmann|Diskussion]]) 09:27, 29. Sep. 2020 (CEST)
::::::Wie so oft sind deine Antworten nebulös und entziehen sich jedem Beweis. Wenn du die Antwort weißt oder wenigstens Hinweise darauf kennst, dann antworte so kurz wie möglich, so lang wie nötig, mit Links auf Wikipedia-Artikel oder andere Quellen, die zum Verständnis beitragen - so steht es im Intro. Natürlich kann eine unerlaubte Überlassung einer Waffe vorliegen. Die Frage bietet dafür aber keinen Anhaltspunkt. Abschließend: Wegen was würden die Teilnehmer verurteilt werden? Wegen illegaler Teilnahme am RR jedenfalls nicht, die Teilnahme daran ist legal. Manipuliert jemand die Waffe in der Absicht, dass der andere zu Tode kommt, dann ist das ggfls. Totschlag oder auch Mord. Hat er die Waffe so manipuliert, dass sie nach auslösen alle Atomwaffenlager der Welt zur Explosion und den Bundestag zum Einsturz bringt, dann erwartet ihn alles mögliche, in D aber keine Veurteilung wg. Teilnahme am RR. <small>-- <span style="text-shadow:grey 0.1em -0.1em 0.1em;"> [[Benutzer:Ian Dury|Ian Dury]]<sup> [[Benutzer Diskussion:Ian Dury|Hit me]]</sup></span>&nbsp;</small> 14:34, 2. Mär. 2017 (CET)
:Wenn Du in den letzten 14 Tagen nicht in Wien, Tirol oder Vorarlberg warst, darfst Du nach D einreisen. Und die Reise von D nach Tirol ist auch problemlos möglich, nur danach darfst Du nicht zurück nach D. --[[Benutzer Diskussion:TheRunnerUp|TheRunnerUp]] 10:09, 29. Sep. 2020 (CEST)
:Und bitte immer beachten, dass nur ein Aufenthalt von 48 Stunden quarantänepflichtig macht. Ein Tagesausflug zum Einkaufen oder Bergwandern nach Tirol ist problemlos möglich. Nur von Besuchen in Innsbruck wird ganz abgeraten. Grüße --[[Benutzer:H-stt|h-stt]] [[Benutzer Diskussion:H-stt|<small>!?</small>]] 21:11, 30. Sep. 2020 (CEST)


:::::::+1, Ich verstehe auch nicht die hartnäckige Ignoranz einiger Beiträge. Die Frage war ''„Ist die Teilnahme an Russisch Roulette legal?“'' Es geht also um die strafrechtliche Verantwortlichkeit der Teilnehmenden bei einem möglichen oder eingetretenen Todesfall. Die waffenrechtliche Diskussion ist aufgesetzt und konstruiert. Wenn z.B. die Waffe dem Opfer gehört und der überlebende Teilnehmer sie niemals in der Hand hatte, spielt das Waffenrecht für den Fall keine Rolle. Ein waffenrechtliches Delikt kann also hinzukommen, muss aber nicht. Ich denke auch, dass es dem Fragesteller um diesen Aspekt nicht ging. Eigentlich ist die Frage schon längst beantwortet.
== "Klebrige" Mullbinden ==
:::::::Ich lese deshalb nochmal vor: ''„Fall 56: – »Russisches Roulette Nr. 1« – A und O gehören zur »Russen-Mafia«. Aus Imponiergehabe spielen sie »Russisches Roulette«: A hält sich einen mit nur einer Patrone geladenen Revolver an die Schläfe, ohne zu wissen, in welcher der acht Kammern der Revolvertrommel sich die Patrone befindet, und drückt ab; er hat Glück. Dann gibt er die Waffe dem O, der sich, nachdem er die Revolvertrommel hat rotieren lassen, ebenfalls den Revolver an die Schläfe setzt und abdrückt. O hat weniger Glück und stirbt. Strafbarkeit des A wegen Fahrlässiger Tötung, wenn beide Gangster nicht mit der Möglichkeit eines tödlichen Ausgangs gerechnet hatten? Die Frage ist zu verneinen: O war das Opfer einer freiveranwortlichen Selbstgefährdung. A war lediglich an dieser Gefährdung beteiligt. Für solche Konstellationen hat der BGH zutreffend entschieden: »Eigenverantwortlich gewollte und verwirklichte Selbstgefährdung unterfallen nicht dem Tatbestand eines Körperverletzungs- oder Tötungsdelikts, wenn das mit der Gefährdung bewusst eingegangene Risiko sich realisiert. Wer lediglich eine solche Selbstgefährdung veranlasst, ermöglicht oder fördert, macht sich nicht wegen eines Körperverletzungs- oder Tötungsdelikts strafbar.« Die Begründung hierfür liegt auf der Hand: Aus der Straflosigkeit der vorsätzlichen Mitverursachung des freiverantwortlichen Suizids und der Selbstverletzung anderer folgt mittels eines Erst-Recht-Arguments die Straflosigkeit der vorsätzlichen Mitverursachung der freiverantwortlichen Selbstgefährdung des Opfers. Soweit die Teilnahme am Suizid oder an der Selbstverletzung straflos ist, kann die Teilnahme an der wissentlichen Selbstgefährdung, auch wenn sich die Gefahr realisiert, nicht als Fahrlässige Tötung oder Fahrlässige Körperverletzung strafbar sein. In casu ist von einer freiverantwortlichen Selbstgefährdung des O auszugehen. Somit ist der Tod des O dem A nicht als sein Werk objektiv zuzurechnen und A hat sich nicht der Fahrlässigen Tötung schuldig gemacht. Als weitere Beispiele für den Ausschluss der objektiven Zurechnung wegen freiverantwortlicher Selbstgefährdung des Opfers werden in Rechtsprechung und Lehre diskutiert: (...) Tödlich ausgehendes Motorrad-Wettrennen zweier Mitglieder einer Bande (...) Abgabe von Heroin an einen Drogenkonsumenten, der sich das Rauschgift injiziert und daran stirbt (...) Beischlaf eines HIV-positiven Mannes mit seiner Freundin, die diese Infektion und das Übertragungsrisiko kennt (...)“'' ([https://books.google.de/books?id=od9tDAAAQBAJ&pg=PP183&lpg=PP183&dq=russisch+roulette+strafbar&source=bl&ots=3906oQB5OU&sig=xt2qM4Naf9LufvNXJ7U_Ingrn6U&hl=de&sa=X&ved=0ahUKEwiYwOHZ_rDSAhXBBSwKHdG6CwQ4ChDoAQhHMAg#v=onepage&q=russisch%20roulette%20strafbar&f=false Robert Esser, Volker Krey: ''Deutsches Strafrecht, Allgemeiner Teil'', Kohlhammer Verlag, Stuttgart, 6, Aufl. 2016], ISBN 978317029882).
:::::::Den jeweiligen Personenartikeln kann man entnehmen: [[Robert Esser]] ist ''„seit Februar 2007 (...) Inhaber des Lehrstuhls für Deutsches, Europäisches und Internationales Strafrecht und Strafprozessrecht sowie Wirtschaftsstrafrecht an der Universität Passau. Er ist zudem Stellvertretender geschäftsführender Direktor des Instituts für internationales und ausländisches Recht – Abteilung für europäisches und internationales Straf- und Strafverfahrensrecht. Im Jahr 2010 wurde zudem die Forschungsstelle "Menschenrechte im Strafverfahren – Human Rights in Criminal Proceedings (HCRP)" gegründet, deren Direktor Robert Esser seit Gründung ist. HRCP ist eine Forschungs-, Fortbildungs- und Beratungsstelle für alle Fragen des internationalen Menschenrechtsschutzes im Strafverfahren. Zu den Aufgaben von HRCP zählt die Auswertung und Systematisierung der Rechtsprechung des Europäischen Gerichtshofs für Menschenrechte (EGMR) in Straßburg sowie der Spruchpraxis des Human Rights Committee und des Committee Against Torture der Vereinten Nationen (UN-Menschenrechtsausschuss / UN-Antifolterausschuss).“'' [[Volker Krey]] war ''„1974 C3-Professor und Wissenschaftlicher Rat an der Universität Bielefeld. 1975 wurde er als einer der Gründungsprofessoren des Fachbereichs Rechtswissenschaft an der Universität Trier zum C4-Professor für Strafrecht, Strafprozessrecht und Methodenlehre der Rechtswissenschaft ernannt. Von 1978 bis 1998 war Krey zudem Richter am 1. Strafsenat des Oberlandesgerichts Koblenz. (...) Seine Lehrbücher zum Allgemeinen und Besonderen Teil des Strafrechts und zum Strafprozessrecht sind weit verbreitet.“'' Der [[Bundesgerichtshof]] erkennt, wie weiter oben bereits diskutiert, zwar fallbezogen, aber letztinstanzlich. Zwei hochkarätige, allgemein anerkannte Juristen. Ein oberster Gerichtshof. Keine einzige Literaturstelle, die sich im Widerspruch oder in der Kontroverse befindet. So what? --[[Spezial:Beiträge/91.40.202.96|91.40.202.96]] 23:50, 2. Mär. 2017 (CET) (hier auch als 80.130.225.92 und 2003:46:xxx unterwegs)


:::::::: [http://www.ingo-heinemann.de RA Ingo Heinemann] ist auch irgendwie ganz oben und findet schon gewisse Kredit-Verträge blöd, auch wenn die ohne Waffen-Einsatz unterschrieben wurden... --[[user:Homer Landskirty|Heimschützenzentrum]] ([[user talk:Homer Landskirty|?]]) 05:19, 3. Mär. 2017 (CET)
Ich hatte gerade mal wieder ein antiseptisches Salbenpflaster auf einer wunden Stelle appliziert. Das befestige ich normalerweise mit "irgendwas" nicht unbedingt hygienischem, weil der Verband letztlich nur dazu dient, das Abfallen des Pflasters zu verhindern und die Kleidung und Umgebung vor Verschmutzung zu schützen und deswegen weder steril noch sauber zu sein braucht. Diesmal griff ich zu einer elastischen Fixierbinde aus einem abgelaufenen Verbandskasten und stellte überrascht fest, daß die nicht "selbstklebend" wie die Verbände war, die üblicherweise bei Ärzten usw. benutzt werden. Na gut, muß ja auch nicht unbedingt: dieses Erste-Hilfe-Material ist ja dazu da, sterile Wundabdeckungen oder Druckverbände nur solange zu fixieren, bis die Rettung den Verletzten in die Notaufnahme gebracht hat - dort wird es sowieso gleich wieder runtergeschnitten. Aber gibt es einen Grund, daß nicht generell Binden aus selbstklebendem Material eingesetzt werden? Wobei: So richtig selbstklebend sind die nun auch nicht - oft wird das Ende noch zusätzlich mit Heftpflaster fixiert. (Ich mache das nicht, ich schneide das Ende der Länge nach auf und verknote die beiden Zipfel um die Extremität herum.) <small>Neulich mußte ich eine Arzthelferin bremsen, die mir partout einen kunstgerechten OP-Wundverband nach Maßgabe des Chirurgen anlegen wollte, obwohl es völlig überflüssig war und nach meiner Methode viel einfacher und haltbarer ging - das ging dann "Aber der Herr Doktor hat gesagt..." und "Das ist aber mein Fuß".</small> --[[Spezial:Beiträge/95.112.134.2|95.112.134.2]] 13:53, 29. Sep. 2020 (CEST)
::::::::: Aber nein, Homer, der hat zwar das zweite Staatsexamen, aber der ist ganz sicher nicht ''„irgendwie ganz oben“''. Und dahin ist es für ihn noch weit. Kein Prägen der aktuellen Rechtssprechung über Jahrzehnte an einem Oberlandesgericht. Kein Lehrstuhl. Kein bedeutender, nichtmal ein nennenswerter Beitrag zur Rechtwissenschaft. Kein Mitherausgeber eines [[Gesetzeskommentar|Kommentars]]. Nach seinen Publikationen lernen nicht über Jahrzehnte hinweg tausende von Jurastudenten. Keine Mitwirkung in bedeutenden Gremien. Aus meiner Sicht also nicht hochkarätig. Und als Konsequenz hat er nichtmal einen Wikipedia-Artikel... Ganz oben ist woanders. --[[Spezial:Beiträge/80.130.231.152|80.130.231.152]] 18:58, 3. Mär. 2017 (CET) (hier auch als 91.40.202.96 und 2003:46:xxx unterwegs)
:Jein.
:Also Binden im Erste Hilfe/Verbandsmaterial waren eigentlich von jeher ohne irgendwelchen Kleb. Binden mit Kleb findet man neuerdings, die sind, wie ich nach einem Kauf realisiert habe, rein als ~orthopädische Bandagen vorgesehen und nicht als Wundverbandsmaterial.
:Wenn ich mich nicht täusche. --[[File:Lemon with white background.jpg|15px|Quetsch mich aus, ... ]] [[Benutzer:Itu|itu]] ([[Benutzer Diskussion:Itu|Disk]]) 20:14, 29. Sep. 2020 (CEST)
:: Mußt Du wohl. Ich habe bei Weißkitteln, z. B. auch beim Blutspenden beim DRK, bisher immer nur Kleb-Binden im Einsatz erlebt, nie als "orthopädisch". Verwechselt Du das vielleicht mit elastischen Binden? --[[Spezial:Beiträge/95.112.134.2|95.112.134.2]] 22:06, 29. Sep. 2020 (CEST)
:::Hm, ich hab hier so ne Kleb-Rolle aus einem Billigladen, die ist als „Haftbandage“ deklariert. --[[File:Lemon with white background.jpg|15px|Quetsch mich aus, ... ]] [[Benutzer:Itu|itu]] ([[Benutzer Diskussion:Itu|Disk]]) 22:20, 29. Sep. 2020 (CEST)
:::: Ich habe keine Ahnung, wie Verbands-Zeuxx auf Fachchinesisch heißt. Wie auch immer: ist mit oder ohne Kleb besser? (Mir will scheinen: "mit", denn das verhindert z. B. auch das Verrutschen der einzelnen Wicklungslagen in Querrichtung; wenn man z. B. eine Binde mittig um das Knie oder den Unterschenkel gewickelt hat, dann bleibt die da auch und findet sich nicht abends lose um den Knöchel wieder, ohne daß man sie mit viel Heftpflaster an der Haut ankleben müßte.) Bleibt jetzt immer noch die Frage, warum sich das nicht im Autoverbandskasten findet - Tradition, Kosten, oder andere Gründe? (Kann da vielleicht mal jemand einen Artikel zu schreiben?) --[[Spezial:Beiträge/95.112.134.2|95.112.134.2]] 00:41, 30. Sep. 2020 (CEST)


== Kölnische Profanität ==
:Haftbinden eignen sich nur für sehr wenige Anwendungen. Meist sind Kurz- oder Langzugbinden ohne Hafteigenschaften besser, weil die sich nicht so leicht in ziehharmonikaartik zusammengeschobener Form verhaken. Direkt auf der Haut lösen die außerdem häufig Reizungen aus. Meist verwendet man die nur äußertes Schicht eines mehrschichtigen Verbands. Laien sind mit der Verwendung häufig überfordert, erst recht im Streß, darum gehört sowas nicht in den EH-Kasten. --[[Spezial:Beiträge/94.219.186.65|94.219.186.65]] 01:31, 30. Sep. 2020 (CEST)


Ich glaube mich an eine kölnische Profanität zu erinnern welche ein Schiff (ggf. Boot) und ein ziehende bzw. schubsende Bewegung beinhaltet. Ungefähr "Deu mer doch den Kaan". Trügt meine Erinnerung und ist etwas über die Herkunft dieser Redewendung bekannt? Ich kann die Frage auch gerne nächste Woche wiederholen.
== Hochprägung bei Kreditkarten ==
Danke,
--[[Spezial:Beiträge/2003:70:EF74:FBD:4986:8EBD:EE03:FA4D|2003:70:EF74:FBD:4986:8EBD:EE03:FA4D]] 20:19, 27. Feb. 2017 (CET)
:[[Kölsch (Sprache)]] kennt „Deu mer doch der Naache!“ (Schiebe doch meinen Kahn an!) mit der standarddeutschen Entsprechung „Leck mich am Arsch!“
:[http://www2.koelsch-akademie.de/liederserver/index.php?inc=0&id=8747&lied=&interpret=Marie-Luise%20Nikuta&texter=&album=&verlag=&kategorie=&seite=0&erw=&erscheinungsjahr=&suchen=suchen Hier] findet man demgegenüber „Däu mer doch der/dr/d'r Naache“ von [[Marie-Luise Nikuta]] als „Lass mich doch in Ruhe“. Die Zeile kommt auch in [http://www.songtexte.com/songtext/black-fooss/immer-wigger-bc7e9a2.html Immer wigger] von den [[Bläck Fööss]] vor. --[[Benutzer:Vsop|Vsop]] ([[Benutzer Diskussion:Vsop|Diskussion]]) 20:50, 27. Feb. 2017 (CET)
::Adam Wrede, ''Neuer Kölnischer Sprachschatz'' (1958), Bd. 2, S. 220, schreibt: „''Däu mer der Naache! Do kanns mer der Naache däue'', ablehnende Antwort: nein, nichts da!“ Der Grad der Derbheit liegt irgendwo zwischen "Lass mich in Ruhe" und "Leck mich am Arsch". Kommt wie immer auf die Situation, den Tonfall, das Verhältnis der Personen zueinander usw. an. --[[Benutzer:Jossi2|Jossi]] ([[Benutzer Diskussion:Jossi2|Diskussion]]) 13:38, 28. Feb. 2017 (CET)
:::In [[Will Hermanns]] ''Aachener Wortschatz'' steht eher neutral 'Du kanns mich ens der Naache döie (abschlägige Antwort)' --[[Benutzer:Elrond|Elrond]] ([[Benutzer Diskussion:Elrond|Diskussion]]) 16:17, 28. Feb. 2017 (CET)


== Käpt´n Ohnefurcht ==
Warum ist die Kreditkartennummer auf Karten eigentlich noch hochgeprägt? Die sogenannten Imprinter, die die Kreditkartennummer mechanisch auf Abrechnungsbelege übertragen, hab ich ewig nicht mehr beim Zahlen mit Kreditkarte erlebt. Sind die überhaupt noch im Einsatz? --[[Spezial:Beiträge/95.175.104.20|95.175.104.20]] 18:25, 29. Sep. 2020 (CEST)
:Ich fand im Netz folgendes: "Are credit card imprinters still used? Some companies use them as a backup option when their electronic credit card scanners break down or there's a power outage. In addition to being an essential backup, manual credit card imprinters can still be relevant in today's high-tech and computerized business world."--[[Benutzer:Dr. Peter Schneider|Dr. Peter Schneider]] ([[Benutzer Diskussion:Dr. Peter Schneider|Diskussion]]) 18:32, 29. Sep. 2020 (CEST)
:"Das war schon immer so" dürfte eine Rolle spielen. Wenn die Prägung fehlt, fehlt halt was. Fühlt sich dann nicht mehr so wertig an, fast so als wäre die Karte auf einmal weiß oder blau statt schwarz oder gold. Ist was für arme Leute, geht also gar nicht. --[[Spezial:Beiträge/2001:1715:9D9B:770:EA4E:84FF:FEFE:92A5|2001:1715:9D9B:770:EA4E:84FF:FEFE:92A5]] 22:19, 29. Sep. 2020 (CEST)
:So ganz hippe Anbieter wie [https://everydaydebit.com/2020/04/07/curve-goes-minimal-with-numberless-cards/ Curve] und bunq geben inzwischen Karten ohne geprägte Nummber aus. --[[Benutzer:FGodard|FGodard]]|[[Benutzer_Diskussion:FGodard|✉]]|[[Benutzer:FGodard/Bewertung|±]] 11:41, 30. Sep. 2020 (CEST)
::Früher wurde in Restaurants und Geschäften die Karte auf einen Prägestempel gelegt und mit dem Darüberbewegen eines Schlittens wurde die Kartennummer auf die Rechnung geprägt. Die Rechnungen wurden dann gesammelt an das Kreditinstitut übersandt. Ich kenne das noch so. Gruß [[Benutzer:W.wolny|W.Wolny]] - [[Benutzer_Diskussion:W.wolny|<small> ''(X)'' </small>]] 12:24, 30. Sep. 2020 (CEST)


Hallo. Ich hab kürzlich eine kurze Szene der Ermordung von [[Jürgen Kanter und Sabine Merz|Jürgen Kanter]] durch die Terrorgruppe [[Abu Sajaf]]; [http://www.derwesten.de/panorama/terrororganisation-abu-sayyaf-entfuehrt-deutschen-segler-id12339859.html geschockt & ohne Worte hab ich das Video abgebrochen.] Unfassbar! Nachdem habe ich anschließend nach ihm (J.K.) googlete, bin darauf gestossen dass beide, Kanter und seine Partnerin 2008 schon einmal [http://www.segeln-magazin.de/selbstmord-auf-see-ex-piratengeiseln-kehren-nach-somalia-zurueck/267 2008 vor Somalia das gleiche Schicksal (jedoch mit einem positiven Ausgang) ereilte], wobei die Bundesregierung (der Staat) ein gewisses Lösegeld zahlte. Nun, nachdem ich heut morgen in den Nachrichten mitbekam dass Kanter exekutiert wurde, erwähnte kein Nachrichten(-sender)programm dieses wichtige Detail. Nachdem ''Abu Sajaf'' dieses mal ein Lösegeld in Höhe von 500- bis 600tausend Euro (Dollar) mit Ultimatum forderte, und diesem nicht nachgekommen wurde, lautet meine Frage: Wollte der Staat dieses Mal kein Lösegeld zahlen weil die Zuständigen für einen solchen Fall sich sagten (oder dachten), ''„wir hatten doch schon mal bezahlt, wenn Herr Kanter nichts daraus gelernt hat, selbst Schuld; nochmal holen wir ihm die Kohle nicht aus dem Feuer“''...!?? . lG und Danke für eine weiterdenkende Antwort. --[[Spezial:Beiträge/2001:7E8:C086:6801:A07C:760F:76AD:9714|2001:7E8:C086:6801:A07C:760F:76AD:9714]] 22:47, 27. Feb. 2017 (CET)
::: Im Ausland gibt es das noch, insbesondere da, wo es keinen oder nur miesen Netzanschluss gibt oder auch in manchen Taxen. So ein Ritsch-Ratsch funktioniert auch bei Kerzenlicht in der Pampa. Somit weiter geprägt!--[[Benutzer:Gadacz|Klaus-Peter ''<small>(<sup>auf</sup><small>und</small><sub>davon</sub>)</small>'']] 12:38, 30. Sep. 2020 (CEST)
:::: {{ping|W.wolny}} Da kennst du mehr als ich. Weltweit kenne ich es mit abrechenbarer Rechnungsprägung nicht. Es gab und gibt Talons mir mind. 2 Durchschlägen (Bank, Verkäufer, Käufer) --[[Benutzer:Gadacz|Klaus-Peter ''<small>(<sup>auf</sup><small>und</small><sub>davon</sub>)</small>'']] 12:43, 30. Sep. 2020 (CEST)
::::: Genau das meinte ich. Hatte mich etwas falsch ausgedrückt :) Gruß [[Benutzer:W.wolny|W.Wolny]] - [[Benutzer_Diskussion:W.wolny|<small> ''(X)'' </small>]] 12:50, 30. Sep. 2020 (CEST)


Erstens würde ich die Meldungen von morgen abwarten, Zweitens könntes Du richtig liegen und Drittens: Du verwechselst die Auskunft mit einem Bloog! --[[Spezial:Beiträge/80.187.114.147|80.187.114.147]] 23:10, 27. Feb. 2017 (CET)
== Lavigny ==
::1. Stimmt. 2. Ok, aber ich habe nicht nach deiner POV gefragt. 3. Die Frage hab ich deshalb HIER gestellt, in der Annahme, dass jemand vielleicht tiefergehende Kenntnisse besitzt, durch irgendwelchen Link das weiterführt bezüglich dieses Themas bzw. jemand zu diesem Thema schon detaillierter nachgeforscht hat. 4. Ich weiss sehr wohl was ein Blog (oder Bloog, oder Blooooooog) ist, und die Auskunft ist keiner. Trotzdem ein Dankeschön. --[[Spezial:Beiträge/88.207.220.156|88.207.220.156]] 00:32, 28. Feb. 2017 (CET)
::: Die Frage hast Du hier gestellt in der Hoffnung auf „tiefergehende Erkenntnisse“? Diese Auskunftsseite handelt von "Allgemeinen Wissen", nicht von tiefergehenden Erkenntnissen zum Verhalten des Staates in einer aktuellen Extremsituation. Selbst wenn hier alles und jedes diskutiert werden sollte und dürfte, so müsste sich hier schon ein eingeweihter Staatsbediensteter äußern, um Dir die Frage da oben zu beantworten. Dass das nicht geschehen wird, weißt Du sicher. Erwarte also bitte nicht zu viel und sei nicht enttäuscht darüber, dass nicht jeder solche Fragen hier als in der Auskunft passend ansieht. Es führt hier zu einem Gespräch über einen schrecklichen Vorfall, aber sicherlich niemals zu einer Antwort auf Deine Frage zu diesem speziellen Vorfall, denn die kann keiner wissen. Spekulationen und Plaudereien sind dann aber eher eine Sache für's WP:Café. Nix für ungut. :-) VG --[[Benutzer:Apraphul|Apraphul]] <small><sup> [[Benutzer Diskussion:Apraphul|Disk]] </sup> <sub> [[Benutzer:Apraphul/WP:SNZ|WP:SNZ]]</sub></small> 10:34, 28. Feb. 2017 (CET)
:Das Auswärtige Amt warnt seit Jahren vor Reisen nach [[Basilan]], [[Mindanao]] und in den [[Sulu-Archipel]]. Das Opfer ist dem Anschein nach wider besseres Wissen in diese Region gereist und hat somit seine Entführung und seinen Tod selbst verschuldet. --[[user:Rotkaeppchen68|R<span style="color:red">ô</span>tkæppchen₆₈]] 00:57, 28. Feb. 2017 (CET)
::Das nennt man Täter-Opfer-Umkehr.--[[Benutzer:Hinnerk11|Hinnerk11]] ([[Benutzer Diskussion:Hinnerk11|Diskussion]]) 01:58, 28. Feb. 2017 (CET)
:::Wenn ich, wissend um die Situation in dieser Gegend selbige bereise, darf ich mich nicht beschweren, wenn der Nervenkitzel böse Realität wird. Ein Bekannter hat vor einigen Jahren Iran, Irak und Afghanistan mit seinem Geländewagen erkundigt. Auf die Gefahr angesprochen, meinte er lapidar: " War schon spannend". Wie auch immer, das Mitleid bei diesen Menschen hält sich bei mir in Grenzen, genauso wie bei anderen Adrenalinjunkies. Wer sich in Gefahr begibt, kommt darin um. --[[Benutzer:Elrond|Elrond]] ([[Benutzer Diskussion:Elrond|Diskussion]]) 10:54, 28. Feb. 2017 (CET)


Es ist ja ganz offensichtlich, dass dem Opfer keine zweite Chance gegeben wurde. Es ist ja ein offenes Geheimnis, dass der Staat in solchen Situationen "Entwicklungshilfe" an diese Gruppen zahlt, damit diese aus lauter Dankbarkeit ihre "Gäste" bewirten und mit Abschiedsgeschenken (ihrem Leben) wieder ziehen lassen. Nur um das böse Wort Lösegeld zu vermeiden, was ja nicht sein kann, da der Staat ja sonst als erpressbar und deutsche Staatsbürger als "sichere Bank" gelten würde. Diese Hilfestellung hat also offenbar Grenzen: Ein Freischuss, dann hat der Unbelehrbare halt Pech gehabt. [[Benutzer Diskussion:morty|Benutzerkennung: '''43067''']] 08:43, 28. Feb. 2017 (CET)
Moin. Ich schreibe gerade einen Artikel zu einer italienischen Burg. In einem Text dazu heißt es:
Mir wird hier zu wenig die Rolle des philippinischen Staates beleuchtet. Wer sagt denn, das die Deutschen nicht auf nem guten Weg waren? Laut Nachrichten wurden aber von Regierungstruppen Stellungen der Abu Sajaf kurz vorher bombardiert, sicher ein Vertrauensbeweis.--[[Benutzer:Scialfa|scif]] ([[Benutzer Diskussion:Scialfa|Diskussion]]) 14:50, 28. Feb. 2017 (CET)
:„Durante il periodo napoleonico, fu annoverato dal Lavigny, nell’elenco preparato per l’arrivo di Giuseppe Bonaparte, tra le opere di interesse artistico.“
:Die Frage geht von falschen Prämissen aus. Die Entscheidungsträger im Staatsdienst sind nicht so einfältig wie viele sie gern sehen. Herrn Kantor wurde nach seiner ersten Freilassung von den zuständigen staatlichen Stellen mit Sicherheit zu verstehen gegeben, daß er sich im Falle des Falles nicht darauf verlassen kann auch ein zweites Mal freigekauft zu werden. --[[Benutzer:Didi_69|DJ]] 20:49, 1. Mär. 2017 (CET)
Ich verstehe, dass Lavigny eine Liste der interessanten Kulturgüter zusammenstellte. Einzige Frage: Wer war Lavigny? Dazu finde ich keine Informationen. Weiß zufällig jemand von euch, wer gemeint ist? Beste Grüße --[[Benutzer:Florean Fortescue|Florean Fortescue]] ([[Benutzer Diskussion:Florean Fortescue|Diskussion]]) 18:36, 29. Sep. 2020 (CEST)


= 28. Februar 2017 =
== Comedian mit dem Thema "Synchronsprecherin" ==


== Ton-/Datenträger für Langzeitarchivierung ==
Hallo, es gab mal eine Comedian, die sich als Synchronsprecherin vorgestellt hat. Ich glaube, der (Bühnen)name war "Susanne Schmidt" oder so ähnlich. Sie hat überzogen gesprochen, so, wie man das von Synchronsprechern bei Filmen kennt. Ich habe durch Googlen nichts gefunden. Erinnert sich jemand? Ist sicherlich schon 20 Jahre her.


Hallo, was eignet sich bei
<!-- Lass die nachfolgende Zeile am ENDE deiner Frage stehen. Sie wird in deine Signatur umgewandelt. -->
# Tonträgern <small>(digital oder analog)</small> sowie
--[[Benutzer:Ziko|Ziko]] ([[Benutzer Diskussion:Ziko|Diskussion]]) 18:42, 29. Sep. 2020 (CEST)
# Datenträgern <small>(digital)</small>
:Du meinst vermutlich [[Käthe Lachmann]]. („Die wohl bekannteste Figur, die Lachmann immer wieder verkörpert, ist die Synchronsprecherin Elke Schmitt, die sich durch eine überzogen affektierte Sprechweise und einen naiv-charmanten Charakter auszeichnet.“) Beste Grüße --[[Benutzer:Florean Fortescue|Florean Fortescue]] ([[Benutzer Diskussion:Florean Fortescue|Diskussion]]) 18:46, 29. Sep. 2020 (CEST)
am besten für Langzeitarchivierung? Die Daten sollen bei dieser theoretischen Überlegung einmalig gespeichert werden, ein wiederbeschreibbares Medium ist also nicht unbedingt nötig. --[[User:Morten Haan|Morten Haan]] &#x1F6E3; [[User:Morten Haan/für Leser|Wikipedia ist für Leser da]] • [[User:Morten Haan/Skin-Entwurf|Skin-Entwurf]] 00:37, 28. Feb. 2017 (CET)
::Danke dir - und ich bin stolz, Teil einer allwissenden Müllhalde zu sein! [[Benutzer:Ziko|Ziko]] ([[Benutzer Diskussion:Ziko|Diskussion]]) 18:49, 29. Sep. 2020 (CEST)


:Hast Du unseren Artikel [[Langzeitarchivierung]] schon entdeckt? --[[user:Rotkaeppchen68|R<span style="color:red">ô</span>tkæppchen₆₈]] 01:11, 28. Feb. 2017 (CET)
== Unendliche Aussage? ==


:: Klar, nur werde ich da nicht so ganz schlau aus der Tabelle, zumal ich nicht weiß, wie aktuell die Daten sind, und analoge Tonträger wie [[Schallplatte]]n ganz fehlen. --[[User:Morten Haan|Morten Haan]] &#x1F6E3; [[User:Morten Haan/für Leser|Wikipedia ist für Leser da]] • [[User:Morten Haan/Skin-Entwurf|Skin-Entwurf]] 14:13, 28. Feb. 2017 (CET)
Gibt es eigentlich eine mathematische Theorie über eine unendliche Aussage, also eine Aussage? Das dürfte im Sinne der Logik kein wohlgeformter Satz sein, aber vielleicht hat das ja mal jemand untersucht? {{unsigniert|2.200.60.220|18:55, 29. Sep. 2020 (CEST)}}
:Ich verstehe nicht, was du meinst. Was soll eine unendliche Aussage sein? Soll der Satz (bzw. der formale Ausdruck) unendlich lang sein? Es gibt Logiken, bei denen man zum Beispiel unendliche viele Einzelaussagen durch ein "und" oder ein "oder" verbinden kann. Meinst du so etwas? --[[Benutzer:Digamma|Digamma]] ([[Benutzer Diskussion:Digamma|Diskussion]]) 21:25, 29. Sep. 2020 (CEST)
::Kleiner! Eine Aussage ist etwas, das von jemandem ausgesagt wird. Anders als Punkte oder Zahlen sind Aussagen sinnhaft. Und da es nicht unendlich viele sinnhafte Kombinationen von Wörtern gibt, kann es auch keine unendlichen Aussagen geben. Irgendwann ist jede Kombinationsmenge erschöpft, beginnt sich zu wiederholen oder wird eben nicht-sinnhaft. --[[Spezial:Beiträge/188.103.248.114|188.103.248.114]] 22:22, 29. Sep. 2020 (CEST)
::: Beweis? --[[Spezial:Beiträge/95.112.134.2|95.112.134.2]] 00:54, 30. Sep. 2020 (CEST)
:::„Und da es nicht unendlich viele sinnhafte Kombinationen von Wörtern gibt […].“ Gegenbeispiel: Die Aussage ''n ist größer als eins'' ist für die unendliche Anzahl an Zahlwörtern, die für natürliche Zahlen größer als eins stehen, richtig. --[[Benutzer:BlackEyedLion|BlackEyedLion]] ([[Benutzer Diskussion:BlackEyedLion|Diskussion]]) 08:39, 30. Sep. 2020 (CEST)
:::Es gibt doch schon unendliche viele natürliche Zahlen. Über die kann man dann auch unendlich viele Aussagen machen. --[[Benutzer:Digamma|Digamma]] ([[Benutzer Diskussion:Digamma|Diskussion]]) 22:06, 30. Sep. 2020 (CEST)
Die [[Cantor-Diagonalisierung]] ist eine Aussage über die Unendlichkeit Aleph<sub>0</sub> (<math>\aleph_0</math> ). Meinst Du so was? --[[Benutzer:Elrond|Elrond]] ([[Benutzer Diskussion:Elrond|Diskussion]]) 22:13, 29. Sep. 2020 (CEST)


:::Es geht um Medien, die selbst bespielt werden können. Bei Schallplatten war das die Ausnahme und betraf Exoten. Der Graveur benötigte etwas weicheres Material. Selbst für [[8-Spur-Kassette]]n gab es Rekorder, da sich um (Endlos)-[[Tonband]] handelt, was aber trotz Graphitbeschichtung höheren Verschleiß hat als Rollentonband. Unser Archiv der Auskunft weiß auch nicht mehr als der Artikel. --<span style="color:#00A000;">Hans Haase ([[BD:Hans Haase|有问题吗]])</span> 16:51, 28. Feb. 2017 (CET)
== Leonardo da Vinci - was wurde verschwiegen ? ==


Zu selbst gebrannten CDs, danach DVDs. Gebrannt zw. 1997 und 2009. Anzahl weit über 100. Gebrannt wurden jeweils Duplikate. Mit Rohlingen und Geräten der Zeit. Kopieren auf Festplatte ab 2011. Bei über 20 war eines der beiden Duplikate mit mehr oder weniger Lesefehlern behaftet. Bei ca. zehn waren beide Kopien nicht mehr sinnvoll lesbar. Hilfreich war bei der Auslesung der Rückgriff auf alte Brenner bzw. alte oder ausgewählte CD/DVD Laufwerke. Ergo: Ich gebe einer CD/DVD acht Jahre, so man zwei davon hat. Grüße --[[Spezial:Beiträge/80.187.97.115|80.187.97.115]] 20:51, 28. Feb. 2017 (CET)
Guten Tag, ich bin so frei und bitte um Eure Hinweise zu Dingen die eventuell verschwiegen wurden. Wir haben einen Artikel zu [[Leonardo da Vinci]], ja da steht vieles drin - nur wie soll man wissen, was eventuell unterschlagen wurde? Wer mag kann auch mal bei [[Wikipedia:Kandidaturen_von_Artikeln,_Listen_und_Portalen#Leonardo_da_Vinci]] rein schauen. LG --[[Benutzer:Tom|Tom]] ([[Benutzer Diskussion:Tom|Diskussion]]) 19:52, 29. Sep. 2020 (CEST)
:Eine recht merkwürdige Frage. Aber falls es darum geht, den Artikel zu verbessern: Beim Einzelnachweis 29 fehlen die Seitenangaben und ich weiß auch nicht, wie man Erfindungen nobilitieren soll. --[[Benutzer:Xocolatl|Xocolatl]] ([[Benutzer Diskussion:Xocolatl|Diskussion]]) 20:03, 29. Sep. 2020 (CEST)
::''Auch sie ''(Fluggeräte, die Hubschraubern gleichen)'' hätten nicht funktioniert, weil Leonardo hierfür das Prinzip der Archimedischen Schraube vorsah, das im dünnen Medium der Luft nicht funktionieren kann, und nicht das heute angewandte Prinzip des Windmühlenflügels.'' ist natürlich Unsinn. Verdächtig wäre ja schon, dass die Windmühlenflügel angeblasen werden, während der Propeller des Hubschraubers selbst "Wind erzeugt". Hubschrauberpropeller haben ein Profil, das der Trägfläche eines Flugzeugs ähnelt. Dadurch entsteht zusätzlich zu der Kraft durch die nach unten beschleunigte Luft oberhalb des Propellers ein Unterdruck, der den Hubschrauber "hochsaugt". Für eine Windmühle wäre so eine - dann in waagerechte Richtung wirkende - Kraft ja sinnlos, solange man die Mühle nicht umreißen möchte. --[[Benutzer:Optimum|Optimum]] ([[Benutzer Diskussion:Optimum|Diskussion]]) 20:58, 29. Sep. 2020 (CEST)
:::Unsinn? Was hat das ~aerodynamische Prinzip mit der anderen räumlichen Orientierung zu tun? --[[File:Lemon with white background.jpg|15px|Quetsch mich aus, ... ]] [[Benutzer:Itu|itu]] ([[Benutzer Diskussion:Itu|Disk]]) 22:28, 29. Sep. 2020 (CEST)
::::Welche andere räumliche Orientierung? --[[Benutzer:Optimum|Optimum]] ([[Benutzer Diskussion:Optimum|Diskussion]]) 23:06, 29. Sep. 2020 (CEST)


:Bei selbstgebrannten CDs und DVDs kommt es extrem stark auf die Qualität der Rohlinge und den Umgang damit an. Ich hatte schon CD-Rs, die nach zwei Jahren unleserlich wurden, weil minderwertiger Kleber verwendet wurde und die Schicht deswegen Blasen warf. Andere CD-Rs wurden durch minderwertigen Lack und Fingerabdrücke oder säurehaltige Filzstifte unbrauchbar. Seitdem fasse ich CD-Rs nur noch am Mittelloch an und beschrifte sie nur noch im durchsichtigen Bereich rings um das Mittelloch. DVD±Rs haben eine deutlich geschütztere Schicht, sodass sie problemlos angefasst und flächig beschriftet werden können, auch mit [[olein]]<nowiki></nowiki>haltigem 1970er-Jahre-Edding aus dem Altbestand meines Arbeitgebers. --[[user:Rotkaeppchen68|R<span style="color:red">ô</span>tkæppchen₆₈]] 23:36, 28. Feb. 2017 (CET)
:::Guter Tip, Danke. Entschwurbelt mit "Die praktische Tauglichkeit seiner Entwürfe zu Fluggeräten ist nach modernen Erkenntnissen zweifelhaft." Eine Seitennummer für Ref 29 kann ich nicht herbeizaubern. Als Ersatz dafür einen [https://books.google.de/books?id=JE5VDwAAQBAJ&lpg=PT1&dq=Matthias%20Eckoldt%3A%20Leonardos%20Erbe&hl=de&pg=PT9#v=onepage&q=Hubschrauber&f=false Einblick] LG --[[Benutzer:Tom|Tom]] ([[Benutzer Diskussion:Tom|Diskussion]]) 21:48, 29. Sep. 2020 (CEST)
::::Zumindest zu seiner Erfindung des Fahrrads haben wir einen eigenen Artikel: [[Angebliche Fahrradzeichnung Leonardo da Vincis]]. --[[Benutzer:Ralf Roletschek|<span style="color:#000000">M<span style="color:#ff0000">@</span>rcela</span>]] [[Bild:Miniauge2.gif|27px]] 22:18, 29. Sep. 2020 (CEST)
::: Ohne mir jetzt Leonardos Hubschrauberentwurf angesehen zu haben: Die optimierten Profile von WKA- und Hubschrauberblättern sind im Prinzip identisch, und zwar auch mit Tragflächenprofilen von Starrflügelflugzeugen. Der wesentliche Unterschied besteht in der Verwindung: Die Umlaufgeschwindigkeit ist natürlich proportional zum Radius, d. h. in der Nähe der Nabe werden sie tangential wesentlich langsamer angeströmt als an den äußeren Enden. Deswegen ist das Profil dort kürzer, dicker und stärker angestellt, und wegen der über den Radius variierenden Anstellung wird das Blatt insgesamt verwunden gebaut. Profile, die Antriebsleistung in eine Beschleunigung des Fluids transformieren, wie z. B. Schiffspropeller, Hubschrauberblätter oder Flugzeugtragflächen, sind so angestellt, daß die gerundete Vorderkante mehr in Richtung Unterdruckseite - beim Flugzeug "oben" - und die spitze Hinterkante in Richtung Überdruckseite zeigt; dadurch wird das Fluid unter Aufwendung mechanischer Antriebsleistung zur Überdruckseite hin sowie in Bewegungsrichtung des Blatts beschleunigt, letzteres führt zum induzierten Widerstand des Tragprofils. Bei WKA-Blättern ist die Anstellung genau umgekehrt, die von der Überdruckseite her in Achsrichtung anströmende Luft wird dadurch abgebremst und entgegen der Bewegungsrichtung des Profils beschleunigt, und die Reaktionskraft treibt den Windmühlenflügel in Bewegungsrichtung an und erzeugt so die nutzbare mechanische Leistung. (Man kann sich leicht überlegen, daß die zur Nabe hin zunehmende negative Anstellung das WKA-Profil mit einer genau entgegengesetzten Verwindung wie das Hubschrauberblattprofil ausgestaltet.) Leonardo dürfte das aerodynamische Prinzip höchstwahrscheinlich nicht gekannt haben, er hatte wahrscheinlich die Modellvorstellung, daß ein Tragprofil in das Fluid hineinschneidet wie ein Messer in die Butter und sich Auftriebskräfte dabei wie an einem Keil, also nach dem Prinzip der Schiefen Ebene senkrecht zur Bewegungsrichtung ergeben. Das heißt aber nicht, daß eine "Archimedische Schraube" als Tragprofil gänzlich ungeeignet wäre. Auch ebene Profile erzeugen bei Anstellung einen Auftrieb, mit genügend Antriebsleistung fliegt auch ein Scheunentor. Nur wäre eine solche Form sehr ineffizient. Kann man Leonardo nun als Hubschraubererfinder ansehen? Da er die grundsätzliche Anordnung - eine um eine vertikale Achse angetriebene Luftschraube kann Auftrieb erzeugen - erkannt hatte: Ja! Da er das Prinzip des aerodynamischen Auftriebs theoretisch nicht durchdrungen hatte: Nein, denn seine Konstruktion wäre praktisch nicht realisierbar gewesen. --[[Spezial:Beiträge/95.116.103.117|95.116.103.117]] 05:12, 1. Okt. 2020 (CEST)


:: Und wie verhalten sich CDs, insbesondere gepresste, im Vergleich zu Schallplatten? --[[User:Morten Haan|Morten Haan]] &#x1F6E3; [[User:Morten Haan/für Leser|Wikipedia ist für Leser da]] • [[User:Morten Haan/Skin-Entwurf|Skin-Entwurf]] 00:16, 1. Mär. 2017 (CET)
== gibt es "schubladisiert"? ==
Ich bin bei [[Panzer#Vorgeschichte]], drittletzter Absatz drüber gestolpert, wollte reflexartig verändern. Aber das Wort [https://de.wikipedia.org/w/index.php?search=schubladisiert&title=Spezial%3ASuche&go=Artikel&ns0=1 wird mehrmals verwendet]. Bei [[Zeittafel Schweiz im Zweiten Weltkrieg]] erscheint es in Fußnote 39, '' Warum hat Hitler seinenausgefeilten Angriffsplan schubladisiert? Eine historische Recherche. In: Weltwoche. 5. Februar 2003'' - also völlig abwegig scheint es nicht? --[[Benutzer:Ralf Roletschek|<span style="color:#000000">M<span style="color:#ff0000">@</span>rcela</span>]] [[Bild:Miniauge2.gif|27px]] 22:15, 29. Sep. 2020 (CEST)


:::Übliche CDs enthalten Aluminium in der Reflektionsschicht. Durch durch die Lackschicht diffundierenden Sauerstoff oxidiert diese allmählich und die Reflektivität nimmt dadurch ab. Damit wird die CD allmählich unleserlich. Weiterhin droht Gefahr durch säurehaltigen oder sonstwie nicht dauerhaften Lack. Die en-wp hat hierzu den Artikel [[:en:Disc rot]]. Gepresste CDs lassen sich aber theoretisch restaurieren, indem die Lack- und Aluschichten chemisch entfernt und der Polycarbonatpressling danach neu vergoldet oder aluminiumbeschichtet wird. Magnetbänder halten auch nicht ewig. Manche Hersteller sparen am Lack, andere an der Trägerfolie. Ton- oder Videobänder aus [[Celluloseacetat]] mussten schon als Totalverlust abgeschrieben werden. Bei anderen Bändern hat zwar die [[biaxial orientierte Polyester-Folie]] durchgehalten, aber der Lack wurde zur Pampe. Außerdem hängt es davon ab, ob Bandmaschine und Bandmaterial für Profi- oder für Heim- und Amateurzwecke vorgesehen sind. Profiband ist hart, Profimagnetköpfe sind weich. Damit verschleißt im Profibetrieb der billig wiederzubeschaffende Magnetkopf und nicht das Magnetbandunikat. Für den Einsatz zuhause wurden Tonband-, Cassetten- und Videogeräte mit harten, verschleißarmen Köpfen und dafür weiches Bandmaterial verkauft. War das Band ausgenudelt, sollte der Heimnutzer bitte ein neues kaufen. Es wäre dem Heimnutzer ebenso unzumutbar, sein Magnetbandgerät regelmäßig zum Kopftausch in die Werkstatt zu geben. Bei Schallplatten kommt es auf das Trägermaterial an und wie oft die Platte mit welchem Tonabnehmer gespielt wurde. Schellackplatten waren mit Steinmehl gefüllt und die Nadeln waren relativ weich und Verbrauchsmaterial. Bei häufigem Abspielen verschliss also nicht die Platte, sondern die Nadel. Vinylplatten sind mit Ruß gefüllt und relativ weich, die Abspielnadeln sind aus Diamant oder Saphir, also hart. Beim Abspielen verschleißt also die Platte und die Nadel überlebt. Schallplattenunikate bestanden aus Lackfolie. Damit besteht prinzipiell die gleiche Zersetzungsproblematik wie bei Magnetbändern. --[[user:Rotkaeppchen68|R<span style="color:red">ô</span>tkæppchen₆₈]] 00:50, 1. Mär. 2017 (CET)
[https://www.dwds.de/wb/schubladisieren Gibt es, aber selten]. [[Benutzer:Ziko|Ziko]] ([[Benutzer Diskussion:Ziko|Diskussion]]) 22:18, 29. Sep. 2020 (CEST)
::Gemäß online-Duden scheint es das tatsächlich zu geben "schwaches Verb, besonders österreichisch, schweizerisch umgangssprachlich" - wie so oft, wieder mal was gelernt... --[[Spezial:Beiträge/2A02:908:2D12:8BC0:FDD4:3B69:B677:D3D7|2A02:908:2D12:8BC0:FDD4:3B69:B677:D3D7]] 22:20, 29. Sep. 2020 (CEST)
:::Und was nicht gibt kann erfunden werden! Das gibt immer! --[[File:Lemon with white background.jpg|15px|Quetsch mich aus, ... ]] [[Benutzer:Itu|itu]] ([[Benutzer Diskussion:Itu|Disk]]) 22:22, 29. Sep. 2020 (CEST)
::::Wie 2A02..., wieder was gelernt. --[[Benutzer:Ralf Roletschek|<span style="color:#000000">M<span style="color:#ff0000">@</span>rcela</span>]] [[Bild:Miniauge2.gif|27px]] 22:44, 29. Sep. 2020 (CEST)
:Bei uns in AT ein durchaus üblicher Ausdruck. --[[Benutzer Diskussion:TheRunnerUp|TheRunnerUp]] 23:11, 29. Sep. 2020 (CEST)
::Meinung eines Schweizers: Zwar üblich, aber eher, wie der Duden schreibt, umgangssprachlich bzw. salopp-journalistisch. Ausser in Zitaten würde ich den Ausdruck hier in der Wikipedia eher nicht verwenden. [[Benutzer:Gestumblindi|Gestumblindi]] 23:22, 29. Sep. 2020 (CEST)
:::Sehr richtig, s. [[WP:GUT#Standardsprache statt Umgangssprache]]. --[[Benutzer:Vsop|Vsop]] ([[Benutzer Diskussion:Vsop|Diskussion]]) 23:27, 29. Sep. 2020 (CEST)
: Das BVerfG erlaubt es in seinen diversen Entscheidungen zur Rechtschreibreform Privatpersonen, von der amtlichen Rechtschreibung abzuweichen und orthographisch wie sprachlich kreativ tätig zu werden (alles andere wäre auch Blödsinn, denn sonst hätte einer der größten deutschsprachigen Dichter, Ernst Jandl, ja gar keine Daseinsberechtigung). An die amtliche Rechtschreibung müssen sich also nur Schulen und Behörden halten. Wenn das Verb „schubladisieren“ erfunden ist, dann existiert es augenscheinlich. Fragt sich, ob sich der Gebrauch durchsetzen oder weitere Verbreitung finden wird. Aber das zeigt sich ja dann. --[[Benutzer:Heletz|Heletz]] ([[Benutzer Diskussion:Heletz|Diskussion]]) 08:49, 30. Sep. 2020 (CEST)
::Die Frage hat doch gar nichts mit Rechtschreibung zu tun. --[[user:Rotkaeppchen68|R<span style="color:red">ô</span>tkæppchen₆₈]] 10:35, 30. Sep. 2020 (CEST)
:: Die deutschsprachige Wikipedia unterwirft sich einigermaßen freiwillig dem Duden. --[[Benutzerin:Ailura|Ailura]] ([[Benutzerin Diskussion:Ailura|Diskussion]]) 13:22, 30. Sep. 2020 (CEST)
::: Nun ist der Duden für die deutsche Sprache in keiner Weise maßgeblich. --[[Spezial:Beiträge/95.116.103.117|95.116.103.117]] 05:31, 1. Okt. 2020 (CEST)
::::Außerdem ist der Duden nicht für Wortfindung bzw Wortbildung zuständig, sondern für Rechtschreibung. Der Duden stellt nicht etwa eine Positivliste amtlich genehmigter Wörter dar. Es gibt noch unzählige andere Wörter, die es nicht in den Duden geschafft haben und deren Benutzung ebenso legal, möglich und sinnvoll ist. --[[user:Rotkaeppchen68|R<span style="color:red">ô</span>tkæppchen₆₈]] 07:15, 1. Okt. 2020 (CEST)
:: Da die Confoederatio Helvetica bedauerlicherweise immer noch nicht um staatsrechtlichen Anschluß an das Deutsche Reich nachgesucht hat - nicht einmal die deutschsprachigen Kantone - konnte der Deutsche Bund dieser so sehnlich zu wünschenden Entwicklung auch nicht stattgegeben, und deswegen besteht nun einmal immer noch keine Zuständigkeit des BVerfG für sprachliche Angelegenheiten in der Schweiz. Derzeit kann eine einheitliche staatliche Umgangsweise mit Angelegenheiten der deutschen Sprache nur durch zwischenstaatliche, völkerrechtlich bindende Vereinbarungen herbeigeführt werden. --[[Spezial:Beiträge/95.116.103.117|95.116.103.117]] 05:31, 1. Okt. 2020 (CEST)
::: Der [[Libyen-Affäre_(Schweiz)#Militärische_Drohungen_und_Verlangen|Gaddafi-Vorschlag von 2009]] wurde sogleich schubladisiert. --[[Benutzer:Aalfons|Aalfons]] ([[Benutzer Diskussion:Aalfons|Diskussion]]) 12:53, 1. Okt. 2020 (CEST)
:::: In diesem Zusammenhang erscheint mir der Begriff eher unpassend, da die Nicht-Umsetzung nicht in der Hand des Planers lag. <small>(Und außerdem: Wo er recht hat...)</small> --[[Spezial:Beiträge/95.116.103.117|95.116.103.117]] 14:31, 1. Okt. 2020 (CEST) PS. Offenbar wurde - mal wieder - das Liechtenstein-Problem übersehen. Dieses Gebilde verdankt seine eigenartige Existenz doch überhaupt lediglich dem Umstand, daß es bereits beim Wiener Kongreß vollkommen vergessen wurde. --[[Spezial:Beiträge/95.116.103.117|95.116.103.117]] 14:31, 1. Okt. 2020 (CEST)
::::: Schubladisieren können alle, denen ein Vorschlag vorgelegt wird. --[[Benutzer:Aalfons|Aalfons]] ([[Benutzer Diskussion:Aalfons|Diskussion]]) 17:39, 1. Okt. 2020 (CEST)
: In den schweizer Medien wird das doch dauernd verwendet. Aber wie [https://books.google.com/ngrams/graph?content=schubladisiert&year_start=1920&year_end=2019&corpus=31&smoothing=3 aus dem Ngram zu erkennen] ist, war es in den 1980er und 1990er Jahren ein ausgesprochenes Modewort. --[[Benutzer:Aalfons|Aalfons]] ([[Benutzer Diskussion:Aalfons|Diskussion]]) 13:00, 30. Sep. 2020 (CEST)


:::Die PVC-Schallplatte an sich hält bei nicht ganz falscher Lagerung ewig. Wie Rotkaeppchen68 aber schon schrieb, verschleißt sie bei jedem Abspielvorgang. Aber eine Platte, die du heute einlagerst, können deine Ururururenkel noch abspielen, sofern sie geeignete Abspielhardware haben (und da gibt es bekanntlich sehr einfache Lösungen, da bei der Abtastung bereits die mechanische Schwingung entsteht und nichts mehr gewandelt werden muss – es reicht ja, eine Ecke einer Postkarte in die rotierende Platte zu hängen). Eine Untersuchung des Instituts für Rundfunktechnik (Nürnberg) aus den 90ern kam auch zu dem Schluss, dass sich das Nadeltonverfahren mit weitem Abstand am besten zur Langzeitarchivierung eignet – erstens wegen der Langlebigkeit des Trägers, zweitens wegen des technisch einfachen Abspielens. --[[Benutzer:Kreuzschnabel|Kreuz]][[Benutzer Diskussion:Kreuzschnabel|schnabel]] 11:12, 1. Mär. 2017 (CET)
::Achtung, Rechtschreibung ;-) - geographische Adjektive auf -er wie "Hamburger" oder eben auch "Schweizer" werden gross geschrieben. [[Benutzer:Gestumblindi|Gestumblindi]] 13:29, 30. Sep. 2020 (CEST)
::: Schprache lebt. --[[Benutzer:Aalfons|Aalfons]] ([[Benutzer Diskussion:Aalfons|Diskussion]]) 13:31, 30. Sep. 2020 (CEST)
:::: Jau, schon bei Tucholsky war mir aufgefallen, dass er diese immer klein geschrieben hat... [[Benutzer:Gestumblindi|Gestumblindi]] 13:35, 30. Sep. 2020 (CEST)


::::Ihr erinnert euch doch bestimmt an Scherben im CD/DVD-Laufwerk? Eine CD/DVD enthält auch Weichmacher. Taugen die nichts mehr oder sind verdampft, war es das. Da kann das Alu ewig halten, wenn das Trägermaterial nichts mehr taugt. Ist der Bruch da, ist man auf Mikroskope und Bildauswertung und Prüfsummen angewiesen, falls das jemand schon je so gebaut hat. --<span style="color:#00A000;">Hans Haase ([[BD:Hans Haase|有问题吗]])</span> 14:07, 1. Mär. 2017 (CET)
Aber was soll schubladisiert bedeuten? Entspricht es der "Rundablage"? Oder wird jemand in eine mehr oder weniger passende Schublade eingeordnet? --[[Benutzer:Bahnmoeller|Bahnmoeller]] ([[Benutzer Diskussion:Bahnmoeller|Diskussion]]) 13:20, 1. Okt. 2020 (CEST)
::::Bei Festplatten altern Bauteile und Schnittstellen werden obsolet. --<span style="color:#00A000;">Hans Haase ([[BD:Hans Haase|有问题吗]])</span> 14:08, 1. Mär. 2017 (CET)
:Danke für die Nachfrage! Ich hatte bis eben auch gedacht, es handelt sich um [[Schubladendenken]], aber vermutlich meint es, etwas in der [[Ignoranz|Schublade verschwinden]] zu lassen... --[[Benutzer:Duckundwech]] 15:36, 1. Okt. 2020 (CEST)
:::::@„Eine CD/DVD enthält auch Weichmacher.“ Dafür hätte ich gerne entweder eine Entschuldigung für die Falschaussage oder einen validen Beleg. --[[user:Rotkaeppchen68|R<span style="color:red">ô</span>tkæppchen₆₈]] 15:19, 1. Mär. 2017 (CET)
:::::--[[Spezial:Beiträge/80.187.96.246|80.187.96.246]] 15:26, 1. Mär. 2017 (CET)Scherben im CD/DVD-Laufwerk? Eine CD/DVD enthält auch Weichmacher. Obwohl jahrzehnte Erfahrung: 1. Da war vielleicht mal was? Hilfts Du mir auf die Sprünge? 2. Keine Ahnung - nie gelesen. --[[Spezial:Beiträge/80.187.96.246|80.187.96.246]] 15:26, 1. Mär. 2017 (CET)


:::: Also ist es nicht nur aus [[Nostalgie|nostalgischen]] Gründen sinnvoll, [[Album (Musik)|Alben]] als Schallplatten herauszubringen sowie zu kaufen. Wie sieht es bei [[Solid-State-Drive]]s aus, gibt es dazu schon Daten? --[[User:Morten Haan|Morten Haan]] &#x1F6E3; [[User:Morten Haan/für Leser|Wikipedia ist für Leser da]] • [[User:Morten Haan/Skin-Entwurf|Skin-Entwurf]] 20:48, 1. Mär. 2017 (CET)
Englisch: ''tabled''. In entweder der amerikanischen oder englischen Version der nordseestämmigen Sprache soll dass mal bedeutet haben, was a) auf den Tisch zu legen, oder b) in der Schublade zu verstecken. Das soll dann sogar mal während des WKI für Missverständnisse gesorgt haben. Ich nehme an, der wenig elegante Deutschismus "schubladisiert" hat hier seinen Ursprung. Ansonsten: die deutsche Sprache wird von "cunning linguists" grundsätzlich für Flexibilität und Erfindungsreichtum gelobt. [[Benutzer:Oalexander|OAlexander]] ([[Benutzer Diskussion:Oalexander|Diskussion]]) 17:22, 1. Okt. 2020 (CEST)
: Für deine Bedeutung (b) habe ich keinen Beleg gefunden, und sie kommt mir auch uneinleuchtend vor, weil sie das Gegenteil von (a) ist. Im [[Oxford_English_Dictionary|OED]] 2 (1989), wer sollte es besser wissen als die, unter den metaphorischen Verwendungen von ''tabled'' nur "entered on a list; listed". --[[Benutzer:Aalfons|Aalfons]] ([[Benutzer Diskussion:Aalfons|Diskussion]]) 17:49, 1. Okt. 2020 (CEST)
::Schau mal unter [[:en:wikt:table#Verb]] nach, dort die Punkte 4, 5 und 7. --[[user:Rotkaeppchen68|R<span style="color:red">ô</span>tkæppchen₆₈]] 17:57, 1. Okt. 2020 (CEST)
::: Tatsächlich. Und [[:en:Table_(parliamentary_procedure)|einleuchtend erklärt]]. --[[Benutzer:Aalfons|Aalfons]] ([[Benutzer Diskussion:Aalfons|Diskussion]]) 18:17, 1. Okt. 2020 (CEST)
::::Ein deutsches Wort mit gegensätzlichen Bedeutungen ist z.B. "sanktionieren". --[[Benutzer:Digamma|Digamma]] ([[Benutzer Diskussion:Digamma|Diskussion]]) 19:59, 1. Okt. 2020 (CEST)
So kann man bei WP den eigenen Wortschatz erweitern, das Wort kannte ich noch gar nicht, finde ich aber toll. Vielleicht lassen sich analoge Verben für nützliche Zwecke kreieren. --[[Benutzer:Dioskorides|Dioskorides]] ([[Benutzer Diskussion:Dioskorides|Diskussion]]) 22:22, 1. Okt. 2020 (CEST)
:Ich kenne das Wort für beide Bedeutungen. Das Österreichische Wörterbuch sagt dazu: "schubladieren = schubladisieren (scherz.); einen Akt s.: seine Erledigung auf unbestimmte Zeit aufschieben." Bedeutung (b) ist [https://www.kunst-worte.de/kunstworte/kunstwort-2396-schubladieren/ hier] beschrieben und findet sich z.B. im Titel eines Bildes von [[Martin Kippenberger]]: "Eifrau, die man Nicht schubladieren kann (Egg women who defies categorisation)"[http://www.artnet.de/k%C3%BCnstler/martin-kippenberger/eifrau-die-man-nicht-schubladieren-kann-egg-women-Ul8kybsjG25DNWcgg45pyw2] oder [https://www.falter.at/zeitung/20030723/sexed-up/1855230005?ver=a hier] im Satz "Unbequeme Wahrheiten, die dem Land von außen vorgehalten werden, schubladiert man regelmäßig als "antisemitisch"." Liebe Grüße, --[[Benutzerin:Haeferl|Häferl]] ([[Benutzerin Diskussion:Haeferl|Diskussion]]) 23:42, 1. Okt. 2020 (CEST)
:Und so gegensätzlich sind die Bedeutungen gar nicht. Wird jemand durch jemand anderen schubladisiert, geschieht das selten im positiven Sinn, ergo hat auch Bedeutung (b) die Folge, dass man sich mit dem Schubladisierten oder dessen Meinung nicht mehr befasst. Liebe Grüße, --[[Benutzerin:Haeferl|Häferl]] ([[Benutzerin Diskussion:Haeferl|Diskussion]]) 23:47, 1. Okt. 2020 (CEST)


:::::SSDs sind auf Geschwindigkeit und Speicherplatzdichte optimiert, nicht auf Dauerhaftigkeit. EEPROMs, um die es sich bei SSDs schlussendlich handelt, halten ihre Ladung bzw Daten nicht ewig. Irgendwann vermindert sich die Ladung auf dem floating gate so stark, dass die darauf gespeicherte Information hin ist. Das hängt von der Einwirkung von Wärme und ionisierender Strahlung ab, wobei auch die [[Hintergrundstrahlung]]/[[Nulleffekt]] etwas ausmacht. Bei MLC-Flash geht das schneller als bei SLC-Flash. Magnetische Festplatten sind da nicht empfindlich. Magnetische Festplatten können allerdings anderweitig altern, beispielsweise durch degradierendes Schmiermittel, Schutzgasverlust, Oxidation, versprödende Gummi- und Kunststoffteile. Besser wäre es, nur die eigentlichen Magnetplatten und dazu einen Bauplan für das Laufwerk aufzubewahren. Da die langzeitarchivierten Daten aber ohnehin nur gelesen und nicht modifiziert oder gelöscht werden sollen, sind wiederbeschreibbare Speicher nicht notwendig. WORM-Medien wie DVD±R, CD-R, BD-R, M-Disc sind da besser geeignet. Bei diesen Medien ist eine Trennung von Laufwerk und eigentlichem Medium üblich, sodass hier die Gefahr der Unlesbarkeit wegen Ausfalls der alten Laufwerkstechnik unwahrscheinlich ist. Ich habe hier über 25 Jahre alte VHS-Bänder, die noch spielbar sind, aber nicht mehr den ursprünglichen VHS-Recorder. --[[user:Rotkaeppchen68|R<span style="color:red">ô</span>tkæppchen₆₈]] 23:37, 1. Mär. 2017 (CET)
== Lied/Musikvideo gesucht ==


:::::: Das heißt also bei Tonträgern ist die PVC-Schallplatte am besten, bei Datenträgern die gepresste [[Blu-ray Disc]] (höhere Kapazität als CD und DVD). Ist das so richtig zusammengefasst? --[[User:Morten Haan|Morten Haan]] &#x1F4BF; [[User:Morten Haan/für Leser|Wikipedia ist für Leser da]] • [[User:Morten Haan/Skin-Entwurf|Skin-Entwurf]] 16:03, 2. Mär. 2017 (CET)
Servus, ich suche ein Lied aus den 1990ern, war irgendwas elektronisches. Das Einzige, an was ich mich tatsächlich erinnere, ist, dass in dem Musikvideo in ner Morgue/Leichenhalle verschiedene Tote lagen, an einer war das Etikett "German Techno". Zusätzlich lag glaube ich noch nen Typ an nem Baum, dem Blut aus den Ohren lief. --<span style="font-family: Segoe Script; font-size:12px;">[[Benutzer:Odeesi|Odeesi]] [[Benutzer_Diskussion:Odeesi|<sub>talk to me</sub>]] [[Benutzer:Odeesi/Bewertung|<sup>rate me</sup>]]</span> 23:23, 29. Sep. 2020 (CEST)


:::::::Wie es mit gepressten DVDs, HD-DVDs und BDs ausschaut, weiß ich nicht. Dort befindet sich die Datenschicht im Innern der zusammengeklebten Schichten. Bei der gepressten CD ist nichts zusammengeklebt, sondern der Pressling wird mit Aluminium (meistens) oder Gold (selten) beschichtet und dann lackiert. Den Lack kriegt man wieder ab, wenn er die Metallschicht versaut. Zusammengeklebte Schichten bekommt man wahrscheinlich nicht mehr getrennt. Wenn DVD oder BD, dann bitte nur einschichtig und mit Gold als Reflektionsschicht, also DVD-5 oder BD-25. --[[user:Rotkaeppchen68|R<span style="color:red">ô</span>tkæppchen₆₈]] 23:37, 2. Mär. 2017 (CET)
= 30. September =


:::::::: Eine BD mit 25GB ist ja schon mal nicht wenig, darauf kann man Video-, Text-, Bild- und Musikdateien sowie beliebige andere Formate speichern. --[[User:Morten Haan|Morten Haan]] &#x1F4BF; [[User:Morten Haan/für Leser|Wikipedia ist für Leser da]] • [[User:Morten Haan/Skin-Entwurf|Skin-Entwurf]] 17:18, 3. Mär. 2017 (CET)
== US-Proxies gehen bei Google Books nicht mehr ==


== [[Nasser Grund]] ... veräppelt Wikipedia den Leser ? ==
In den letzten Tagen konnte ich keine US-Proxies bei Google Books nutzen. Zenmate Free funktioniert mit HathiTrust ordnungsgemäß, zeigt aber bei Books die Version ohne Proxy an. Die getesteten kostenlosen Webproxies lassen im Hintergrund die Vollansicht erkennen, im Vordergrund lädt aber irgendetwas und ein weißes Feld erscheint, das alle weiteren Links verhindert. Auf die Einzelseite kann ich mit dem Analysetool (Chrome: Element untersuchen) zugreifen, aber gibt es einen Weg, ans PDF zu kommen oder eine andere Problemlösung? Einige von euch haben kostenpflichtige Proxies - wie sieht das da aus? Danke

Guten Tag. Ich hab den Artikel [[Nasser Grund]] gelesen. Was inhaltlich zu Gewässerschichtungen beschrieben wird kommt mir plausibel vor. Von dem Lemma hab ich allerdings noch nie etwas gehört. "Nasser Grund" dort wo sowieso schon alles übernässt ist? Will da jemand den Leser veräppeln? Kennt jemand diesen Begriff oder einen Besseren für diese [[Dichteanomalie]] die auch in [[Thermische Schichten]] beschrieben wird?--[[Benutzer:Tom|Tom]] ([[Benutzer Diskussion:Tom|Diskussion]]) 08:24, 28. Feb. 2017 (CET)
: Wenn ich das richtig lese, geht es um die Bezeichnung, die die Marine (angeblich) für das Phänomen benutzt, deswegen auch der Zusammenhang mit U-Booten. Die Quelle muss man wohl in Papierform einsehen. --[[Benutzer:Ailura|Ailura]] ([[Benutzer Diskussion:Ailura|Diskussion]]) 08:32, 28. Feb. 2017 (CET)
:: Habe ich mitmal das Suchen im Internet verlernt oder ist darüber tatsächlich rein gar nichts zu finden? VG --[[Benutzer:Apraphul|Apraphul]] <small><sup> [[Benutzer Diskussion:Apraphul|Disk]] </sup> <sub> [[Benutzer:Apraphul/WP:SNZ|WP:SNZ]]</sub></small> 08:43, 28. Feb. 2017 (CET)
:::Vielleicht wäre es besser, einen Löschantrag zu stellen. --[[Benutzer:Schlesinger|Schlesinger]] [[Benutzer Diskussion:Schlesinger|<sup>schreib!</sup>]] 08:46, 28. Feb. 2017 (CET)
::::Bis auf den Begriff "Nachbar"schichten stimmt es. Es geht um verschiedene Tiefen. --[[Benutzer:Ralf Roletschek|M@rcela]] [[Bild:Miniauge2.gif|27px]] 08:48, 28. Feb. 2017 (CET)
:::::Na, dann wirst du uns gleich einen Beleg dafür liefern und in den Artikel einfügen. Dir liegt diese"Marine-Enzyklopädie A bis Z 2003" aus dem Tosa-Verlag, eher bekannt für Karl May, vor? --[[Benutzer:Schlesinger|Schlesinger]] [[Benutzer Diskussion:Schlesinger|<sup>schreib!</sup>]] 08:52, 28. Feb. 2017 (CET)
::::::Wenn nicht, gibt es diesen Schmöker <nowiki>ISBN 3854927576</nowiki> gebraucht für 5 euro bei ZVAB.com.--[[user:Rotkaeppchen68|R<span style="color:red">ô</span>tkæppchen₆₈]] 09:05, 28. Feb. 2017 (CET)
:::::::Ich habe Löschantrag gestellt.--[[Benutzer:Meloe|Meloe]] ([[Benutzer Diskussion:Meloe|Diskussion]]) 09:08, 28. Feb. 2017 (CET)
::::::::Na dann hat sich ja die Quellensuche erledigt. --[[Benutzer:Ralf Roletschek|M@rcela]] [[Bild:Miniauge2.gif|27px]] 09:18, 28. Feb. 2017 (CET)
:::::::::Merkwürdige Auffassung. Hindert dich jemand daran, mit neuen, dann aber seriösen Quellen, einen besseren Text zu verfassen, der wenigstens ansatzweise die Zusammenhänge erklärt? --[[Benutzer:Schlesinger|Schlesinger]] [[Benutzer Diskussion:Schlesinger|<sup>schreib!</sup>]] 09:22, 28. Feb. 2017 (CET)
::::::::::Ja, destruktives Verhalten. --[[Benutzer:Ralf Roletschek|M@rcela]] [[Bild:Miniauge2.gif|27px]] 10:16, 28. Feb. 2017 (CET)
:::::::::::Destruktiv wäre sofortiges Löschen gewesen, du kannst dich konstruktiv in der LD beteiligen. --[[Spezial:Beiträge/84.129.180.227|84.129.180.227]] 10:40, 28. Feb. 2017 (CET)
::::::::::: (BK) Naja, den LA hätte ich jetzt auch nicht gestellt und auch nicht schon jetzt erwartet. So wirklich schlecht begründet ist er allerdings nicht, so dass ich das nicht als destruktives Verhalten bezeichnen würde. Wenn einem der LA nicht gefällt, könnte man dann ja erst recht Belege suchen, um dem LA-Steller den LA ggf. um die Ohren zu hauen. ;-) Zurück zum Thema: Ich würde, selbst bei völligem Fehlen des Begriffs im Internet, nicht behaupten wollen und schon gar nicht beweisen können, dass es den Begriff nie gegeben hat. Aber auch Begriffe und Ausdrücke sollten eine gewisse Relevanz besitzen, um einen Wiki-Artikel zu haben; wir sind ja kein Wörterbuch. Eine Relevanz/Gebräuchlichkeit/Bekanntheit müsste sich aber doch irgendwo im Internet widerspiegeln. Das scheint nicht der Fall zu sein. VG --[[Benutzer:Apraphul|Apraphul]] <small><sup> [[Benutzer Diskussion:Apraphul|Disk]] </sup> <sub> [[Benutzer:Apraphul/WP:SNZ|WP:SNZ]]</sub></small> 10:47, 28. Feb. 2017 (CET)
::::::::::::Als LA-Steller: Wer mag, möge einen Artikel akustische Schattenzone anlegen, der wäre belegbar. Mag sein, mag nicht sein, dass das im Seemannsjargon auf Ubooten mal "Nasser Grund" genannt wurde. Sofern die genannte Quelle der einzige Beleg dafür wäre, wäre dass auch dann nicht relevant, wenn es zutreffen würde. Dazu müsste der Ausdruck schon eine gewisse Verbreitung erlangt haben, was zu beweisen wäre. Da das Phänomen existiert, kann jeder, der es haben will, dazu, unter neuem Lemma, einen neuen Artikel anlegen. Ich wüsste nicht, was der existierende Artikel außer dem möglichen Beleg einer Wortverwendung in einem Soldatenjargon, dazu beizutragen hätte.--[[Benutzer:Meloe|Meloe]] ([[Benutzer Diskussion:Meloe|Diskussion]]) 11:05, 28. Feb. 2017 (CET)
:::::::::::::Im Lehrbuch der "Navigation auf Seewasserstraßen" von 1974 existiert kein Seemannsjargon. Aber ist ja erfolgreich eliminiert. Erstmal Löschen, so kommen wir sicher weiter. --[[Benutzer:Ralf Roletschek|M@rcela]] [[Bild:Miniauge2.gif|27px]] 12:22, 28. Feb. 2017 (CET)
:::::::::::::: Es ist ''gar nichts'' gelöscht worden. Das betreffende Artikelchen ist in eine BKS umgewandelt worden und dessen ursprünglicher Inhalt befindet sich jetzt in der [[Liste seemännischer Fachwörter (N bis Z)]], für den Leser auffindbar über die BKS. Du kannst dich also wieder entspannen und dir ein trockenes Höschen anziehen... --[[Benutzer:Gretarsson|Gretarsson]] ([[Benutzer Diskussion:Gretarsson|Diskussion]]) 13:18, 28. Feb. 2017 (CET)

:@all: Herzliches Dankeschön für die Aufklärung dieser nassen Angelegenheit. Grüße --[[Benutzer:Tom|Tom]] ([[Benutzer Diskussion:Tom|Diskussion]]) 15:44, 1. Mär. 2017 (CET)

== [[Gottfried Wilhelm Leibniz]] ==

Ich will ein paar Werke von Leibniz lesen. Hat hier jemand Vorschläge, womit ich am besten anfange? Zum Hintergrund: Ich studiere Mathematik und bin über Gödel auf Leibniz gestoßen, und möchte nun wissen, was ihn an Leibniz so fasziniert hat. --[[Spezial:Beiträge/132.230.195.232|132.230.195.232]] 09:40, 28. Feb. 2017 (CET)
:Als Mathematiker könnten Dich wohl die [[Gottfried_Wilhelm_Leibniz#Monadentheorie|Monaden]] interessieren. --[[Benutzer:Elrond|Elrond]] ([[Benutzer Diskussion:Elrond|Diskussion]]) 11:06, 28. Feb. 2017 (CET)
::Ach ja, weil Du Gödel ansprachst, dann sind vielleicht auch seine Arbeiten zur [[Gottfried_Wilhelm_Leibniz#Logik|Logik]] von Interesse. --[[Benutzer:Elrond|Elrond]] ([[Benutzer Diskussion:Elrond|Diskussion]]) 11:08, 28. Feb. 2017 (CET)
:Auch Russell interessierte sich für Leibniz. Interessant ist wohl seine Darstellung der Logik, über die ich mal gelesen habe, indem die Prämissen Primzahlen waren und die Schlussfolgerung die Multiplikation aus diesen. <small>(''nicht [[Hilfe:Signatur|signierter]] Beitrag von'' [[Spezial:Beiträge/188.101.73.75|188.101.73.75]] ([[Benutzer Diskussion:188.101.73.75|Diskussion]])<nowiki/> 22:02, 28. Feb. 2017 (CET))</small>

== Speicherdauer von mehr oder minder öffentlichen Überwachungskamera-Aufnahmen ==

Wie lange kann man damit rechnen, dass die Bilder von ''mehr oder minder'' öffentlichen Überwachungskameras in Österreich gespeichert werden? Im konkreten Falle geht es um eine Kamera bei der Österreichischen Post, die möglicherweise vor ca. zwei Wochen ein Vergehen gefilmt hat, und um Zeitdruck für das Einleiten von Schritten (das müsste über den Filialleiter beim Sicherheitsdienst der Österreichischen Post beantragt werden). Das "mehr oder minder" soll einerseits zum Ausdruck bringen, dass die entsprechende Kamera zwar in den Räumichkeiten der Postfiliale ist, aber in einem 24 Stunden lang zugänglichen Bereich; andererseits, dass die Post mittlerweile ein privatisiertes Unternehmen ist, aber zweifelsohne doch noch geprägt von ihren früheren staatlichen Strukturen und weiterhin mit großer öffentlicher Bedeutung. --[[Benutzer:Anonyme Frage|AF]] ([[Benutzer Diskussion:Anonyme Frage|Diskussion]]) 10:48, 28. Feb. 2017 (CET)
:Aufnahmen müssen normalerweise nach 72 Stunden gelöscht werden. --[[Benutzer:Schaffnerlos|Schaffnerlos]] ([[Benutzer Diskussion:Schaffnerlos|Diskussion]]) 10:53, 28. Feb. 2017 (CET)
::Im Ernst? Was ist die entsprechende Vorschrift? --[[Benutzer:Anonyme Frage|AF]] ([[Benutzer Diskussion:Anonyme Frage|Diskussion]]) 12:39, 28. Feb. 2017 (CET)
:::DSG-Novelle 2010 § 50b (2). Eine längere Aufbewahrungsdauer ist möglich, aber zu begründen. Sicherheitsbehörden dürfen überhaupt nur 48 Stunden speichern, die Wiener Linien zum Beispiel halten es genauso. --[[Benutzer:Schaffnerlos|Schaffnerlos]] ([[Benutzer Diskussion:Schaffnerlos|Diskussion]]) 12:54, 28. Feb. 2017 (CET)
::::Ok, danke schön. --[[Benutzer:Anonyme Frage|AF]] ([[Benutzer Diskussion:Anonyme Frage|Diskussion]]) 15:58, 28. Feb. 2017 (CET)

== Nutzerdaten in Bibliotheken archiviert ==

Hallo allerseits,

werden in Universitäts- und Stadtbibliotheken Daten darüber archiviert, wer wann welches Buch ausgeliehen hat? Wenn ja, gibt es diese Praxis schon lange? Kann man so beispielsweise (in Einzelfällen) irgendwie herausfinden, welche Bücher (willkürliches Beispiel) Gottfried Benn mal aus der Universitätsbibliothek ausgeliehen hatte? Danke im Voraus, besten Gruß :) --[[Spezial:Beiträge/80.132.174.44|80.132.174.44]] 12:09, 28. Feb. 2017 (CET)

:Früher gab es in Büchern die Leihkarten, wo Entleiher, Leihdatum und Rückgabedatum vermerkt waren (die Karten wurde bei der Entleihe aus dem Buch genommen und in einen Karteikasten gesteckt). Die EDV-Speicherung wird die Sache vereinfachen und bestimmt werden die Daten nicht gleich bei Rückgabe gelöscht, allein aus dem Grund, dass man eine erst später entdeckte Beschädigung ggf. nachvollziehen kann. Über die Speicherdauer ist mir aber nichts bekannt, irgendwo wird der Datenschutz da Grenzen setzen. --[[Benutzer:Tsungam|Magnus]] [[BD:Tsungam|(Diskussion)]] 12:16, 28. Feb. 2017 (CET)

:: Man müsste dann irgend eine Einverständniserklärung unterzeichnet haben, auf der man darauf hingewiesen wird, dass die Daten elektronisch blablablabla... Du musst also nur einmal hingehen und die die Bedingungen ansehen. [[Benutzer:Yotwen|Yotwen]] ([[Benutzer Diskussion:Yotwen|Diskussion]]) 12:28, 28. Feb. 2017 (CET)
::::Das passiert üblicherweise beim Einschreiben, dass man sich mit irgendwelchen Sachen einverstanden erklärt. Wird wie immer beim Kleingeschriebenen nicht gelesen und somit nicht wahrgenommen. --[[Benutzer:Elrond|Elrond]] ([[Benutzer Diskussion:Elrond|Diskussion]]) 13:33, 28. Feb. 2017 (CET)
::: Natürlich, die guten alten Leihkarten! Ich danke euch. --[[Spezial:Beiträge/80.132.174.44|80.132.174.44]] 12:30, 28. Feb. 2017 (CET)

In Deutschland könnte Gottfried Benn eine Anfrage nach dem Informationsfreiheitsgesetz des jeweiligen Bundeslandes stellen, dann sollte er Auskunft über die zu seiner Person im System der Bibliothek gespeicherten Daten erhalten. --[[Benutzer:Jaroslaw Jablonski|Jaroslaw Jablonski]] ([[Benutzer Diskussion:Jaroslaw Jablonski|Diskussion]]) 13:51, 28. Feb. 2017 (CET)

:Ich nehme mal an, es geht um [[Gottfried Benn|diesen Gottfried Benn]]. Der kann keine Anfragen mehr stellen. --[[Benutzer:Digamma|Digamma]] ([[Benutzer Diskussion:Digamma|Diskussion]]) 14:27, 28. Feb. 2017 (CET)
::Nun gut, der sicher nicht, falls er gemeint gewesen sein sollte. --[[Benutzer:Jaroslaw Jablonski|Jaroslaw Jablonski]] ([[Benutzer Diskussion:Jaroslaw Jablonski|Diskussion]]) 14:35, 28. Feb. 2017 (CET)
:::Uiuiui. Wieder mal viel Unwissen unterwegs. Wir reden zunächst um die Zeit zwischen sagen wir 1905 und 1956. Da wäre zunächst zu klären, welche Bücherei es war. Zweitens, wie war der Verleihmodus. Irgendwelcher elektronischer Kram scheidet da aus. Ob Bibs Nutzerkarteikarten aus dieser Zeit mittlerweile eingescannt haben, wäre zu klären. Fraglich aber ist , ob solche Karteikarten überhaupt archiviert werden. Warum sollten Bibs das tun? Ob man zu Benns Zeiten schon Nutzererklärungen im datenschutzrechtlichen Sinne unterschrieb, wage ich überdies zu bezweifeln. Vielleicht vorher mal mit dem Fragegegenstand befassen und das Problem im zeitlichen Kontext sehen, da klärt sich vieles von allein.--[[Benutzer:Scialfa|scif]] ([[Benutzer Diskussion:Scialfa|Diskussion]]) 14:44, 28. Feb. 2017 (CET)
::::Kein Grund, hier rumzugiften. --[[Benutzer:Jaroslaw Jablonski|Jaroslaw Jablonski]] ([[Benutzer Diskussion:Jaroslaw Jablonski|Diskussion]]) 14:51, 28. Feb. 2017 (CET)
Jeder hat so seine Wahrnehmung. Oder möchtest du mir erzählen, das Benn was mit dem Informationsfreiheitsgesetz anfangen kann?--[[Benutzer:Scialfa|scif]] ([[Benutzer Diskussion:Scialfa|Diskussion]]) 15:14, 28. Feb. 2017 (CET)
:Benn war mir nicht bekannt, sonst hätte ich auf das Informationsfreiheitsgesetz nicht hingwiesen. Den Oberlehrer kannst du stecken lassen, dafür bin ich mittlerweile zu alt. --[[Benutzer:Jaroslaw Jablonski|Jaroslaw Jablonski]] ([[Benutzer Diskussion:Jaroslaw Jablonski|Diskussion]]) 15:20, 28. Feb. 2017 (CET)
::Ich schätze, dass man von Anfang an bis in die erste Hälfte des 20. Jahrhunderts [[Ausleihbuch|Ausleihbücher]] führte, die man natürlich aufhob, da sich darin ja auch die Einträge über nicht zurückerhaltene Bücher fanden. Danach dürfte man – zu Benns Lebzeiten – zum System der Ausleihe mittels Leih- und [[Buchkarte]]n übergegangen sein, das bis zum Ende des 20. Jahrhunderts üblich war. --[[Benutzer:Pp.paul.4|Pp.paul.4]] ([[Benutzer Diskussion:Pp.paul.4|Diskussion]]) 15:33, 28. Feb. 2017 (CET)
:::Frage aber ist doch, ob diese archiviert werden. Kann ich mir ehrlich gesagt nicht vorstellen.--[[Benutzer:Scialfa|scif]] ([[Benutzer Diskussion:Scialfa|Diskussion]]) 15:58, 28. Feb. 2017 (CET)
::::Vertu Dich da mal nicht, Bibliothekare lassen sich lieber die linke Hand abhacken, als solche Schätze zu vernichten. --[[Benutzer:Elrond|Elrond]] ([[Benutzer Diskussion:Elrond|Diskussion]]) 16:10, 28. Feb. 2017 (CET)
:::::Vielleicht sind aber auch Bibliothekare weisungsgebunden von höherer Stelle. Ich habe jedenfalls mal in eigener Sache bei der Stadtbücherei nachgefragt (um meiner Erinnerung etwas auf die Sprünge zu helfen), in der ich als junger Mensch viel ausgeliehen hatte, und die hatten die Leihkarten schon vor 20 Jahren, oder so, vernichtet. [[Benutzer:Geoz|Geoz]] ([[Benutzer Diskussion:Geoz|Diskussion]]) 18:17, 28. Feb. 2017 (CET)
:Viele UBs haben ask a librarian service, einen virtuellen helpdesk, die oft sehr hilfsbereit und auskunftsfreudig sind. Sonst https://fragdenstaat.de ? --[[Benutzer:Asteroidenbergbauer|Asteroidenbergbauer]] ([[Benutzer Diskussion:Asteroidenbergbauer|Diskussion]]) 18:28, 28. Feb. 2017 (CET)
:Ich habe mal ein Werk konsultiert und mich auf der dazugehörenden Karte eingetragen; einige Einträge über meinem hatte Goethe (ja, der Goethe) unterzeichnet, der den Band auch dort benutzt hatte. Ich hab's nicht rumerzählt, wusste aber schon, dass Goethes Beschäftigung mit dem Werk bereits bekannt war. [[Benutzer:Bink22|Bink22]] ([[Benutzer Diskussion:Bink22|Diskussion]]) 20:05, 28. Feb. 2017 (CET)

Nun ja! Gehe mal in eine Landesbibel, suche im Katalog unter: Spengfallen, Rohrbomben, Schmutziges, Fernsteuerung etc. und lasse Dir das alles in den Lesesaal bringen. - Natürlich ist Gottfried Benn die andere Seite. Grüße --[[Spezial:Beiträge/80.187.96.246|80.187.96.246]] 16:14, 1. Mär. 2017 (CET)

== Jausenbesteck ==

An die Österreicher: Was ist der Unterschied zwischen einem Jausenbesteck und einem normalen Besteck? --[[Spezial:Beiträge/92.107.205.150|92.107.205.150]] 15:10, 28. Feb. 2017 (CET)
:Ich kenne seit meiner Kindheit nur Jausenmesser. Die haben eine Klinge mit Wellenschliff und eine zweizinkige Spitze, so dass man man sie auch als Gabel verwenden kann. [[Benutzer:Rainer Zenz|Rainer Z]] [[Benutzer Diskussion:Rainer Zenz| ...]] 15:26, 28. Feb. 2017 (CET)
::Glaskugel als Nicht-Österreicher: Die Google-Bildersuche zeigt in den ersten Treffern jeweils nur Messer und Gabel (mit rustikalem Holzgriff). Ein [[Essbesteck]] hat mehr Teile. Gruss --[[Benutzer:Nightflyer|Nightflyer]] ([[Benutzer Diskussion:Nightflyer|Diskussion]]) 15:36, 28. Feb. 2017 (CET)
:::Ich habe auch gerade nachgeschaut: Die von mir beschriebenen Messer scheinen ausgestorben zu sein. Wundert mich sehr. [[Benutzer:Rainer Zenz|Rainer Z]] [[Benutzer Diskussion:Rainer Zenz| ...]] 15:47, 28. Feb. 2017 (CET)
:::::[https://www.willhaben.at/iad/kaufen-und-verkaufen/d/jausenmesser-von-solingen-2x-190496781/ solche] Messer vermeine ich auf einem Weihnachtsmarkt bei einem Messerstand gesehen zu haben. Der Preis lag deutlich noch im einstelligen €-Bereich. --[[Benutzer:Elrond|Elrond]] ([[Benutzer Diskussion:Elrond|Diskussion]]) 16:04, 28. Feb. 2017 (CET)
[[File:Kaesemesser weißer Griff-2.jpg|mini|kein Jausenmesser]]
::::Außerhalb Österreichs wäre das eher ein Käsemesser. --[[user:Rotkaeppchen68|R<span style="color:red">ô</span>tkæppchen₆₈]] 15:58, 28. Feb. 2017 (CET)
:::::Jausebesteck ist eher etwas Einfaches, Rustikales. Gestannztes Blech mit Griffen aus Holz oder Horn. [https://www.grube.de/hubertus-jausenbesteck-4-teilig-77-575/], [https://www.servusmarktplatz.com/de/p/Traunviertler-Jausenbesteck/SM119111/], neuerdings auch Plaste [http://www.pizzini.at/shop/jausenbesteck-weiss-10tlg/p-2542.html]. --[[Benutzer:Ralf Roletschek|M@rcela]] [[Bild:Miniauge2.gif|27px]] 16:02, 28. Feb. 2017 (CET)
Ich dachte, es wäre vielleicht eher der Knauf am Ende. Hier ein Link eines Össi-Herstellers:
https://www.hack.at/deutsch-1/online-shop/ --[[Spezial:Beiträge/92.107.205.150|92.107.205.150]] 16:13, 28. Feb. 2017 (CET)
:Ha! Mein uraltes Jausenmesser mit zwei Zinken ist sogar von Hack (das kann man gerade noch erkennen). Die Form stellen sie aber wohl nicht mehr her. Dabei hätte ich gerne einen Nachfolger, weil die Klinge nix mehr ist. [[Benutzer:Rainer Zenz|Rainer Z]] [[Benutzer Diskussion:Rainer Zenz| ...]] 18:51, 28. Feb. 2017 (CET)
::Der Unterschied scheint vor allem in der Klinge zu liegen. Ein "normales" Besteckmesser ist stumpfer, das Jausenmesser scheint mehr in Richtung Steakmesser zu gehen, ist also wohl stärker auf die Schnittleistung ausgelegt, so dass man harte Brotrinde, epökeltes und geräuchertes Fleisch und Hartkäse mit Rinde damit besser schneiden kann. Also kann das meiste damit schneiden, wozu man sonst vorbereitend in der Küche die typischen Küchenmesser verwenden würde, also Brotmesser, Tranchiermesser etc. Die Gemeinsamkeit zum normalen Essbesteck ist die kurze Klinge, die Form ist abweichend spitzer, so dass man z. B. in ein Brot einstechen kann, um eine Scheibe abzuschneiden, was mit einem normalen Besteckmesser praktisch nicht geht.--[[Benutzer:Giftzwerg 88|Giftzwerg 88]] ([[Benutzer Diskussion:Giftzwerg 88|Diskussion]]) 22:09, 28. Feb. 2017 (CET)
:::Bei der Jause muss man selber eine Scheibe vom Laib Brot abschneiden und außerdem den Spreck in kleine Streifen schneiden. Diese zwei Aufgaben muss ein Jausenmesser erfüllen können. Gurkerl, bzw. Pfefferoni aus dem Glasl fischen eventuell auch noch und den Butter verstreichen ebenso. Also das sind die Anforderungen. --[[Spezial:Beiträge/109.100.148.154|109.100.148.154]] 22:28, 28. Feb. 2017 (CET)
::::Und was machst Du mit den geschnittenen Speckstreifen. (Übrigens werden die nicht in Streifen sondern in Teile geschnitten) Dies müssen aufgespiest und zum Mund geführt werden. Also Spitze, so oder doppelso. :-) --[[Spezial:Beiträge/80.187.96.246|80.187.96.246]] 15:19, 1. Mär. 2017 (CET)
:::::Teile? What the hack meinst du mit Teile? Bist du ein Piefke, ein Preiß? Wenn ja, warum schreibst du zu dem hier Thema mit? Ja ehrlich, warum? --[[Spezial:Beiträge/109.100.148.154|109.100.148.154]] 16:29, 1. Mär. 2017 (CET)
:Ich muss mich korrigieren. Was ich bisher für ein Jausenmesser gehalten habe, findet man als Tomatenmesser im Handel. Die gibt es weiterhin mit solchen zwei Spitzen. Da war ich, was die Bezeichnung angeht, wohl lange auf dem Holzweg. [[Benutzer:Rainer Zenz|Rainer Z]] [[Benutzer Diskussion:Rainer Zenz| ...]] 13:06, 2. Mär. 2017 (CET)

== Online-Plattform für Büchereiexemplare ==

Kennt jemand eine Online-Plattform, die auch Bücher von Bibliotheken aufkauft? Momox hat mir das verneint.--[[Benutzer:Scialfa|scif]] ([[Benutzer Diskussion:Scialfa|Diskussion]]) 15:16, 28. Feb. 2017 (CET)
:Das [[Zentrales Verzeichnis Antiquarischer Bücher|ZVAB]] könnte ein Ansprechpartner sein. Dort kann man den einzelnen Antiquariaten die dem Netz angeschlossen sind Angebote machen. --[[Benutzer:Elrond|Elrond]] ([[Benutzer Diskussion:Elrond|Diskussion]]) 15:57, 28. Feb. 2017 (CET)
:: Bei abebooks und anderen finden sich oft ehemalige Bibliotheksexemplare mit Ausscheidestempeln, siehe auch [[Deakzession]].Steht dann auch in der Buchbeschreibung.--[[Benutzer:Asteroidenbergbauer|Asteroidenbergbauer]] ([[Benutzer Diskussion:Asteroidenbergbauer|Diskussion]]) 18:49, 28. Feb. 2017 (CET)
:::Warum sollten das die üblichen Ankäufer nicht annehmen? Es geht doch nur um die Frage ISBN oder nicht. Ansonsten gewöhnliche Antiquare. Abebooks, ZVAB, Booklooker sind keine Händler die ankaufen sondern nur Marktplätze, ein erheblicher Unterschied.--[[Benutzer:Antemister|Antemister]] ([[Benutzer Diskussion:Antemister|Diskussion]]) 19:34, 28. Feb. 2017 (CET)
::::Es kommt sehr stark darauf an, was für eine Art Bibliothek die Bücher anbietet. Wenn es eine öffentliche Bibo ist, sind die Bücher zumeist Altpapier, auch bei Fachbibliotheken nehmen meist nur spezialisierte Antiquare diese Bücher an, wenn überhaupt. Daher ist das Schwarze Brett von ZVAB & Co die beste Möglichkeit. Große Reichtümer sind sowieso mit großer Wahrscheinlichkeit keine zu erwarten, es sei denn, es handelt sich um gesuchte Raitäten. --[[Benutzer:Elrond|Elrond]] ([[Benutzer Diskussion:Elrond|Diskussion]]) 19:50, 28. Feb. 2017 (CET)
:::::Die Antiquariate haben ein Netzwerk, mit dessen Hilfe man an ausgemusterte Bücher oder Bestände aus aufgelösten oder zusammengelegten Bibliotheken kommt. Ich habe sogar schon mal ein Buch aus einer englischen Unibibliothek über Amazon gekriegt. Es hat immer noch die Signatur, die Leihkarte und eine Scancode drin. Wer auf die "gebrauchten" bei Amazon geht kommt so manchmal auf solche Fundstücke. Es wird aber nicht gesagt, wo das Exemplar jeweils herkommt.--[[Benutzer:Giftzwerg 88|Giftzwerg 88]] ([[Benutzer Diskussion:Giftzwerg 88|Diskussion]]) 00:02, 1. Mär. 2017 (CET)
Es geht um Gesetzeskommentare, wie z.B. den Fischer StGB in der Vorauflage (63.) in größerer Stückzahl. Die Schwarte kostete was an die 90 Euro, nach einem Jahr noch sehr gut erhalten. Momox weigert sich, weil nach eigener Aussage zuviel Bib-Exemplare von Endkunden "retourniert" werden. Ich weigere mich aber, solche Vermögenswerte einfach ins Altpapier zu schmeißen. Vorsorglich: es gibt auch keine Massen an Referendaren, die händeringend danach suchen, zumal deren Ansprüche auch arg gestiegen sind.--[[Benutzer:Scialfa|scif]] ([[Benutzer Diskussion:Scialfa|Diskussion]]) 13:50, 1. Mär. 2017 (CET)

== Lange Haare von Männern in der Antike (Naher Osten) ==

[[File:Turin shroud positive and negative displaying original color information 708 x 465 pixels 94 KB.jpg|mini]]
War es eigentlich in der Antike im Nahen Osten üblich, dass die Männer meistens lange Haare trugen? Jesus wird ja fast immer so dargestellt.
--[[Spezial:Beiträge/2A01:C23:C014:8B00:F432:9081:855E:7770|2A01:C23:C014:8B00:F432:9081:855E:7770]] 18:26, 28. Feb. 2017 (CET)
:Nicht ganz dein Thema, aber kürzlich lief im [[Deutschlandfunk|DLF]] ein [http://ondemand-mp3.dradio.de/file/dradio/2017/02/26/haarige_geschichten_biblische_frisuren_und_modernes_dlf_20170226_0835_ec56de9e.mp3 haariger Beitrag.] --[[Benutzer:Stefan|Stefan »Στέφανος«]]&#8201;[[Benutzer Diskussion:Stefan|⸘…‽]] 18:55, 28. Feb. 2017 (CET)
::Wir haben tatsächlich einen Artikel [[Langes Haar]]. Dort vermerkt auch die berüchtigte Stelle aus 1 Kor. 11, 14, nach der es für Männer "von Natur aus" eine Schande ist, ihre Haare lang wie Frauenhaar wachsen zu lassen. Für den Nahen Osten bin ich ziemlich ahnungslos, aber die männliche Haartracht war in der Antike insgesamt recht häufig wechselnder Mode unterworfen: Archaisches Griechenland lang, kurz markierte den Sklaven; später eher kurz; Rom in der alten Republik (sehr) kurz, später länger und gestylt, oft nach der Fasson des aktuellen Herrschers. Grüße [[Benutzer:Dumbox|Dumbox]] ([[Benutzer Diskussion:Dumbox|Diskussion]]) 19:11, 28. Feb. 2017 (CET)
::::Ferner haben wir [[Dreadlocks#Rastafari]] mit noch mehr einschlägigen bzw. einschlägig gemachten Bibelstellen. --[[Benutzer:Edith Wahr|Edith Wahr]] ([[Benutzer Diskussion:Edith Wahr|Diskussion]]) 19:15, 28. Feb. 2017 (CET)
:::::Wobei ausgerechnet Paulus selbst einmal das [[Nasiräer]]-Gelübde geleistet und (zumindest zeitweise) gehalten haben soll. [[Benutzer:Geoz|Geoz]] ([[Benutzer Diskussion:Geoz|Diskussion]]) 19:37, 28. Feb. 2017 (CET)

:Bildliche Darstellungen Jesu (und seiner Zeitgenossen) gibt es erst ab dem 4ten Jahrhundert. Nach generellem Konsensus, gestützt durch einige Bibelzitate, den kontemporären Haarstilen der Römer und Hellenen aber auch Untersuchungen skelettaler Funde trugen Juden dieser Epoche kurzes Haar. Da die Verfasser der Evangelien Jesus wahrscheinlich nie sahen gibt es auch im Neuen Testament keine Beschreibungen der Erscheinung Jesu. --[[Benutzer:Cookatoo.ergo.ZooM|Cookatoo.ergo.ZooM]] ([[Benutzer Diskussion:Cookatoo.ergo.ZooM|Diskussion]]) 20:27, 28. Feb. 2017 (CET)

:Hier eine Büste eines “fast” Zeitgenossen Jesu, Flavius Josephus. [[Datei:Josephusbust.jpg|mini|Römische Büste, die auf Flavius Josephus gedeutet wurde.]] --[[Benutzer:Cookatoo.ergo.ZooM|Cookatoo.ergo.ZooM]] ([[Benutzer Diskussion:Cookatoo.ergo.ZooM|Diskussion]]) 20:31, 28. Feb. 2017 (CET)

Haarlänge in der Antike, als Athen und Rom: Wo wären die brauchbaren Quellen? --[[Spezial:Beiträge/80.187.97.115|80.187.97.115]] 21:12, 28. Feb. 2017 (CET)
:Fürs erste Nachschlagen z. B. der Kleine oder Neue Pauly. Dort die literarischen Belegstellen, dazu natürlich die zeitgenössischen Plastiken, Vasenmalereien etc. Grüße [[Benutzer:Dumbox|Dumbox]] ([[Benutzer Diskussion:Dumbox|Diskussion]]) 21:15, 28. Feb. 2017 (CET)
::Danke! Mir ist der alte, neue und KLP (hat man sich gekauft) geläufig. DU zu kurz, sehr kurz - zu Rom - ist mir zu kurz! :-) Grüße --[[Spezial:Beiträge/80.187.97.115|80.187.97.115]] 21:22, 28. Feb. 2017 (CET)
:::<small>War ja auch streng genommen nicht zum Thema. Grüße [[Benutzer:Dumbox|Dumbox]] ([[Benutzer Diskussion:Dumbox|Diskussion]]) 21:26, 28. Feb. 2017 (CET)</small>
::::<small>OK! Nochmal gelesen - wenn Du meinst dass dies auf Jesus bezogen war, so muss ich Dir recht geben. Grüße </small>--[[Spezial:Beiträge/80.187.97.115|80.187.97.115]] 21:36, 28. Feb. 2017 (CET)
:::::Die Haarlänge und Haarfarbe des J. v. N. war kein Thema, über das irgendeiner der Autoren des 1. Jh. geschrieben hat. Alles was wir heute dazu haben beruht in irgendeiner Form auf der künstlerischen Freiheit. Zum historischen Background: Es ist natürlich ziemlich eindeutig, dass die Römer dieser Zeit eher auf kurze Haare und Rasur standen, was aber nicht auf die unterdrückten Nationen zutreffen muss. Denkbar ist also, dass aus diesen sich manche als politische "Statements" Haartrachten und Bartmoden einfallen ließen, die dazu kontrastierten. Es ist wie in den Kontaktanzeigen: Alles ist möglich, nichts muss.--[[Benutzer:Giftzwerg 88|Giftzwerg 88]] ([[Benutzer Diskussion:Giftzwerg 88|Diskussion]]) 21:51, 28. Feb. 2017 (CET)
:S. auch: Brian Palmer: ''[http://www.slate.com/articles/life/explainer/2011/12/was_jesus_christ_s_hairstyle_normal_for_his_time_.html The Jesus Hairstyle: Did a lot of men have flowing locks in ancient Judea?]'', in ''slate'', Nov. 2011. --[[Benutzer:Edith Wahr|Edith Wahr]] ([[Benutzer Diskussion:Edith Wahr|Diskussion]]) 22:19, 28. Feb. 2017 (CET)
:::<small>Wassjez?!? [http://www.i-am-bored.com/2015/10/korean-jesus-ill-do-anything-you-say-pic.html Und blaue Augen hatte er wohl auch nicht]? <small style="color:grey"><b>'''GEEZER'''</b></small><sup>[[BD:Grey Geezer|<span style="color:grey"> … nil nisi bene</span>]]</sup> 08:26, 1. Mär. 2017 (CET) </small>
Die Antike war ein Zeitraum von mindestens 1000 Jahren (500 vor und 500 nach Christus). Bei den Römern waren lange Haare und Bart lange Zeit ein Zeichen für zurückgebliebene Barbaren, deshalb rasierte sich der römische Bürger und auch der römische Legionär täglich und trug die Haare kurz. Das war auch ein Statussymbol, man konnte es sich leisten, hatte scharfe Klingen und Zeit genug für diese Form der Körperflege. Im ersten Jahrhundert nach Christus, in der frühen Kaiserzeit, ändert sich diese Mode aber, etwa bei den bärtigen Kaisern [[Hadrian (Kaiser)|Hadrian]] und [[Marc Aurel]], mit Unterbrechung des immer glatt rasiert dargestellten [[Trajan]] (98 bis 117). Die dann in Mode gekommenen Bärte waren jedoch immer noch gepflegt und eher kurz, auch wenn Vollbart. Also setzte sich eher der klassisch griechische Einfluss bei den Römern durch, als der barbarisch-germanische oder orientalische. Es gibt natürlich detaillierte kulturgeschichtliche Studie zu der Frage, aber so grob kann man sagen, ab dem ersten Jahrhundert nach Chr. werden Vollbart und etwas längere Haare in Rom modern. Diese kulturelle Prägung der Römer war aber so stark, dass sich in der Westkirche nie die alttestamentalische Vorstellung vom Samson-Mythos durchsetzte, also dass man sich die Haare nicht abschneiden dürfe, während ostkirchliche Popen bis heute lange Haare und lange Bärte haben, ebenso wie islamische Imame. Der gute römische westkirchliche Abendländer ist hingegen kurzhaarig und rasiert, nicht wegen dem Christentum, sondern weil die römische Tradition hier stärker war als die hebräische, griechische oder nahöstliche. --[[Spezial:Beiträge/109.100.148.154|109.100.148.154]] 22:52, 28. Feb. 2017 (CET)

Zur Erinnerung: die Ursprungsfrage lautete ''War es eigentlich in der Antike '''im Nahen Osten''' üblich, dass die Männer meistens lange Haare trugen?''. Was Rom und die Römer betrifft ist sicher interessant, aber hier nur bedingt hilfreich. --[[Benutzer:Elrond|Elrond]] ([[Benutzer Diskussion:Elrond|Diskussion]]) 23:00, 28. Feb. 2017 (CET)
:Durch Kaiser Konstantin und die spätere Geschichte sind wir aber in Westeuropa extrem stark römisch geprägt, egal ob katholisch, evangelisch oder anglikanisch, und nur sehr wenig griechisch, jüdisch oder sonstwie östlich. Und wie gesagt, es gibt keine künstlerische Jesus-Darstellung vor Kaiser Konstantin, die sich erhalten hätte. Die langen Haare Jesu sind also samt und sonders künstlerische Imagination, bzw. ein Topos um ihn eben nicht als Römer darzustellen. Einen Römer erkennt man im Bild (Ikon) eben durch die kurzen Haare und den rasierten Bart. Jesus sollte aber eben nicht als Römer dargestellt werden und damit die Beobachter das gleich erkennen, verpassten ihm die Künstler lange Haare. Ob er diese wirklich hatte, wissen wir nicht. Das Turiner Grabtuch ist ja auch eine mittelalterliche Reliquien-Fälschung und somit keine valide Quelle. --[[Spezial:Beiträge/109.100.148.154|109.100.148.154]] 23:22, 28. Feb. 2017 (CET)
[[Datei:Dürer - Selbstbildnis im Pelzrock - Alte Pinakothek.jpg|thumb|ei guck/ecce homo: nochne imitatio christi]]
:::<small>Das Grabtuch ist vielleicht eine Fälschung, aber dafür ein echter Dürer, ein Selbstbildnis natürlich, wie ich unlängst [http://www.simonandschuster.com/books/The-Relic-Master/Christopher-Buckley/9781501125768 in diesem schönen Schmöker] las, den hiermit & unter uns Pfarrerstöchtern als Strandlektüre empfehlen mag. --[[Benutzer:Edith Wahr|Edith Wahr]] ([[Benutzer Diskussion:Edith Wahr|Diskussion]]) 00:03, 1. Mär. 2017 (CET) </small>
:::: Wenn mich nicht alles täuscht, dann war unter germanischen Stämmen das Haar ein bürgerliches Statussymbol und der rasierte Kopf ein Zeichen von Leibeigenschaft. Reste davon haben sich bis ins frühe Mittelalter erhalten, als beispielsweise [[Karl der Kahle]] jahrelang rasiert wurde, um zu verhindern, dass er Kaiser werden konnte. Und damals hatte die Kirche noch jede Menge Konkurrenten auf dem Kontinent und vom nächsten Schisma war man noch weit entfernt. Es kann durchaus sein, dass die künstlerische Darstellung Anleihen aus dem germanischen Alltag zog. Daher haben unsere Jesus-Bilder ja auch europäische Züge und Hautfarbe. [[Benutzer:Yotwen|Yotwen]] ([[Benutzer Diskussion:Yotwen|Diskussion]]) 09:12, 1. Mär. 2017 (CET)
:::::Die Frage der Haartracht von Jesus hat [[Hans Belting]] in Bild und Kult ausführlicher thematisiert. Er weist (ich zitiere jetzt nur aus dem Gedächtnis) auf zahlreiche unterschiedliche Darstellungsweisen in den ältesten Bildern hin, aus denen der "klassische" langhaarige Jesus mit Mittelscheitel sich erst nach und nach entwickelte. Wichtigste Anregung dafür seien die Bildnisse des Jupiter gewesen.--[[Benutzer:Meloe|Meloe]] ([[Benutzer Diskussion:Meloe|Diskussion]]) 10:30, 1. Mär. 2017 (CET)
::::::Wie die beiden Vorschreiber. Die Evangelisten wie auch Paulus hatten womöglich schlichtweg kein Interesse an einer Beschreibung des genauen Aussehens Jesu, hervorhebenswert war allenfalls die Annahme einer Gestalt als Mann bzw. Sohn. Interessanterweise stützen sich einige Darstellungen oder Verkörperungen Jesu, die wir heute nicht als gutaussehend bezeichnen würden, auf Jesajas Prophezeiungen: „Er hatte keine schöne und edle Gestalt, sodaß wir ihn anschauen mochten. Er sah nicht so aus, dass wir Gefallen fanden an ihm. Er wurde verachtet und von den Menschen gemieden, ein Mann voller Schmerzen, mit Krankheit vertraut. Wie einer, vor dem man das Gesicht verhüllt, war er verachtet; wir schätzten ihn nicht.“ Das verläßt aber den Boden des Themas Haarlänge. --[[Benutzer:Turris Davidica|Turris Davidica]] ([[Benutzer Diskussion:Turris Davidica|Diskussion]]) 12:48, 1. Mär. 2017 (CET)
:::::::<small> In der Offenbarung sind seine Haare weiß (wie Wolle) - nach all dem Stress - aber auch keine Längenangabe. <small style="color:grey"><b>'''GEEZER'''</b></small><sup>[[BD:Grey Geezer|<span style="color:grey"> … nil nisi bene</span>]]</sup> 14:36, 1. Mär. 2017 (CET) </small>
::::::::<small> Wissenschaftlich betrachtet ist festzustellen, dass man hierzu nichts aussagen kann. Belastbare Quellen fehlen schlicht. --[[Spezial:Beiträge/80.187.96.246|80.187.96.246]] 16:22, 1. Mär. 2017 (CET)</small>
:::::::::<small>Es lässt sich feststellen, dass DU keine Ahnung hast, liebe(r) 80.187.96.246. --[[Spezial:Beiträge/109.100.148.154|109.100.148.154]] 16:35, 1. Mär. 2017 (CET)</small>
::::::::::<small>'''Na''' dann bau mal Deine neueren belastbaren Quellen ein. Verate aber welche und wo. --[[Spezial:Beiträge/80.187.96.246|80.187.96.246]] 17:47, 1. Mär. 2017 (CET)</small>
:::::::::::<small>Im Gegensatz zu anderen Zeitepochen und anderen Kulturräumen ist die Quellenlage zur Römischen Antike wahrlich nicht mager, siehe etwa dem [[Corpus Inscriptionum Latinarum]], die über das Mittelalter überlieferte lateinische Literatur, tausende archäologische Funde (zB Mosaike), etc. Wir wissen über die Völker des römischen Reiches so ziemlich alles, was sie zum Mittagessen gegessen haben und welche Unterhose sie getragen haben, wohingegen wir über die Völker außerhalb des Reiches (Skythen, Pikten, Berber, Sudanesen, Germanen, Hunnen, etc.) nur fragmentarische Informationen haben, die wenn vorhanden meist auch über römische Filiation überliefert sind. Von Attila dem Hunnenkönig haben wir zB kein einziges zeitgenössisches Portrait, keine Texte, das wenige was wir über die Hunnen wissen, wissen wir von den Römern, nicht von den Hunnen selber. --[[Spezial:Beiträge/109.100.148.154|109.100.148.154]] 21:42, 1. Mär. 2017 (CET)</small>
::::::::::::<small> Alles gut und schön, aber wissenschaftliche Quellen, zudem belastbar, über Haarlängen gibt es nicht. Und darum geht es hier. Übrigens: Ich habe auch den ordentlichen Hauptschulabschluß. </small>--[[Spezial:Beiträge/80.187.123.130|80.187.123.130]] 22:30, 1. Mär. 2017 (CET)
:::::::::::::<small>Gratuliere! --[[Spezial:Beiträge/109.100.148.154|109.100.148.154]] 22:57, 1. Mär. 2017 (CET)</small>

== Phosphorsäure und Eisen ==

Wenn ich auf eine nicht verrostete Stelle aus Eisen Phosphorsäure streiche, gibt das dann eine Reaktion? Geht um verrostste Stellen an einer Heizung, die ich großflächig abgeschliffen habe. Zur Prophylaxe für noch ev. verbliebene Rostporen möchte ich mit Phosphorsäure arbeiten. Die Frage ist nun, ob das eher kontraproduktiv ist, bevor ich die Grundierung auftrage.
--[[Spezial:Beiträge/91.14.11.96|91.14.11.96]] 23:14, 28. Feb. 2017 (CET)
:Ja, es gibt eine Reaktion. Das Eisen reagiert mit der Phosphorsäure zu Eisenphosphaten. Dabei wird das Eisen passiviert. Das ist nicht kontraproduktiv, sondern nützlich, weil korrosionsschützend. --[[user:Rotkaeppchen68|R<span style="color:red">ô</span>tkæppchen₆₈]] 23:27, 28. Feb. 2017 (CET)
::Zur weiteren Information. Phosphorsäure ist stark ätzend (Arbeitsschutz!) und andere Metalle reagieren auch damit, meist auf eine Art, die nicht gewünscht ist. Dann müssen alle Säurereste peinlich genau vom Werkstück entfernt werden, denn Säurereste können anschließend genau das Gegenteil von dem bewirken, was es soll, nämlich dass das Werkstück rostet. Reine Phosphorsäure ist nebenbei nicht so wirksam wie kommerzielle Rostumwandler, weil die noch etliche Hilfsmittel enthält wie [[Tensid]]e, Verdickungsmittel (damit die Säure länger einwirken kann), Reaktionsbeschleuniger, Korrosionsschutzmittel etcpp. --[[Benutzer:Elrond|Elrond]] ([[Benutzer Diskussion:Elrond|Diskussion]]) 23:45, 28. Feb. 2017 (CET)
:::Abwaschen steht aber in der Bedienungsanleitung jedes Rostumwandlers. Seinerzeit hat die Stiftung Warentest übrigens Coca-Cola als Rostumwandler mit „gut“ bewertet. --[[user:Rotkaeppchen68|R<span style="color:red">ô</span>tkæppchen₆₈]] 23:52, 28. Feb. 2017 (CET)
:::::Wenn der Frager aber keinen konfektionierten Rostumwandler hat, sondern ne Pulle mit Phosphorsäure ohne weitere Informationen, dann ist dieser Hinweis m.E. aber wichtig. --[[Benutzer:Elrond|Elrond]] ([[Benutzer Diskussion:Elrond|Diskussion]]) 10:37, 1. Mär. 2017 (CET)
::::Sollte man nicht eher Coca-Cola Light (oder Zero) verwenden? Enthält auch Phosphorsäure und Citronensäure, ist aber wegen dem fehlenden Zucker nicht so klebrig. --[[Benutzer:MrBurns|MrBurns]] ([[Benutzer Diskussion:MrBurns|Diskussion]]) 09:58, 1. Mär. 2017 (CET)
:::::Die Klebrigkeit ist doch gerade erwünscht. Sonst läuft die Flüssigkeit ab, ohne Gelegenheit zu haben, einzuwirken. --[[user:Rotkaeppchen68|R<span style="color:red">ô</span>tkæppchen₆₈]] 11:04, 1. Mär. 2017 (CET)

== [[Maine-Coon-Katze]] ==

Expertenfrage: Sind in diesem Artikel die Bildunterschriften in der Form korrekt (inhaltlich, sprachlich, typografisch), insbesondere was die jeweiligen Farbbezeichnungen angeht?--[[Benutzer:Hubon|Hubon]] ([[Benutzer Diskussion:Hubon|Diskussion]]) 23:28, 28. Feb. 2017 (CET)
:In der Box würde ich ändern:
:* epistatisches weiß, schwarz (''brown'' in US-Sprachgebrauch), blau, rot, creme, jeweils mit oder ohne Silber und mit oder ohne Scheckungs-Weiß
::in
:* epistatisches Weiß, Schwarz (''brown'' in US-Sprachgebrauch), Blau, Rot, Creme, jeweils mit oder ohne Silber und mit oder ohne Scheckungs-Weiß
::Es geht hier um die Farben an sich und wird nicht adjektivisch verwendet (rotes Was, blaues Wer). <small style="color:grey"><b>'''GEEZER'''</b></small><sup>[[BD:Grey Geezer|<span style="color:grey"> … nil nisi bene</span>]]</sup> 09:21, 1. Mär. 2017 (CET)
:::Das denke ich auch. Aber auch bei den Bildern ist wohl das Meiste auf Englisch und noch dazu kursiv gesetzt. Bin aber kein Fachmann – vielleicht soll das so? Ich lasse da, glaube ich, lieber mal die Finger von, bevor ich noch was falsch mache...--[[Benutzer:Hubon|Hubon]] ([[Benutzer Diskussion:Hubon|Diskussion]]) 15:42, 1. Mär. 2017 (CET)

Eindeutig groß. Dem Autor dieser Bildunterschrift kamen spätestens bei "Scheckungs-Weiß" ja offenbar selber Zweifel. Warum sonst sollte das dann großgeschrieben werden?<br>
Die in Kursiv gesetzten englischsprachigen Bezeichnungen kann man m.E. so lassen. --[[Spezial:Beiträge/93.212.230.216|93.212.230.216]] 16:56, 1. Mär. 2017 (CET)
:Habe es angepasst. --[[Benutzer:Digamma|Digamma]] ([[Benutzer Diskussion:Digamma|Diskussion]]) 20:30, 1. Mär. 2017 (CET)
::Danke dir. Aber dann müsste doch bei den Bildunterschriften wie bspw. beim 1. Bild unter der Infobox auch großgeschrieben werden („Typvoller Maine-Coon-Kater mit auffälligen Lynxtips in '''''b'''lack-mackarel-tabby-white''“), oder?--[[Benutzer:Hubon|Hubon]] ([[Benutzer Diskussion:Hubon|Diskussion]]) 03:55, 2. Mär. 2017 (CET)
:::Mit den engl. Bezeichnungen ist es hoffnungslos (aber nicht alternativlos). In GoogleBooks (sehr viel BoD überr Coons) mal groß, mal klein.
:::Entweder findet man ein Standardwerk zur Orientierung - oder man fragt sich in jedem Fall: "Ist die Farbe gemeint oder ist es adjektivisch." <small style="color:grey"><b>'''GEEZER'''</b></small><sup>[[BD:Grey Geezer|<span style="color:grey"> … nil nisi bene</span>]]</sup> 08:03, 2. Mär. 2017 (CET)
::::Ich denke, die englischen Bezeichnungen können als Zitatwörter nach englischer Rechtschreibung klein geschrieben werden. --[[Benutzer:Digamma|Digamma]] ([[Benutzer Diskussion:Digamma|Diskussion]]) 10:22, 2. Mär. 2017 (CET)

= 1. März 2017 =

== [[Kleinsche Flasche]] ==

Weiß jemand, oder kann jemand herausfinden, wann und von wem die Bezeichung "Kleinsche Flasche" oder englisch "Klein bottle" zum ersten Mal verwendet wurde? Vielen Dank. --[[Benutzer:Digamma|Digamma]] ([[Benutzer Diskussion:Digamma|Diskussion]]) 11:23, 1. Mär. 2017 (CET)
: Verstehe nicht. Ist denn der Satz im Artikel falsch? → ''„[...] wurde erstmals 1882 von dem deutschen Mathematiker Felix Klein beschrieben“'' VG --[[Benutzer:Apraphul|Apraphul]] <small><sup> [[Benutzer Diskussion:Apraphul|Disk]] </sup> <sub> [[Benutzer:Apraphul/WP:SNZ|WP:SNZ]]</sub></small> 11:55, 1. Mär. 2017 (CET)

::Mir geht es um die Bezeichnung. Klein hat meines Wissens nicht die Bezeichung "Flasche" verwendet, schon gar nicht "Kleinsche Flasche". Bei Hilbert/Cohn-Vossen wird die Fläche "Kleinsche Fläche" genannt. --[[Benutzer:Digamma|Digamma]] ([[Benutzer Diskussion:Digamma|Diskussion]]) 12:32, 1. Mär. 2017 (CET)
:::Obere Abschaetzung: <s>1893</s>, siehe [https://books.google.co.th/books?id=Ao0bAAAAMAAJ Annual Report, S. 29] MfG -- [[Benutzer:Iwesb|Iwesb]] ([[Benutzer Diskussion:Iwesb|Diskussion]]) 12:38, 1. Mär. 2017 (CET) <small>gestrichen, s.u. -- [[Benutzer:Iwesb|Iwesb]] ([[Benutzer Diskussion:Iwesb|Diskussion]]) 01:42, 2. Mär. 2017 (CET)</small>

::::Wow. Danke. --[[Benutzer:Digamma|Digamma]] ([[Benutzer Diskussion:Digamma|Diskussion]]) 14:32, 1. Mär. 2017 (CET)
::::<small>Das ist keine geeignete Abschätzung. Die verlinkten ''Annual reports of the Dayton Public Library and Museum., v.32-94 1891/1892-1954/1957 incompl.'' stammen aus den Jahren 1891 bis 1957. --[[Benutzer:Pp.paul.4|Pp.paul.4]] ([[Benutzer Diskussion:Pp.paul.4|Diskussion]]) 16:47, 1. Mär. 2017 (CET)</small>
:::::Weiterhin interessiert mich aber, welcher Autor die Bezeichnung zuerst verwendet hat. Und außerdem die erstmalige Verwendung der deutschen Bezeichnung "Kleinsche Flasche". --[[Benutzer:Digamma|Digamma]] ([[Benutzer Diskussion:Digamma|Diskussion]]) 16:07, 1. Mär. 2017 (CET)
::::::Stimmt, Pp.paul.4 hat recht, das Erscheinungsdatum bei Google Books ist irrefuehrend. Das liefert uns "lediglich" <=1957. In [https://books.google.co.th/books?id=tWobCAAAQBAJ&pg=PA161 diesem Buch, S. 161f.] ist ein Scan eines Briefes von E. Witt an G. Herglotz von Januar 1943, dort heisst es "Kleinscher Schlauch" (und wird uebersetzt als "Klein bottle"). <small>Das ist eine Superfrage. Beim Versuch, diese "Flasche" zu fuellen, wird keiner gluecklich :-) </small> MfG -- [[Benutzer:Iwesb|Iwesb]] ([[Benutzer Diskussion:Iwesb|Diskussion]]) 01:42, 2. Mär. 2017 (CET)
:::::::Google-Suche liefert Ergebnisse aus Zeitschriftenbänden, die regelmäßig nicht unter dem Erscheinungsjahr des Aufsatzes, sondern unter dem der Reihe verhaftet sind. Bei den Schnipseln sind Erscheinungsjahr, Titel oder Autor des Aufsatzes häufig kaum zu bestimmen. Dass, wie die englische Wikipedia unter Berufung auf [https://books.google.de/books?id=F5qIAwAAQBAJ&lpg=PA94&hl=de&pg=PA95#v=onepage&q&f=false diese Quelle] andeutet, Klein bottle eine Fehlübersetzung von Kleinsche Fläche sein könnte, glaube ich nicht. Jeder Leser und Übersetzer von Arbeiten aus diesem Gebiet ist mit dem Wort „Fläche“ vertraut und weiß, was es bedeutet. Ich halte das englische "Klein bottle" entweder für eine schiefe Übersetzung von Kleinscher Schlauch ([[Neuer Wein in alten Schläuchen]]) oder für eine kreative Neuschöpfung. Kleinsche Flasche ([https://books.google.de/books?hl=de&id=mhsJAQAAMAAJ&dq=Kleinsche+Flasche+Gamow&focus=searchwithinvolume&q=Kleinsche+Flasche+Gamow 1958]) halte ich, da der englische Ausdruck "Klein bottle" ([https://books.google.de/books?id=fu-SjQudB_0C&lpg=PP1&vq=klein%20bottle&hl=de&pg=PA62#v=onepage&q=klein%20bottle&f=false 1947?]) zeitlich voranzugehen scheint, für eine Übersetzung des englischen Ausdrucks. --[[Benutzer:Pp.paul.4|Pp.paul.4]] ([[Benutzer Diskussion:Pp.paul.4|Diskussion]]) 15:59, 3. Mär. 2017 (CET)

::::::::Die in [[:en:Klein bottle]] angegebene Quelle steht dort seit [https://en.wikipedia.org/w/index.php?title=Klein_bottle&diff=382177300&oldid=382164726 September 2010]. Die Quelle ist von 2009. Die Behauptung mit der Falschübersetzung steht aber schon seit [https://en.wikipedia.org/w/index.php?title=Klein_bottle&diff=11337884&oldid=10957917 März 2005] im Artikel. Es ist somit nicht auszuschließen, dass der Buchautor bei Wikipedia abgeschrieben hat. --[[Benutzer:Digamma|Digamma]] ([[Benutzer Diskussion:Digamma|Diskussion]]) 16:43, 3. Mär. 2017 (CET)

::::::::[https://books.google.de/books?id=3UOGBwAAQBAJ&pg=PA272&dq=hilbert+cohn+vossen+anschauliche+geometrie+kleinsche&hl=de&sa=X&ved=0ahUKEwjx7Y6k2rrSAhUFSBQKHZK-B8IQuwUIIDAA#v=onepage&q=hilbert%20cohn%20vossen%20anschauliche%20geometrie%20kleinsche&f=false In Hilbert, Cohn-Vossen: ''Anschauliche Geometrie''] von 1932 steht "Kleinsche Fläche" und mindestens an einer Stelle auch "Kleinscher Schlauch". In der englischen Übersetzung [https://books.google.de/books?id=7WY5AAAAQBAJ&printsec=frontcover&dq=hilbert+cohn+vossen+geometry+imagination+klein&hl=de&sa=X&ved=0ahUKEwjqwYOI2brSAhVMXRQKHUSACQkQuwUIIDAA#v=onepage&q=Klein&f=false ''Geometry and the Imagination''] von 1952 steht "Klein bottle". --[[Benutzer:Digamma|Digamma]] ([[Benutzer Diskussion:Digamma|Diskussion]]) 17:04, 3. Mär. 2017 (CET)
:::::::::Ein weiteres verstörendes Merkmal der Google-Buchsuche. Wenn ich von deinem Link (S. 272) zur Folgeseite (S. 273) mit dem Kleinschen Schlauch gehe, erhalte ich den Hinweis „Anzeigebeschränkung“, verlinke ich S. 273 direkt, erhalte ich die [https://books.google.de/books?id=3UOGBwAAQBAJ&lpg=PP1&hl=de&pg=PA273#v=onepage&q&f=false Seite 273]. Was mich auch wundert, wieso merken du, Digamma, und andere, schon nach wenigen Minuten, wenn ich auf einem ein oder mehrere Tage alten Thread antworte? --[[Benutzer:Pp.paul.4|Pp.paul.4]] ([[Benutzer Diskussion:Pp.paul.4|Diskussion]]) 17:53, 3. Mär. 2017 (CET)

== Gedicht - aus welchem Jahr ==

Kann jemand feststellen, aus welchem Jahr das Gedicht [http://www.zum.de/Faecher/Materialien/dittrich/Lyrik/Sprichwoerter.htm "Sprichwörter"] von [[Fred Endrikat]] stammt? Quelle wäre ggfs. nett. --[[Spezial:Beiträge/85.178.195.31|85.178.195.31]] 12:56, 1. Mär. 2017 (CET)
:[http://www1.wdr.de/radio/wdr5/sendungen/liegen-bleiben/was-ist-die-welt-100.html Angeblich] aus „Liederliches und Lyrisches“ (Buchwarte Berlin 1942). Wann es geschrieben wurde, dürfte kaum verlässlich festzustellen sein. Siehe Nachwort bei https://www.lwl.org/literaturkommission-download/Bibliothek_Westfalica/Endrikat_Lesebuch.pdf. --[[Benutzer:Vsop|Vsop]] ([[Benutzer Diskussion:Vsop|Diskussion]]) 14:01, 1. Mär. 2017 (CET)

::Vielen Dank für die Informationen. Ich bin sehr unsicher, ob die Angabe des WDR stimmt, denn in allen Inhaltsverzeichnissen, die ich online zu „Liederliches und Lyrisches“ finden kann, taucht "Sprichwörter" nicht auf [http://gedichte.xbib.de/gedichtband_Liederliches+und+Lyrisches_Endrikat%2C+Fred%2C32,64.htm beispielsweise hier] --[[Spezial:Beiträge/85.178.195.31|85.178.195.31]] 14:56, 1. Mär. 2017 (CET)
:::Whow, "sehr" unsicher... Und was bringt dich zu der Annahme, dass der [http://gedichte.xbib.de/impressum.html Christian Ritter aus Heidesee] und die Familie Kindermann aus Radeberg sorgfältiger arbeiten als die professionellen Journalisten einer öffentlich-rechtlichen Sendeanstalt? Haben die beim WDR sich das ausgedacht oder gibt es im Web jede Menge Seites, die schnell und falsch voneinander abschreiben? --[[Spezial:Beiträge/62.226.227.152|62.226.227.152]] 15:42, 1. Mär. 2017 (CET)
:::: Hmmmm - ob das Projekt Gutenberg des Spiegel-Verlags bei den guten Menschen aus Heidesee und Radeberg abschreibt oder all die Ritters und Kindermanns in Hamburg, weiß ich natürlich nicht - jedenfalls taucht das Gedicht auch nicht in der [http://gutenberg.spiegel.de/buch/liederliches-und-lyrisches-6839/2 Gutenberg.de-Zusammenstellung] auf. --[[Spezial:Beiträge/85.178.195.31|85.178.195.31]] 16:45, 1. Mär. 2017 (CET)
:::::Na, dann schreib doch auch genau das als Begründung für deine Skepsis. Dann lohnt sich auch genaueres Hinsehen. Wenn ich es richtig sehe wird der Beitrag des WDR referenziert mit: Liederliches und Lyrisches/ Create Space. Create Space ist eine Self-Publishing-Plattform von Amazon. Die Quelle ist zwar korrekt angegeben, aber damit für unsere Frage nicht weniger unseriös wie Familie Kindermann. Über eurobuch.com sehe ich, dass das "wirkliche" Buch in diversen Ausgaben der 1940er Jahre antiquarisch erhältlich ist. Wenn es ein Antiquar in deiner Nähe hat lässt er dich vielleicht reingucken oder es sagt dir am Telefon ein netter Antiquar, was er findet und nicht findet (die Adressen der Antiquariate findet man sehr gut bei zvab.com). Weiterhin ist als Taschenbuch 2015 eine Neuausgabe erschienen (Endrikat, Fred: Liederliches und Lyrisches - Verse vom vergnüglichen Leben - , ISBN: 9783958220683), die sich von der ursprünglichen Ausgabe unterscheiden könnte. Das käme als Ursache für den Widerspruch auch in Frage. Gruß --[[Spezial:Beiträge/2003:46:A0F:5600:E22A:82FF:FEA0:3113|2003:46:A0F:5600:E22A:82FF:FEA0:3113]] 19:20, 1. Mär. 2017 (CET) (hier auch als 62.226.227.152 unterwegs) Nachtrag: GBS kennt auch eine Ausgabe: Leda Verlag, 1967 (sic!), ISBN 3764521023, 9783764521028. --[[Spezial:Beiträge/2003:46:A0F:5600:E22A:82FF:FEA0:3113|2003:46:A0F:5600:E22A:82FF:FEA0:3113]] 19:32, 1. Mär. 2017 (CET) Korrektur / Ergänzung: Die Berliner [http://d-nb.info/451109163 Blanvalet-Ausgabe] reicht bis mindestens 1951 zurück ("Neuausg., 180. - 185. Tsd.", bei eurobuch.com sehe ich sogar 174.-185. Tsd. 1950). Und neben der [http://d-nb.info/1075007194 Omnium-Ausgabe] von 2015 gibt es noch eine [http://d-nb.info/1082767204 Renovamen-Ausgabe] von 2016. Die DNB datiert die erste [http://d-nb.info/573006121 Buchwarte-Ausgabe, 1.-20. Tsd.] auf 1940 und nicht wie oben [[Benutzer:Vsop|Vsop]] herausfand, 1942. 1940 deckt sich auch mit den Angaben im Projekt Gutenberg. Nach [https://www.lwl.org/literaturkommission-download/Bibliothek_Westfalica/Endrikat_Lesebuch.pdf Joachim Wittkowsk, S. 129], den Vsop oben anführt, erreicht das Bändchen noch 1940 das 227. Tausend. --[[Spezial:Beiträge/62.226.227.152|62.226.227.152]] 22:50, 1. Mär. 2017 (CET)

== lotte gegen dortmund ==

Was kam gestern in der ARD um 20.15 anstelle des abgesagten Spiels Lotte gegen Dortmund?
Gruß


<!-- Lass die nachfolgende Zeile am ENDE deiner Frage stehen. Sie wird in deine Signatur umgewandelt. -->
<!-- Lass die nachfolgende Zeile am ENDE deiner Frage stehen. Sie wird in deine Signatur umgewandelt. -->
--[[Spezial:Beiträge/217.251.201.249|217.251.201.249]] 14:19, 1. Mär. 2017 (CET)
--[[Benutzer:Historiograf|Historiograf]] ([[Benutzer Diskussion:Historiograf|Diskussion]]) 03:17, 30. Sep. 2020 (CEST)
:Ein anderes Pokalspiel. --[[Benutzer:MrBurns|MrBurns]] ([[Benutzer Diskussion:MrBurns|Diskussion]]) 14:20, 1. Mär. 2017 (CET)
:Danke![[Spezial:Beiträge/217.251.201.249|217.251.201.249]] 14:22, 1. Mär. 2017 (CET)
:: Die ARD behauptet: [http://www.daserste.de/programm/index~_s-327da4d4-3f2c-4e8d-b391-d793e6426c5b_pd-20170228.html ein alter Tatort]. --[[Benutzer:Wrongfilter|Wrongfilter]] [[Benutzer Diskussion:Wrongfilter|...]] 14:22, 1. Mär. 2017 (CET)
:::Na erstmal haben sie 45 Min. (LIVE !) vor Ort über das Nichtspiel gesprochen. Zur besten Sendezeit! <small style="color:grey"><b>'''GEEZER'''</b></small><sup>[[BD:Grey Geezer|<span style="color:grey"> … nil nisi bene</span>]]</sup> 14:28, 1. Mär. 2017 (CET)
:::::Das war doch zum Schutz der Angestellten des Senders. Was glaubst Du was losgewesen wäre, wenn ein zu spät einschaltender erst mal einen Tatort gesehen hätte, oder sonstewas? So viele Todesdrohungen kann man keinem Menschen an einem Servicetelefon zumuten. So sahen die erst mal - nichts, gespickt mit irgendwelchen Kommentaren, aber den Zuschauern war klar, dass da irgendwas mit dem Spiel nicht so läuft wie erwartet. Den Stress in den vielen Wohnzimmern möchte ich allerdings trotzdem nicht mitgekriegt haben. --[[Benutzer:Elrond|Elrond]] ([[Benutzer Diskussion:Elrond|Diskussion]]) 14:37, 1. Mär. 2017 (CET)
::::::Dafür hätte wohl ein [[Insert]] gereicht. Also unten im Bild ein Text, wo sowas steht wie "Das Pokalspiel Lotte - Dortmund wurde wegen Schneefall abgesagt" (eventuell in [[Laufschrift]], falls die Schrift sonst zu klein wäre). --[[Benutzer:MrBurns|MrBurns]] ([[Benutzer Diskussion:MrBurns|Diskussion]]) 14:44, 1. Mär. 2017 (CET)
:::::::Du glaubst gar nicht, was aufgeregte Fans zu übersehen im Stande sind! Kein Scherz, sondern erlebte Realität. --[[Benutzer:Elrond|Elrond]] ([[Benutzer Diskussion:Elrond|Diskussion]]) 15:12, 1. Mär. 2017 (CET)
::::(BK)Ich glaub eher, der Tatort wäre das Ersatzprogramm gewesen, wenn (z.B. wegen großflächigen Unwettern) alle Pokalspiele mit entsprechender Ankickzeit (also 28.02., 20:45 oder früher, waren nur 2) entfallen wären. Jedenfalls lief wie ich kurz bei ARD vorbeigezappt habe (ich glaub um ca. 21:00) dort definitiv ein Fußballspiel. War aber wohl nur eine Aufzeichnung oder Zusammenfassung, da kein anderes Match am Hauptabend war, jedenfalls laut [[DFB-Pokal_2016/17#Viertelfinale]]. Müsste demnach eine Aufzeichnung oder Zusammenfassung von Frankfurt - Bielefeld gewesen sein.
::::@Geezer: ich hab definitiv Fußballer herumrennen gesehen. --[[Benutzer:MrBurns|MrBurns]] ([[Benutzer Diskussion:MrBurns|Diskussion]]) 14:33, 1. Mär. 2017 (CET)
::::: Das war die Zusammenfassung des Spiels Frankfurt gegen Bielefeld (das war schon vorher zu Ende, da der Anstoß um 18.30 Uhr war). Danach kam ein Tatort. Gruß --[[Benutzer:Mikered|Mikered]] ([[Benutzer Diskussion:Mikered|Diskussion]]) 14:54, 1. Mär. 2017 (CET)
Google hilft: [http://www.express.de/sport/fussball/zu-viel-schnee-fuer-lotte-dortmund-was-die-ard-statt-des-abgesagten-pokal-spiels-zeigte-25942178]. --[[Benutzer:Ailura|Ailura]] ([[Benutzer Diskussion:Ailura|Diskussion]]) 14:57, 1. Mär. 2017 (CET)

Erst lief die Sportschau live aus Lotte mit Alexander Bommes und Mehmet Scholl, die über das nicht stattfindende Spiel sprachen. Dann kam die ausführliche Zusammenfassung des Spiels Frankfurt-Bielefeld und danach ein kurzer Ausblick auf die heutige Partie Bayern-Schalke. Dann kam dann der Berliner Tatort „Gefährlicher Glaube“ oder so ähnlich, während dem hin und wieder unten links die Einblendung kam, dass das Spiel „wegen Schneetreibens“ (oder so ähnlich) ausgefallen sei (tatsächlicher Grund war wohl eher eine allgemeine Unbespielbarkeit des Platzes, das Schneetreiben am Abend setzte dem Ganzen nur die Krone auf – als die Sportschau lief hatte es jedenfalls schon aufgehört zu schneien), danach dann die Tagesthemen, die eigentlich während der Halbzeit des Lotte-Dortmund-Spiels hätten ausgestrahlt werden sollen. Ich ''weiß es sicher'', denn ich saß die ganze Zeit (moderat angepisst) vor der weiter nebenbei laufenden Glotze (frag mich aber nicht, wer bei dem Tatort die Leiche und wer der Täter war)... --[[Benutzer:Gretarsson|Gretarsson]] ([[Benutzer Diskussion:Gretarsson|Diskussion]]) 15:27, 1. Mär. 2017 (CET)
:Danke für alle Antworten, besonders für die letzte von Gretarsson! [[Spezial:Beiträge/217.251.201.249|217.251.201.249]] 15:29, 1. Mär. 2017 (CET)
:Der Tatort hieß „Blinder Glaube“ (Nr. 703 vom 31. August 2008). --[[user:Rotkaeppchen68|R<span style="color:red">ô</span>tkæppchen₆₈]] 15:46, 1. Mär. 2017 (CET)
:: <small>Kommt hin, es ging da auch um Blinde bzw. um Medizintechnik den Sehsinn betreffend...
:: @217.251.201.249: Keine Ursache. --[[Benutzer:Gretarsson|Gretarsson]] ([[Benutzer Diskussion:Gretarsson|Diskussion]]) 15:51, 1. Mär. 2017 (CET)</small>
:::Zugegeben off topic: Was ich aber aber immer wieder bewundere, ist die Vielfalt der Live-Spiele, die das Erste überträgt! Beispiel: 2. Hauptrunde: Bayern-Augsburg; Achtelfinale Bayern-Wolfsburg und Lotte-BVB. 2015 sah es so aus: 2. Hauptrunde Wolfsburg-Bayern, Achtelfinale Bayern-Darmstadt und Augsburg-BVB; Viertelfinale: Bochum-Bayern und Stuttgart-BVB.--[[Benutzer:IP-Los|IP-Los]] ([[Benutzer Diskussion:IP-Los|Diskussion]]) 16:18, 1. Mär. 2017 (CET)
:::: Da die Anhänger/Sympathisanten von Bayern und Dortmund wohl die größten Gruppen unter den deutschen Fußballfans stellen, kommt es mit dieser Spielauswahl sicher den Wünschen/Erwartungen der relativen Mehrheit der fußballinteressierten ARD-Gucker nach. Kann aber auch sein, dass es hier den Spielansetzungen des DFB „ausgeliefert“ ist, weil es nur die Übertragungsrechte für die 20.45-Uhr-Partien hat. Ist dann halt ’ne Frage, inwieweit die ARD bei den Ansetzungen Einfluss auf den DFB hat. Davon abgesehen kann ja die ARD nix dafür, dass Dortmund und die Bayern die letzten Jahre immer mindestens bis ins Viertelfinale gekommen sind. Darüber hinaus darf man sich dann noch für eine Erfindung namens Pay-TV im Speziellen und Privatfernsehen im Allgemeinen bedanken, mit denen der ÖRR um die Übertragungsrechte konkurriert, für die der Preis vom DFB somit entspechend hoch angesetzt werden kann. Wäre er niedriger und die Konkurrenz geringer, könnte man sich die Übertragungsrechte für mehr Spiele sichern, sodass man die eine oder andere Partie ohne Beteiligung der Bayern oder Dortmunds dann evtl. bei den Dritten oder auf einem der Digitalkanäle der beiden „Großen“ ÖRR verfogen könnte. Aber dann beschwerten sich wahrscheinlich wieder die Fußball-Nichtmöger, warum denn der ÖRR ständig Fußball überträgt... --[[Benutzer:Gretarsson|Gretarsson]] ([[Benutzer Diskussion:Gretarsson|Diskussion]]) 18:23, 1. Mär. 2017 (CET)
:::::Gerade beim ÖRR, der sich ja hauptsächlich durch Zwangsgebühren und nicht quotenabhängige Werbung finanziert (erst recht nicht zur [[Hauptsendezeit|Primetime]], weil da Werbepausen auf ARD/ZDF verboten sind), könnte man sich etwas mehr Ausgeglichenheit erwarten. Ansonsten weiß ich nicht wie es beim DFB-Pokal ist aber generell ist es im Fußball heute üblich, dass der Ausrichter nur die möglichen Beginnzeiten sowie eventuell die Zahl der Spiele pro Beginnzeit festlegt und der Fernsehsender dann bestimmt, welches Spiel zu welcher Beginnzeit läuft. --[[Benutzer:MrBurns|MrBurns]] ([[Benutzer Diskussion:MrBurns|Diskussion]]) 19:00, 1. Mär. 2017 (CET)
:::::: Ich wage zu bezweifeln, dass das so ist (woher beziehst du dieses Wissen?), zumindest dass hier die ARD das letzte Wort hat. Die Spiele werden auch auf Sky übertragen und die machen Werbung in der Halbzeitpause. Es ist also für Sky lukrativer als für die ÖR, wenn die Partien der Bayern und des BVB um 20.45 angepfiffen werden. Ist aber gut möglich, dass der DFB von sich aus die quotenträchtigeren Spiele auf die attraktiveren Anpfiffzeiten legt... --[[Benutzer:Gretarsson|Gretarsson]] ([[Benutzer Diskussion:Gretarsson|Diskussion]]) 21:36, 1. Mär. 2017 (CET)
:::::::Sky verdient hauptsächlich an den Abonnenten, die Werbung ist auch nur ein Nebengeschäft. Aber natürlich ist für Sky natürlich auch eine attraktivere Anpiffzeit besser, weil die Abonnenten werden ja nicht dazu gezwungen, Abonnenten zu werden und zu bleiben, vor allem ist es für Sky aber gut, wenn das Spiel nicht im sog. [[Free-TV]] läuft, weil was bringt das Abo wenn man das Match auch ohne Abo woanders schauen kann? Der einzige Grund warum es überhaupt noch spiele im Free-TV gibt ist dass die Vereine nicht nur an den TV-gGldern verdienen, sondern auch an Sponsorengeldern und für die Sponsoren sind Spiele im Free-TV wegen der höheren [[Reichweite (Medien)|Reichweite]] interessante. --[[Benutzer:MrBurns|MrBurns]] ([[Benutzer Diskussion:MrBurns|Diskussion]]) 07:32, 2. Mär. 2017 (CET)


:(BK) Die Entscheidung liegt bei der ARD. Das Problem ist folgendes: Das ist keine Vielfalt, sondern schlichtweg verkapptes Bayern- bzw. BVB-TV! Beispiel? Bayern-Augsburg. Es hätte aber auch die Partien Hoffenheim-Köln oder Nürnberg-Schalke gegeben. Diese wären deutlich spannender gewesen. Das hätte man nicht erahnen können? In diesem Falle schon. Es ist doch immer das Gleiche: der übermächtige Goliath Bayern gegen den David. Ab dem Halbfinale werden beide Spiele übertragen. Ab da sähe man Bayern also sowieso mit hoher Wahrscheinlichkeit (siehe Ergebnisse der letzten DFB-Pokale). Was ist mit Hamburg-Gladbach? Das hätte man so ansetzen können, daß es eben statt Lotte-BVB übertragen wird. Aber nein, es spielt der BVB gegen einen Drittligisten! Nichts gegen Lotte, aber Hamburg-Gladbach mutet ausgeglichener und damit interessanter an. Nun rate mal, wie oft Bayer Leverkusen und Gladbach zusammen im ZDF in der Champions League am Mittwoch zu sehen waren, wie oft Bayern und der BVB? (bei 6 Spieltagen 1x Gladbach, 2x Bayern und 3x Dortmund, also 1:5).
:Einfache Lösung: einmal alle Browser durchprobiert, ob die einen Unterschied machen? Cache & Cookies gelöscht? -- [[Benutzer:Southpark|southpark]] 12:53, 30. Sep. 2020 (CEST)
:Also: ''Davon abgesehen kann ja die ARD nix dafür, dass Dortmund und die Bayern die letzten Jahre immer mindestens bis ins Viertelfinale gekommen sind.'' Nein, aber sie können etwas dafür, daß schon ''vorher'' fast ausschließlich Bayern gezeigt wird. Sie könnten für die Viertelfinalspiele wenigstens ''ein'' Spiel auswählen, an dem nicht Bayern oder der BVB beteiligt ist. Die ARD kann ''zwei'' Spiele übertragen. Eine echte Chance hätten andere Vereine nur noch, wenn durch die Auslosung Bayern und der BVB schon da gegeneinander spielten. Mir geht es hier als neutraler Beobachter schlichtweg um Ausgewogenheit, die aber von den öffentlich-rechtlichen Sendern eben nicht mehr geleistet wird.
:''Wäre er niedriger und die Konkurrenz geringer, könnte man sich die Übertragungsrechte für mehr Spiele sichern, sodass man die eine oder andere Partie ohne Beteiligung der Bayern oder Dortmunds dann evtl. bei den Dritten oder auf einem der Digitalkanäle der beiden „Großen“ ÖRR verfogen könnte.'' Man könnte aber auch einfach mal andere Vereine berücksichtigen! Es zahlen alle Rundfunkgebühren, nicht nur Bayern- und BVB-Fans, sondern auch Fans anderer Vereine, einfache Fußballinteressierte und Fußballhasser. Da sollte man dann doch schon mal auf etwas mehr Vielfalt achten, denn "vielfältige Inhalte" kann ich da beim besten Willen nicht mehr erkennen.--[[Benutzer:IP-Los|IP-Los]] ([[Benutzer Diskussion:IP-Los|Diskussion]]) 19:35, 1. Mär. 2017 (CET)
::Zumal gerade das Phänomen, dass der BVB eindeutig der zweitpopulärste Verein in Deutschland ist, m.E. ein relativ neues Phänomen ist. Ich denke vor 10 Jahren war der BVB noch nicht viel populärer als z.B. Schalke. Da frag ich mich schon auch, ob der BVB diesen Popularitätssprung nicht auch dem TV zu verdanken hat... --[[Benutzer:MrBurns|MrBurns]] ([[Benutzer Diskussion:MrBurns|Diskussion]]) 19:49, 1. Mär. 2017 (CET)


::: @MrBurns: Viele heutige Dortmund-Fans sind es wahrscheinlich schon seit der Hitzfeld-Ära in den 90ern, unter Sammer wurde Dortmund dann auch 2002 nochmal Meister. Da ist also schon eine gewisse Kontinuität vorhanden. Der aktuelle Popularitätsschub der Dormunder geht eindeutig auf die Klopp-Ära zurück, der aus einer Truppe voller junger Nobodys ein Meisterteam formte. Sowas macht Eindruck auch bei Anhängern anderer Vereine. Hinzugehen und zu behaupten, dass an deren aktueller Popularität auch das Fernsehen seinen Anteil hätte, schmälert m.E. die Leistungen, die man von dieser Mannschaft in den letzten Jahren geboten bekommen hat. Welcher Bundesligaverein kann denn von sich behaupten, schon mal Real Madrid in einem Pflichtspiel mit 3:0 besiegt zu haben? Ich bin kein erklärter BVB-Fan, schaue mir aber aktuell, so ich die Zeit habe, jedes Live-Spiel dieses Clubs gerne an, was ich so über den HSV, Gladbach, Leverkusen, Wolfsburg, Mainz oder Schalke sicher nicht sagen kann. --[[Benutzer:Gretarsson|Gretarsson]] ([[Benutzer Diskussion:Gretarsson|Diskussion]]) 22:10, 1. Mär. 2017 (CET)
== Waschmaschine Trockner Verbindungsrahmen ==


::Ich denke, ich habe oben im ersten Satz den wohl gewichtigsten Grund genannt, warum Bayern und der BVB präsenter bei den Live-Übertragungen der ÖR sind als andere Vereine. Dass mag denen nicht schmecken, die ebenjene anderen Vereine favorisieren, indes ist dies hier der falsche Ort, sich darüber zu echauffieren. Schreib halt ’ne Hassmail an die Programmverantwortlichen. Ich kann’s nicht ändern und ich hab auch nichts entschuldigt, nur zu erklären versucht. --[[Benutzer:Gretarsson|Gretarsson]] ([[Benutzer Diskussion:Gretarsson|Diskussion]]) 22:10, 1. Mär. 2017 (CET)
Hallo allerseits, habe eine WaMa (Bauknecht, ca. 60x55) und bald einen Trockner (Bosch, 60x60), nun wollte ich den Trockern auf die WaMa stellen. Dafür gibt es ja sog. Verbindungsrahmen (VR). Problem ist nur: für die WaMa sagt der Hersteller, man solle den VR xy nehmen, auf dem passen alle Trockner. Beim Trockner sagt der Hersteller, unbedingt "seinen" VR nehmen, der passe auf alle WaMas.


:::Ich wollte lediglich aufzeigen, daß die ÖR hier eben nicht ausgewogen auswählen, mehr nicht. Der Grund, warum man hier auf Eintönigkeit setzt, ist klar. Deshalb habe ich auch versucht, Deine weiteren Vermutungen richtigzustellen. Aber dieses extreme Ungleichgewicht fällt nicht nur mir als wie gesagt neutraler Beobachter auf, [http://www.focus.de/kultur/kino_tv/focus-fernsehclub/tv-kolumne-dfb-pokal-schon-wieder-bayern-im-tv-die-angsthasen-entscheidung-der-ard-nervt_id_6120238.html sondern] durchaus [http://www.faz.net/aktuell/sport/fussball/der-dfb-pokal-zeigt-ausschliesslich-bayern-und-bvb-14845237.html auch anderen].
Da die WaMa etwas weniger tief ist als der Trockner frage ich mich, ob das überhaupt geht, und wer nun "Recht" hat? Danke Euch. --[[Spezial:Beiträge/2A02:810D:1600:1A5C:A5AB:74FF:6250:139A|2A02:810D:1600:1A5C:A5AB:74FF:6250:139A]] 10:08, 30. Sep. 2020 (CEST)
:::Ach ja: Schalke hat Real 4:3 im Bernabeau-Stadion geschlagen und wäre damit fast weitergekommen (womit niemand gerechnet hätte) - das war auch eine reife Leistung (ohne die des BVB schmälern zu wollen). Da zeigt sich aber: nicht nur der BVB spielt tollen Fußball, und manchmal wäre es doch auch mal ganz angenehm, andere Mannschaften sehen zu können (so wie eben das Hinspiel zwischen Schalke und Real damals).--[[Benutzer:IP-Los|IP-Los]] ([[Benutzer Diskussion:IP-Los|Diskussion]]) 00:47, 2. Mär. 2017 (CET)
:Ich hab meins mit anschraubbaren Füßen verbunden. Suche z.B. nach Wasch-Trockner-Verbindungsbausatz. Hinterlässt natürlich Löcher, aber ist relativ universell, muss nur zu den Füßen des Trockners passen. --[[Benutzer:Windharp|Windharp]] ([[Benutzer Diskussion:Windharp|Diskussion]]) 13:03, 1. Okt. 2020 (CEST)


== Tod von Kirsten Erl ==
== Mit welchem Sinn ortet die Elster das Fleisch im Boden? ==


Wie ich gelesen habe, ist die TV Richterin Kirten Erl am 10.02.1017 verstorben.
Die Elster landet auf dem Rasen und beginnt fleissig zu hacken. Bald zieht sie Fleisch heraus - ob Wurm oder Larve weiss ich nicht. Das Loch ist einige cm tief. Welches Sinnesorgan des Vogels ermöglicht diese richtungsgenaue Ortung unter dem Boden? Geruch, Gehör oder Taktilität?
Wird das eingetragen auf der Seite von Frau Erl? In Wikipedia!
Hier der Link der Traueranzeige
https://waz.trauer.de/Traueranzeige/Kirsten-Erl


VIELEN Dank
<!-- Lass die nachfolgende Zeile am ENDE deiner Frage stehen. Sie wird in deine Signatur umgewandelt. -->
<!-- Lass die nachfolgende Zeile am ENDE deiner Frage stehen. Sie wird in deine Signatur umgewandelt. -->
--[[Spezial:Beiträge/84.182.211.160|84.182.211.160]] 15:15, 30. Sep. 2020 (CEST)
--[[Spezial:Beiträge/5.146.169.43|5.146.169.43]] 14:32, 1. Mär. 2017 (CET)
:Reicht das als Quelle? Es könnte ja zufällig eine nicht prominente Kirsten Erl mit dem selben Geburtsdatum geben. Weder [[Kirsten (Vorname)|Kirsten]] noch [[Erl]] sind seltene Namen. --[[Benutzer:MrBurns|MrBurns]] ([[Benutzer Diskussion:MrBurns|Diskussion]]) 14:38, 1. Mär. 2017 (CET)
:<halbwissen type="bin_kein_ornithologe"> Der Geruchssinn ist nur bei wenigen Vogelarten hoch genug für die Jagd entwickelt. Beute im Boden wird über Gehör und Tastsinn (z.B. in der Zunge, wenn der Schnabel im Boden steckt) geortet. </halbwissen> --[[Benutzer:Kreuzschnabel|Kreuz]][[Benutzer Diskussion:Kreuzschnabel|schnabel]] 15:40, 30. Sep. 2020 (CEST)
:::Und zu [[Dülmen]] haben beide dann auch irgendwie eine Beziehung?! --[[Benutzer:Elrond|Elrond]] ([[Benutzer Diskussion:Elrond|Diskussion]]) 14:41, 1. Mär. 2017 (CET)
::Trag es doch selber ein [[Kirsten Erl]] --[[Benutzer:Elrond|Elrond]] ([[Benutzer Diskussion:Elrond|Diskussion]]) 14:39, 1. Mär. 2017 (CET)
:::Ich hab das mal erledigt. Name + Geburtsdaten + Ort (Dülmen/Essen) sollten als Identitätsmerkmale ausreichen. Ansonsten ist von Fr. Erl aus der Zeit nach ihrer TV-Präsenz auch nicht viel zu finden. --[[Benutzer:Tsungam|Magnus]] [[BD:Tsungam|(Diskussion)]] 15:46, 1. Mär. 2017 (CET)
::::Ich sehe es bis heute nicht redaktionell bestätigt. Wer kann eine Anzeige aufgeben? Die Seite gehört einem Medienunternehmen aus Bremen. Google blendet einige Suchtreffer aus. --<span style="color:#00A000;">Hans Haase ([[BD:Hans Haase|有问题吗]])</span> 17:23, 1. Mär. 2017 (CET)
:::::[[wp:Q]] fordert keine redaktionelle Bestätigung, sondern nur „überprüfbare Informationen aus zuverlässiger Literatur“. Insofern ist eine auf einem Trauerportal wiedergegebene Todesanzeige einer renommierten Zeitung erstens überprüfbar und zweitens zuverlässig, zumal diese Zeitung aufgrund ihrer Geschichte selbst von Rübenkresseanhängern konkludent als zuverlässig eingestuft wird. --[[user:Rotkaeppchen68|R<span style="color:red">ô</span>tkæppchen₆₈]] 02:03, 2. Mär. 2017 (CET)


Ich weiss aus eigener Erfahrung, das seriöse Zeitungen das überprüfen. Traueranzeige des Unternehmens ging nur nach Angabe des Bestattungsunternehmens, das dann die Tatsachen bestätigt hat. --[[Benutzer:Eingangskontrolle|Eingangskontrolle]] ([[Benutzer Diskussion:Eingangskontrolle|Diskussion]]) 10:05, 2. Mär. 2017 (CET)
:: Ich tippe auf die Augen + Erfahrung - wie auch bei der Amsel. Zu hören gibt es da selten was und der Geruchssinn ist, wie der Kreuzschnabel schon sagt, bei Vögeln eher unterentwickelt. Man erkennt ja feuchte und trockene Stellen sowie [[Zeigerpflanzen]] - auch im anderen Sinne, daß bestimmte Tierchen immer im Wurzelbereich bestimmter Planzen rumgraben. --[[Benutzer:Elop|Elop]] 16:31, 30. Sep. 2020 (CEST)


== Google Instant ==
:::Ich habe einen wissenschaftlichen Beweis! [https://www.gocomics.com/peanuts/1961/11/27] --[[Benutzer:Kreuzschnabel|Kreuz]][[Benutzer Diskussion:Kreuzschnabel|schnabel]] 17:05, 30. Sep. 2020 (CEST)
::::[https://education.abc.net.au/newsandarticles/blog/-/b/3410946/curious-kids-how-do-magpies-detect-worms-and-other-food-underground Sehen, hören]. [[Benutzer:Odore di spirito adolescenziale|Odore di spirito adolescenziale]] ([[Benutzer Diskussion:Odore di spirito adolescenziale|Diskussion]]) 13:30, 1. Okt. 2020 (CEST)


Die Autovervollständigung beim Eintippen in das kleine Suchfeld meines Browsers (aktueller Firefox) oben rechts funktioniert nicht mehr, obwohl ich nichts geändert habe und "Suchen mit Google" eingestellt ist. Auch bei deaktiviertem Adblock tut sich nichts. Woran könnte das liegen?--[[Benutzer:Hubon|Hubon]] ([[Benutzer Diskussion:Hubon|Diskussion]]) 15:39, 1. Mär. 2017 (CET)
== Wurden durch den Lockdown Krankheiten ausgerottet? Zumindest in einigen Regionen temporär? ==
:Hast du das schon durchgearbeitet: [https://support.google.com/websearch/answer/186645?hl=de]? --[[Benutzer:Nurmalschnell|Nurmalschnell]] ([[Benutzer Diskussion:Nurmalschnell|Diskussion]]) 15:58, 1. Mär. 2017 (CET)
::Ja, danke. Bei mir steht "aktiviert". Und bei der Google-Seite selbst funktioniert es auch, aber eben nicht (wie zuvor) bei dem kleinen Suchfeld oben rechts. ???--[[Benutzer:Hubon|Hubon]] ([[Benutzer Diskussion:Hubon|Diskussion]]) 16:22, 1. Mär. 2017 (CET)
:::Hast Du Autocomplete in den Firefox-Einstellungen aktiviert?[http://kb.mozillazine.org/Disabling_autocomplete_(Firefox)] --[[user:Rotkaeppchen68|R<span style="color:red">ô</span>tkæppchen₆₈]] 18:29, 1. Mär. 2017 (CET)
::::Jetzt geht's wieder! Die Lösung steht auf [https://support.mozilla.org/t5/Firefox-Display-and-Appearance/Use-popular-search-suggestions-in-the-Firefox-Search-bar/td-p/525#w_enabling-or-disabling-search-suggestions dieser Seite]. Bloß weiß ich jetzt nicht wirklich, warum sich das offenbar von alleine umgestellt hat. Kann so etwas denn auch ohne eigenes Zutun passieren oder übernimmt hier etwa langsam die Netz-KI die Weltherrschaft...?--[[Benutzer:Hubon|Hubon]] ([[Benutzer Diskussion:Hubon|Diskussion]]) 04:01, 2. Mär. 2017 (CET)


== Hochschulregeln der Birzait Universität ==
Wurde durch den [[COVID-19]]-[[Lockdown]] bestimmte Krankheiten/Infektionen eventuell ausgerottet bzw. zumindest temporär in einigen Ländern stark auf beinahne null gesenkt? Auf die Frage bin gekommen, als ich [https://www.neues-deutschland.de/artikel/1142198.kaum-grippefaelle-auf-der-suedhalbkugel.html las, dass die Grippewelle auf der Südhalbkugel] dank Hygiene/Lockdown ausgefallen ist.
--[[Spezial:Beiträge/2003:EC:BF21:5185:8CD1:81AC:C481:CBBF|2003:EC:BF21:5185:8CD1:81AC:C481:CBBF]] 21:55, 30. Sep. 2020 (CEST)


Im Artikel zur Birzait-Universität wird dargestellt, dass die Universität "Judenrein" zu sein hat, und das "aufgrund einer entsprechenden Rechtsnorm der Hochschule", so dass kein Jude die Universiät betreten darf. Dazu habe ich keine Regelungen auf den Seiten der Universität gefunden. Weiß einer von Ihnen, wo ich diese Hochschulregel finden kann? Über eine Auskunft wäre ich sehr dankbar.
: Derartiges wurde AFAIK bislang nicht berichtet. Stark zurückgegangen sind in einigen Regionen Todesfälle durch Autounfälle und gewalttätige Übergriffe. Der Rückgang der Luftverschmutzung dürfte sich positiv auf div. Lungenerkrankungen und einige Formen von Krebs ausgewirkt haben, aber dafür fehlen noch Datenauswertungen. Influenza ist hierzulande schon in der letzten Saison runter gegangen, sogar schon vor dem Lockdown, weil die Leute von sich aus Kontakte vermieden haben. In der kommenden Saison wird das ähnlich sein, daß Influenza schwach ausfällt und tlw. durch Covid-19 ersetzt wird, aber ausgerottet wird davon höchstwahrscheinlich nichts. --[[Spezial:Beiträge/178.4.185.67|178.4.185.67]] 13:20, 1. Okt. 2020 (CEST)
--[[Spezial:Beiträge/88.76.146.115|88.76.146.115]] 16:32, 1. Mär. 2017 (CET)
:: "Ausgerottet" nicht, sonst würde es keine folgende Wellen geben, meines Wissens gelang das der Menschheit bisher nur bei [[Pocken]] durch eine wirksame Impfung. Selbst heute trifft die Pest noch zeitweise auf, wie kürzlich etwa in China. Beim Thema Grippe/Influenza kommt noch dazu, dass es dort zahlreiche Mutationen bzw Virenstämme gibt, "die Grippe/die Influenza" gibt es einfach nicht. Aber es stimmt, dass auch auf der Nordhalbkugel die letzte Welle frühzeitig schon im Februar/März 2020 beendet wurde.--[[Spezial:Beiträge/2A02:908:1B54:47A0:4464:7E2E:F77F:AB06|2A02:908:1B54:47A0:4464:7E2E:F77F:AB06]] 20:13, 1. Okt. 2020 (CEST)
::: Ausrotten funktioniert deswegen nicht, weil Influenza eine aviäre Zoonose ist. Aber es ist eine interessante Frage, ob die Coronamaßnahmen die Reproduktionsrate soweit unter eins drücken können, daß die saisonale Grippewelle deswegen praktisch ausfällt. Mutationen: Ich hatte Ärger mit dem Hausarzt - dessen Schuld - und deswegen Kontakt mit der Ärztekammer. Dort argumentierte man logisch, der Arzt könnte wohl sanktioniert werden, aber ich kaum wollen, dann von ihm weiter behandelt zu werden, also sollte ich mir doch auf jeden Fall einen neuen suchen. Ich hatte dann Bedenken, wenn ich nicht alsbald geimpft würde, könnte der Influenzaimpfstoff ausgehen, wie es im Frühjahr der Fall war, als Spahn die Pneumokokkenimpfung empfohlen hatte. Die Antwort war, man ginge nicht von einem drohenden Mangel aus, und außerdem sollte besser später als früh geimpft werden, weil sich das Virus noch verändere. Verstehe ich nicht: Das Virus wohl schon, aber der Impfstoff doch nicht mehr... --[[Spezial:Beiträge/95.116.103.117|95.116.103.117]] 23:46, 1. Okt. 2020 (CEST)


Kannst Du mal einen Link reinstellen! Birzait-Universität finde ich nicht. --[[Spezial:Beiträge/80.187.96.246|80.187.96.246]] 16:42, 1. Mär. 2017 (CET)
== Formaldehyd Kohlehydrat ==


:Über Trottel-Institutionen, die sich fälschlich "Universität" nennen, diskutieren wir hier nicht. So einfach ist das. --[[Spezial:Beiträge/109.100.148.154|109.100.148.154]] 16:43, 1. Mär. 2017 (CET)
Zählt Formaldehyd zu den Kohlehydraten? Immerhin enthält es Kohlenstoff, Wasserstoff und Sauerstoff im stöchiometrischen Verhältnis 1:2:1, so wie die als Zucker bekannten Kohlehydrate. eine Aldose ist es ebenfalls und eine Carbonylgruppe enthält es auch. --[[Spezial:Beiträge/2003:F7:DF17:7D00:2847:5EEC:4251:9D8C|2003:F7:DF17:7D00:2847:5EEC:4251:9D8C]] 23:50, 30. Sep. 2020 (CEST)
::[[Universität Bir Zait]] ... <small style="color:grey"><b>'''GEEZER'''</b></small><sup>[[BD:Grey Geezer|<span style="color:grey"> … nil nisi bene</span>]]</sup> 16:46, 1. Mär. 2017 (CET)
:Nein. Kohlenhydrate haben die Summenformel C<sub>n</sub>H<sub>2n</sub>O<sub>n</sub> mit n ≥ 3. --[[Benutzer:Blutgretchen|Blutgretchen]] ([[Benutzer Diskussion:Blutgretchen|Diskussion]]) 00:11, 1. Okt. 2020 (CEST)
::Präziser: ''Allgemein liegen Kohlenhydrate vor, wenn in einem Stoff mindestens eine Aldehydgruppe bzw. Ketogruppe und mindestens zwei Hydroxygruppen zu finden sind.'' --[[Benutzer:Elrond|Elrond]] ([[Benutzer Diskussion:Elrond|Diskussion]]) 12:24, 1. Okt. 2020 (CEST)
:(BK)Linkservice: [[Universität Bir Zait]]. Von "judenrein" steht dort nichts. In dem als Einzelnachweis angeführten Artikel von Amira Hass steht nicht, dass Juden im Allgemeinen die Universiät nicht betreten dürften, sondern jüdische Israelis. --[[Benutzer:Digamma|Digamma]] ([[Benutzer Diskussion:Digamma|Diskussion]]) 16:48, 1. Mär. 2017 (CET)
:::Dann ist [https://pubchem.ncbi.nlm.nih.gov/compound/6453984 das hier] (C4H8O3) ein Kohlenhydrat !?! [[Benutzer:Odore di spirito adolescenziale|Odore di spirito adolescenziale]] ([[Benutzer Diskussion:Odore di spirito adolescenziale|Diskussion]]) 13:40, 1. Okt. 2020 (CEST)
''...dass Juden im Allgemeinen die Universiät nicht betreten dürften, sondern jüdische Israelis'' Wo ist denn der Unterschied? Übrigens ist die HP schwer erreichbar, Seitenwechsel fast unmöglich. Also bislang kein eigener Eindruck. --[[Spezial:Beiträge/80.187.96.246|80.187.96.246]] 17:00, 1. Mär. 2017 (CET)
::::Nö, da fehlt ein Sauerstoff. --[[Benutzer:Elrond|Elrond]] ([[Benutzer Diskussion:Elrond|Diskussion]]) 21:27, 1. Okt. 2020 (CEST)
:Unterschied: Angehörige der Besatzungsmacht. --[[Benutzer:Digamma|Digamma]] ([[Benutzer Diskussion:Digamma|Diskussion]]) 17:12, 1. Mär. 2017 (CET)
::::::Nach der Aussage
:Naja. 14,3 Millionen Juden weltweit, davon 6,2 Millionen Israelis. Mathe? [[Benutzer:Marcus Cyron|Marcus Cyron]] [[Benutzer Diskussion:Marcus Cyron|Reden]] 18:17, 1. Mär. 2017 (CET)
::::::Präziser: ''Allgemein liegen Kohlenhydrate vor, wenn in einem Stoff mindestens eine Aldehydgruppe bzw. Ketogruppe und mindestens zwei Hydroxygruppen zu finden sind'' wäre es ein Kohlenhydrat, denn es liegen eine Aldehydgruppe und zwei Hydroxygruppen vor. [[Benutzer:Odore di spirito adolescenziale|Odore di spirito adolescenziale]] ([[Benutzer Diskussion:Odore di spirito adolescenziale|Diskussion]]) 08:39, 2. Okt. 2020 (CEST)
:::::::Dann extra für Dich: ''Kohlenhydrate haben die Summenformel C<sub>n</sub>H<sub>2n</sub>O<sub>n</sub> mit n ≥ 3'' '''und''' ''Es liegen Kohlenhydrate vor, wenn in einem Stoff mindestens eine Aldehydgruppe bzw. Ketogruppe und mindestens zwei Hydroxygruppen zu finden sind'' Es gibt durchaus Verbindungen, die die Summenformel C<sub>n</sub>H<sub>2n</sub>O<sub>n</sub> haben und n ≥ 3, aber die Bedingung ''mindestens eine Aldehydgruppe bzw. Ketogruppe und mindestens zwei Hydroxygruppen'' nicht erfüllen. Die gehören dann nicht zu den Kohlenhydraten. --[[Benutzer:Elrond|Elrond]] ([[Benutzer Diskussion:Elrond|Diskussion]]) 11:13, 2. Okt. 2020 (CEST)
:::::@IP 2003:F7 …Gegenbeispiel für deine Formel ist schon der gewöhnliche Zucker, die [[Saccharose]]. Die KH sind nicht nur über die Summenformel definiert, vgl. [[Kohlenhydrat#Etymologie]]. --[[Benutzer:Dioskorides|Dioskorides]] ([[Benutzer Diskussion:Dioskorides|Diskussion]]) 21:59, 1. Okt. 2020 (CEST)
::::::Die Formel C<sub>n</sub>H<sub>2n</sub>O<sub>n</sub> gilt streng für [[Monosaccharide]], [[Oligosaccharide|Oligo-]] oder [[Polysaccharide]] leiten sich davon ab, dass durch Abspaltung ([[Kondensationsreaktion|Kondensation]]) von Wasser mehr oder weniger lange Ketten gebildet werden. Die gehören dann aber trotzdem zu den Kohlehydraten. --[[Benutzer:Elrond|Elrond]] ([[Benutzer Diskussion:Elrond|Diskussion]]) 11:13, 2. Okt. 2020 (CEST)


Oh, meine Wortwahl war wohl nicht so ordentlich, Ich muß also eine Unterscheidung zwischen Juden aus Israel und normalen Juden machen? Das ändert jedoch nichts an meiner Frage: Wo finde ich die Quelle, dass Juden aus Israel die Bir Zait - Universiätät nicht betreten dürfen?--[[Spezial:Beiträge/88.76.146.115|88.76.146.115]] 17:06, 1. Mär. 2017 (CET)
= 1. Oktober =
:Ich vermute mal, dass die das nicht auf die Homepage schreiben werden. --[[Benutzer:Digamma|Digamma]] ([[Benutzer Diskussion:Digamma|Diskussion]]) 17:15, 1. Mär. 2017 (CET)a
:''...Unterscheidung zwischen Juden aus Israel und normalen Juden machen?'' Was sind denn "normale Juden" Hallo! --[[Spezial:Beiträge/80.187.96.246|80.187.96.246]] 17:35, 1. Mär. 2017 (CET)
::Die HP der Uni ist inzwischen bekannt und bei mir eof hierzu!! --[[Spezial:Beiträge/80.187.96.246|80.187.96.246]] 17:40, 1. Mär. 2017 (CET)


Bevor das hier noch ausartet, erlaube ich mir, (auch) nochmal auf die Frage zu verweisen. Kann also jemand eine solche Regelung bestätigen und weiß auch, wo sie steht? Fall ja, dann bitte kurz erläutern. Falls nein, ist das ja nicht schlimm. :-) VG --[[Benutzer:Apraphul|Apraphul]] <small><sup> [[Benutzer Diskussion:Apraphul|Disk]] </sup> <sub> [[Benutzer:Apraphul/WP:SNZ|WP:SNZ]]</sub></small> 17:43, 1. Mär. 2017 (CET)
== "./." in Kontenblättern etc. als Verdeutlichung von "hier steht nichts" ==
:Damit wir nicht alle zugleich suchen: Die Universität hat zahlreiche ihrer Satzungen auf der Website veröffentlicht. Bei oberflächlicher Durchsicht (per Google-Übersetzung) der vielversprechendsten Dokumente habe ich eine entsprechende Regelung nicht gefunden. Die Grundordnung der Universität ist allerdings nur in der Fassung von 2015 bereitgestellt [https://ritaj.birzeit.edu/university-laws/?root_node_id=100#] - nachdem der Vorfall 2014 war und die Regelung laut einer der Quellen "seit zwei Dekaden" bestand, kann es natürlich sein, dass sie nach dem Vorfall (bzw. wegen des Vorfalls) geändert wurde. Die Grundordnung stammt wohl von 1979 und wurde erstmals 1989 geändert - mein heißester Kandidat wäre also die zweite Fassung. --[[Benutzer:Rudolph Buch|Rudolph Buch]] ([[Benutzer Diskussion:Rudolph Buch|Diskussion]]) 18:46, 1. Mär. 2017 (CET)


: Eben, der entsprechende Satz ist im Konjunktiv (indirekte Rede) geschrieben und macht damit deutlich, dass er keine absoluten Wahrheiten verkündet. Dass Frau Hass von der Konferenz ausgeschlossen wurde, ist wohl ein unbestrittenes Faktum, und dass es deshalb war, weil sie Israelin ''und'' Jüdin ist, ist nun nicht allzu abwegig... --[[Benutzer:Gretarsson|Gretarsson]] ([[Benutzer Diskussion:Gretarsson|Diskussion]]) 18:39, 1. Mär. 2017 (CET)
Ich habe in alten (handschriftlichen) Kontenbüchern u.Ä. immer wieder gesehen, dass leere Felder mit ./. markiert wurden. Klar, da kann keiner mehr etwas reinschreiben, so wie auf Schecks ja auch "- 300 DM -" steht, damit keiner noch eine Eins dazuschreibt (plus dem ausgeschriebenen Wort Dreihundert). Aber weder die allwissende Müllhalde noch wir haben etwas zu dem ./., schade. Mich würde mal interessieren, wie das "entstanden" ist. So eteas Ähnliches sieht man auf diesem Bild: [https://c8.alamy.com/compde/ec8r5w/nahaufnahme-eines-buchhalters-uberprufen-die-zahlen-in-einer-alten-handschriftlichen-buchhaltung-ledger-summen-in-pfund-schillinge-und-pence-ec8r5w.jpg] [[Benutzer:Flossenträger|Flossenträger]] 11:56, 1. Okt. 2020 (CEST)
::Nur eben nicht belegt und wenn man dies noch eine Ecke weitertreiben lässt, ist Wiki der Beleg dafür!! --[[Spezial:Beiträge/80.187.96.12|80.187.96.12]] 20:56, 1. Mär. 2017 (CET)
:Für "hier nix mehr" wird eigentlich die [[Buchhalternase]] genommen?! . Das ./. kenne ich als alternatives Minus-Zeichen. (Wobei ich bei beiden DIngen keine Ahnung habe woher/warum) ...[[Benutzer:Sicherlich|<span style="color:#348853">Sicherlich</span>]] <sup> [[Benutzer Diskussion:Sicherlich|Post]] </sup> 12:06, 1. Okt. 2020 (CEST)
::[[Minuszeichen]] kennt ./. auch. Aber leider kann man keine Weiterleitung anlegen (zumindest ich nicht ;o) ) ...[[Benutzer:Sicherlich|<span style="color:#348853">Sicherlich</span>]] <sup> [[Benutzer Diskussion:Sicherlich|Post]] </sup> 12:10, 1. Okt. 2020 (CEST)
::: Kenne das auch aus archivischen Amts- und Rechnungsbüchern und habe gerade in einer Fachgruppe nachgefragt. --[[Benutzer:Aalfons|Aalfons]] ([[Benutzer Diskussion:Aalfons|Diskussion]]) 12:48, 1. Okt. 2020 (CEST)
::::Auch bekannt als juristisches Gegen, zum Beispiel enthalten in [[DIN 5008]], zur trennenden Auflistung der Prozessbeteiligten. --[[Benutzer:BlackEyedLion|BlackEyedLion]] ([[Benutzer Diskussion:BlackEyedLion|Diskussion]]) 12:58, 1. Okt. 2020 (CEST)
::Also das Minuszeichen (DIN 5008:2020-03, Abschnitt 10.2 Subtraktionszeichen) ohne einen Wert kommt eher nicht in Frage. Wenn aus dem Kontext der Aufzeichnungen hervorgeht, dass damit Felder entwertet wurden, dann hat es sich damit. Die Frage zur Entstehung ist berechtigt. IMHO hat es zu jeder Zeit gewisse Konventionen gegeben etwas zu schreiben. In der Stenographie ist es gestattet (und üblich gewesen) Sonderzeichen nach eigenem Gusto zu verwenden. LG --[[Spezial:Beiträge/80.187.106.99|80.187.106.99]] 13:08, 1. Okt. 2020 (CEST)
:Die Buchhalternase (lustiger Name) kenne ich nur für große Flächen / viele Felder z.B. Am Ende der Rechnung um das Blatt "vollzumachen". Allerdings bin ich auch kein Buchhalter, deswegen bin ich da etwas vorsichtig mit ist so oder so... [[Benutzer:Flossenträger|Flossenträger]] 13:01, 1. Okt. 2020 (CEST)
::Die Buchhalternase verwende ich z.B. im Praktiukm, um bei Versuchsende beim Vortestat die Protokollblätter und Laborjournale zu markieren, damit später keine weiteren Daten nachgetragen werden können. --[[Benutzer:Elrond|Elrond]] ([[Benutzer Diskussion:Elrond|Diskussion]]) 13:18, 1. Okt. 2020 (CEST)
:::ja, ich kenne es auch nur für mehrere Zeilen ...[[Benutzer:Sicherlich|<span style="color:#348853">Sicherlich</span>]] <sup> [[Benutzer Diskussion:Sicherlich|Post]] </sup> 13:45, 1. Okt. 2020 (CEST)
::Zur Eingangsfrage der Zeichenfolge '''"./."'''. Vermutlich sind die Beispiele aus den Kontoblättern älter als die „Richtlinien zur Organisation der Buchführung“ vom 11. 11. 1937. Die [https://books.google.de/books?id=z0Z8lR3LeZcC&lpg=PA46&dq=%22Vakatstrich%22&hl=de&pg=PA47#v=onepage&q&f=false Buchhalternase aka Vokatstrich] nutze ich gern in ellenlangen Formularen um nicht unnütze Kreuzchen zu machen. --[[Spezial:Beiträge/80.187.110.113|80.187.110.113]] 14:12, 1. Okt. 2020 (CEST)


::Der Nebensatz "da sie sich als israelische Jüdin aufgrund einer entsprechenden Rechtsnorm der Hochschule nicht dort aufhalten dürfe" ist seit meiner heutigen [https://de.wikipedia.org/w/index.php?title=Universit%C3%A4t_Bir_Zait&diff=163137189&oldid=153539706 Änderung] im Konjunktiv. Davor stand da "darf". Ich habe "darf" durch "dürfe" ersetzt, weil die einzige Quelle der Zeitungsartikel ist, in dem Amira Hass auch nur die Aussagen von Mitarbeitern der Uni zitiert.
Ist ein [[Auslassungszeichen]] für "keine Eintragung" Punkt [[Virgel]] Punkt --[[Spezial:Beiträge/178.115.131.248|178.115.131.248]] 14:18, 1. Okt. 2020 (CEST)
::Also: Beleg ist der Haaretz-Artikel von Amira Hass, aber der gibt auch nur wieder, dass andere behaupten, es gäbe so eine Bestimmung. Belegt ist aber, dass Amira Hass unter Berufung auf so eine Bestimmung vom Campus verwiesen wurde. --[[Benutzer:Digamma|Digamma]] ([[Benutzer Diskussion:Digamma|Diskussion]]) 21:10, 1. Mär. 2017 (CET)


::: Hättest du es nicht geändert, hätte ich es getan. So wie der Satz jetzt dort steht, kann er stehen bleiben. Wenn sich Frau Hass auf Mitarbeiter der Uni bezieht, dann ist das erst mal so hinzunehmen. Die werden ihr ja hoffentlich keinen Unsinn erzählt haben. Zudem ist Hass offensichtlich Links und als Mitarbeiterin von Haaretz nun nicht als zionistisch und antipalästinensisch einzustufen, im Gegenteil, dürfte sie zu den israelischen Juden gehören, die von zionistischen Hardlinern gerne als „jüdische Selbsthasser“ tituliert werden. Im Übrigen kann man [http://www.telospress.com/academic-freedom-in-palestinian-universities/ hier] nachlesen, dass [[Ilan Pappe]] für einen Vortrag an die Bir-Zait-Uni eingeladen wurde und aufgrund der betreffenden Vorschrift diesen „off campus“ halten musste. Hass war also nicht die einzige Betroffene, sondern leider „nur“ im Vorfeld schlecht informiert oder ein „Opfer“ des Umstandes, dass man wegen ihr allein nicht die Konferenz einfach woanders abhalten konnte. --[[Benutzer:Gretarsson|Gretarsson]] ([[Benutzer Diskussion:Gretarsson|Diskussion]]) 22:52, 1. Mär. 2017 (CET)<small>; nachträgl. erg. --[[Benutzer:Gretarsson|Gretarsson]] ([[Benutzer Diskussion:Gretarsson|Diskussion]]) 00:12, 2. Mär. 2017 (CET)
::Die Verwendung von Zeichen in der Buchhaltung wurde/ist schon normiert. Hier in [https://books.google.de/books?id=5fwKJ5pv-LEC&dq=%22Abschlussstrich%22%20%22Buchhaltung%22&hl=de&pg=PA64#v=onepage&q&f=false „Lection 12“ (1877)] werden Verwendung von Abschlussstrich und Schrägstrich erklärt. --[[Spezial:Beiträge/80.187.110.113|80.187.110.113]] 14:27, 1. Okt. 2020 (CEST)
</small>


:Ich merke mal an, dass Ramallah in der Area A liegt, jüdischen Israelis dort der Zutritt also normalerweise von israelischer Seite ohnehin untersagt ist. Insofern wird es nicht gerade viele zufällige Besucher geben, für die man die Regelung öffentlich machen müsste. --[[Benutzer:Chricho|Chricho]] [[BD:Chricho|¹]] [//de.wikipedia.org/w/?title=BD:Chricho&amp;action=edit&amp;section=new ²] [[Benutzer:Chricho/Keine_Verbesserung|³]] 14:23, 2. Mär. 2017 (CET)
Vermutung aus einer Archivfachgruppe: ./. ging aus . für 0 hervor und der Strich trennt verschiedene Einheiten. 5/8 waren fünf Kronen acht Groschen, ./8 nur die acht Groschen, ./. nix. Eigene Überlegung: Wenn die Null als Punkt geschrieben wurde, konnte sie trotz unsauberer Schreibweise auch nicht mit einer anderen Zahl verwechselt werden. --[[Benutzer:Aalfons|Aalfons]] ([[Benutzer Diskussion:Aalfons|Diskussion]]) 17:36, 1. Okt. 2020 (CEST)


== Grenzschutz in Österreich ==
In der Musiknotation ist es ein [[Faulenzer (Musik)]] in der Buchhaltung ein "[[Kaufmännisches Minus]]" --[[Spezial:Beiträge/178.115.131.248|178.115.131.248]] 22:05, 1. Okt. 2020 (CEST)


Ich habe jetzt eine längere Zeit gesucht, finde es aber nicht. Welche Institution entspricht in Österreich der Bundespolizei in Deutschland, wem obliegt die Grenzsicherung?
== Wer/was hat ein Dekolleté? ==
--[[Benutzer:Marcus Cyron|Marcus Cyron]] [[Benutzer Diskussion:Marcus Cyron|Reden]] 18:10, 1. Mär. 2017 (CET)
:[[:en:Template:Border guards]] weiß es. --[[user:Rotkaeppchen68|R<span style="color:red">ô</span>tkæppchen₆₈]] 18:22, 1. Mär. 2017 (CET)
::Vielleicht kann man das noch bei [[Österreichische Grenzübergänge in die Nachbarstaaten]] eintragen. --[[Benutzer:Stefan|Stefan »Στέφανος«]]&#8201;[[Benutzer Diskussion:Stefan|⸘…‽]] 18:24, 1. Mär. 2017 (CET)
::Leider steht es nicht wirklich im Artikel. Da wird einmal ein Gesetz erwähnt, das war es aber auch schon. Bin echt überrascht, daß sich das in unserer Wikipedia bislang nicht wirklich als Aussage findet. [[Benutzer:Marcus Cyron|Marcus Cyron]] [[Benutzer Diskussion:Marcus Cyron|Reden]] 18:28, 1. Mär. 2017 (CET)
:::Das macht die [http://www.bmi.gv.at/cms/bmi_fremdenpolizei/ Fremdenpolizei]. --[[Benutzer:Ralf Roletschek|M@rcela]] [[Bild:Miniauge2.gif|27px]] 18:30, 1. Mär. 2017 (CET)
:::(BK) Hoffentlich war meine [[Special:Diff/163141287|Änderung]] jetzt nicht völlig falsch. [[User:Plani|Plani]] bitte auf den Plan, [[User:Plani|Plani]] bitte. --[[Benutzer:Stefan|Stefan »Στέφανος«]]&#8201;[[Benutzer Diskussion:Stefan|⸘…‽]] 18:31, 1. Mär. 2017 (CET)
::::Das ist freilich alles nicht ganz hundertprozentig korrekt. Der Schutz der österreichischen Außengrenzen obliegt seit der Zusammenführung von [[Zollwache]] und Polizei dem Wachkörper [[Bundespolizei (Österreich)|Bundespolizei]], also der "ganz normalen" Polizei. Das ist freilich nur die halbe Wahrheit, weil spezialisierte AGM (Schengen-Ausgleichsmaßnahmen-)Dienststellen gegründet wurden, deren Beamte sich großteils aus ehemaligen Mitgliedern der Zollwache rekrutierten. Diese sind schwerpunktmäßig mit der Grenzüberwachung betraut. Nachdem Österreich aber ausschließlich von anderen Schengen-Mitgliedsstaaten umgeben ist, sind tatsächliche Grenzkontrollen an Grenzübergängen eine Ausnahme. Wie der Name "Schengen-Ausgleichsmaßnahmen" schon erahnen lässt, finden Kontrollen eher im Grenzhinterland durch die sog. [[Schleierfahndung]] statt. Die einzigen tatsächlichen Grenzkontrollen in diesem Sinne gibt es noch an den österreichischen Flughäfen, wobei diese eben von Beamten der Bundespolizei besetzt sind. Ach ja, die Angelegenheiten des Zollwesens an Flughäfen und im Grenzhinterland übernimmt übrigens nicht die Bundespolizei, sondern die [[Operative Zollaufsicht]]. Ich hoffe, mit dieser Antwort geholfen zu haben. Beste Grüße, [[Benutzer:Plani|Plani]] ([[Benutzer Diskussion:Plani|Diskussion]]) 11:10, 2. Mär. 2017 (CET)
::::: + der Einsatz des [[Bundesheer|ÖBH]] [http://www.bundesheer.at/cms/artikel.php?ID=8301], [https://kurier.at/chronik/burgenland/assistenzeinsatz-sie-stehen-wieder-an-der-grenze/197.945.799], oder ist der vorbei? --[[Spezial:Beiträge/213.147.162.52|213.147.162.52]] 13:02, 2. Mär. 2017 (CET)
::::::Das Österreichische Bundesheer steht fallweise (Stichwort: Flüchtlingskrise) an der Grenze im Assistenzeinsatz für die Bundespolizei. Grundsätzlich zum Schutz der Grenze zuständig ist das Bundesheer aber nicht. Beste Grüße, [[Benutzer:Plani|Plani]] ([[Benutzer Diskussion:Plani|Diskussion]]) 13:31, 2. Mär. 2017 (CET)


== Schraubglas öffnen ==
Kürzlich habe ich weiblichen Verwandten erzählt, eine bestimmte Bluse habe "kein Dekolleté" gehabt - was bei ihnen für Lacher sorgte, weil ihrer Meinung nach wenn schon eine ''Frau'' ein Dekolleté hat - die Bluse selbst hat dann einen ''Ausschnitt''. Ist das sprachlich so getrennt? Der Artikel [[Dekolleté]] sieht es derzeit nicht so. --[[Benutzer:KnightMove|KnightMove]] ([[Benutzer Diskussion:KnightMove|Diskussion]]) 12:22, 1. Okt. 2020 (CEST)
: Verwandtschaft kann man sich nicht aussuchen. Bildungsnotstand!--[[Benutzer:Gadacz|Klaus-Peter ''<small>(<sup>auf</sup><small>und</small><sub>davon</sub>)</small>'']] 12:28, 1. Okt. 2020 (CEST)
::Siehe https://dict.leo.org/franz%C3%B6sisch-deutsch/d%C3%A9collet%C3%A9: ''décolleté'' „der Halsausschnitt“. --[[Benutzer:BlackEyedLion|BlackEyedLion]] ([[Benutzer Diskussion:BlackEyedLion|Diskussion]]) 13:00, 1. Okt. 2020 (CEST)
:Ich denke, die Bedeutung des Begriffes hat sich von (auch Ausschnitt) zu "Oberweite" verschoben. Ich kenne ich auch fast nur als Synonym für die Oberweite und würde den Begriff auch eher nur auf Frauen beziehen. Aber nun ja, waren vermutlich etwas jüngeren Alters die Damen, oder? [[Benutzer:Flossenträger|Flossenträger]] 13:04, 1. Okt. 2020 (CEST)
::Nicht wirklich - schon komfortabel in der zweiten Lebenshälfte. Aber ganz ehrlich, einen schönen weiblichen Vorbau würde ich nie als Dekolleté bezeichnen, wenn Stoff drüber ist. Macht das die Jugend heute? --[[Benutzer:KnightMove|KnightMove]] ([[Benutzer Diskussion:KnightMove|Diskussion]]) 13:12, 1. Okt. 2020 (CEST)
Wenn der Herr Geheimrath schrieb, dass sich ihm! der Busen hebt war das allen klar, was gemeint ist, denn der Busen ist (dichterisch und veraltet, ok) auch die männliche Brust bzw. die Stelle zwischen den Brüsten. So teste ich bei Neunmalklugen gerne die sprachliche Kompetenz bzw. den Wortschatz. Aber was verlangt man manchmal von den Mitmenschen! ;-) --[[Benutzer:Elrond|Elrond]] ([[Benutzer Diskussion:Elrond|Diskussion]]) 13:13, 1. Okt. 2020 (CEST)
:Die Duden-Definition ist eindeutig: ''tiefer Ausschnitt an Damenkleidern o. Ä., der Schultern, Brustansatz oder Rücken frei lässt''.
:Selbst ein Mann, der ein ausgeschnittenes Kleid mit reichlich Brusthaaren trägt, hat ein Dekolleté. Eine Google-Bildersuche macht das klar.
:Das ist so wie mit einer Teekanne: Es ist eine Teekanne, ob da nun Tee oder Weisswein - oder garnichts drin ist. [[Benutzer:Odore di spirito adolescenziale|Odore di spirito adolescenziale]] ([[Benutzer Diskussion:Odore di spirito adolescenziale|Diskussion]]) 13:49, 1. Okt. 2020 (CEST)
Das Wort stammt ja aus dem Französischen und bedeutet so viel wie kragenlos, ausgeschnitten oder im technischen Sinn (décolletage) abstechen, abdrehen. Frau kragenlos oder ausgeschnitten? Wohl eher weniger ;0]. --[[Benutzer:Chriusha|Хрюша]] <small> <sup>[[Benutzer Diskussion:Chriusha|? ! ? !]]</sup> </small> 14:03, 1. Okt. 2020 (CEST)


Wenn man ein Schraubglas, z.Bsp. saure Gurken gerade ebends, beim ersten Versuch nicht aufgedreht bekommt, und dann mit der Hand gegen den Glasboden schlägt, und es geht dann auf, was passiert bei diesem Vorgang, daß das Glas dann aufgeht?--[[Benutzer:Sigismund Sülzheimer|Sigismund Sülzheimer]] ([[Benutzer Diskussion:Sigismund Sülzheimer|Diskussion]]) 18:19, 1. Mär. 2017 (CET)
Zweifellos bezeichnet Dekolleté den Ausschnitt z.B. der Bluse, es wird aber auch, und das schon lange, euphemistisch für alles verwendet, was in einem Ausschnitt zu sehen ist, also der Bereich zwischen den Brüsten und deren innenliegende Seiten sowie die Haut oberhalb davon - ähnlich, wie sich ja auch die Bikinizone durchgesetzt hat, auch wenn es gar nicht um Bikinis geht. Wie so oft bei Euphemismen ist es dann nur eine Frage der Zeit, bis die euphemistische Bezeichnung die ursprüngliche überlagert. --[[Benutzer:Studmult|Studmult]] ([[Benutzer Diskussion:Studmult|Diskussion]]) 14:17, 1. Okt. 2020 (CEST)
:Dazu gibts anscheinend unterschiedliche Theorien.[http://www.zeit.de/1997/47/stimmt47.txt.19971114.xml/komplettansicht] --[[Benutzer:MrBurns|MrBurns]] ([[Benutzer Diskussion:MrBurns|Diskussion]]) 18:39, 1. Mär. 2017 (CET)
:::Ich kenne das auch nur als Begriff für den unbedeckten "Vorbau", eben das, was der Stoff freilässt. [[Benutzer:Flossenträger|Flossenträger]] 14:20, 1. Okt. 2020 (CEST)
::Es liegt an der Veränderung des Luftdrucks im Glas. Wenn man dagegen schlägt wird der Deckel entweder von oben oder von unten ganz leicht gewölbt. Dadurch kann ein bisschen Luft eindringen und der Unterdruck, der im Glas herrscht und das Aufmachen vorher erschwert hat wird ausgleichen. Hier ist eine Quellenangabe Das Buch der 100 Fragen und Antworten S.310,311 Verlag :LOEWE ISBN 3-7855-2381-5 1993 Dort steht das auch drin ,bloß hat das Glas dort noch eine Folie zwischen der Marmelade und deem Deckel. --[[Benutzer:Fiver, der Hellseher|Fiver, der Hellseher]] ([[Benutzer Diskussion:Fiver, der Hellseher|Diskussion]]) 18:48, 1. Mär. 2017 (CET)
::Vgl. "Maurerdekolleté" --[[Spezial:Beiträge/178.115.131.248|178.115.131.248]] 14:28, 1. Okt. 2020 (CEST)
:::Danke für die Infos. --[[Benutzer:Sigismund Sülzheimer|Sigismund Sülzheimer]] ([[Benutzer Diskussion:Sigismund Sülzheimer|Diskussion]]) 19:05, 1. Mär. 2017 (CET)
::::Es hilft auf den Schraubdeckel zuschlagen. Die Dichtung verharzt und verklebt. Der Druck wird von den in das Glasgewinde ragenden Blechnasen im Deckel gehalten. Durch den Schlag geben diese nach, da das Metall gespannt ist. Ein Druckunterschied kann durch den Schlag auf die Dichtung spätestens durch Verformung verringert werden. --<span style="color:#00A000;">Hans Haase ([[BD:Hans Haase|有问题吗]])</span> 21:08, 1. Mär. 2017 (CET)
Protipp: Mit einem Buttermesses oder gleichfalls geeigneten Gegenstand an einer Stelle den Deckel seitlich leicht anhebeln. Überlicherweise macht es dann "pfflluuuupppfff" und der Unterdruck ist ausgeglichen, der Deckel kinderleicht abschraubbar. Hat eine extrem bessere Erfolgsquote als das Glas zu prügeln und weniger schmerzhaft an den Händen ist es auch (wenn man nicht mit dem Messer abrutscht). [[Benutzer Diskussion:morty|Benutzerkennung: '''43067''']] 15:01, 2. Mär. 2017 (CET)
::Das ist zwar auch meine Methode der Wahl, speziell bei Sachen die kleben können wie Konfitüre & Co, allerdings hab ich schon erlebt, daß das Glas der Klügere war und nachgab, dann ist das Glas und der Inhalt meist hin. --[[Benutzer:Elrond|Elrond]] ([[Benutzer Diskussion:Elrond|Diskussion]]) 15:41, 2. Mär. 2017 (CET)
: Man muss da nicht wie blöde auf den Glasboden einprügeln. Zumindest bei 720-ml-Gläsern reicht ein ''einfacher'' beherzter Schlag mit dem Handballen völlig aus. Das tut nicht weh und geht drei mal schneller als erstmal einen geeigneten Gegenstand rauszusuchen, dann mit diesem am Deckel rumzuhantieren um sich am Ende vielleicht noch ein Auge auszustechen... --[[Benutzer:Gretarsson|Gretarsson]] ([[Benutzer Diskussion:Gretarsson|Diskussion]]) 15:24, 2. Mär. 2017 (CET)
::Früher stach man ein Loch in den Deckel. Die Ahle dazu hatte man sowieso in der Küchenschublade, wegen der Dosenmilch für den Kaffee. --[[user:Rotkaeppchen68|R<span style="color:red">ô</span>tkæppchen₆₈]] 15:28, 2. Mär. 2017 (CET)


::: Womit dann aber der Deckel hin ist... Und heutzutage wissen viele wahrscheinlich nicht mal, wie man „Ahle“ schreibt oder wie eine aussieht, geschweige denn, dass sie eine besitzen... --[[Benutzer:Gretarsson|Gretarsson]] ([[Benutzer Diskussion:Gretarsson|Diskussion]]) 15:31, 2. Mär. 2017 (CET)<small>; nachträgl. erg. 15:34, 2. Mär. 2017 (CET)</small>
: (BK) Als Dekolleté bezeichnet man einen tiefen Ausschnitt am Hals eines weiblichen Kleidungsoberteils. Bei angezogenem Oberteil läßt dieser Ausschnitt Teile der Brüste und des Busens unbedeckt. Dekolleté, Bluse, Schritt und Loch haben gemeinsam, daß sie sich auf den nicht vorhandenen Teil von etwas beziehen: Dekolleté auf das Fehlen von Stoff, Busen auf den Bereich zwischen den weibl. Brüsten, Schritt auf den Bereich zwischen den Beinen und Loch auf ein von beliebiger Umrandung umschlossenes Fehlen von etwas. Bei einer schambesetzten Sprache liegt es nahe, indirekt zu bleiben. Anstatt die Körperteile direkt zu benennen, wird der gemeinten Bereich lediglich umschrieben. Quillt also die obere Hälfte der Brüste aus dem Halsausschnitt, geziemt es sich dem wohlerzogenen Gegenüber nicht, diese direkt zu loben, wohl aber das Fehlen von Blusenstoff im gemeinten Bereich. Die Bezeichnung "Busen" als Umschreibung für Brüste hat sich schon dermaßen tief in den alltäglichen Gebrauch eingegraben, daß viele nicht einmal mehr wissen, was damit eigentlich gemeint ist, so daß sich die klangvolle Bezeichnung für das Nichts zwischen den Brüsten zunehmend zu einem Synonym für die Brüste als solche wandelt. (Beim Herrn Geheimrat könnte man auch argumentieren, daß sich das Nichts zwischen seinen Brüsten in Ermangelung selbiger über die gesamte Breite des Brustkorbs erstreckt;)
::::Tesa drauf, tschüs. --[[user:Rotkaeppchen68|R<span style="color:red">ô</span>tkæppchen₆₈]] 15:53, 2. Mär. 2017 (CET)
: Die Bedeutung von Dekolleté kann sich hingegen ''nicht'' in Richtung [[Oberweite]] verschoben haben, da dies eine Bezeichnung für den Brustumfang ist. Mit den Maßen für die Oberweite kann ein Schneider kein Dekolleté schneidern et vice versa. Zur "Jugend von heute" schwappt aus den soz. Medien eine Menge an Nonsens rüber. Dekolleté schwimmt da tlw. mit und klingt so edel und gehoben, daß manche das in ihren Sprachgebrauch übernehmen, ohne den geringsten Schimmer zu haben, was damit gemeint ist, aber es klingt irgendwie sexy, ohne vulgär zu wirken. So mutiert dann bspw. auch der "weibl. Pornostar mit den Doppel-F Brüsten" zu einer "intelligenten jungen Frau mit wunderschönem Dekolleté, die selbstbewußt ihren Körper zeigt." Der Unterschied ist: Erstgenannte ist schmuddelig, mit der sollte man sich nicht abgeben, während die Zweitgenannte Gala tauglich ist und sogar im Kinderfernsehen erscheinen kann als Role Model für eine erfolgreiche Businessfrau. Sprache machts's möglich. --[[Spezial:Beiträge/178.4.185.67|178.4.185.67]] 14:20, 1. Okt. 2020 (CEST) <small>P.S.: Die eigentliche Bedeutung von "Vorbau" setze ich als bekannt voraus und sehe daher vorerst von einer näheren Erläuterung ab.</small>
::::: Ja, genau, erst fummel ich die Ahle raus um ein Loch in den Deckel zu machen, dass ich anschließend dann mit Tesafilm wieder zukleben darf. Kann ich mir mit der „Schlag-Methode“ ''beides'' schenken ;-) --[[Benutzer:Gretarsson|Gretarsson]] ([[Benutzer Diskussion:Gretarsson|Diskussion]]) 16:02, 2. Mär. 2017 (CET)
::Bei "schönes Dekolleté" ist ja eigentlich ziemlich eindeutig, was damit gemeint ist: dass der Ausschnitt schön ist bzw. das was man durch den Ausschnitt sieht, da der Ausschnitt selbst ja nur das Fehlen von Stoff in diesem Bereich ist. Ob dafür die Oberweite besonders hoch sein muss ist einen andere Frage, ich persönlich finde z.B. überdurchschnittliche Oberweiten attraktiv, aber nicht [https://www.flickr.com/photos/mlnapol/4722550589/ extreme Oberweiten].
::<small>Als Schritt bezeichnet man nicht das Fehlen von etwas, sondern den Bereich zwischen den Beinen bei einem Menschen oder einer Hose. Jede Hose hat einen Schritt, wenn dort etwas fehlt hat die Hose ein Loch im Schritt, wenn bei einem Mann etwas im schritt fehlt ist der Mann je nachdem was alles fehlt ein [[Kastrat]] oder ein [[Eunuch]].
::Und welche eigentliche Bedeutung von Vorbau meinst du? [[Vorbau_(Begriffsklärung)|Hier]] sind einige genannt, wobei die Liste nicht vollständig ist, siehe z.B. [https://www.sportlexikon.com/skispringen-aufsprunghuegel]. --[[Benutzer:MrBurns|MrBurns]] ([[Benutzer Diskussion:MrBurns|Diskussion]]) 16:20, 1. Okt. 2020 (CEST)</small>
:Ich kenne es so, daß man dann vom Dekolleté spricht, wenn ein Teil des Busens erkennbar ist. --[[Benutzer:Ralf Roletschek|<span style="color:#000000">M<span style="color:#ff0000">@</span>rcela</span>]] [[Bild:Miniauge2.gif|27px]] 19:56, 1. Okt. 2020 (CEST)


kurze werbeeinblendung: [https://www.amazon.de/dp/B002SAL24I/ Vacumex]! --[[Benutzer:JD|JD]] [[Benutzer Diskussion:JD|{æ}]] 15:58, 2. Mär. 2017 (CET)
== Lautstärke von Kühlschränken ==
:Stimmt, so ein ähnliches Ding haben wir bei der Wohnungsauflösung irgend einer Großmutter auch gefunden. Es ruht in den Tiefen einer Küchenschublade, genauso wie ein [http://www.leifheit.de/produkt/details/dosenoeffner-edelstahl.html professioneller Deckelöffner (Abb. ähnlich)]. --[[Benutzer:Elrond|Elrond]] ([[Benutzer Diskussion:Elrond|Diskussion]]) 16:27, 2. Mär. 2017 (CET)
:Nochn Trick: heißes Wasser über den Deckel laufen lassen. Deckel dehnt sich aus, die Luft drunter auch. [[Benutzer:Rainer Zenz|Rainer Z]] [[Benutzer Diskussion:Rainer Zenz| ...]] 19:16, 2. Mär. 2017 (CET)
::Wenn Luft unter dem Deckel ist, hat der Hersteller irgendetwas falsch gemacht. Normalerweise sollte Vakuum bzw sehr dünner Wasserdampf (7–11 hPa bei Kühlschranktemperatur, 23–32 hPa bei Raumtemperatur) unter dem Deckel sein. --[[user:Rotkaeppchen68|R<span style="color:red">ô</span>tkæppchen₆₈]] 22:14, 2. Mär. 2017 (CET)


== Alternativen zur Mondlandung ==
Die Lautstärke von Kühlschränken wird in dB(A) angegeben. Ist das die Lautstärke im Betrieb oder gemittelt über eine Stunde (d.h. der Mittelwert ist durch die Stillstandszeiten verfälscht)?


Vor Jahren habe ich mal in einem Vortrag von [[Jesco von Puttkamer (NASA)|Jesco von Puttkamer]] gehört das die NASA Kennedy eine Liste von 10 (evtl. auch nur sechs) Projekten vorgelegt hat wie man auf den Sputnikschock reagieren könne um die "Schande" nur zweiter zu sein auszugleichen.<br /> Neben der dann tatsächlich realisierten Mondlandung weis ich noch als Alternativen eine Raumstation und eine Nuklearexplosion auf dem Mond [[Projekt A119|(Projekt A119)]]. Weis jemand was die anderen waren?
Was sagen die Normen? --[[Spezial:Beiträge/81.10.221.11|81.10.221.11]] 14:10, 1. Okt. 2020 (CEST)
--[[Benutzer:Dreifachaxel|Dreifachaxel]] ([[Benutzer Diskussion:Dreifachaxel|Diskussion]]) 18:43, 1. Mär. 2017 (CET)
:Das passt alles nicht zusammen. Sputnick-Schock 1957, NASA-Gründung 1958 und Kennedy erst 1962 Präsident.--[[Spezial:Beiträge/2.240.16.75|2.240.16.75]] 19:39, 1. Mär. 2017 (CET)
::Die erste Reaktion auf den [[Sputnik-Schock]] war dass die Amis versucht haben, einen Mann in den [[Weltraum]] zu schicken. Erst wie ihnen da auch die Russen zuvorgekommen sind (siehe [[Juri Gagarin]]), hat man sich Alternativen überlegt und sich dann bald für die Mondlandung entschieden, dann kam Kennedys berühmte Rede. Fun-Fact: es gibt die Vermutung, dass die Russen bei ihrem Mondlandungsprojekt nur gescheitert sind, weil [[Baikonur]] nicht so nahe am Äquator liegt wie das [[Kennedy Space Center]] und die Russen deshalb wegen dem höheren Energieaufwand für die [[Bahnkorrektur]] eine größere Rakete brauchten als die Amis hätten bauen müssen und [[N1 (Rakete)|diese Rakete]] haben sie dann nicht mehr so richtig hinbekommen. Andererseits war die [[Saturn V]] so überdimensioniert, dass sie sogar noch größer war als die N1, die Amerikaner hätten ja auch nicht unbedingt mit [[Apollo 11|mit 3 Mann in den Mondorbit fliegen müssen]]. --[[Benutzer:MrBurns|MrBurns]] ([[Benutzer Diskussion:MrBurns|Diskussion]]) 20:01, 1. Mär. 2017 (CET)
:::Das [[Sowjetisches bemanntes Mondprogramm|sowjetische bemannte Mondprogramm]] ist eher deswegen gescheitert, weil die sowjetischen Raumfahrtingenieure der ihnen durch die Konzeption der N1-Rakete gestellte Herausforderung nicht gewachsen waren. Der modifizierte Entwurf für die erste Stufe sah 30 (in Worten dreißig) gleichzeitig brennende Haupttriebwerke vor. Das Konkurrenzprodukt Saturn V hatte insgesamt in allen drei Stufen nur insgesamt elf Triebwerke. --[[user:Rotkaeppchen68|R<span style="color:red">ô</span>tkæppchen₆₈]] 00:14, 2. Mär. 2017 (CET)
::::Ja, das war wohl das Hauptproblem, da die Amerikaner es geschafft haben, eine Rakete zu bauen, die auch für eine Mondmission von Baikonur stark genug gewesen wäre. Allerdings ist nicht klar, ob die Russen es nicht doch geschafft hätten, wenn ihr Weltraumbahnhof auch so weit südlich gewesen wäre und sie daher weniger Triebwerke gebraucht hätten (auch wenns noch immer viel mehr gewesen wären als die Saturn V hat). --[[Benutzer:MrBurns|MrBurns]] ([[Benutzer Diskussion:MrBurns|Diskussion]]) 04:23, 2. Mär. 2017 (CET)
:::::PS: Laut der Formel von [http://www.braeunig.us/space/orbmech.htm], Kapitel "Orbit Plane Changes", entspricht eine Inklinationsänderung von 19° (entspricht ungefähr dem Unterschied zwischen Baikonur und dem Kenndy Space Center) eine Geschwindigkeitsänderung, die ca. 33% der aktuellen Geschwindigkeit entspricht. Da man das nicht beim Start macht sondern möglichst weit oben, wo die Geschwindigkeit geringer ist, ist der Unterschied insgesamt aber nicht so groß. --[[Benutzer:MrBurns|MrBurns]] ([[Benutzer Diskussion:MrBurns|Diskussion]]) 05:14, 2. Mär. 2017 (CET)
::::::Für einen Mondflug braucht man doch gar keine Bahn, die in der Äquatorebene liegt, sondern eine, die in der Mondbahnebene liegt, was ungefähr die Ekliptikebene ist. --[[Benutzer:Digamma|Digamma]] ([[Benutzer Diskussion:Digamma|Diskussion]]) 10:33, 2. Mär. 2017 (CET)
:::::::Das ist im Prinzip richtig, aber mit den damaligen Steuerungsmöglichkeiten war wohl nur ein Äquatorkurs praktikabel. --[[Benutzer:MrBurns|MrBurns]] ([[Benutzer Diskussion:MrBurns|Diskussion]]) 10:36, 2. Mär. 2017 (CET)


:Auf [[Wettlauf ins All#Vom „Wettlauf ins All“ zum „Wettlauf zum Mond“]] (und dem Rest des Artikels) steht ein bisschen was zum Thema. Am 20. April 1961 schickte Kennedy an seinen Vize [[Lyndon B. Johnson]] die Notiz: „Haben wir eine Chance, die Sowjets zu schlagen, indem wir ein Labor im Weltraum errichten oder durch einen Flug um den Mond oder durch eine Rakete zur Landung auf dem Mond, die einen Menschen hin- und zurückbringt?“ Johnson, gleichzeitig auch Leiter des National Aeronautics and Space Council, bejahte diese Fragen. --[[Benutzer:Neitram|<span style="color:#008800">Ne</span><span style="color:#005555">it</span><span style="color:#005588">ra</span><span style="color:#0000FF">m</span>]]&nbsp;[[Benutzer_Diskussion:Neitram|<span style="font-size:large">✉</span>]] 09:54, 2. Mär. 2017 (CET)
:Im Betrieb. (Sage ich, nicht die Normen. Woher soll der Hersteller wissen, in welchen Intervallen bei mir der Kompressor läuft? Das hängt von einigen lokalen Variablen ab.). --[[Benutzer:Kreuzschnabel|Kreuz]][[Benutzer Diskussion:Kreuzschnabel|schnabel]] 15:22, 1. Okt. 2020 (CEST)
:: Der Artikel ist mir natürlich bekannt :-) Meine Frage war aber welche anderen Alternativen man in Erwägung gezogen hatte.<br /> An die IP ganz oben ("passt alles nicht zusammen"): Es war keine Rede davon das die Liste einen Tag nach Sputnik vorgelegen habe... --[[Benutzer:Dreifachaxel|Dreifachaxel]] ([[Benutzer Diskussion:Dreifachaxel|Diskussion]]) 14:13, 2. Mär. 2017 (CET)
::Für die [[Energieverbrauchskennzeichnung]] werden einige Annahmen zum Benutzungsverhalten gemacht. Da könnte man problemlos eine komplett sinnlose mittlere Geräuschemission bestimmen. Mittelt man da jetzt Schallleistung, Schalldruck oder Schalldruckpegel? --[[user:Rotkaeppchen68|R<span style="color:red">ô</span>tkæppchen₆₈]] 16:00, 1. Okt. 2020 (CEST)
:: [[Stiftung Warentest]] prüft mit schöner Regelmässigkeit auch Kühlgeräte und macht Angaben zur Lautstärke. Sie haben in ihren Testbeschreibungen auch sicher ein Prüfverfahren und die Annahmen beschrieben. [[Benutzer:Yotwen|Yotwen]] ([[Benutzer Diskussion:Yotwen|Diskussion]]) 16:18, 1. Okt. 2020 (CEST)
::Mir hat ein Techniker erklärt, die Lautstärke wird unter "Laborbedingungen" ermittelt und es sei daher normal, dass mein Kühlschrank im normalen Betrieb lauter ist als im Verkaufsprospekt angegeben. Für mich haben seitdem die Lautstärkeangaben die gleiche Aussagekraft wie die Verbrauchswerte bei Autos. --[[Spezial:Beiträge/2001:16B8:3006:FE00:E502:61BC:A98D:B9E5|2001:16B8:3006:FE00:E502:61BC:A98D:B9E5]] 18:23, 1. Okt. 2020 (CEST)
:::Nur weil etwas sinnlos ist, heisst nicht, des es nicht so gemacht wird. Vor allem dann, wenn dabei günstige Angaben im Verkaufsprospekt herauskommen. Die Deutsche Bahn pflegte Lärmemissionen auch wegzumitteln (fünf Minuten Höllenlärm und fünfundfünfzig Minuten Stille ergeben, im Mittel, kaum mehr als ein leichts Säuseln).--[[Benutzer:Meloe|Meloe]] ([[Benutzer Diskussion:Meloe|Diskussion]]) 08:46, 2. Okt. 2020 (CEST)


== An die Lateiner unter uns ==
== Dollar Wechselkurs Template ==


Habe heute Mittag den Artikel [[Conditio Jacobaea]] gelesen. Seitdem beschäftigen mich zwei Fragen:
Irgendwo gibt es eine Template (oder wie das auch immer auf WP:DE heisst) die man in Artikel einfügen kann, die besagt wieviel 100 Dollar von 1955 oder so, heute in Euro wert sind. Kann mir da bitte einer einen Wegweiser geben? Besten Dank! [[Benutzer:Oalexander|OAlexander]] ([[Benutzer Diskussion:Oalexander|Diskussion]]) 17:08, 1. Okt. 2020 (CEST)
# Müsste ''Jacobaea'' nicht kleingeschrieben werden?
# Woher leitet sich das ''ae'' in ''Jacobaea'' ab? Müsste es nicht einfach ''Jacoba'' oder ''Jacobia'' heißen?
Dank + Gruß, [[Spezial:Beiträge/2A02:8109:2C0:B54:10D0:631F:28B2:E93F|2A02:8109:2C0:B54:10D0:631F:28B2:E93F]] 19:43, 1. Mär. 2017 (CET)
:Groß oder klein gibt sich nichts. Ae stellt den Genitiv dar. Nach so um 400 nach gelten andere Spielregeln. :-) --[[Spezial:Beiträge/80.187.96.12|80.187.96.12]] 20:10, 1. Mär. 2017 (CET)
::"Hammer" doch alles: [[https://de.wikipedia.org/wiki/Lateinische_Grammatik#A-Deklination]] --[[Spezial:Beiträge/80.187.96.12|80.187.96.12]] 20:18, 1. Mär. 2017 (CET)
:::Also hört mal! Es geht um das Suffix -ae; das ist eine Adjektivbildung "zu ... gehörig" nach griechischem Vorbild. Und die Endung lautet -a, für Mom. Sg. fem. Grüße [[Benutzer:Dumbox|Dumbox]] ([[Benutzer Diskussion:Dumbox|Diskussion]]) 20:20, 1. Mär. 2017 (CET)
::::wir reden über Conditio Jacobaea? Oder? --[[Spezial:Beiträge/80.187.96.12|80.187.96.12]] 20:23, 1. Mär. 2017 (CET)
:::::Ich jedenfalls schon. Name Jacobus, abgeleitetes Adjektiv Jacob-ae-us, Femininum dazu Jacobaea. Groß übrigens wg. Eigenname, nach Konvention auch bei Adjektiven, wie im Englischen. Grüße [[Benutzer:Dumbox|Dumbox]] ([[Benutzer Diskussion:Dumbox|Diskussion]]) 20:55, 1. Mär. 2017 (CET)
[BK] - Genau: über ''Conditio Jacobaea''. ''Jacobaea'' ist das vom Namen ''Jacobus'' abgeleitete Adjektiv und hat, was das ''ae'' angeht, nach [[Benutzer:Dumbox|Dumbox]] ein griechisches Vorbild. Kann es damit zusammenhängen, dass der ursprünglich hebräische Name (Jaákov) über den griechischen "Zwischenwirt" (Jakobos) ins Lateinische (Jakobus) gekommen ist und von daher die Adjektivbildung sich am Griechischen orientiert? Im Lateinischen finden sich doch auch andere von Namen abgeleitete Adjektivformen, zB [[Gregoriana]]. Andererseits: [[Caesarea Philippi]] ... *denk, *kratz, *schweig ... [[Spezial:Beiträge/2A02:8109:2C0:B54:10D0:631F:28B2:E93F|2A02:8109:2C0:B54:10D0:631F:28B2:E93F]] 21:00, 1. Mär. 2017 (CET)
:Im Griechischen sind häufige Zugehörigkeitssuffixe -ios und -aios; -aios aber meist bei a-Stämmen. Mir scheint aber -aios auch besonders bei nichtgriechischen Namen vorzukommen. Vielleicht hat Aramäisch (man beachte die Wortbildung!) ein entsprechendes Suffix ([[Nisbe]]), das man einfach übernehmen konnte. Lateinisch wäre -icus, -ius, -anus, inus, -ensis und einige mehr. Grüße [[Benutzer:Dumbox|Dumbox]] ([[Benutzer Diskussion:Dumbox|Diskussion]]) 21:26, 1. Mär. 2017 (CET)
:::Dann beantwortet mal dem Fragesteller seine Fragen hinreichend. Grins + Grüße --[[Spezial:Beiträge/80.187.96.12|80.187.96.12]] 21:03, 1. Mär. 2017 (CET)
::::::@[[Benutzer:Dumbox|Dumbox]]: Gratias ago! [[Spezial:Beiträge/2A02:8109:2C0:B54:F15E:767A:45B6:6350|2A02:8109:2C0:B54:F15E:767A:45B6:6350]] 08:37, 2. Mär. 2017 (CET)
:::::::Mein gesammeltes Wissen zur Groß- und Kleinschreibung in neulateinischen Texten (aus einsichtigen Ursachen hatten die klassisch-lateinischen Texte kein Problem damit) hatte ich vor einiger Zeit im Artikel „Bipontum“ (mit Absicht hier nicht verlinkt) eingebracht. Vielleicht beginnt ja mal jemand einen Artikel zur neulateinischen Orthographie? --[[Benutzer:Pp.paul.4|Pp.paul.4]] ([[Benutzer Diskussion:Pp.paul.4|Diskussion]]) 09:13, 2. Mär. 2017 (CET)


== Warum befindet sich der CVR im Heck des Flugzeugs? ==
:{{FZW}} und {{Vorlage|Wechselkurs}} oder {{Vorlage|Inflation}}. --[[Benutzer:Tsungam|Magnus]] [[BD:Tsungam|(Diskussion)]] 17:11, 1. Okt. 2020 (CEST)
:: Ich werde versuchen, mir das zu merken und bedanke mich ansonsten für die äusserst rapide und hilfreiche Antwort. Bester Gruß, [[Benutzer:Oalexander|OAlexander]] ([[Benutzer Diskussion:Oalexander|Diskussion]]) 17:27, 1. Okt. 2020 (CEST)


Warum befindet sich der Cockpit Voice Recorder im Heck des Flugzeuges? In der Vergangenheit ist es mehrfach vorgekommen, dass der CVR vorzeitig ausfiel, weil die Kabel, die vom Cockpit zu ihm führen, z.B. durch Feuer beschädigt wurden. Wenn er sich im Cockpit befände, könnte so etwas nicht passieren.
== Aussprache Nachname "Haß" ==


--[[Spezial:Beiträge/188.118.248.227|188.118.248.227]] 20:56, 1. Mär. 2017 (CET)
Kleine Frage: Wie wird eigentlich der Nachname "Haß" ausgesprochen, mit kurzen oder langen ''a'' (''Hass'' vs. ''Ha(a)s''). Man will sich ja nicht blamieren (es handelt sich um eine Personalleiterin bei einem Nebenerwerb; bisher nur schriftlicher Kontakt). Schätze letzteres, aber wieso kommt es dann zu dem ''ß'' im Namen?
:Bei einem Absturz könnte sich der Vorderteil des Flugzeugs in den Boden bohren. Damit würde der CVR erstens besonders stark beschädigt und zweitens besonders unzugänglich. Deswegen macht man das nicht. --[[user:Rotkaeppchen68|R<span style="color:red">ô</span>tkæppchen₆₈]] 21:01, 1. Mär. 2017 (CET)


::Es wurden schon 12 Meter Tiefe in den Boden geschafft. Da bleibt nichts mehr übrig. --<span style="color:#00A000;">Hans Haase ([[BD:Hans Haase|有问题吗]])</span> 21:04, 1. Mär. 2017 (CET)
:::Flugzeuge schlagen nicht immer mit der Nase voran auf. Sie können auch zuerst mit dem Heck aufschlagen. Gibt es Statistiken die zeigen, dass die Wahrscheinlichkeit, dass das Cockpit zu hart aufschlägt größer ist als dass das mit dem Heck passiert? --[[Benutzer:MrBurns|MrBurns]] ([[Benutzer Diskussion:MrBurns|Diskussion]]) 21:17, 1. Mär. 2017 (CET)
:::(BK) Weil das Heck das Teil des Flugzeuges ist, wo auf bezogen auf zig Unfällen einfach die geringste Wahrscheinlichkeit an eines Totalschaden des CVR zu erwarten ist. Und auch reisst das Heck beim Aufprall gerne ab, erst recht wenn es als erste Teil aufkommt, und schafft es dadurch meist nicht im Hautfeuer zu landen. Auch bei Abstürzen über Gewässer ist meist das Heck, das Teil das als ganzes geborgen werden kann (Da kriegt man relativ einfach ein Seil darum das nicht abrutscht). Schau dir einfach auch mal die Flugzeuge an die nach der Notladung usw. ausgebrannt sind, da bei sieht das Heck sehr oft noch "gut" aus, im Vergleich vom Rest vom Flugzeug jedenfalls ([[Asiana-Airlines-Flug 214]], [[Emirates-Flug 521]]). Das CVR muss ja einiges aushalten können. Kurzum das CVR ist auch darauf ausgelegt alle Unfall Szenarien, die dem Heck zustossen könnten, zu überleben. An anderer Stelle eingebaut, müsste es noch massiver gebaut sein. --[[Benutzer:Bobo11|Bobo11]] ([[Benutzer Diskussion:Bobo11|Diskussion]]) 21:20, 1. Mär. 2017 (CET)
::::Die FAR-Vorschrift besagt, dass das Flugzeug je ein CVR und FDR (Flight Data Recorder) haben muss. Einer muss dann möglichst nah am Cockpit und einer muss möglichst weit weg vom Cockpit eingebaut werden. Es ist den Konstrukteuren überlassen, welcher wo ist. Alles andere ist phantasiereiche Spekulation unserer beiden ausgewiesenen Laien der Auskunft aus A und CH.--[[Spezial:Beiträge/2003:75:AF59:2800:A5B2:B441:10F4:20A3|2003:75:AF59:2800:A5B2:B441:10F4:20A3]] 22:33, 1. Mär. 2017 (CET)
:::::Dann erklär mal, wieso das dann mit dem Zusammenfassen im CVDR geht? Das wäre dann ja nicht Zulassfähigkeit. --[[Benutzer:Bobo11|Bobo11]] ([[Benutzer Diskussion:Bobo11|Diskussion]]) 23:07, 1. Mär. 2017 (CET)
::::::Bei den kombinierten Geräten besagt die Vorschrift, dass sie möglichst weit vom Cockpit eingebaut werden müssen, bzw. mindestens hinter den Tragflächen. Das Kriterium ist das Feuer durch den Sprit in den Tragflächen und nicht irgendwelche Crash-/ Überlebensbetrachtungen. Diese Beschleunigungen haben die Geräte, egal wo sie eingebaut sind auszuhalten. --[[Spezial:Beiträge/2003:75:AF59:2800:F07D:E864:5F8:F7DF|2003:75:AF59:2800:F07D:E864:5F8:F7DF]] 23:29, 1. Mär. 2017 (CET)
:::::::Stimmt leider nicht, der Einbauart hat sehr starken Einfluss darauf welche Beschleunigung überhaupt auf das Gerät einwirken kann. Und je mehr Blech es zwischen Felswand und Rekorder hat desto länger ist der Bremsweg. Und das heisst nicht anderes, als das die kinetische Energie wird über einen längerer Zeitraum abgebaut werden kann. Und Flugzeuge werden äussert selten von hinten gerammt. Und selbst wenn ist es die Geschwindigkeitsdifferenz die für Geräte-Beschleunigung Ausschlag gebend ist. Mir wäre gerade kein Unfall bekannt wo das Heck mit mehr als Lande/Start-Geschwindigkeit zuerst aufgekommen wäre. Mit Reiseflug Geschwindigkeit war es immer Nase voran. Und da bei Berechnung der Kinetische Energie die Geschwindigkeit im Quadrat Einfluss nimmt, ist diese durchaus ein nicht zu vernachlässigende Faktor. Also welches Unfallszenerio sich mit welcher Geschwindigkeit abspielt. --[[Benutzer:Bobo11|Bobo11]] ([[Benutzer Diskussion:Bobo11|Diskussion]]) 07:56, 3. Mär. 2017 (CET)
::::::::Die aufgezeichneten Daten müssen 3400g aushalten und auslesbar bleiben, da ist es egal, wo sie im Rumpf eingebaut sind. Hör auf dich auch als Flugzeugbauer zu produzieren, das gibt einen Absturz.--[[Spezial:Beiträge/2003:75:AF44:4D00:5D27:EEB3:CBAE:4858|2003:75:AF44:4D00:5D27:EEB3:CBAE:4858]] 13:06, 3. Mär. 2017 (CET)
:::::::::Dafür, dass du so vollmundige Behauptungen aufstellst (davon mindestens eine mit Sicherheit falsche, falls du dieselbe Person wie der Kommentierer von vorgestern 22:33 bist), wäre es schon mal an der Zeit, dass du Belege (Links) beibringst. Zufällig scheinen nämlich die Spekulationen der alpinen und sonstigen Bevölkerung in diesem Fall durchaus zutreffend zu sein, vgl. auch [http://aviation.stackexchange.com/questions/11627/why-are-flight-recorders-generally-located-in-the-tail]. --[[Spezial:Beiträge/91.89.146.92|91.89.146.92]] 15:44, 3. Mär. 2017 (CET)
:::::Deine Aussage stimmt so nicht. In den FAR wird im Normalfall für den CVR die Position hinten ("as far aft as practicable") vorgeschrieben, siehe [http://www.risingup.com/fars/info/part23-1457-FAR.shtml FAR Sec. 23.1457 — Cockpit voice recorders (e)(1)]. --[[Spezial:Beiträge/91.89.146.92|91.89.146.92]] 15:44, 3. Mär. 2017 (CET)
::::::Das habe ich nicht infrage gestellt. Der Flugzeugkonstrukteur, weiß aber, dass der Grund alleinig aus der Gefahr vor dem brennenden Sprit in der Tragfläche resultiert und nicht der Schutz vor dem crash. Da gibt es für die Recorder genauere und begründete Vorschriften bei RTCA/ EUROCAE. Die sucht euch selbst raus, es reicht, wenn ich es weiß.--[[Spezial:Beiträge/2003:75:AF44:4D00:2806:45E6:AD03:CBA5|2003:75:AF44:4D00:2806:45E6:AD03:CBA5]] 16:34, 3. Mär. 2017 (CET)
:::::::<small>Achso, das hast du "nicht infrage gestellt". Dann warst Du wohl doch nicht die IP, die bzgl. CVR und FDR [https://de.wikipedia.org/w/index.php?title=Wikipedia:Auskunft&diff=prev&oldid=163149994 "Es ist den Konstrukteuren überlassen, welcher wo ist"] geschrieben hatte. ;) --[[Spezial:Beiträge/91.89.146.92|91.89.146.92]] 18:43, 3. Mär. 2017 (CET)</small>
:Noch etwas zum Fliegerischen: Ein Aufschlag am Heck weist oft darauf hin, dass die Piloten eine Landung versucht hatten. Dafür sind Landeklappen und Fahrwerk ausgefahren, und die Geschwindigkeit erheblich reduziert. Schlägt das Flugzeug mit der Nase ein, geschieht die Kollision mit einer sehr hohen Geschwindigkeit. Das ganze Flugzeug dient dann als Knautschzone für die Recorder.--[[Benutzer:Keimzelle|Keimzelle]] <small>[[Benutzer_Diskussion:Keimzelle| talk ]]</small> 09:48, 2. Mär. 2017 (CET)
::Oder anders ausgedrückt: Ein Flugzeug bewegt sich grundsätzlich vorwärts. Somit ist das Heck am weitesten entfernt vom theoretisch Punkt des Aufschlags. Schlägt das Flugzeug nicht mit dem Bug auf, hat es zuvor die Richtung gewechselt und dabei fast zwangsläufig die Geschwindigkeit verringert. --[[Benutzer:Nobody perfect|Nobody Perfect]] ([[Benutzer Diskussion:Nobody perfect|Diskussion]]) 16:21, 2. Mär. 2017 (CET)
:::Richtig. Ein Flugzeug das vom Himmel fällt, ist in der Regel "nur" mit Fallgeschwindigkeit unterwegs (Dynamische Komponente von oben nach unten). Der klassische CFIT-Unfall (Controlled Flight Into Terrain = kontrollierter Flug in den Boden) ist Nase voran (jedenfalls im Bezug auf die (Haupt-)Richtung der (beschleunigte) Massenbewegung). --[[Benutzer:Bobo11|Bobo11]] ([[Benutzer Diskussion:Bobo11|Diskussion]]) 07:40, 3. Mär. 2017 (CET)
::::Er kann es nicht lassen, sich als Laie weiter was anzulesen und die falschen Schlüsse als Obergutachter (Richtig!) herauszudrücken. "Klassische" CFIT gibt es auf der Welt gerade mal 1-2 Unfälle pro Jahr, also verschwindend klein.--[[Spezial:Beiträge/2003:75:AF44:4D00:5D27:EEB3:CBAE:4858|2003:75:AF44:4D00:5D27:EEB3:CBAE:4858]] 11:59, 3. Mär. 2017 (CET)

== Welche Zeitung kann das gewesen sein? ==

Ich bin hinter Informationen zur Familie Mahlberg her. [http://www.numbers-stations.com/cia/ROEDER,%20MANFRED/ROEDER,%20MANFRED_0013.pdf Hier], auf S. 55, gibt es einen Abschnitt zu Stella Mahlberg, in dem die Lebensdaten von [[Paul Mahlberg]] handschriftlich eingetragen sind. Findet jemand heraus, welches "Communist newspaper in Stuttgart" er damals herausgegeben haben soll? --[[Benutzer:Xocolatl|Xocolatl]] ([[Benutzer Diskussion:Xocolatl|Diskussion]]) 21:11, 1. Mär. 2017 (CET)
:Der Link ist keine Zeitung HSB Rote Kapelle? --[[Spezial:Beiträge/80.187.123.130|80.187.123.130]] 21:30, 1. Mär. 2017 (CET)
::Die verlinkte pdf hat weder as it noch nach eigener Seitenzahl den Verweis zu Stuttgart auf 55 im Text. Seiten wurden gelesen. Hat Zeit gekostet. Was willst Du? --[[Spezial:Beiträge/80.187.123.130|80.187.123.130]] 21:38, 1. Mär. 2017 (CET)
:::Ich habe irgendwie das Gefühl, dass du weder meine Frage noch den zugehörigen Text gelesen hast. Was ich will, ist eine Antwort auf die Frage, wie die angeblich kommunistische Zeitung geheißen hat, die angeblich Paul Mahlberg angeblich in Stuttgart herausgegeben hat. Schrieb ich aber schon oben. --[[Benutzer:Xocolatl|Xocolatl]] ([[Benutzer Diskussion:Xocolatl|Diskussion]]) 21:42, 1. Mär. 2017 (CET)
::::Ja, auf Seite 55 der pdf. Da gibts aber sowas nicht! --[[Spezial:Beiträge/80.187.123.130|80.187.123.130]] 21:49, 1. Mär. 2017 (CET)
:::::Schau genauer hin. --[[Benutzer:Tsungam|Magnus]] [[BD:Tsungam|(Diskussion)]] 21:51, 1. Mär. 2017 (CET)
:::::Ja habs verstandem --[[Spezial:Beiträge/80.187.123.130|80.187.123.130]] 21:56, 1. Mär. 2017 (CET)
::::::So eine Zeitung gabs in S nicht. Wahrscheinlich ist was anderes gemeint. Allerdings lese ich noch. Grüße
:::::::Englischer Text nach 45. Handschriftliche Einträge kennst Du wohl. PM etc. Also S war zur Nazizeit die Stadt der Auslandsdeutschen. Wenn Dir die handschriftlichen Einträge zu Namen was sagen, waren sie dem Randgebiet. um D herum anzusiedeln?--[[Spezial:Beiträge/80.187.123.130|80.187.123.130]] 22:09, 1. Mär. 2017 (CET)
:::::::Rote Kapelle ist Dir klar? Grüße --[[Spezial:Beiträge/80.187.123.130|80.187.123.130]] 22:13, 1. Mär. 2017 (CET)
::::::::Ohne Antwort lass ich es ''still as bevor'' verweit auf dieser Seite weg von S - R: O --[[Spezial:Beiträge/80.187.123.130|80.187.123.130]] 22:23, 1. Mär. 2017 (CET)
:::::::::Es wird behauptet, dass die Information zur Herausgabe der Zeitung falsch ist.[http://forum.ahnenforschung.net/showthread.php?t=99299&page=3] --[[Benutzer:BlackEyedLion|BlackEyedLion]] ([[Benutzer Diskussion:BlackEyedLion|Diskussion]]) 23:59, 1. Mär. 2017 (CET)
::::::::::Was heißt damals? Von wann stammt der Bericht? --[[Benutzer:BlackEyedLion|BlackEyedLion]] ([[Benutzer Diskussion:BlackEyedLion|Diskussion]]) 00:12, 2. Mär. 2017 (CET)
:::::::::::Dass es in Stuttgart keine kommunistische Zeitung gegeben hätte, ist falsch. ''Unsere Stimme'' wurde in Stuttgart herausgegeben, siehe [[:en:Paula Acker]]. --[[Benutzer:BlackEyedLion|BlackEyedLion]] ([[Benutzer Diskussion:BlackEyedLion|Diskussion]]) 00:18, 2. Mär. 2017 (CET)
::::::::::::@ BlackEyedLion, von wann genau der Bericht stammt, weiß ich nicht, aber irgendwo habe ich gelesen, dass Stella Mahlberg sich 1947 umgebracht haben soll, das wäre dann also ein Terminus post. Und Terminus ante wäre 1955, weil da Paul Mahlberg eine andere Beschäftigung hatte. Die Beiträge im forum.ahnenforschung habe ich natürlich gelesen, ich will ja eben darüber hinaus kommen. Aber erstmal danke für deine Bemühungen, ich werde mir mal den Acker-Artikel anschauen. Gruß --[[Benutzer:Xocolatl|Xocolatl]] ([[Benutzer Diskussion:Xocolatl|Diskussion]]) 00:23, 2. Mär. 2017 (CET)
:::::::::::::In Kurt Koszyk, Geschichte der deutschen Presse. 4. Pressepolitik für Deutsche 1945–1949 könnte dazu etwas stehen. Zu Paula Acker: [https://www.bundesstiftung-aufarbeitung.de/wer-war-wer-in-der-ddr-%2363%3B-1424.html?ID=5]. --[[Benutzer:BlackEyedLion|BlackEyedLion]] ([[Benutzer Diskussion:BlackEyedLion|Diskussion]]) 00:25, 2. Mär. 2017 (CET)
::::::::::::::Mhm... da muss ich wohl mal einen Bibliotheksurlaub machen. ''Unsere Stimme'' scheint mir so gefühlsmäßig nicht recht zu passen, aber wer weiß. Ist ja eine illustre Familie gewesen. --[[Benutzer:Xocolatl|Xocolatl]] ([[Benutzer Diskussion:Xocolatl|Diskussion]]) 01:53, 2. Mär. 2017 (CET)
:::::::::::::::Ist zwar nicht gefragt, aber zur Info, falls es jemanden wundert: DOB steht für ''date of birth'' (Geburtsdatum). --[[Benutzer:Pp.paul.4|Pp.paul.4]] ([[Benutzer Diskussion:Pp.paul.4|Diskussion]]) 09:31, 2. Mär. 2017 (CET)
::::::::::::::::Das Archiv der Partei ist übrigens im Stiftung Archiv der Parteien und Massenorganisationen der DDR im Bundesarchiv erhalten. --[[Benutzer:BlackEyedLion|BlackEyedLion]] ([[Benutzer Diskussion:BlackEyedLion|Diskussion]]) 09:50, 2. Mär. 2017 (CET)
<linksrück>Es gab auch noch die ''Volksstimme'' {{ZDB-ID|126019-4}}, Tages-Ausgabe {{ZDB-ID|126566-0}}, Wochen-Ausgabe {{ZDB-ID|126567-2}}. --[[Benutzer:HHill|HHill]] ([[Benutzer Diskussion:HHill|Diskussion]]) 11:19, 2. Mär. 2017 (CET)
:Danke, ja, die klingt von der Beschreibung her für mich irgendwie passender, ich habe aber zumindest online auch keine Zusammenhänge mit Mahlberg finden können. Wäre der Wendelin Niedlich damals schon in Stuttgart gewesen... war er aber nicht. --[[Benutzer:Xocolatl|Xocolatl]] ([[Benutzer Diskussion:Xocolatl|Diskussion]]) 14:40, 2. Mär. 2017 (CET)
::Zumindest hat er 1946 ein [https://wlb.ibs-bw.de/aDISWeb/app?service=direct/0/Home/$DirectLink&sp=S127.0.0.1:23002&sp=SAKSWB-IdNr065800974 Buch bei einem Stuttgarter Verlag] herausgebracht. --[[Benutzer:HHill|HHill]] ([[Benutzer Diskussion:HHill|Diskussion]]) 14:48, 2. Mär. 2017 (CET)

= 2. März 2017 =

== Standleitung ins Netz vs. SDSL ==

Laienfrage an die Netzwerker: Wie unterscheidet sich eine Standleitung ins Internet von SDSL? Danke für die Antwort. <small>(''nicht [[Hilfe:Signatur|signierter]] Beitrag von'' [[Spezial:Beiträge/188.101.73.75|188.101.73.75]] ([[Benutzer Diskussion:188.101.73.75|Diskussion]])<nowiki/> 00:20, 2. Mär. 2017 (CET))</small>
:„Standleitung“ ist ein veralteter Begriff aus dem Zeitalter der [[Leitungsvermittlung]]. Das Internet und die Zugänge dahin sind heutzutage ausschließlich [[Paketvermittlung|paketvermittelt]]. SDSL ist nur eine von vielen Techniken, Internetdaten vom Hauptverteiler zum Kunden zu bringen. Allerdings vergeben viele Fernmeldedienstleister für SDSL-Anschlüsse feste IP-Adressen. Bei ADSL-, VDSL- oder DOCSIS-Anschlüssen ohne Zwangstrennung gibt es das mit Einschränkungen auch, ohne dass diese Verbindungen als „Standleitung“ bezeichnet werden. --[[user:Rotkaeppchen68|R<span style="color:red">ô</span>tkæppchen₆₈]] 00:31, 2. Mär. 2017 (CET)
::Man kann durchaus auch heute noch eine Standleitung z.B. zwischen zwei Firmenstandorten im selben Ort bekommen. Da rennen dann Telekomiker rum und klemmen tatsächlich Kabel um. Überregional kann man sich auch Glasfasern mieten usw. Ist alles eher was für den dickeren Geldbeutel. -- [[Benutzer:Janka|Janka]] ([[Benutzer Diskussion:Janka|Diskussion]]) 01:02, 2. Mär. 2017 (CET)
:::<small>Dickeren Geldbeutel trifft es genau. Mein Arbeitgeber hatte vor ein paar Jahren die Aufgabe, zwei 240 Meter Luftlinie entfernte Standorte des Unternehmens telefon- und LAN-mäßig zu verbinden. Ich schlug als allererstes Richtfunk vor, der eine Gebäudeeigentümer schlug Telekom-Standleitung oder alternativ selbstverlegtes Erdkabel vor. Das Erdkabel hätte einen fünf- bis sechsstelligen Betrag gekostet, die Telekommietleitung erwies sich als unbezahlbar. Da wurde dann doch eine 5-GHz-WLAN-Bridge mit Richtantennen und VoIP für Telefon und Fax installiert. Nach wenigen Wochen stellte sich heraus, dass der zweite Unternehmensstandort wenige hundert Meter entfernt den Arbeitsablauf erheblich beeinträchtigte und der zweite Standort wurde wieder aufgegeben. Die Richtfunkinstallation existiert heute noch, allerdings ungenutzt – es fühlte sich niemand dazu berufen, die Hardware zu demontieren und bei Ebay zu veräußern. So war das ganze reine Geldverschwendung. --[[user:Rotkaeppchen68|R<span style="color:red">ô</span>tkæppchen₆₈]] 01:37, 2. Mär. 2017 (CET)</small>

== Spanische Strafzumessung ==

Den Amis ist ja so einiges zuzutrauen (aber auch da könnte man die Frage mal stellen). Aber warum sind in Spanien - obwohl Menschen legal nur 40(?) Jahre ihres Lebens im Knast verbringen dürfen - Strafzumessungen möglich, die (rückwärts gerechnet) teilweise bis ins [[Mesolithikum]] oder [[Paläolithikum]] reichen würden? Zum Beispiel hat [[Jamal Zougam]] genau 42.922 Jahre Haftstrafe bekommen.
Eine weitere Frage wäre: Wie kommen bei diesen hohen - rein symbolischen - Zahlen die dennoch auf das Jahr genau präzise Strafzumessung zustande, so z.B. dass [[Otman el-Gnaoui]] (obwohl wir im mittleren fünfstelligen Bereich sind!) genau zwei Jahre Haft mehr bekam als Jamal Zougam? --[[Benutzer:ObersterGenosse|ObersterGenosse]] ([[Benutzer Diskussion:ObersterGenosse|Diskussion]]) 01:45, 2. Mär. 2017 (CET)
:Das hat meines Wissens nach damit zu tun, dass diese Rechtssysteme die weitere Vollstreckung der Strafe nach dem Tod und einer dann folgenden Wiedergeburt oder andersartigen Wiederkehr sichern sollen, dass letztendlich die "Persönlichkeit" oder die "Seele" und nicht nur ein von diesen Entitäten bewohnter Körper bestraft wird. Ich weiß jetzt allerdings nicht, ob (in den USA) "Life without parole" (Lebenslang ohne Bewährung) die längere oder kürzere Haftstrafe ist als bsp. "3500 Jahre Gefängis", ich nehme mal an, dass "Lebenslang" formaljuristisch als kürzer gilt (da es ja auch bsp. 5-fach Lebenslang gibt - [[Ariel Castro]] war so ein Verurteilter). Letztendlich ist das eine Verquickung von Moral- und Glaubensprinzipen, die nicht in jedem Weltbild rational erklärbar sind. Ganz praktisch könnte das noch den Hintergrund haben, dass die verhängte Strafdauer Einfluss auf Bewährungsmöglichkeiten und Strafstatistiken (die nach verhängten Haftzeiten ausgegeben werden können) nimmt. Grüße, [[Benutzer:Grand-Duc|Grand-Duc]] ([[Benutzer Diskussion:Grand-Duc|Diskussion]]) 02:42, 2. Mär. 2017 (CET)
::Wo steht, dass ein Mensch (außerhalb des Geltungsbereichs unseres Grundgesetzes und des Einflussbereichs unseres Bundesverfassungsgerichts) legal nur 40 Jahre im Gefängnis verbringen darf?
::Die Strafzumessung über die erwartete Lebenswahrscheinlichkeit hinaus ergibt sich in den USA und andernorts in der Regel durch bloße Multiplikation und hat mit der gesetzlich vorgesehenen Strafe zu tun, nichts mit einer Strafe über den Tod hinaus. Sinnvoll ist das natürlich nicht - es sei denn, man will eine Strafe auch für den Fall einer teilweisen Begnadigung sicherstellen (bei "zweimal lebenslänglich" kann ein mal erlassen werden, und der Verurteilte bleibt trotzdem bis zum Ableben inhaftiert, oder bei 40.000 Jahren kann die Strafe ohne praktische Folgen auf 10% reduziert werden). --[[Benutzer Diskussion:Snevern|Snevern]] 06:30, 2. Mär. 2017 (CET)
:::Keine Multiplikation, sondern eine Addition, oder? Siehe [[Absorptionsprinzip (Recht)]] bzw. [[Konkurrenz (Strafrecht Deutschlands)]], nach meinem Verstädnis wird in Deutschland jemand der 30 mal das gleiche Verbrechen begeht dafür einmal bestraft (wenn auch i.d.R. eher am oberen Ende des möglichen Strafmaßes) während z.B. in den USA die 30 Einzelstrafen aufaddiert werden - oder? --[[Benutzer:Studmult|Studmult]] ([[Benutzer Diskussion:Studmult|Diskussion]]) 07:41, 2. Mär. 2017 (CET)
::::In den USA wird üblicherweise addiert, in Deutschland wird eine Gesamtstrafe berechnet, wirklich genaue Vorschriften dazu wie das gemacht wird, scheint es nicht zu geben, es wird nicht einfach addiert, aber die kann durchaus höher sein als die höchstmögliche Strafe für eine einzelne Tat, siehe §§ 53-55 [https://www.gesetze-im-internet.de/stgb/BJNR001270871.html StGB]. --[[Benutzer:MrBurns|MrBurns]] ([[Benutzer Diskussion:MrBurns|Diskussion]]) 07:50, 2. Mär. 2017 (CET)
:::::Klar ist es eine Addition, denn jede Multiplikation ist ja nichts anderes als eine wiederholte Addition. Wenn ich für 3 Mordopfer eines Anschlags jeweils eine Einzelstrafe von 100 Jahren auswerfe, dann komme ich zu 100 + 100 + 100 Jahren (Addition) oder zu 3 x 100 Jahren (Multiplikation) Gesamtstrafe.
:::::Die Regeln für die Bildung einer Gesamtstrafe in den USA kenne ich nicht; in Deutschland ist das in §§ 52 ff. StGB geregelt. --[[Benutzer Diskussion:Snevern|Snevern]] 09:21, 2. Mär. 2017 (CET)
::::::Dad ist korrekt, aber die Regelungen in §§ 53 ff sind nicht sehr genau. Ske legen eigenntlich nur Ober- und Untergrenzen fest. Z.B. für 15 Straftaten die jeweils als einzuge Tat mit 1 Jahr bestraft werden würden, sind gemäß Wortlaut zwischen 1 Jahr und 15 Jahren und 11 Monaten möglich. Die tatsächliche Strafzumessung ist also wohl mehr auf Richtergewohnheiten aufgebaut als auf dem Gesetzestext.--[[Benutzer:MrBurns|MrBurns]] ([[Benutzer Diskussion:MrBurns|Diskussion]]) 00:54, 3. Mär. 2017 (CET)

Bisher war der Gedanke von [[Benutzer:Grand-Duc|Grand-Duc]] der Beste. Im Falle von Reinkarnationen muss man die (gesamte?) Haftstrafe absitzen!

Aber sonst bezog ich mich speziell auf '''spanisches''' Strafzumessungsrecht, und auch nur bezüglich dieses Rechtskreises stand in en:WP, dass man nur 40 Jahre im Knast sitzen darf. Wie ist also die (Gesamt-)Strafenbildung im spanischen Strafzumessungsrecht (im Codigo Penal, oder gibt's da was eigenes?) geregelt? --[[Benutzer:ObersterGenosse|ObersterGenosse]] ([[Benutzer Diskussion:ObersterGenosse|Diskussion]]) 00:59, 3. Mär. 2017 (CET)
::Denk dir vorsichtshalber mal ein [[Citation needed]] an meine Aussage, ich weiß nämlich nicht mehr, wo ich das aufgeschnappt habe. Dafür habe ich aber einen interessanten [http://www.bbc.com/news/blogs-magazine-monitor-23495284 BBC-Artikel] zum Thema gefunden. Grüße, [[Benutzer:Grand-Duc|Grand-Duc]] ([[Benutzer Diskussion:Grand-Duc|Diskussion]]) 03:10, 3. Mär. 2017 (CET)

== Tetris-Rätsel... (erl.) ==

Etwas für die Informatiker: [http://www.tetrisfriends.com/help/tips_Battle.php Auf dieser Seite] wird ein Tetris-Multiplayer-Modus beschrieben, bei dem man gegen einen bzw. fünf Gegner auf Zeit spielen kann. Was mich wundert: Ich kann das Spiel unterbrechen und später wieder fortsetzen, was jedoch bspw. im [http://www.tetrisfriends.com/games/Live/game.php Arena-Modus] nicht mehr möglich ist. Warum geht das aber im Zwei- bzw. Sechs-Spieler-Modus? Die Erklärung kann doch eigentlich nur die sein, dass man hier gegen eine KI spielt, oder? Für aufschlussreiches Feedback stets dankbar, euer--[[Benutzer:Hubon|Hubon]] ([[Benutzer Diskussion:Hubon|Diskussion]]) 04:08, 2. Mär. 2017 (CET)

PS: Gerade habe ich das nochmals getestet und gegen [http://www.tetrisfriends.com/users/profile.php?id=824769 diesen „Nutzer“] gespielt, der aber nichts getan hat, sodass die Steine einfach ungeordnet aufeinander gefallen sind. Wenn es sich aber tatsächlich um eine KI handelte, warum würde man dann ein vergleichsweise so aufwändiges Profil mit [fiktiven] Freunden etc. erstellen? Wenn aber nicht, wie soll das mit der Pause dann bitte bei einem echten, menschlichen User funktionieren? Das Ganze erscheint mir zugegeben immer rätselhafter, je mehr ich darüber nachdenke...--[[Benutzer:Hubon|Hubon]] ([[Benutzer Diskussion:Hubon|Diskussion]]) 04:29, 2. Mär. 2017 (CET)
:Wie das mit der Pause bei einem echten, menschlichen Gegner funktioniert? Die einfachste Möglichkeit wäre: einer drückt auf Pause, wenns weitergeht drückt einer auf Fortsetzen, was man natürlich normalerweise per Chat abspricht. --[[Benutzer:MrBurns|MrBurns]] ([[Benutzer Diskussion:MrBurns|Diskussion]]) 05:33, 2. Mär. 2017 (CET)
:Siehe [http://harddrop.com/forums/index.php?showtopic=1434 hier]. --[[user:Rotkaeppchen68|R<span style="color:red">ô</span>tkæppchen₆₈]] 11:10, 2. Mär. 2017 (CET)
::Ja, aber das Verrückte ist: In diesen Modi gibt es keinen Chat, bei dem man eine solche Unterbrechung absprechen könnte! Und wie gesagt: Man wird auf der Seite rund um die Uhr einen Spielpartner finden – so wie heute Nacht, wo aber einfach die Tetrimos meines Gegenübers aufeinander gefallen sind, ohne dass dieses aktiv eine Bewegung ausgeführt hat, sodass ich das Spiel im Nu gewann. Ist doch seltsam, nicht?--[[Benutzer:Hubon|Hubon]] ([[Benutzer Diskussion:Hubon|Diskussion]]) 14:26, 2. Mär. 2017 (CET)
:::Auf der verlinkten Seite steht der Grund dafür: „You're playing against replays of other players, it isn't true multiplayer.“ --[[user:Rotkaeppchen68|R<span style="color:red">ô</span>tkæppchen₆₈]] 15:48, 2. Mär. 2017 (CET)
::::Was heisst das jetzt genau? "replays of other players" verstehe ich nicht. gruß [[Spezial:Beiträge/79.224.193.227|79.224.193.227]] 17:16, 2. Mär. 2017 (CET)
:::::Jemand spielt Tetris und sein Spiel wird aufgezeichnet. Dieses aufgezeichnete Spiel wird dann eingeblendet, während du spielst. Was du siehst ist also die Aufzeichnung eines Spiels. Der Spieler ist gar nicht mehr anwesend. --[[Benutzer Diskussion:Snevern|Snevern]] 18:40, 2. Mär. 2017 (CET)
:::::: Und so wie es aussieht, war der schon bei der Aufzeichnung nicht mehr anwesend, weil er entweder das Interesse verloren hatte oder weil seine Verbindung zur Website abgerissen ist (kommt öfter vor), weshalb sich die Blöcke einfach uneingepasst stapeln... --[[Benutzer:Gretarsson|Gretarsson]] ([[Benutzer Diskussion:Gretarsson|Diskussion]]) 19:13, 2. Mär. 2017 (CET)
[[Datei:Ice Cream dessert 02.jpg|miniatur|hochkant|Zum Dank eine Runde virtuelles Eis für alle! ;-)]]
:::::::Oh, wie ich euch danke! Das muss ich wohl übersehen haben. Dieses Rätsel hätte mich fast verrückt gemacht... Also nochmals ein '''großes Dankeschön''' für die Aufklärung – damit habt ihr mir die Nacht gerettet!!! <small>(Zur Feier des Tages probiere ich gleich noch mal mein Können und denke dabei an euch!)</small> ;-) Herzlichst, euer--[[Benutzer:Hubon|Hubon]] ([[Benutzer Diskussion:Hubon|Diskussion]]) 01:15, 3. Mär. 2017 (CET)
{{erledigt|1=--[[Benutzer:Hubon|Hubon]] ([[Benutzer Diskussion:Hubon|Diskussion]]) 01:15, 3. Mär. 2017 (CET)}}

== [[Unterführung]] und [[Personentunnel]] ==

Was ist der Unterschied zwischen beiden Sachen? Ist letzteres die spezifische Bezeichnung für Tunnel unter Bahnhöfen, selbst wenn sie in offener Bauweise gebaut und nicht länger als 80 Meter sind, was laut ersterem Artikel nicht als Tunnel gilt? [[Spezial:Beiträge/129.13.72.198|129.13.72.198]] 09:07, 2. Mär. 2017 (CET)
:Ein Tunnel muss nicht unter der Oberfläche liegen.--[[Benutzer:Wikiseidank|Wikiseidank]] ([[Benutzer Diskussion:Wikiseidank|Diskussion]]) 09:22, 2. Mär. 2017 (CET)
::Da schreibt der Artikel [[Tunnel]] aber was anderes. Außerdem: Eine Unterführung auch nicht, siehe erstes Bild im Artikel. [[Spezial:Beiträge/129.13.72.198|129.13.72.198]] 10:11, 2. Mär. 2017 (CET)
::: Intuitiv sehe ich das nur als Frage der Perspektive: Ein Tunnel führt "durch etwas hindurch", eine Unterführung "unter etwas hindurch". Im ersten Fall kann es ein relevantes Objekt über dem Tunnel geben, aber es ist nicht zwingend. Im zweiten Fall muss es ein relevantes Objekt geben, unter dem etwas geführt wird. [[Benutzer:Yotwen|Yotwen]] ([[Benutzer Diskussion:Yotwen|Diskussion]]) 12:18, 2. Mär. 2017 (CET)

== Buch Hiob: Relation Gott - Satan ==

Leider wird auf auf der Disk.Seite des Artikels "Ijob" (Das Buch Hiob im Alten Testament) meine Frage nicht bemerkt. Deshalb versuche ich es hier: In dem Artikel über das geplagte und elende Leben des Hiob fällt der Satz ''"Daraufhin erlaubt Gott es Satan, Ijob zu prüfen."'' Für mich als Leser wird dabei die schlichte Relation Gut vs. Böse bzw. Gott vs. Satan außer Kraft gesetzt. Satan erscheint daher als Diener Gottes, der beim "Allmächtigen" um Erlaubnis bitten muss. Werden nicht Teile des christlichen Weltbildes und die Rolle des "Verführers" dadurch in Frage gestellt? Sprich: Ist der Satz nun falsch (oder falsch formuliert) oder ist dieser Teil der Interpretation des Verfassers (des Artikels) falsch? Ich bin Agnostiker, habe aber die Bibel gelesen. Und dies ist nur ein Teil meiner "Verwunderung" über das "Buch der Bücher". Aber mir erscheint dieser Satz falsch formuliert. Oder entspricht dies der allgemeinen Interpretation der Geschichte Hiobs? Was dann wiederum Fragen über die Relation Gott - Satan aufwirft. Danke schon mal für Meinungen.--[[Spezial:Beiträge/87.184.129.106|87.184.129.106]] 10:17, 2. Mär. 2017 (CET)
: Im Kontext ist die Geschichte doch klar. Und das Verhältnis Gott-Satan geht aus dem Artikel auch hervor. Der eine sagt, Hiob sei nicht oder nur aus bestimmten Gründen fromm und der andere sagt, das Hiob unter allen Umständen fromm sei bzw. bliebe. Und dann prüfen sie Hiob (machen also ihre Wette). Ob ich den Satz mit dem Verb "erlauben" schreiben würde, weiß ich nicht. (Was steht denn in der Bibel?) Aber der Satz für sich alleine verändert doch nicht die Darstellung der damaligen Sicht bzw. Wahrnehmung von Gut und Böse. Ich, als jemand, der--[[Spezial:Beiträge/87.184.129.106|87.184.129.106]] 13:27, 2. Mär. 2017 (CET) als Kind von Gott und dem Teufel auch immer nur im Sinne von Gut und Böse erzählt bekommen hat, staune sowieso, dass die da so locker miteinander sprechen, und dem armen Hiob die Pest aufs Auge drücken, nur um zu beweisen, wie fromm er doch ist/sein kann. Dieses Erstaunen meinerseits hängt aber nicht an diesem einen von Dir kritisierten Satz. VG --[[Benutzer:Apraphul|Apraphul]] <small><sup> [[Benutzer Diskussion:Apraphul|Disk]] </sup> <sub> [[Benutzer:Apraphul/WP:SNZ|WP:SNZ]]</sub></small> 10:40, 2. Mär. 2017 (CET)

::Der Unterschied ist, dass du wohl gläubiger Christ bist und ich Agnostiker. Und es ist in der Tat für mich dieser eine Satz, der mich stutzig macht. Nicht in Bezug auf die Hiobsgeschichte an sich, sondern genau die Aussage des Satzes, dass Satan "um Erlaubnis bitten/fragen muss". Für mich bedeutet dies in der Form, in der der Satz existiert, eine Diskrepanz in der Beziehung Gott - Satan. Wahrscheinlich ist schlicht die Formulierung falsch. Aber wenn man nicht gläubig ist, fallen einem solche Kleinigkeiten auf, da man nicht "glaubt", sondern "nicht weiß" und daher Sätze mit Skepsis liest. Das Anzweifeln von religiösen Aussagen (und der Satz definiert für mich eine Aussage) ist ein Grundsatz des Agnostizismus. Daher reicht ein unverständlicher Satz, um wieder weitere Fragen auf zu werfen. Ich kann auch noch mal in die Bibel schauen. Abseits meiner philosophischen Auffassung, ist es aber auch die sprachliche Komponente, die auffällt, die für mich fragliche Aussage des einen Satzes. Daher auch meine Frage: Ist der Satz, der eine (vermeintliche) Tatsache darlegt, in der vorliegenden Form und dem Wort "erlauben", richtig oder falsch?--[[Spezial:Beiträge/87.184.129.106|87.184.129.106]] 11:03, 2. Mär. 2017 (CET)

:: Man sollte das Buch Hiob nicht mit christlichen Massstab messen, da es jüdischen Ursprungs ist. Das eine ist, das derSatan nicht wirklich entscheidend für die Aussage des Buches ist, wichtig ist die Treue Hiobs zu Gott und die Widersprüche seiner Freunden. Wie es zu den Unglücken kam spielt m.E keine so große Rolle. Das andere ist dass sich die Vorstellungen über den Satan/Diabolos sicher seit der Entstehung des Buche geändert hat, sowohl im Judentum als auch im Christentum. Siehe auch [[Teufel#Judentum]] [[Spezial:Beiträge/178.3.144.88|178.3.144.88]] 10:57, 2. Mär. 2017 (CET)

:::@178.3..., einmal das; und, @87.184..., wäre ich wer weis wie gläubig, könnte ich Dir sicher eher sagen, wie der Satz gemeint ist und ob er passend ist oder nicht. Kann ich aber nicht. Während Du Dich an einen Satz hängst (nicht böse gemeint) und sich Dir damit und deswegen noch weitere Fragen stellen, sehe ich die Aussagen zu der Geschichte im Artikel im Kontext, also als Ganzes. Und da werfen sich für mich dann nicht wirklich viele Fragen auf. Einzelne Sätze ausserhalb ihres Zusammenhangs dürfen m. M. n. nicht zu streng gesehen werden. Wenn das Wort "erlauben" hier tatsächlich irreführend sein sollte und wenn man zudem bedenkt, dass Gott es nicht selbst tun will und es aber auch nicht verhindern will, so könnte man vielleicht mit "zulassen" anstelle von "erlauben" umschreiben. Das dürfte man sicher im Artikel entsprechend so korrigieren. VG --[[Benutzer:Apraphul|Apraphul]] <small><sup> [[Benutzer Diskussion:Apraphul|Disk]] </sup> <sub> [[Benutzer:Apraphul/WP:SNZ|WP:SNZ]]</sub></small> 11:14, 2. Mär. 2017 (CET)

::::Ich habe nur die King James Bibel vorliegen, werde aber noch einmal genau lesen. Ich hänge mich wirklich an den einen Satz, da er irrtierend wirkt und sehr viel in Frage stellt. Vor allem für "Nichtgläubige". Und zumindest in meiner Auslegung sagt der Satz sehr viel über die Beziehung Gott - Satan aus. Wenn der Satz falsch ist, werde ich ihn entfernen bzw. anpassen. PS: Ich wollte dich nicht als Gläubigen abstempeln, mitnichten. Ich dachte nur, da du als Kind damit "konfrontiert" wurdest... ;-) --[[Spezial:Beiträge/87.184.129.106|87.184.129.106]] 13:28, 2. Mär. 2017 (CET)
::::: <small>Kein Problem. :-) Ich bin mittlerweile in einem Alter, in dem man öfter schon mal die (geglaubte) bloße Möglichkeit, dass es mehr geben könnte zwischen Himmel und Erde als der Mensch erklären kann, sehr begrüßt. Es hilft manchmal, Dingen einen Sinn (oder mögliche Erklärungen) zu geben, an die man in der rein wissenschaftlich akzeptierten Welt ansonsten vielleicht verzweifeln würde. Von daher dürfte man mich auch gerne als gläubig "abstempeln". :-) VG --[[Benutzer:Apraphul|Apraphul]] <small><sup> [[Benutzer Diskussion:Apraphul|Disk]] </sup> <sub> [[Benutzer:Apraphul/WP:SNZ|WP:SNZ]]</sub></small> 13:58, 2. Mär. 2017 (CET)</small>

::::::<small>Na, dann... ;-)--[[Spezial:Beiträge/87.184.129.106|87.184.129.106]] 14:04, 2. Mär. 2017 (CET)</small>

::::Sorry, aber die Frage von OP ist schon berechtigt. Man kann biblische Texte selbstverständlich analysieren wie jeden anderen Text auch, und dabei spielt dann natürlich jedes Detail eine Rolle. Dass eine Figur "nicht wirklich entscheidend für die Aussage" ist, mag im Hinblick auf den Glauben richtig sein, aber einer Textanalyse steht das nicht im Weg. Ich bin mir sicher, dass speziell zu diesem "Detail" hier schon viel geschrieben worden ist, kann aber leider selbst nicht weiterhelfen. --[[Spezial:Beiträge/89.246.198.251|89.246.198.251]] 11:53, 2. Mär. 2017 (CET)
::::::Bin ich OP? Und was heißt OP... :-)?--[[Spezial:Beiträge/87.184.129.106|87.184.129.106]] 14:06, 2. Mär. 2017 (CET)
::::::: <small>Jawohl, Du bist der Fragensteller, der Abschnittsersteller oder Abschnittseröffner, der neudeutsch Threadopener (TO) oder neudeutsch kurz Opener (OP). Na, wie auch immer - Du bist gemeint. ;-) VG --[[Benutzer:Apraphul|Apraphul]] <small><sup> [[Benutzer Diskussion:Apraphul|Disk]] </sup> <sub> [[Benutzer:Apraphul/WP:SNZ|WP:SNZ]]</sub></small> 14:11, 2. Mär. 2017 (CET)</small>

::::::::<small>Ok. Ich bin ein OP, aber auch ein TO. Gut zu wissen. Man lernt nie aus, auch in unserem, sorry, meinem Alter nicht... ;-)--[[Spezial:Beiträge/87.184.129.106|87.184.129.106]] 14:42, 2. Mär. 2017 (CET)</small>
:::::::::<small>"OP" heißt Original Poster, siehe [[Liste von Abkürzungen (Netzjargon)#O]]. --[[Benutzer:Neitram|<span style="color:#008800">Ne</span><span style="color:#005555">it</span><span style="color:#005588">ra</span><span style="color:#0000FF">m</span>]]&nbsp;[[Benutzer_Diskussion:Neitram|<span style="font-size:large">✉</span>]] 17:15, 2. Mär. 2017 (CET)</small>

:::::Aber dann muss man den Text lesen und nicht die Inhaltsangabe in der Wikipedia.
:::::Hilfreich ist vielleicht der Artikel [[Satan]]. --[[Benutzer:Digamma|Digamma]] ([[Benutzer Diskussion:Digamma|Diskussion]]) 12:27, 2. Mär. 2017 (CET)

::::::Dann ist aber der Satz im Artikel Ijob falsch und sollte geändert werden. Denn schließlich sollte Wikipedia das (paraphrasiert) wiedergeben, was textlich in der Bibel steht und dabei keine Eigeninterpretation liefern. Es ist nur ein Satz, aber er fällt einem, wenn man über einigermaßen kompetentes Textverständnis und linguistisches sowie literatur-analytisches Grundwissen verfügt, ins Auge. Schließlich sagt er in der jetzigen Form weitaus mehr als, als man meinen mag. --[[Spezial:Beiträge/87.184.129.106|87.184.129.106]] 13:38, 2. Mär. 2017 (CET)

:::::::In meiner Bibel (Einheitsübersetzung von 1980) steht
::::::::"Der Herr sprach zum Satan: Gut, all sein Besitz ist in deiner Hand, nur gegen ihn selbst streck deine Hand nicht aus! Darauf ging der Satan weg vom Angesicht des Herrn." {{Bibel|Ijob|1|12|EU}}
:::::::Ist das mit dem Satz "Daraufhin erlaubt Gott es Satan, Ijob zu prüfen." richtig wiedergegeben? --[[Benutzer:Digamma|Digamma]] ([[Benutzer Diskussion:Digamma|Diskussion]]) 16:30, 2. Mär. 2017 (CET)

:::::::::Im Artikel [[Ijob]], Absatz "Inhalt: Handlung: Frömmigkeit:" steht der letzte Satz so. Und um diesen Satz geht es. Wohlgemerkt, es ist KEIN Bibelzitat, sondern die Inhaltsangabe (und leider auch Interpretation) des Artikelschreibers.--[[Spezial:Beiträge/87.184.129.106|87.184.129.106]] 20:11, 2. Mär. 2017 (CET)

::::::::::Das ist mir auch klar. Deshalb ja meine Frage: Ist deiner Meinung nach die zitierte Aussage aus der Bibel ("Der Herr sprach ...") durch den Satz im Artikel ("Daraufhin erlaubt ...") richtig wiedergegeben? Beruht dein Unbehagen nur auf der Formulierung im Artikel, oder spürt du es auch bei der Originalformulierung der Bibel? --[[Benutzer:Digamma|Digamma]] ([[Benutzer Diskussion:Digamma|Diskussion]]) 20:22, 2. Mär. 2017 (CET)

:::::::::::Der interpretative Satz im Artikel ist mein Stein des Anstoßes, wenn man so will. Ich dachte, das wäre im Verlauf der Diskussion bereits durchgeklungen. Der Satz ist, nachdem ich noch mal in die Bibel (KJB) geschaut habe, falsch. Den gibt es, soweit ich es überblicken kann und ich lese jetzt nicht das ganze Buch Hiob noch einmal, so nicht. Also ist der Satz des Schreibers im Artikel "Ijob" überflüssig. Der philosophische Ansatz über die Relation Gott - Satan hat nur Berechtigung, wenn der Satz so auch irgendwo auftauchen würde. Soweit ich jetzt erkenne, sind wir hier zum Schluss gekommen (und diese Disk. läuft mehr oder weniger den ganzen Tag), dass es die falsche Interpretation des Schreibers ist, die nicht passt. Wobei das ein User (letzter Abschnitt) wieder anders sieht und die von mir angesprochene Frage zur Position Satans im hirarchischen Aufbau der auftretenden "Charaktere" wieder ins Spiel bringt. Und nein, ich spüre kein "Unbehagen" bei der biblischen Originalformulierung. Als Agnostiker spüre ich beim Lesen der Bibel relativ wenig.--[[Spezial:Beiträge/87.184.159.68|87.184.159.68]] 21:47, 2. Mär. 2017 (CET)

::::::::::::Im Buch Hiob steht: "Der Herr sprach zum Satan: Gut, all sein Besitz ist in deiner Hand, nur gegen ihn selbst streck deine Hand nicht aus! Darauf ging der Satan weg vom Angesicht des Herrn." Meiner Meinung nach kann man das schon mit "er erlaubte" wiedergeben. Was ist es denn deiner Meinung nach sonst?
--[[Benutzer:Digamma|Digamma]] ([[Benutzer Diskussion:Digamma|Diskussion]]) 22:15, 2. Mär. 2017 (CET)

:::::::::::::Ein Rat, eine Abmachung oder ein Kompromiss. ''"Daraufhin erlaubt Gott es Satan, Ijob zu prüfen."'' ist jedoch die Behauptung, ein Höherstehender würde seinem Untergebenen etwas zugestehen, was er aber auch ablehnen kann. Was andeutet, dass es eine hirarchische Befehls- und Ordnungsstruktur gibt. Die jedoch in der Bibel, soweit ich mich erinnere, nach dem "Höllensturz", also dem "Rauswurf" der aufständischen Engel aus dem Himmel, so nicht gegeben ist. Ich finde aber auch Benutzer Meloes (s.u.) Einschätzung interessant, die genau diese Frage der Hirarchie aufwirft.--[[Spezial:Beiträge/87.184.159.68|87.184.159.68]] 10:27, 3. Mär. 2017 (CET)

:::::::: Klingt für mich nach einer ausdrücklichen Erlaubnis. <small>Aber vielleicht sollte man zur Sicherheit noch einen Rechtsanwalt hinzuziehen (SCNR).</small> --[[Benutzer:Gretarsson|Gretarsson]] ([[Benutzer Diskussion:Gretarsson|Diskussion]]) 17:10, 2. Mär. 2017 (CET)

:::::::: Luther verwendet da noch n Konjunktiv, weil er wohl noch Latein konnte... [https://www.biblegateway.com/passage/?search=Hiob+1:12&version=LUTH1545]... in [[Constantine (Film)]] mit Keanu Reeves *hechel* hat der Satan irgendwie auch noch n Wörtchen mitzureden, indem er Krankheiten lindert, die ansonsten mit dem Leben nicht länger vereinbar sind (also schon ziemlich mächstig...)... ansonsten sind Menschen-Experimente Gott ja nich fremd: Adam (mit der dicken langen Schlange), Kain (der wilde Brutalo), und mehr weiß ich nich (aber ob Gott nun ne Schlange bastelt oder irgendwie Zwietracht säht oder dem Satan suggeriert, er habe fast freie Hand, ist doch irgendwie immer das Gleiche...)... wir haben uns in der Schule immer gefragt, warum wir an irgendwas Schuld sein sollten, wenn Gott das alles so unnötig kompliziert gemacht hat... --[[user:Homer Landskirty|Heimschützenzentrum]] ([[user talk:Homer Landskirty|?]]) 18:04, 2. Mär. 2017 (CET)

::::::::: Die Frage ist doch nicht, ob der Übersetzer Latein kann, sondern was die bessere Übersetzung ist. --[[Benutzer:Digamma|Digamma]] ([[Benutzer Diskussion:Digamma|Diskussion]]) 18:16, 2. Mär. 2017 (CET)
::::::::: <small>In der Vulgata steht übrigens der Indikativ "sunt". --[[Benutzer:Digamma|Digamma]] ([[Benutzer Diskussion:Digamma|Diskussion]]) 20:25, 2. Mär. 2017 (CET)</small>

:::::::::: dann ist das „sei“ wohl irgendwie kein Konjunktiv? bevor man sich fragt, ob ne Erlaubnis passt, sollte man erstmal rausfinden, ob es ne Erlaubnis ist... darum sollte man möglichst dicht am Original sein... --[[user:Homer Landskirty|Heimschützenzentrum]] ([[user talk:Homer Landskirty|?]]) 21:56, 2. Mär. 2017 (CET)

::::::::::Um diese Frage geht es ursprünglich auch gar nicht. Sondern darum, ob der Satz "Daraufhin erlaubt Gott es Satan, Ijob zu prüfen.", der so im Artikel Ijob steht (Kein Bibelzitat!), so Bestand hat. Der Schreiber hat seine eigene Interpretation in diesen einzigen Satz gepackt. Und da geht es jetzt darum, diesen zu löschen oder umzuformulieren.--[[Spezial:Beiträge/87.184.129.106|87.184.129.106]] 20:17, 2. Mär. 2017 (CET)

:::::::::::Als Agnostiker bin ich der Meinung, dass man sich bei der Inhaltsangabe nicht davon leiten lassen sollte, ob die Aussage zum christlichen Weltbild passt. --[[Benutzer:Digamma|Digamma]] ([[Benutzer Diskussion:Digamma|Diskussion]]) 20:34, 2. Mär. 2017 (CET)

::::::::: Ändert für mich nicht wirklich etwas daran, dass es nach einer ausdrücklichen Erlaubnis klingt. „''So sei es''“ ist auch nur eine etwas hochtrabende Formulierung für „''Deal!''“ oder „''Top, die Wette gilt!''“ Der Konjunktiv ist da nicht überzubewerten... --[[Benutzer:Gretarsson|Gretarsson]] ([[Benutzer Diskussion:Gretarsson|Diskussion]]) 19:20, 2. Mär. 2017 (CET)

::::::::::Im Gegenteil: Der Konjunktiv bedeutet eine ausdrückliche Aufforderung ("soll sein"). --[[Benutzer:Digamma|Digamma]] ([[Benutzer Diskussion:Digamma|Diskussion]]) 19:23, 2. Mär. 2017 (CET)

:Ich verstehe die biblische Erzählung so: Gott ist allmächtig, Satan nicht. Daher diskutiert Satan mit Gott vorab - vor allen Engeln - damit Gott nicht die folgenden Handlungen Satans aus der Position des Stärkeren heraus einfach verhindert oder rückgängig macht. Das heißt aber für mich noch nicht, dass Satan ein Diener Gottes wäre; sein Handeln wird von Gott bloß geduldet, damit am Ende für die beobachtenden Engel klar wird, dass Gott nicht nur der Stärkere ist, sondern dass Satan mit seinen Behauptungen unrecht hatte. Und der Allmächtige schafft trotzdem, zumindest im Nachhinein. auch für den leidenden Menschen noch einen versöhnlichen Ausklang. --[[Benutzer:DemFriedrichGrafzuliebeangemeldet|DFGza]] ([[Benutzer Diskussion:DemFriedrichGrafzuliebeangemeldet|Diskussion]]) 13:33, 2. Mär. 2017 (CET)
:::Aber wenn Gott allmächtig ist, warum muss er dann dieses perfide Spiel spielen? Er weiß dann doch schon wie es ausgeht/ausgehen würde. Mir waren Sadisten immer schon suspekt, genauso wie Personen, die sich immer wieder beweisen lassen müssen, was für tolle Hechte sie sind. Vielen Dank! --[[Benutzer:Elrond|Elrond]] ([[Benutzer Diskussion:Elrond|Diskussion]]) 15:19, 2. Mär. 2017 (CET)
::Aber wenn Gott allmächtig ist, warum duldet er Satan und vernichtet ihn nicht? Weil Satan zum christlichen Weltbild dazugehört? Weil Gott die Hölle und ihren Herrscher braucht, um den Menschen zu strafen und ihn die Furcht vor dem Bösen fühlen lässt? Insofern sehe ich Satan als Teil von Gottes Plan, was dann wiederum bedeuten könnte, das Satan kann reiner Gegenspieler, sondern trotz aller Gegensätze, unter der allmächtigen Gewalt Gottes steht. Und im Einzelfall das tun muss, was Gott sagt. Natürlich ist das alles rein hypothetisch gesprochen. Aber vielleicht ist es jetzt ein bißchen zu viel des Guten... ;-) --[[Spezial:Beiträge/87.184.129.106|87.184.129.106]] 13:49, 2. Mär. 2017 (CET)

::"Satan erscheint daher als Diener Gottes, der beim Allmächtigen um Erlaubnis bitten muss." Diese Aussage ist völlig korekt, sie gilt nach amtskirchlich christlichen Verständnis sowieso für alle Handlungen Satans. Das Buch Hiob ist dazu noch ein Sonderfall, weil es die historisch älteste Darstellung der Materie ist. Er ist hier (noch) nicht der nahezu allmächtige Fürst der Finsternis, sondern eher sowas wie ein Ankläger-Engel an Gottes Hofstaat. Die Rolle des Teufels ist innerhalb der christlichen Tradition einschneidenden Änderungen unterworfen gewesen und auch innerhalb der Bücher der Bibel nicht widerspruchsfrei. Zur vertiefenden Lektüre empfohlen: Kurt Flasch: Der Teufel und seine Engel. Die neue Biographie. Beck Verlag, München 2015, ISBN 978-3-406-68412-8.--[[Benutzer:Meloe|Meloe]] ([[Benutzer Diskussion:Meloe|Diskussion]]) 13:54, 2. Mär. 2017 (CET)

Spätestens seit der Aufklärung wissen wir, dass wir unser Weltbild nicht auf der Basis eines einzigen Buches aufbauen sollten. Bibliotheken führen die Bibel, aber sie führen auch 1.000.000 andere Bücher. Aus der Bibel kann man spannende Dinge ziehen, aber aus all den anderen Büchern eben auch. Die Widersprüchlichkeit der Bibel ist analog zur Widerspruchlichkeit dessen was andere Bücher bëinhalten, etwa die Schriften von Naturwissenschaftlern und Philosophen. Da sie, wie alle Bücher dieser Welt, von Menschen geschrieben wurde, sollte Dich die Widersprüchlichkeit der Bibel doch nicht wirklich verwundern? [[Spezial:Beiträge/62.44.134.189|62.44.134.189]] 06:40, 3. Mär. 2017 (CET)
:<small><small>Ich halte es nicht für realitätsfern. Die meisten Chefs (die ich kennenlernen durfte) haben doch einen Adlatus fürs Grobe. Das war sicherlich auch früher so - und da hat man es halt mit hineingeschrieben. Heisst ja nicht umsonst "der Versucher". <small style="color:grey"><b>'''GEEZER'''</b></small><sup>[[BD:Grey Geezer|<span style="color:grey"> … nil nisi bene</span>]]</sup> 07:44, 3. Mär. 2017 (CET) </small></small>

: Du schreibst reichlich am Thema vorbei. Es geht darum, ob ein Satz, der die falsche Aussage hat, in einen Artikel gehört oder nicht. Als Agnostiker wundert mich nicht viel, aber vieles verstehe ich nicht, weil es rational eigentlich keinen Sinn macht. Deshalb bin ich wohl auch Agnostiker.--[[Spezial:Beiträge/87.184.159.68|87.184.159.68]] 10:02, 3. Mär. 2017 (CET)

Ich habe den Satz im Artikel [[Ijob]] jetzt umformuliert. Sollte es jemand anders sehen, dann bitte nicht einfach löschen oder zurück setzen, sondern hier argumentativ diskutieren. Danke.--[[Spezial:Beiträge/87.184.159.68|87.184.159.68]] 12:07, 3. Mär. 2017 (CET)

:Zunächst geht es darum, ob der Inhalt von {{B|Ijob|1|12}} mit "Gott erlaubt Satan" korrekt wiedergegeben ist. Ich denke dass ja. Der erste Satz hat kein Verb "alles was ihm [gehört] [ist] in deiner Hand". Den kann man als Feststellung "es ist" oder als Erlaubnis "es sei" auffassen, wobei ersteres wörtlicher ist. Der zweite Satz aber schränkt den ersten mit einem [[Jussiv]] ein ("du mögest deine Hand nicht gegen ihn ausstrecken"), was eben nicht eine Feststellung, sondern eine Erlaubnis einschränkt.

:Generell ist das Verhältnis Gott–Satan in der Bibel nie als purer Dualismus gleicher Partner beschrieben, sondern geht immer von der Souveränität Gottes aus, die der Satan nicht hat, z.B. {{B|Sach|3}}, wo der Satan in einer Vision als Ankläger erscheint, der jedoch nicht zu Wort kommt, weil sich Gott nicht für die Anklage interessiert. Auch im Buch Ijob interessiert sich niemand mehr für den Satan, wo der doch eine Wette verloren hat. Ansonsten ist das Bild Satans in der Bibel durchaus recht schillernd; das ist aber Thema des Artikels [[Satan]] und gehört nicht nach [[Ijob]]. --[[Benutzer:Lantani|Lantani]] ([[Benutzer Diskussion:Lantani|Diskussion]]) 13:26, 3. Mär. 2017 (CET)

== Übersetzer/-ung des Coleridge Gedichts "Ballade vom Alten Seemann" ==

Vorweg: ich habe bereits eine Übersetzung dieses Gedichts, aber diese hier finde ich wesentlich besser, habe aber nur diese wenigen Zeilen:
* "'..Gott schütze dich, alter Seefahrer,
* Was war es, das da dich verdroß?
* Was raubt dir das Heil?' - 'Mit meinem Pfeil
* Schoß ich den Schicksalsvogel ALBATROS!'"
Google bringt mich nicht weiter. Wo könnte ich ''diese'' Übersetzung komplett finden oder nach dem Übersetzer suchen? Das beschäftigt mich schon länger. Danke. --[[Spezial:Beiträge/95.208.26.144|95.208.26.144]] 11:30, 2. Mär. 2017 (CET)
:[https://books.google.at/books?id=zTO6AAAAIAAJ&q=pfeil Hier] sehe ich schon mal den Ausschnitt (allerdings ohne den "Schicksalsvogel"). --[[Benutzer:Wrongfilter|Wrongfilter]] [[Benutzer Diskussion:Wrongfilter|...]] 11:43, 2. Mär. 2017 (CET)
:Hast Du [https://forum.donald.org/read.php?1,5897 das hier (klick)] gesehen? Besonders der zweite Post ... VG --[[Benutzer:Apraphul|Apraphul]] <small><sup> [[Benutzer Diskussion:Apraphul|Disk]] </sup> <sub> [[Benutzer:Apraphul/WP:SNZ|WP:SNZ]]</sub></small> 11:53, 2. Mär. 2017 (CET)
:(BK und Kampf mit dem Spamfilter) Der von mir genannte Ausschnitt stammt aus dem Aufsatz "Der rassistische Albatros" von Immanuel Wallerstein (gefunden durch Suche nach "Albatros" in dem Buch/der Zeitschrift), dazu findet sich ein pdf [http://www.studienverlag.at/data.cfm?vpath=openaccess/oezg-32003-wallerstein&download=yes hier] , damit kann man in Anmerkung 2 nachschauen und findet die Angabe des Übersetzers: Heinz Pollitzer (Frankfurt am Main 1993). (An Erika Fuchs dachte ich übrigens auch schon...) --[[Benutzer:Wrongfilter|Wrongfilter]] [[Benutzer Diskussion:Wrongfilter|...]] 12:01, 2. Mär. 2017 (CET)
:Abschließend: [[Heinz Politzer]] mit einem ''l''. Ich finde keine Ausgabe von 1993, nur von 1963 und 1968, die [https://www.zvab.com/buch-suchen/titel/der-alte-seefahrer/autor/taylor-coleridge/ antiquarisch] nicht schwer zu finden sind. --[[Benutzer:Wrongfilter|Wrongfilter]] [[Benutzer Diskussion:Wrongfilter|...]] 12:10, 2. Mär. 2017 (CET)
:: Der Link von Aprahul hatte mich auch schon eher weitergebracht, weil es auch den Übersetzer enthält. Das pdf hatte ich gleich danach gefunden. Der antiquarische Link führte bei mir nicht zu dieser Website. Danke für die vielen Antworten. [[Spezial:Beiträge/95.208.26.144|95.208.26.144]] 12:38, 2. Mär. 2017 (CET)
: @wrongfilter: tschuldigung, ich hatte nicht den Link angeklickt, sondern die Adresse händisch kopiert und keine Seite angezeigt bekommen.[[Spezial:Beiträge/95.208.26.144|95.208.26.144]] 12:41, 2. Mär. 2017 (CET)

[[Ferdinand Freiligrath]]: [[:s:Benutzer:A. Wagner/DER ALTE MATROSE]] ([[The Rime of the Ancient Mariner]]). --[[Benutzer:Vsop|Vsop]] ([[Benutzer Diskussion:Vsop|Diskussion]]) 15:42, 2. Mär. 2017 (CET)
:Das ist nicht die gesuchte Übersetzung. Bei Freiligrath lautet die betreffende Strophe: "Vor bösen Geistern schütz’ dich Gott,//Du alter Schiffsgenoß!//Was stierst du? – mit der Armbrust mein//Schoß ich den Albatros!". --[[Benutzer:Wrongfilter|Wrongfilter]] [[Benutzer Diskussion:Wrongfilter|...]] 15:45, 2. Mär. 2017 (CET)
::<small>Ich kenne da noch die Donald-Duck-Variante: "Weh mir Frevler, daß ich schoß den Schicksalsvogel Albatros! Dreimal wehe, daß ich traf, denn nun trifft mich des Schicksals Straf!" --[[Benutzer:Neitram|<span style="color:#008800">Ne</span><span style="color:#005555">it</span><span style="color:#005588">ra</span><span style="color:#0000FF">m</span>]]&nbsp;[[Benutzer_Diskussion:Neitram|<span style="font-size:large">✉</span>]] 17:13, 2. Mär. 2017 (CET)</small>
::Politzer 1963 https://books.google.de/books?id=kXk4AAAAIAAJAA. Dass man eine Eindeutschung, die nicht mal den Rhythmus halten kann, „wesentlich besser“ finden könnte als die von Freiligrath, die damit keine Schwierigkeiten hat, hatte ich mir bisher nicht vorstellen können. --[[Benutzer:Vsop|Vsop]] ([[Benutzer Diskussion:Vsop|Diskussion]]) 17:16, 2. Mär. 2017 (CET)
: @wrongfilter: Die Übersetzung von Freiligrath kannte ich bisher gar nicht, danke für den Tipp. --[[Spezial:Beiträge/95.208.26.144|95.208.26.144]] 19:59, 2. Mär. 2017 (CET)
:: Der Dank muss [[Benutzer:Vsop|Vsop]] gelten. --[[Benutzer:Wrongfilter|Wrongfilter]] [[Benutzer Diskussion:Wrongfilter|...]] 20:33, 2. Mär. 2017 (CET)
Übersetzt wurde der Mariner in jüngerer Zeit u.a. von [[Wolfgang Breitwieser]] (1963), [[Siegfried Schmitz]] (1967), [[Edgar Mertner]] (1973), [[Günter Kunert]] (1983)...--[[Benutzer:Edith Wahr|Edith Wahr]] ([[Benutzer Diskussion:Edith Wahr|Diskussion]]) 21:23, 2. Mär. 2017 (CET)
@ Wrongfilter: Ja, natürlich, ich hatte es erst gesehen, als ich es bereits eingegeben hatte. Also @ Vsop: danke, ebenso an @ Edith Wahr. --[[Spezial:Beiträge/95.208.26.144|95.208.26.144]] 23:52, 2. Mär. 2017 (CET)

== Post-sowjetische Staatsbürgerschaft ==

Gestern habe ich einen Foreign-Policy-Artikel gelesen (<strike>ist aktuell noch [http://foreignpolicy.com/2017/02/28/the-curious-case-of-the-soviet-citizen-trapped-in-2017-soviet-union-russia-ex-convict-released-passport-europe-eurasia/ frei verfügbar], wird aber bald hinter einer Paywall verschwinden</strike> Sorry, nicht gesehen, ist schon weg…), in dem es um einen Mann geht, der kurz vor Auflösung der Sowjetunion in ein russisches Gefängnis kam und jetzt entlassen wurde. Er hat naturgemäß die damaligen Fristen verpasst, die russische Staatsbürgerschaft direkt im Anschluss zu erwerben, und da er ursprünglich aus Kasachstan kommt, wollen die russischen Behörden vor Ort ihn jetzt nicht die russische Staatsbürgerschaft geben. So weit, so gut. Mit Hilfe der [[:en:Citizenship of Russia|englischen Wikipedia]] (und dabei auf sie vertrauend) konnte ich auch gut nachvollziehen, warum er 1991 nicht automatisch russischer Bürger geworden ist und wohl jetzt auch ziemlich schlechte Karten hat, die russische Staatsbürgerschaft zu bekommen. Der Artikel [[:en:Kazakhstani nationality law|zu Kasachstan]] ist leider … wenig aussagekräftig. Zwei Fragen:
*1.) Kann jemand erklären, wie die Lage in Kasachstan aussieht, und ob der Mann genau so schlechte Chancen auf eine kasachische wie auf eine russische Staatsbürgerschaft hat?
*2.) Als spektakulär später Fall ist das sicherlich ein Einzelfall, aber ich kann mir vorstellen, dass es nach dem Zusammenbruch der Sowjetunion doch ziemlich oft zu „Entstaatlichungen“ von Ex-Sowjetbürger gekommen sein muss. Der englische WP-Artikel zur russischen Staatsbürgerschaft hat ein bisschen was dazu. Auf jeden Fall scheint es ja keine Automatismen gegeben haben. Gab es denn damals viele Fälle, in denen entweder Fristen versäumt worden sind oder aber die Betroffenen vielleicht auch absichtlich verzichtet haben, Anträge zu stellen? Wie groß war die Zahl der Ex-Sowjetbürger ohne neue Staatsbürgerschaft, und wie viele davon gibt es noch heute?
--[[Spezial:Beiträge/92.225.69.252|92.225.69.252]] 12:00, 2. Mär. 2017 (CET)
:Tja. Die Lage der ''[[Nichtbürger]]'' ist besonders krass in [[Nichtbürger (Lettland)|Lettland]], Estland und Litauen. Bleibt die Staatsangehörigkeit unklar, kann zuerst ein ''[[Fremdenpass]]'' beantragt werden.--[[Benutzer:ElmarG|ElmarG]] ([[Benutzer Diskussion:ElmarG|Diskussion]]) 13:04, 2. Mär. 2017 (CET)

== Alexander Lion ==

Wann starb [[Alexander Lion]]? (Aktuell steht er auf der Hauptseite unter "Was geschah am 2. März?".) Siehe [[Diskussion:Alexander_Lion#Sterbeort_und_-datum]] -- ich mache für heute Feierabend, vielleicht möchten andere die Suche weiterführen. --[[Benutzer:Neitram|<span style="color:#008800">Ne</span><span style="color:#005555">it</span><span style="color:#005588">ra</span><span style="color:#0000FF">m</span>]]&nbsp;[[Benutzer_Diskussion:Neitram|<span style="font-size:large">✉</span>]] 17:05, 2. Mär. 2017 (CET)

== ZFC-Axiome als normale Aussagesätze auffassen ==

Warum werden die Axiome von ZFC (Zermelo-Fraenkel-Mengenlehre mit Auswahlaxiom) als Axiome eingeführt? Macht es nicht mehr Sinn, sie als normale Aussagesätze einzuführen und zu beweisen? Den Satz des Pythagoras führt man ja auch nicht als Axiom ein, sondern beweist ihn.
<!-- Lass die nachfolgende Zeile am ENDE deiner Frage stehen. Sie wird in deine Signatur umgewandelt. -->
<!-- Lass die nachfolgende Zeile am ENDE deiner Frage stehen. Sie wird in deine Signatur umgewandelt. -->
--[[Benutzer:Thomas Limberg 1986|Thomas Limberg 1986]] ([[Benutzer Diskussion:Thomas Limberg 1986|Diskussion]]) 17:29, 2. Mär. 2017 (CET)
--[[Spezial:Beiträge/2A02:908:1B54:47A0:4464:7E2E:F77F:AB06|2A02:908:1B54:47A0:4464:7E2E:F77F:AB06]] 19:51, 1. Okt. 2020 (CEST)
:Am besten höflich fragen. Langes a ist zu vermuten, das sz vermutlich ein Erbe der barocken Mode der Buchstabenhäufung. Aber man weiß es nicht. Grüße [[Benutzer:Dumbox|Dumbox]] ([[Benutzer Diskussion:Dumbox|Diskussion]]) 20:14, 1. Okt. 2020 (CEST)
:+1 Nett fragen, die Dame wird es wahrscheinlich kennen. Es gibt auch regionale Unterschiede wie man (Namen ausspricht. Beispiel mögen die Namen Vois und Broich oder Fußbroich sein, der im Rheinland (wo diese Namen auch herkommen) mit langem 'o', also Voos und Brooch ausgesprochen werden und nicht mit 'oi'. --[[Benutzer:Elrond|Elrond]] ([[Benutzer Diskussion:Elrond|Diskussion]]) 20:26, 1. Okt. 2020 (CEST)
::+1 nett fragen, dürfte aber vermutlich von ''[[Hass|Haß]]'', Hauptwort zu ''hassen'' kommen. Namen machen ja üblicherweise keine Rechtschreibreformen mit. -- [[Benutzer:Aspiriniks|Aspiriniks]] ([[Benutzer Diskussion:Aspiriniks|Diskussion]]) 20:37, 1. Okt. 2020 (CEST)
:::Nö, es kommt von „der Hesse“. --[[Spezial:Beiträge/84.190.201.61|84.190.201.61]] 20:52, 1. Okt. 2020 (CEST)
::: (BK) Danke für Antworten, bin allerdings nach Recherchen auch auf [https://www.wn.de/Archiv/2011/12/In-Hasse-steckt-der-Rufname-Hasso dieses], ergo Abstammung vom Vornamen [[Hasso]] oder auch [[Hesse (Begriffsklärung)|Hesse]] gestoßen. Klärt natürlich nicht eine verbindliche Aussprache, macht aber einiges verständlicher. Was auch für die Hessen spräche wäre evtl etwa auch der Flurname [[Haßberge]]. Was mich auch etwas verunsicherte war zudem die Aussprache von [[Maß]] (in schweizerdeutsch geschrieben mit ''ss'', was mich als tatsächlicher Hesse dann doch immer etwas verwirrte ;)--[[Spezial:Beiträge/2A02:908:1B54:47A0:4464:7E2E:F77F:AB06|2A02:908:1B54:47A0:4464:7E2E:F77F:AB06]] 20:55, 1. Okt. 2020 (CEST)
::::Die Herkunft des Namens oder gar die Etymologie hilft da nicht weiter. Manche Leute werden eben „falsch“ ausgesprochen, so wie es nach der Herkunft eigentlich nicht sein dürfte. Beispiel: ''Kubicki'' (bekannter Politiker) müsste eigentlich „Kubitzki“ ausgesprochen werden, heiß aber trotzdem „Kubikki“, und ihm gegenüber würde ich das auch niemals anders aussprechen. --[[Benutzer:Dioskorides|Dioskorides]] ([[Benutzer Diskussion:Dioskorides|Diskussion]]) 21:50, 1. Okt. 2020 (CEST)


:::::Bekanntes Beispiel ist der ehemalige britische Premierminister Lord Home, der [hjuum] ausgesprochen wird. Der Grund soll folgender sein: Adelige hatten früher höhere millitärische Ränge. Wenn sich die Soldaten um ihr Feldzeichen sammeln sollten, wurde dazu auch laut der Name des millitärischen Anführers gerufen. Das wäre bei "home" ziemlich ungünstig. --[[Benutzer:Optimum|Optimum]] ([[Benutzer Diskussion:Optimum|Diskussion]]) 00:06, 2. Okt. 2020 (CEST)
: Es gibt keine Aussagen, auf die die Sätze zurückgeführt werden könnten. An irgend einem Punkt musst du ein Axiom aufstellen, auf welches sich spätere Aussagen berufen. So kannst du den Satz des Pythagoras auf die ZFC-Axiome zurückführen. Oder besser [[Axiom]]. [[Benutzer:Yotwen|Yotwen]] ([[Benutzer Diskussion:Yotwen|Diskussion]]) 17:36, 2. Mär. 2017 (CET)


:: Wow, echt schnelle Reaktion hier !!! Aber:
:::::: Da möchte ich gerne den meisten meiner Vorrednern zustimmen: Einfach danach fragen. Beim ersten Mal hat man einen Freibrief, das ist wie bei einem Telefonat, wenn man beim ersten Mal den Namen nicht gleich verstanden hat, der Gegenüber wird gerne antworten, auch auf mehrfache Nachfragen. Nur beim zweiten Mal (Telefonat) wirds peinlich. Also: nur zu! --[[Benutzer:Hexakopter|Hexakopter]] ([[Benutzer Diskussion:Hexakopter|Diskussion]]) 00:17, 2. Okt. 2020 (CEST)
:: Zitat: "Es gibt keine Aussagen, auf die die Sätze zurückgeführt werden könnten.". Das sehe ich irgendwie anders. Nehmen wir mal das Leermengenaxiom. Warum nicht <math> \emptyset </math> definieren als <math> \forall A : A \notin \emptyset </math>, dann beweisen, dass diese Definition konsistent ist, und schon hat man bewiesen, dass <math> \exists B \forall A : A \notin B </math>, hat also das Leermengenaxiom, als Aussagesatz aufgefasst, bewiesen. --[[Benutzer:Thomas Limberg 1986|Thomas Limberg 1986]] ([[Benutzer Diskussion:Thomas Limberg 1986|Diskussion]]) 17:47, 2. Mär. 2017 (CET)


::Nicht fragen, sondern einfach „Has“ aussprechen (also nicht „Hass“ und „Haas“) und warten bis sie mal den Namen selbst sagt. Es darf auch noch ein paar Geheimnisse geben. Oder anrufen und anhören, dann aufhängen. :-) --[[Benutzer:&#61;|&#61;]] ([[Benutzer Diskussion:&#61;|Diskussion]]) 01:30, 2. Okt. 2020 (CEST)
:::Wieso soll das ein Existenzbeweis für die leere Menge sein? --[[Benutzer:Digamma|Digamma]] ([[Benutzer Diskussion:Digamma|Diskussion]]) 17:54, 2. Mär. 2017 (CET)


::::Na, weil mit <math> \emptyset </math> gibt es ein B, für das gilt <math> \forall A : A \notin B </math>.--[[Benutzer:Thomas Limberg 1986|Thomas Limberg 1986]] ([[Benutzer Diskussion:Thomas Limberg 1986|Diskussion]]) 18:00, 2. Mär. 2017 (CET)
== Art der Überwachung laut Schweizer Gesetz ==


:::Dafür brauchst du erst einmal die Möglichkeit „Objekte zu definieren“. Das kannst du mit einer [[Klassenlogik]]. Wenn du da aber einfach beliebige Definitionen zulässt, bekommst du die [[Russel’sche Antinomie]]. Sich schlichtweg auf konsistente Definitionen einschränken geht nicht, weil es im Allgemeinen unentscheidbar ist, ob eine Definition konsistent ist. Also muss man schon angeben, was für Klassendefinitionen man zulassen möchte – und landet damit bei etwas ähnlichem wie den Axiomen von ZFC. --[[Benutzer:Chricho|Chricho]] [[BD:Chricho|¹]] [//de.wikipedia.org/w/?title=BD:Chricho&amp;action=edit&amp;section=new ²] [[Benutzer:Chricho/Keine_Verbesserung|³]] 18:06, 2. Mär. 2017 (CET)
Mir ist nicht ganz klar ob es sich beim folgenden Fall um eine angepasste Überwachung, Echtzeitüberwachung, rückwirkende Überwachung oder fortlaufende Überwachung handelt: ''Eine Verkehrsdatenerhebung sämtlicher Nutzerinnen und Nutzer, welche keinen Tatverdacht bedingt. Also eine flächendeckende Speicherung bestimmter Kommunikationsdaten eines Nutzers, welche zu einem späteren Zeitpunkt eingefordert werden können.'' Ich denke aber eher, dass es sich um eine Echtzeitüberwachung oder fortlaufende Überwahcung handelt, da sämtliche Daten erfasst werden. Ich könnte da aber auch falsch liegen. Und wie sieht es das Schweizer Gesetz?--[[Benutzer:Specialities57|Specialities57]] ([[Benutzer Diskussion:Specialities57|Diskussion]]) 20:16, 1. Okt. 2020 (CEST)


:::: Zitat: "...weil es im Allgemeinen unentscheidbar ist, ob eine Definition konsistent ist.". Dazu: Im konkreten Fall der Leermenge (und weiterer Axiome) geht das aber schon, würde ich mal behaupten. Also, warum es nicht tun? --[[Benutzer:Thomas Limberg 1986|Thomas Limberg 1986]] ([[Benutzer Diskussion:Thomas Limberg 1986|Diskussion]]) 18:12, 2. Mär. 2017 (CET)
== Windows-Blickpunkt ==


::::: Nein, auch im konkreten Fall geht es nicht. Denn dafür müsstest du wissen, dass die Regel „jede definierbare Menge existiert, für die diese Existenz konsistent ist“ selber konsistent ist (Konsistenz muss ja auch Konsistenz mit allen Regeln heißen). Noch die kleinste Existenzaussage hinge dann von der Konsistenz des gesamten Systems ab. --[[Benutzer:Chricho|Chricho]] [[BD:Chricho|¹]] [//de.wikipedia.org/w/?title=BD:Chricho&amp;action=edit&amp;section=new ²] [[Benutzer:Chricho/Keine_Verbesserung|³]] 18:28, 2. Mär. 2017 (CET)
Bei Windows 10 wird jeden Tag ein neues Bild als Startbildschirm dargestellt (Windows Blickpunkt). Wo kann ich herausfinden, welches Motiv gerade dargestellt wird? Aktuell handelt es sich um eine Straße, bei der sich eine Straße kreisförmig teilt und wieder zusammengeführt wird. Das ganze erfolgt auf einer Brücke über Wasser.--[[Benutzer:Salino01|Salino01]] ([[Benutzer Diskussion:Salino01|Diskussion]]) 21:20, 1. Okt. 2020 (CEST)
:Ich habe da einen Text "Gefällt Ihnen, was Sie sehen?". Wenn man da draufklickt, wird das Motiv in der Bing-Suche angezeigt. --[[Benutzer:Tsungam|Magnus]] [[BD:Tsungam|(Diskussion)]] 21:53, 1. Okt. 2020 (CEST)
::Habe gerade eine Anleitung gefunden: https://www.antwort.net/windows-blickpunkt-bilder-infos-zum-ort-fotograf-finden-so-gehts.html --[[Benutzer:Salino01|Salino01]] ([[Benutzer Diskussion:Salino01|Diskussion]]) 07:39, 2. Okt. 2020 (CEST)


:::::: Ich denke, an dieser Stelle macht es wenig Sinn, weiterzudiskutieren. Am besten, ich formuliere erst einmal meinen Konsistenzbeweis, und dann kannst du mir ja die genaue Stelle zeigen, an der eine Inkonsistenz (oder mangelnder Beweis der Konsistenz) vorliegt. Aber danke erst einmal für die Hinweise! --[[Benutzer:Thomas Limberg 1986|Thomas Limberg 1986]] ([[Benutzer Diskussion:Thomas Limberg 1986|Diskussion]]) 18:37, 2. Mär. 2017 (CET)
== Wovon hängt es ab, ob englische Oberhäupter deutsch oder englisch ausgesprochen werden? ==


::::::: Ein Hinweis: Es ist doch davon auszugehen (vllt. hängt das von formalen Details ab, aber so wie ich dein System verstehe, wird das passieren, wenn wir davon ausgehen, dass dein System keine offensichtliche Inkonsistenz hat und es hinreichend stark ist, um klassische Arithmetik etc. zu betreiben), dass die Situation eintritt, dass es zwei Definitionen für Mengen gibt, sodass die Existenz jeder einzelnen annehmbar ist, jedoch die Existenz beider gleichzeitig zu einem offensichtlichen Widerspruch führt. Ich denke etwa an die Mengen „alle x, sodass x sich selbst nicht enthält oder dein φ“ und „alle x, sodass x nicht sich selbst enthält oder nicht φ“, wobei φ eine Aussage ist, die sich in deinem System nicht beweisen und nicht widerlegen lässt. Solch eine gibt es nach dem Unvollständigkeitssatz, wenn dein System zumindest etwas Arithmetik kann. Unter der Annahme, dass dein System konsistent ist, wäre auch die Annahme der Existenz je einer dieser beiden Mengen konsistent mit deinem System. Dann müssten ja aber beide in deinem System existieren. Eine von den beiden würde dann aber zur Russel’schen Antinomie führen. Also haben wir einen Widerspruch. Das heißt, wenn dein System nicht widersprüchlich ist, muss es sehr schwach sein, Induktionsbeweise über arithmetische Aussagen mit + und · dürften außer in wenigen Fällen nicht möglich sein. --[[Benutzer:Chricho|Chricho]] [[BD:Chricho|¹]] [//de.wikipedia.org/w/?title=BD:Chricho&amp;action=edit&amp;section=new ²] [[Benutzer:Chricho/Keine_Verbesserung|³]] 18:58, 2. Mär. 2017 (CET)
[[Heinrich VIII. (England)|Heinrich VIII]], [[Georg_VI._(Vereinigtes_Königreich)|Georg VI]]--[[Benutzer:Wikiseidank|Wikiseidank]] ([[Benutzer Diskussion:Wikiseidank|Diskussion]]) 21:33, 1. Okt. 2020 (CEST)
:Von der persönlichen Vorliebe der Sprecher. Man darf beides. Heute ist die englische Aussprache wohl häufiger als früher. --[[Benutzer:Dioskorides|Dioskorides]] ([[Benutzer Diskussion:Dioskorides|Diskussion]]) 21:45, 1. Okt. 2020 (CEST)
::Letzthin höre ich auch immer häufiger anmutige Mischformen wie "Dschordsch der Sechste". <small>Mich graust's dabei, aber andere scheinen es als Zeichen besonderer Weltläufigkeit zu sehen.</small> --[[Spezial:Beiträge/2A02:908:2D12:8BC0:74E8:380B:F9EB:FC96|2A02:908:2D12:8BC0:74E8:380B:F9EB:FC96]] 22:20, 1. Okt. 2020 (CEST)
:::Man könnte höchstens fragen, wieso das gerade bei englischen Monarchen so zweigleisig läuft. Ludwig XIV. oder Peter d.Gr. werden im dt. Sprachraum nur äußerst selten in ihrer Landessprache ausgesprochen. --[[Benutzer:Dioskorides|Dioskorides]] ([[Benutzer Diskussion:Dioskorides|Diskussion]]) 23:58, 1. Okt. 2020 (CEST)


Nur noch ein Hinweis: Deine Regel „jede definierbare Menge existiert, für die diese Existenz konsistent ist“ ist selbst auch ein Axiom/[[Axiomenschema]], das man annehmen kann oder eben auch nicht, aber nicht beweisen (wenn man denn nicht andere Axiome hat). Dieses Axiom anzunehmen hat den Nachteil, dass es sehr kompliziert ist (viel komplizierter als etwa das Leermengenaxiom). Auch den Satz des Pythagoras beweist man ausgehend von Axiomen. --[[Benutzer:Chricho|Chricho]] [[BD:Chricho|¹]] [//de.wikipedia.org/w/?title=BD:Chricho&amp;action=edit&amp;section=new ²] [[Benutzer:Chricho/Keine_Verbesserung|³]] 19:03, 2. Mär. 2017 (CET)
:Wobei die britischen Georgs ja ursprünglich Deutsche aus Hannover und Sachsen-Coburg Gotha waren (die ersten sprachen kaum englisch).
:Sonst mag es historische Gründe geben. Früher hat man Namen meistens übersetzt. Hatten wir gerade schon mal: [[Wikipedia:Auskunft/Archiv/2020/Woche_38#Elizabeth_vs_Elisabeth|Elizabeth_vs_Elisabeth]]. --[[Benutzer:Optimum|Optimum]] ([[Benutzer Diskussion:Optimum|Diskussion]]) 00:25, 2. Okt. 2020 (CEST)


: Axiome sollten Sätze sein, die man logisch nicht weiter zerlegen kann. Den Satz des Pythagoras kann man weiter logisch zerlegen. Dies nennt man dann Beweis. Das Leermengenaxiom kann man meines Erachtens logisch zerlegen, weshalb ich es nicht als Axiom einführen würde, sondern als zu beweisenden Satz. --[[Benutzer:Thomas Limberg 1986|Thomas Limberg 1986]] ([[Benutzer Diskussion:Thomas Limberg 1986|Diskussion]]) 19:20, 2. Mär. 2017 (CET)
:Die persönliche Vorliebe der Sprecher kann auch von Darstellungsbedürfnissen beeinflusst sein. Trödelhändler und Auktionäre nutzen u.U. eigene Sprachstile um sich selbst oder Verkaufsobjekte zu qualifizieren.
:"Dschordsch der Sechste" ist auf diesem Bild. Das sagt eventuell der Trödler.
:"Lui Kattorzse", "Lui Känghse" oder "Lui Säshse" heisst es dann beim Auktionator für den Stil von franz. Möbeln ;-) LG --[[Spezial:Beiträge/80.187.106.248|80.187.106.248]] 07:19, 2. Okt. 2020 (CEST)


: Weiterhin, Zitat: "Deine Regel ... hat den Nachteil, dass es sehr kompliziert ist". Dazu: Ich finde nicht, dass es sehr kompliziert ist. Und es kann ja auch für den Beweis der anderen Axiome (als normale Sätze aufgefasst) (Paarmengen-, Vereinigungs-, Unendlichkeitsaxiom u.s.w.) verwendet werden. In Summe ist es also viel einfacher! --[[Benutzer:Thomas Limberg 1986|Thomas Limberg 1986]] ([[Benutzer Diskussion:Thomas Limberg 1986|Diskussion]]) 19:34, 2. Mär. 2017 (CET)
::in Wikipeda [https://de.wikipedia.org/w/index.php?title=Spezial:Suche&limit=500&offset=0&profile=default&search=Kunstlexikon+P.W.+Hartmann&advancedSearch-current={}&ns0=1 heftigst als Referenzwerk] genutzt: ''Das grosse Kunstlexikon von P.W. Hartmann''. Darin zur Benennung von Stilrichtungen die schönsten Kombinationen. z.B. [http://www.beyars.com/kunstlexikon/lexikon_9677.html „William and Mary Style] , englischer Kunststil des späten 17. Jahrhundert, benannt nach König Wilhelm III. von Oranien (Reg. 1689-1702) und seiner Gemahlin Maria, einer Tochter König Jakobs II.“ --[[Spezial:Beiträge/80.187.109.133|80.187.109.133]] 07:42, 2. Okt. 2020 (CEST)
Wenn mein Gedächtnis mich nicht trügt, dann wurde im ''Ebbinghaus'' Axiome als "implizite Definitionen" bezeichnet. Sprich: Nachdem Cantors Mengenbegriff gescheitert ist, sagt man "Eine Menge ist das, was den folgnden Axiomen entspricht:" Die Axiome sind sozusagen ein Kriterienkatalog für die "Objekte", über die man reden will. <small>(''nicht [[Hilfe:Signatur|signierter]] Beitrag von'' [[Spezial:Beiträge/188.101.64.217|188.101.64.217]] ([[Benutzer Diskussion:188.101.64.217|Diskussion]])<nowiki/> 21:55, 2. Mär. 2017 (CET))</small>


== Namenszusatz "von" ==
= 2. Oktober =


Gibt es einen Grund, wieso man den Namenszusatz "von" bei einigen Personen weglässt, wenn man nur deren Nachnamen erwähnt, bei anderen aber nicht? Wieso heisst es "[[Alec von Graffenried|Von Graffenried]] wuchs in Bern auf", aber "Wegen des breiten Spektrums seiner Fähigkeiten galt [[Albrecht von Haller|Haller]] als Universalgelehrter."?
== Tod eines Präsidenten in der Endphase des US-Wahlkampf ==
--[[Spezial:Beiträge/85.3.130.75|85.3.130.75]] 18:23, 2. Mär. 2017 (CET)
:{{FZW}}
:Vielleicht gibt es keinen Grund, vielleicht will mancheiner aber auch zwischen Namensbestandteil (Vorsatzwort) und [[Adelsprädikat]] unterscheiden. --[[Benutzer:Tsungam|Magnus]] [[BD:Tsungam|(Diskussion)]] 19:18, 2. Mär. 2017 (CET)
:: Es betrifft nicht nur die Wikipedia, die beiden Personen werden immer so genannt, siehe z.B. "[http://www.derbund.ch/bern/stadt/von-graffenried-gibt-einblick-in-spendenliste/story/11011815 Von Graffenried gibt Einblick in Spendenliste]" und "[http://www.derbund.ch/zeitungen/kultur/Hallers-ungeheurer-Wissensdrang/story/28791899 Hallers ungeheurer Wissensdrang]". --[[Spezial:Beiträge/2A02:1205:5038:24B0:514E:527D:E3A1:2E0C|2A02:1205:5038:24B0:514E:527D:E3A1:2E0C]] 21:59, 2. Mär. 2017 (CET)
:::Je „berühmter“ jemand ist, umso eher wird auf das „von“ verzichtet. Beispiel: Müller wurde kurz vor seinem Tode geadelt, also heißt die Straße später nach ihm todsicher „Von-Müller-Straße“; aber wer kennt eine „Von-Schiller“-oder „Von-Goethe“-Straße? --[[Benutzer:Dioskorides|Dioskorides]] ([[Benutzer Diskussion:Dioskorides|Diskussion]]) 00:25, 3. Mär. 2017 (CET)
:::: Danke! --[[Spezial:Beiträge/85.3.130.34|85.3.130.34]] 11:38, 3. Mär. 2017 (CET)


Bei heutigen Namensträgern handelt es sich ja lediglich um einen Namen. Da das "von" heute kein Adelsprädikat oder dergleich ist, kann man es auch nicht weglassen, denn Eigennamen sind unveränderlich. Das gilt für Buchtitel, Produktnamen, Nachnamen usw. usf. Da Eigennamen nicht den Rechtschreibregeln unterliegen, müsste man das "von" in einem heutigen Namen konsequent auch klein schreiben, so wie man ja auch "iPhone" nicht "Iphone" schreiben kann – auch am Satzanfang. Bei historischen Namen kann man anders argumentieren. Hier ist das "von" ein offizieller Titel, den man dann ggf. auch weglassen kann. "Humboldt unternahm umfangreiche Forschungsreisen". Damals war das "von" nicht Teil des Eigennamens "Humboldt", sondern wurde ihm vorangestellt. [[Spezial:Beiträge/62.44.134.189|62.44.134.189]] 06:34, 3. Mär. 2017 (CET)
Was würde rechtlich passieren, bzw. ist rechtlich vorgeschrieben wenn ein US-Präsidenten Kandidat während der Endphase des Wahlkampfes stirbt? --[[Spezial:Beiträge/2A01:598:808A:B763:B58E:4EE1:DF06:8DDC|2A01:598:808A:B763:B58E:4EE1:DF06:8DDC]] 07:30, 2. Okt. 2020 (CEST)
: Vielen Dank für die ausführliche Erklärung! Müsste man von Graffenried folglich z.B. in einem Quellenverzeichnis unter V aufführen, d.h. ''von Graffenried, Alec'' anstatt ''Graffenried, Alec von''? --[[Spezial:Beiträge/85.3.130.34|85.3.130.34]] 11:38, 3. Mär. 2017 (CET)
: gibt es dafür rechtliche Regelungen? Die wahlunterlagen sind ja jetzt fest... rückt dann einfach Pence nach? Wenn er gewählt wurde ja, was ist aber im Wahlkampf? [[Spezial:Beiträge/2A01:598:808A:B763:B58E:4EE1:DF06:8DDC|2A01:598:808A:B763:B58E:4EE1:DF06:8DDC]] 07:45, 2. Okt. 2020 (CEST)
::Es gibt für die Sortierung DIN-Normen und wahrscheinlich Schweizer Normen, an die sich große Sortierbetriebe wie Telefonbuchverlage oder Bibliotheken halten. Nach dem Vorbild von {{HLS|25954|Graffenried, Christoph von|Autor= Karin Marti-Weissenbach}} kämen die Graffenrieds vielleicht unter G. --[[Benutzer:Pp.paul.4|Pp.paul.4]] ([[Benutzer Diskussion:Pp.paul.4|Diskussion]]) 16:30, 3. Mär. 2017 (CET)
::Wenn der nominierte Kandidat vor dem Zusammentritt des Electoral College stirbt, stellt die jeweilige Partei einen Ersatzkandidaten auf. Man geht davon aus, dass es der Kandidat für die Vizepräsidentschaft ist, aber das ist nicht genau vorgeschrieben. Die Wahlmänner/-frauen sollen dann für diesen Kandidaten abstimmen, aber auch dazu gibt es je nach Staat unterschiedliche Regelungen. Ist der Kandidat gewählt (vom Electoral College) und vom Kongress bestätigt, treten die bekannten Erbfolgeregelungen in Kraft. Richtig spannend wird es, wenn der Kandidat stirbt, nachdem er gewählt ist, aber noch vor der Bestätigung. Was dann passiert, weiß so recht keiner. [[Benutzer:Dumbox|Dumbox]] ([[Benutzer Diskussion:Dumbox|Diskussion]]) 07:54, 2. Okt. 2020 (CEST)
:::Es gibt da einen Roman von Tom Clancy, in dem der Superduperadmirals-<s>Vize</s>Präsident (sieht im Film aus wie [[Samuel L. Jackson]] kurz (6 Tage) vor der Wahl ermordet wird und deshalb ein vaterlandsverräterischer Feigling (aka Demokrat) Präsident wird - nur so kann ein echter Republikaner verlieren. Aber so richtig genau wird Clancy dabei nicht. Н[[Benutzer:Syrcro/Automatik|а]]ктаффэ 08:56, 2. Okt. 2020 (CEST)
:::::<small>@[[Benutzer:Syrcro]]: Ja, Clancy sollte politisch zu den republikanischen Falken gezählt werden. Dennoch muss im Hinblick auf dein Framing trotzdem gesagt werden, dass die Figur ''„Robby Jackson“'', auf die Du anspielst, als Afro-Amerikaner angelegt und von einem White-Power-Rassisten erschossen wurde. Der im Roman dann eingesetzte Präsident, ''„Edward Kealty“'', ist ansonsten im [[:en:Ryanverse]] ein Typ à la Trump im Hinblick auf „Grab 'em by the pussy“. Er wurde ansonsten als Vizepräsident eines mutmaßlichen Republikaners, ''„Roger Durling“'', in [[Ehrenschuld]] eingeführt. Grüße, [[Benutzer:Grand-Duc|Grand-Duc]]<small> ist kein Großherzog</small> ([[Benutzer Diskussion:Grand-Duc|Diskussion]]) 10:41, 2. Okt. 2020 (CEST)</small>
::::::Dass Samuel L. Jackson selten alte weiße Männer spielt, habe ich als Wissen bei der Mitlesern vorausgesetzt. Н[[Benutzer:Syrcro/Automatik|а]]ктаффэ 12:11, 2. Okt. 2020 (CEST)


== Lateiner gesucht (mal wieder...) ==
it will instantly disappear and be replaced by something even more bizarre and inexplicable. - There is another theory which states that this has already happened. --[[Spezial:Beiträge/77.6.54.254|77.6.54.254]] 07:59, 2. Okt. 2020 (CEST)
: Trump wird möglicherweise darauf bestehen, dass die Wahl bis nach seiner Wiederauferstehung verschoben wird. Als US-Gott und Erlöser steht ihm das zu. Seine honorige Familie vertritt ihn so lange kommissarisch. Möglich und etwas irdischer wäre aber eine koreanische Lösung, wie beim ewigen Oberhäuptling Kim-Il-sung. Als ewiger Präsident sieht er sich jetzt schon.--[[Benutzer:Gadacz|Klaus-Peter ''<small>(<sup>auf</sup><small>und</small><sub>davon</sub>)</small>'']] 08:56, 2. Okt. 2020 (CEST)
::Auch wenn ich mir nichts sehnlicher wünsche als dass Trump aus dem Amt gejagt und für Straftaten zur Rechenschaft gezogen wird - gesundheitlich wünsche ich ihm alles Gute, die "[[Thomas Klestil|österreichische Lösung]]" muss nicht sein. Zumal die Frage ist, welche Auswirkungen Mitleids- und Sympathieffekte auf das Wählerverhalten haben, was eine gesellschaftlich tragische Ironie wäre bei einem Präsidenten, dessen fehlgeleitete Covid-19-Politik ihm selbst zum Verhängnis werden könnte. -- [[Spezial:Beiträge/79.91.113.116|79.91.113.116]] 09:40, 2. Okt. 2020 (CEST)
::[[Benutzer:Gadacz|Klaus-Peter]], wo kann man das denn nachlesen, dass das, was du da beschreibst, überhaupt möglich ist oder gar rechtlich so vorgeschrieben ist? Genau das ist nämlich gefragt worden. Hast du oben das Intro gelesen? "Du weißt die Antwort oder kennst wenigstens Hinweise darauf? Dann antworte so kurz wie möglich, so lang wie nötig, mit Links auf Wikipedia-Artikel oder andere Quellen, die zum Verständnis beitragen." Oben kannst du auch lesen, dass hier nicht die Ausbreitung von Meinungen oder eigenen Theorien das Ziel ist, sondern die Verbreitung von belegbarem Wissen. --[[Spezial:Beiträge/77.8.119.94|77.8.119.94]] 10:28, 2. Okt. 2020 (CEST)
{{Erledigt|1=Frage beantwortet, Spekulatius bitte woanders. --[[Spezial:Beiträge/77.8.119.94|77.8.119.94]] 10:28, 2. Okt. 2020 (CEST)}}


Hallo, ich bitte um sinngemäße Übersetzung der folgenden Grabinschrift (Hinweis: es geht um [[Avo (Corvey)]]):
== Frage zum Thema Schiefer und Sedimentgestein ==
{{Zitat|Hoc rogo quisquis adis limina sacra,<br />
Quem calcas pedibus noscito quis fim.<br />
Abbas Avo sui nomine dictus:<br />
Nunc cinis, in cinerem hic resolutus<br />
Tempus judicii specto supremi:<br />
Hinc Christi domini te per amorem<br />
Pro me, quaeso, preces fundere crebras<br />
Ob hoc me petii hic sepeliri,<br />
Quinto Idus abii ipso Novembris.}}
Danke. Textquelle, falls ich mich verguckt/verschrieben habe: {{Google Buch|BuchID=FWiRKn8rQ9cC|Seite=PA96|Linktext=Volltext}} --[[Benutzer:Tsungam|Magnus]] [[BD:Tsungam|(Diskussion)]] 19:08, 2. Mär. 2017 (CET)
: OT? Jedenfalls Info auf die Schnelle: mit dem Topos "Asche zu Asche" (nunc cinis...) auch die ''Epitaphia Corbeiensia II'', in: ''MGH Poetae 4'', hg. v. K. Strecker, S. 1041,3., von [http://www.inschriften.net/essen-stadt/inschrift/nr/di081-0025.html#content hier]. [https://books.google.de/books?hl=de&id=IqI-AAAAYAAJ|Digitalisat zum Proxystöbern], weiß nicht, ob sie auf Übersetzungen hinweisen. --[[Benutzer:Aalfons|Aalfons]] ([[Benutzer Diskussion:Aalfons|Diskussion]]) 19:21, 2. Mär. 2017 (CET)

:"Darum bitte ich, wer immer du bist, der an die heiligen Schwellen tritt/, du sollst erkennen, wer ich bin (''s''im), auf den du mit deinen Füßen trittst./ Ich war (''f''ui) der Abt, Avo genannt:/ Jetzt bin ich Asche, und hier in Asche verfallen erwarte ich die Zeit des höchsten Gerichts:/ Drum bitte ich dreifach (i. S. v. sehr?), um Christi des Herrn Liebe willen,/ für mich häufige Gebete zu verrichten/Deswegen bat ich darum, hier begraben zu werden./ Ich starb am 9. November." Grüße [[Benutzer:Dumbox|Dumbox]] ([[Benutzer Diskussion:Dumbox|Diskussion]]) 19:44, 2. Mär. 2017 (CET)
::Danke sehr! --[[Benutzer:Tsungam|Magnus]] [[BD:Tsungam|(Diskussion)]] 19:48, 2. Mär. 2017 (CET)

:::{{ping|Tsungam|Aalfons}} {{MGH|Poetae|4,2.3|1041|1042}}. --[[Benutzer:HHill|HHill]] ([[Benutzer Diskussion:HHill|Diskussion]]) 22:21, 2. Mär. 2017 (CET)
::::Ah prima, das bietet zwei bessere Lesarten: ''Haec'' am Anfang könnte sich auf ''limina sacra'' beziehen: "an ''diese'' heiligen Schwellen"; und das doofe ''ter'' wird zum schlichten ''te'': "Ich bitte dich häufige Gebete zu verrichten". Grüße [[Benutzer:Dumbox|Dumbox]] ([[Benutzer Diskussion:Dumbox|Diskussion]]) 22:27, 2. Mär. 2017 (CET)

:::::Wo hast du "ter" gelesen? --[[Benutzer:Digamma|Digamma]] ([[Benutzer Diskussion:Digamma|Diskussion]]) 22:33, 2. Mär. 2017 (CET)
::::::In der verlinkten Textvorlage des OP. Grüße [[Benutzer:Dumbox|Dumbox]] ([[Benutzer Diskussion:Dumbox|Diskussion]]) 22:34, 2. Mär. 2017 (CET)
:::::::Danke. Ich hatte nur den getippten Text gesehen. --[[Benutzer:Digamma|Digamma]] ([[Benutzer Diskussion:Digamma|Diskussion]]) 22:36, 2. Mär. 2017 (CET)
::::::::Die MGH bieten den emendierten Text, sagen in den Anmerkungen noch manches darüber, vermerken dort die Lesarten und verweisen auf die [http://digital.ub.uni-paderborn.de/ihd/content/pageview/1740521 Nachzeichnung der Inschrift] und die Möglichkeit der nachmittelalterlichen Entstehung. --[[Benutzer:Pp.paul.4|Pp.paul.4]] ([[Benutzer Diskussion:Pp.paul.4|Diskussion]]) 08:05, 3. Mär. 2017 (CET)
::Es sei noch angemerkt, dass der Text öfters benutzt wurde. Im [[Kloster Liesborn]] bei Münster erhielt Abt [[Heinrich von Kleve]] 1490, also 600 Jahre nach Avo, [https://books.google.de/books?id=TQE27ANVWSEC&pg=PA245 eine fast identische Grabinschrift.] Da steckt also noch eine eigene Geschichte hinter. --[[Benutzer:Aalfons|Aalfons]] ([[Benutzer Diskussion:Aalfons|Diskussion]]) 14:04, 3. Mär. 2017 (CET)
:::Den verlinkten Band gibt es übrigens auch direkt bei der [[Germania Sacra]] [http://personendatenbank.germania-sacra.de/books/view/33/259 online]. --[[Benutzer:HHill|HHill]] ([[Benutzer Diskussion:HHill|Diskussion]]) 14:56, 3. Mär. 2017 (CET)
::::Zur Frage nach dem Alter der Avo-Inschrift äußern sich die MGH ja verhalten. Mit den Buchstabenformen der oben verlinkten Nachzeichnung und dem verlinkten Textfund von 1490 halte ich die Avo-Inschrift für frühneuzeitlich, keinesfalls für karolingisch. Ob Gelehrtenfleiß (Matthias Borbonius widmete 1595 jedem Kaiser ein Gedicht [https://books.google.de/books?id=ZFBTAAAAcAAJ&dq=inauthor%3A%22Matthias%20Borbonius%22&hl=de&pg=PT3#v=onepage&q&f=false], warum sollte nicht ein Gelehrter in gleicher Art jedem Abt ein Gedicht widmen) oder weniger fromme Gelehrtenfälschung dahintersteckt? --[[Benutzer:Pp.paul.4|Pp.paul.4]] ([[Benutzer Diskussion:Pp.paul.4|Diskussion]]) 16:44, 3. Mär. 2017 (CET)
::::: Und [https://books.google.de/books?id=XnIAAAAAcAAJ&pg=PA501 hier] für Nichtmitglieder des MGH-Dechiffriersyndikats das Überlieferungsproblem mit dem Avo-Epitaph auf deutsch. Rein nach Beleg (1587) ist die Liesborner Inschrift die nachgewiesen ältere. --[[Benutzer:Aalfons|Aalfons]] ([[Benutzer Diskussion:Aalfons|Diskussion]]) 17:43, 3. Mär. 2017 (CET)

== Resteverwertung Entenbrust ==

Wie kann man die verbliebene Hälfte einer übriggebliebenen, rosarot gelungenen Entenbrust am nächsten Tag noch anders verwerten als sie scheibchenweise aufs Butterbrot zu legen?
<!-- Lass die nachfolgende Zeile am ENDE deiner Frage stehen. Sie wird in deine Signatur umgewandelt. -->
--[[Benutzer:BN-PG 2263|BN-PG 2263]] ([[Benutzer Diskussion:BN-PG 2263|Diskussion]]) 20:02, 2. Mär. 2017 (CET)
:Klassisch: Das gute, alte Resteragout? Hoffentlich schimpft Oliver S. Y. nicht... ;) [[Benutzer:Dumbox|Dumbox]] ([[Benutzer Diskussion:Dumbox|Diskussion]]) 20:07, 2. Mär. 2017 (CET)
::Danke. Schon mal eine gute Idee, mit Reis, Möhrchen und einer mit einem Müller-Th. abgelöschten und mit Eigelb abgebundenen Soße, fiele mir da so ein. Aber vielleicht hat [[Benutzer:Oliver S.Y.|Oliver S.Y.]] eine noch bessere Idee? -- [[Benutzer:BN-PG 2263|BN-PG 2263]] ([[Benutzer Diskussion:BN-PG 2263|Diskussion]]) 20:20, 2. Mär. 2017 (CET)
::BK. <small>DAFÜR wird mich Oliver wohl sicher schimpfen ;-) ...</small> Schlicht und einfach scheibchenweise in der Mikrowelle aufwärmen? Ente wird nach meiner Erfahrung dabei nicht besonders zäh, wenn man ein klein wenig Flüssigkeit drübergibt und nicht mit voller Leistung wärmt, sondern etwas langsamer. --[[Benutzer:Apierta]] 20:22, 2. Mär. 2017 (CET)

Eine schöne kräftige Gemüsesuppe mit dem Fleisch anreichern? Für ein [[Rillettes]] dürfte die Menge zu klein sein. --[[Benutzer:Elrond|Elrond]] ([[Benutzer Diskussion:Elrond|Diskussion]]) 20:59, 2. Mär. 2017 (CET)

[[Datei:Entenverwertung2.jpg|mini|Nochmals vielen Dank an alle, die einen Radschlag für mich hatten. Das Bild zeigt, wie die Geschmacksfrage letztlich beantwortet wurde. Nach der Entdeckung einer Rotkohlkonserve knapp jenseits des Haltbarkeitsdatums im Vorratsschrank lag die Kombination mit Bratkartoffeln nahe. (Rotkohl kam ganz zum Schluss nur für's Foto in die Pfanne.) -- [[Benutzer:BN-PG 2263|BN-PG 2263]] ([[Benutzer Diskussion:BN-PG 2263|Diskussion]]) 19:50, 3. Mär. 2017 (CET)]]
Hallo! Ne, wenn man sowas daheim macht schimpfe ich sicher nicht, nur bei solchem Pfusch in der Gastronomie oder Kochbüchern. Meistens ist dieses "rosarot" ja klar denaturiert, weshalb ein erneutes Garen nicht nötig ist. Wenn es um Kochtips geht, wie wäre es mal auf nem Enten-Club-Sandwich statt Hühnerbrust. Dafür nur der Länge nach halbieren oder dritteln, nicht in Scheiben schneiden, ist schließlich echtes Fleisch. Womit man beim nächsten typischen Gericht ist, Geflügelcocktail, Fleisch fein würfeln, und mit Gemüse und Dressing anmachen, kurz marinieren, aufs Brot oder als Vorspeise lecker. Dem Wetter entsprechend kann man dafür auch gut die ersten Frühlingskräuer nehmen, und statt Mayo eine Vinaigrette, Leute wie ich mögen Ente mit Senf ganz gern. Wenn es warm sein soll, dann lege man die Scheiben auf eine Pizza, nachdem sie aus dem Ofen kam. Die Restwärme reicht völlig. Und wenn Mikrowelle, bitte einfach vorher etwas Flüssigkeit durch eine Marinade oder einem leichten gewürzten Rotwein zugeben.[[Benutzer:Oliver S.Y.|Oliver S.Y.]] ([[Benutzer Diskussion:Oliver S.Y.|Diskussion]]) 21:01, 2. Mär. 2017 (CET)
:Dünne Scheiben ganz kurz in heißer Butter erwärmen, dazu béarnaise. --[[Benutzer:Ralf Roletschek|M@rcela]] [[Bild:Miniauge2.gif|27px]] 21:11, 2. Mär. 2017 (CET)
::... womit die Entenbrust dann zum bloßen Butter- und Saucenträger degradiert worden wäre. Dann besser Kartoffeln in Butter ertränken und die Ente stattdessen der Katze überlassen. Scnr, --[[Benutzer_Diskussion:Zxmt|Zxmt]] <sup>[[Benutzer:Zxmt|Nutze Dein Stimmrecht!]]</sup> 20:00, 3. Mär. 2017 (CET)

== Hat Berlin eigentlich einen mittelalterlichen Stadtkern? ==

Im Vergleich zu München ist Berlin ja rein architektonisch betrachtet stocklangweilig. München bietet im Zentrum südländisches "Theater", während Berlin ein deutsches New York sein möchte, es aber nicht schafft. Irgendwie tragisch. Breite Straßen, da ein bisschen Klassizismus, da und dort ein paar moderne Bauten, bunt durcheinander gewürfelt, wie das in einem Stadtmilieu ist, das - schon rein historisch betrachtet - beziehungslos und bindungslos ist. Ein wirkliches Ensemble existiert nicht. Meine Frage: Gibt es eigentlich ein "altes" Berlin? Und wo ist das Zentrum? Am Gendarmenmarkt (???).

<!-- Lass die nachfolgende Zeile am ENDE deiner Frage stehen. Sie wird in deine Signatur umgewandelt. -->
--[[Spezial:Beiträge/217.238.153.230|217.238.153.230]] 20:21, 2. Mär. 2017 (CET)
:Ich glaube, [[Alt-Berlin]] beantwortet Deine Frage(n). Gruß, --[[Benutzer:Apierta]] 20:36, 2. Mär. 2017 (CET)
:(BK)[[Alt-Berlin]] und [[Cölln]] sind der eigentlich Siedlungskern der Stadt Berlin. Einfach mal [[Geschichte Berlins]] lesen. --[[Benutzer:Bobo11|Bobo11]] ([[Benutzer Diskussion:Bobo11|Diskussion]]) 20:38, 2. Mär. 2017 (CET)
::"mittellterlicher Stadtkern" Nicht wirklich. Berlin ist in dem Sinne keine "logisch gewachsene" Stadt, sondern eine Art "Sammlung". Zudem steht Berlin für den permanenten Wandel, Beispiel: Bei U-Bahnarbeiten vor dem [[Berliner Rathaus]] fand man unterirdisch die Reste eines alten Rathauses an fast gleicher Stelle. Berlin baute schon immer neues auf den Ruinen des alten - Schloss/Palast der Republik/Humboldtforum. Der Denkmalschutz versucht sein bestes, aber ein zusammengehöriges Stadtensemble war zu keiner Zeit erhaltbar. Wenn du durchs [[Nikolaiviertel]] gehst, vermutest du einen historischen Stadtkern. Allerdings ist es komplett zu DDR-Zeiten aufgebaut worden, da stand nur noch eine Ruine der Nikolaikirche. "Spießige Altstadthistorie" wirst du in Berlin nicht finden, auch wenn es Ansätze dafür in einigen Randbezirken gibt.--[[Benutzer:Wikiseidank|Wikiseidank]] ([[Benutzer Diskussion:Wikiseidank|Diskussion]]) 20:44, 2. Mär. 2017 (CET)

Also Berlin hat von 1848 bis 1914 und 1918 bis 1939 jeweils genausoviele Gebäude im Zentrum abgerissen, wie es dann im Krieg zerstört wurde. Der anschließende Wiederaufbau in Ost- und Westberlin taten ein Übrigens. ABER, es wurde kein mittelalterlicher Stadtkern zerstört, sondern Gebäude die meist erst am Ende des 17.Jahrhunderts entstanden. Bin mir nicht so sicher, ob Münchens Stadtkern wirklich so zutreffend ist. Erhaltende Stadtkerne und deren Strukturen findest Du eher in den beiden anderen historischen Städten Spandau und Köpenick, die im Mittelalter eine ebenso große Bedeutung hatten. Ansonsten will Berlin gar kein deutsches New York sein, das wollen nur paar Zugezogene Gernegrößen, egal ob sie nun aus Sachsen, Saarland, Hamburg oder aktuell der Schweiz kommen. Die haben feuchte Träume, weil sie denken, Investoren für ihre Legopläne zu finden. Berlin ist maßgeblich durch Schinkel und seine Schüler geprägt worden, genauso wie das Stadtbild bereits Mitte des 19.Jh. festgelegt wurde. Wenn Du Milleu willst, die Spandauer Vorstadt ist immer noch ziemlich authentisch, und mit Erklärungen die Stadtgeschichte nachzuvollziehen. Wenn man heute ein "Zentrum" definieren will, ist es das Dreieck Potsdamer Platz/Leipziger Platz/Pariser Platz. Das andere ist die City West und der Alexanderplatz (auch wenn er es nicht ist^^). Darum hat sich in Berlin ja gerade dieses Kiezgefühl herausgebildet, weil diese Straßenzüge Insellagen bildeten, wo die Bevölkerung einer deutschen Kleinstadt über- und nebeneinander wohnte. Einige von diesen Vierteln sind darum auch bis heute noch so empfindbar. Aber auch die findet man nur mit Führungen von Eingeweihten, nicht durch die Standardrouten von Reiseführern.[[Benutzer:Oliver S.Y.|Oliver S.Y.]] ([[Benutzer Diskussion:Oliver S.Y.|Diskussion]]) 20:54, 2. Mär. 2017 (CET)
:Es kommt hinzu, daß das Mittelalter schon fast vorüber war, als Berlin eine nennenswerte Stadt wurde. --[[Benutzer:Ralf Roletschek|M@rcela]] [[Bild:Miniauge2.gif|27px]] 21:40, 2. Mär. 2017 (CET)
:: Ist bei München übrigens auch nicht anders. Im Mittelalter waren in Bayern andere Städte vorne, Regensburg bspw. --[[Benutzer:Gretarsson|Gretarsson]] ([[Benutzer Diskussion:Gretarsson|Diskussion]]) 01:01, 3. Mär. 2017 (CET)

:Mittelalter ist das nicht mehr, aber Stadtmauern / Stadtkern sind da abzulesen. [[File:ZLB-Berliner Ansichten-Januar.jpg|thumb|Map of Berlin in 1688]] --[[Benutzer:Cookatoo.ergo.ZooM|Cookatoo.ergo.ZooM]] ([[Benutzer Diskussion:Cookatoo.ergo.ZooM|Diskussion]]) 22:24, 2. Mär. 2017 (CET)

Einen schicken mittelalterlichen Stadtkern - sollte der Frager '''darauf''' aus sein - gibt es glaube ich oft in kleine(re)n Städten, so in Memmingen... --[[Benutzer:ObersterGenosse|ObersterGenosse]] ([[Benutzer Diskussion:ObersterGenosse|Diskussion]]) 23:04, 2. Mär. 2017 (CET)
:„Mittelalterlich“ war nur der Stadtkern im ummauerten Bereich auf der nebenstehenden Karte. Die eigentlich mittelalterliche Struktur ist dabei höchstens das Straßennetz, selbst die Mauer erst 17./18. Jh. Berlin entwickelte sich eigentlich erst ab dem beginnenden 18 Jh.: Dorotheenstadt, Friedrichstadt etc. Man sollte sich auch nicht täuschen: auch in den „mittelalterlich“ wirkenden Städten sind außer den Kirchen und wenigen Profanbauten die meisten anderen alten Gebäude erst ab dem 16.Jh. entstanden, als das MA schon vorbei war. --[[Benutzer:Dioskorides|Dioskorides]] ([[Benutzer Diskussion:Dioskorides|Diskussion]]) 00:20, 3. Mär. 2017 (CET)

Ach, die euroskeptische, „reaktionär-religiotische“ (nicht von mir) 217.238er IP aus Oberbayern (Tacherting) mal wieder, mit ihrem Markenzeichen: Ganz viel Meinung und ganz wenig Ahnung. Warum wusste ich das schon, bevor ich den OP zuende gelesen hatte? (Im Café übrigens ebenfalls mit einem [https://de.wikipedia.org/wiki/Wikipedia:Caf%C3%A9#Warum_sieht_Bayern_so_.22geschleckt.22_und_ordentlich_aus Ich-hol-mir-einen-auf-Bayern-runter-Thread] am Start)... --[[Benutzer:Gretarsson|Gretarsson]] ([[Benutzer Diskussion:Gretarsson|Diskussion]]) 01:01, 3. Mär. 2017 (CET)
:{{Antwort|Dioskorides}} Dazu kommt, dass [[Sichtfachwerk]], was viele mit mittelalterlichen Stadtkernen assoziieren, auch lange nach Ende des Mittelalters gebaut wurde. Bei mir im Wohnort gibt es ein echtes Sichtfachwerkhaus, was erst nach 2012 gebaut wurde. Die Schlossbergschule in [[Neuenbürg]] wurde in den 1950er-Jahren als verputzter Fachwerkbau neu gebaut. --[[user:Rotkaeppchen68|R<span style="color:red">ô</span>tkæppchen₆₈]] 01:16, 3. Mär. 2017 (CET)

Wobei ich die Frage der IP aber berechtigt und interessant finde. Wenn man sich die Stadtpläne anderer großer deutscher Städte, wie etwa Köln oder Frankfurt, anschaut, erkennt man ja die ursprünglichen Stadtkerne noch sehr deutlich, auch wenn der Bombenkrieg und der anschließende „zeitgemäße“ Wiederaufbau von der eigentlichen Bausubstanz wenig übriggelassen haben. In Berlin ist das nicht so. Warum? Eine besonders krasse Folge des sozialistischen Städtebaus in der Hauptstadt der DDR, der etwa die [[Marienkirche (Berlin-Mitte)|Marienkirche]] als einsame gotische Insel in einem Meer von Hochhäusern und Grünanlagen beließ? Oder sind die Gründe schon früher zu suchen, v.a. im 18. Jahrhundert, als die vorherige Residenz eines Provinzfürsten innerhalb weniger Generationen zur Hauptstadt eines neuen europäischen Königreiches aufstieg (siehe Artikel [[Einwohnerentwicklung von Berlin]]: 6.000 Einwohner im Jahr 1648, 20.000 im Jahr 1688, 55.000 im Jahr 1709 und über 100.000 im Jahr 1747) und man womöglich auch damals schon bestehende Stadtstrukturen beseitigte? --[[Benutzer:Steffen Löwe Gera|slg]] ([[Benutzer Diskussion:Steffen Löwe Gera|Diskussion]]) 02:49, 3. Mär. 2017 (CET)
:Die Berliner Innenstadt war 1945 total zerstört. Beim Wiederaufbau musste/wollte man im Osten wirklich nicht so große Rücksicht auf bestehende Grundstücksgrenzen nehmen wie im Westen, und deshalb war es wohl leichter, Veränderungen im Straßennetz vorzunehmen. Du hast Recht, in vielen Städten ist das Straßennetz das konservativste Strukturelement, nicht die Bausubstanz. --[[Benutzer:Dioskorides|Dioskorides]] ([[Benutzer Diskussion:Dioskorides|Diskussion]]) 06:27, 3. Mär. 2017 (CET)

Es gibt mit dem Nikolaiviertel in Berlin natürlich einen historischen Stadtkern. Später jedoch, zu Zeiten preußischer Größe, baute man sich gewissermaßen ein neues Zentrum. Alle vier preußischen Grundpfeiler waren vertreten: Die Kultur mit dem Alten Museum, das Zeughaus mit dem Militär, der Protestantismus mit dem Berliner Dom und das Schloss mit dem König. Heute wird das Schloss dort neu gebaut, allerdings missachtet man die Bedeutung des Platzes. Zum einen weil an dieser Stelle heute die Straße Unter den Linden viel befahrener ist und dadurch diese historische Stelle durchschneidet, zum anderen, weil das Schloss dort gebaut wurde und nicht etwa ein Symbol für die Demokratie wie etwa ein neues Parlament oder Kanzerlamt. Zum wird das Schloss eine Art Museum, was es auf der anderen Seite ja schon gibt. Eine vertane Chance jedenfalls, Berlin wieder einen passigen Stadtkern zu schenken. [[Spezial:Beiträge/62.44.134.189|62.44.134.189]] 06:29, 3. Mär. 2017 (CET)

::Sry, das ist Unfug! Das Nikolaiviertel war 1980 platt. Es wurde mit Plattenbauten und Einzelprojekten ohne historische Authenzität wieder errichtet. Ansonsten war es nur ein Teil des Stadtkern, nicht der Stadtkern. Dessen Ausmaße kann man wirklich sehr leicht durch die Spree, das [[Heilig-Geist-Spital (Berlin)]] am ehemals nördlichen Stadtrand, der S-Bahntrasse bzw. Dircksenstraße und der Ruine vom [[Graues Kloster (Berlin)|Grauen Kloster]] nachvollziehen. Die Fläche war also wesentlich größer. Und viele mittelalterliche Straßenzüge lassen sich ja bis heute auch noch nachvollziehen. Das alte Museum wird mitten in den Lustgarten des Schlosses auf die Insel platziert. Das hat nichts mit den westlichen Vorstädten aus dem 18. Jahrhundert zu tun. Genauso ist es eher Beweis für das neue Kulturverständnis von FW III gewesen, der seine kurz zuvor gekauften (nicht eroberten) Sammlungen unterbringen wollte. Außerdem krankt Deine These daran, daß zu diesem Ensemble die katholische Hedwigskathedrale gehört, denn das war der spezielle Kit Preußens, nicht der Protestantismus ansich, sondern die Gottefürchtigkeit und eine Form von bescheidenem "preußischen" Katholizismus, welcher schon wegen der Wiedererstehung im 18.Jahrhundert kaum etwas mit der barocken, ausladenen Form in Mittel- und Süddeutschland zu tun hatte. Das Schloss war mitten in der Stadt, der König sah über die Spree, und sah die Wäscherinnen Berlins. Nicht der Bezug zu den Linden ist das Problem, damals wie heute eigentlich nur eine Ausfallstraße ohne Nutzwert, sondern der Bezug zum Berliner Stadtkern. Einfach mal nachschauen, wo die preußische Geheimpolizeil und Hauptpost standen. Die Preußenkönige waren da mehr Berliner, als mancher heute glauben mag. [[Benutzer:Oliver S.Y.|Oliver S.Y.]] ([[Benutzer Diskussion:Oliver S.Y.|Diskussion]]) 10:26, 3. Mär. 2017 (CET)

:Als Vergleich: Vom mittelalterlichen Stadtkern ist in London oder Paris auch nicht viel mehr als der Tower oder die Kathedralen erhalten. In Paris hat man im 19. Jh ganz gezielt „aufgeräumt“, die alte Bausubstanz entsorgt und das Straßennetz neu gestaltet. --[[Spezial:Beiträge/84.135.140.238|84.135.140.238]] 07:48, 3. Mär. 2017 (CET)
::Städte des heutigen Deutschland waren im Mittelalter bis auf wenige Ausnahmen so klein, daß sie heute eher als Dorf gelten würden. Berlin hat erst im 17. Jahrhundert 5000 Einwohner erreicht. --[[Benutzer:Ralf Roletschek|M@rcela]] [[Bild:Miniauge2.gif|27px]] 10:35, 3. Mär. 2017 (CET)

:::Sry Ralf, auch das ist Quark, einfach in der Wikipedia nachzuvollziehen. Bereits um 1400 hatte Berlin 7000 Einwohner, 1576 (also im 16.Jahrhundert) wurden 11.000 erreicht. Damit lag man in Ostdeutschland vor Leipzig und Rostock, gleichauf mit Dresden, nur Magdeburg hatte da mehr als 20.000 EW (Grenze Kleinstadt zu Mittelstadt). In den anschließenden 65 Jahren mit dem 30 Jährigen Krieg ging diese Zahl wie in weiten Teilen Deutschlands auf 6000 zurück. Aber kurz vor dem Krieg 1613 wurden 12.000 erreicht. Magdeburg wurde fast ausgelöscht, 1639 gerade noch 450 EW, keine 2 Prozent.[[Benutzer:Oliver S.Y.|Oliver S.Y.]] ([[Benutzer Diskussion:Oliver S.Y.|Diskussion]]) 11:35, 3. Mär. 2017 (CET)

::::Meines Wissens gibt es aber in Deutschland keine Stadt, die in der Gründerzeit so »explodiert« ist wie Berlin. Wobei ganze Stadtteile quasi auf die grüne Wiese gesetzt wurden und dann erst zusammenwuchsen. Auf manchen Zille-Fotos kann man das erahnen. Darin könnte die seltsame Abwesenheit eines echten Zentrums begründet sein. Paris dagegen wuchs wesentlich langsamer und organischer durch Erweiterungen der Stadtmauern und schließlich Bebauung jenseits der geschleiften Mauern. Was wiederum dazu führte, dass die Infrastruktur schließlich radikal erneuert und erweitert werden musste. [[Benutzer:Rainer Zenz|Rainer Z]] [[Benutzer Diskussion:Rainer Zenz| ...]] 13:14, 3. Mär. 2017 (CET)

== Siliconallergie? ==

Ich hänge an Sauerstoff, die Zufuhrschläuche samt "Nasenbrille" sind aus Silicon. An den Berührungspunkten über den Ohren und im Nasenraum machen sie mich wund. Was gibt es für Alternativen? Das Sanitätshaus (Lieferant) erzählt mir nur blabla. Auf den Sauerstoff bin ich angewiesen. Gruß -- [[Benutzer:Dr.cueppers|Dr.cueppers]] - [[Benutzer Diskussion:Dr.cueppers|Disk.]] 20:36, 2. Mär. 2017 (CET)
:Könnte es etwas bringen, wenn man den Brillen- und Zuleitungssatz eines anderen Herstellers probiert? Oder an den Kontaktstellen mit einem anderen (Kunst)Stoff umwirken. Ev. ein Schrumpfschlauch, die gibt es auch aus Polyolefinen oder PTFA, wobei letzteres natürlich arg hart sein und mechanisch reizen kann. --[[Benutzer:Elrond|Elrond]] ([[Benutzer Diskussion:Elrond|Diskussion]]) 20:56, 2. Mär. 2017 (CET)
::Babypuder könnte helfen. Jedenfalls kann es nicht schaden. --[[Benutzer:Ralf Roletschek|M@rcela]] [[Bild:Miniauge2.gif|27px]] 21:42, 2. Mär. 2017 (CET)
:Es gibt außer aus Silikon auch Brillen aus PVC und auch welche aus Kraton. Kraton ist sehr weich und soll gut verträglich sein. Es beugt allergischen Reaktionen und Druckstellen vor. Sollten Sie auch eine Sehbrille tragen, ist die O2-Brille (Kickinger-Dirmeier-Brille) vielleicht einen Blick wert. Die Sauerstoffversorgung ist dabei in die normale Brille integriert. --[[Spezial:Beiträge/83.135.224.233|83.135.224.233]] 21:46, 2. Mär. 2017 (CET)
:: Danke für die Antworten: Sehbrille habe ich, brauche sie aber nur außerhalb der Wohnung - und die verlasse ich kaum noch. Nach PVC und Kraton werde ich fragen.Gruß -- [[Benutzer:Dr.cueppers|Dr.cueppers]] - [[Benutzer Diskussion:Dr.cueppers|Disk.]] 22:17, 2. Mär. 2017 (CET)
:::Es gibt auch welche aus Vinyl. Je nach dem kann es aber auch ein Hilfsstoffe sein, auf den du allergisch reagiert wie z.b. Desinfektionsmittel.--[[Benutzer:Bobo11|Bobo11]] ([[Benutzer Diskussion:Bobo11|Diskussion]]) 22:34, 2. Mär. 2017 (CET)
::::<small>PVC ist Vinyl. --[[user:Rotkaeppchen68|R<span style="color:red">ô</span>tkæppchen₆₈]] 22:45, 2. Mär. 2017 (CET)</small>
:::::Und PVC ist nicht besonders hautfreundlich, Polyvinyl'''chlorid''', siehe [[Polyvinylchlorid#Gesundheitliche_Gefahren]]. --[[Benutzer:Ralf Roletschek|M@rcela]] [[Bild:Miniauge2.gif|27px]] 22:58, 2. Mär. 2017 (CET)
:::::::Mit Verlaub, aber wenn es solche Bestecke aus PVC geben sollte, werden sie keine Phthalate enthalten sondern Adipate oder Sebacate. Die sind deutlich bedenkenärmer. Freies Vinylchlorid schließe ich mal aus. --[[Benutzer:Elrond|Elrond]] ([[Benutzer Diskussion:Elrond|Diskussion]]) 23:27, 2. Mär. 2017 (CET)
::::::Da hilft die Schläuche und die empfindlichen Stellen der Haut mit Bepanten einzufetten.--[[Spezial:Beiträge/2003:75:AF10:D00:D152:2BCA:EA18:39D9|2003:75:AF10:D00:D152:2BCA:EA18:39D9]] 23:11, 2. Mär. 2017 (CET)
::[[:en:Kraton (polymer)|Kraton]] ist eine Art [[Styrol-Butadien-Kautschuk]]. --[[user:Rotkaeppchen68|R<span style="color:red">ô</span>tkæppchen₆₈]] 23:21, 2. Mär. 2017 (CET)
:{{ping|Dr.cueppers}} Bist Du Dir sicher, dass das eine allergische Reaktion ist und nicht etwa nur eine mechanische Druckstelle? --[[user:Rotkaeppchen68|R<span style="color:red">ô</span>tkæppchen₆₈]] 23:27, 2. Mär. 2017 (CET)
::Darauf hätte ich auch getippt. Die "Verspannung" so justieren, dass es gerade genug Auflagedruck gibt. (Das gilt auch für Apnoe-Masken...) <small style="color:grey"><b>'''GEEZER'''</b></small><sup>[[BD:Grey Geezer|<span style="color:grey"> … nil nisi bene</span>]]</sup> 07:33, 3. Mär. 2017 (CET)

:Andere Maske probieren, Wattepads unterlegen... [http://www.paradisi.de/Health_und_Ernaehrung/Erkrankungen/Schlafapnoe/Forum/38399.php hier] gibt es Tipps von Nutzern solcher Masken, die damit Probleme hatten. --[[Spezial:Beiträge/2003:76:E12:952A:EC00:6C66:CA30:F78F|2003:76:E12:952A:EC00:6C66:CA30:F78F]] 01:03, 3. Mär. 2017 (CET)
:: {{ping|user:Rotkaeppchen68}} Wenn ich mir über ''Allergie oder was Anderes'' sicher gewesen wäre, hätte ich die Überschrift nicht mit Fragezeichen versehen! Ich weiß ja eben nicht, wo das herkommt! Beim Lieferanten hatte ich heute einen etwas kompetenteren Gesprächspartner, der mir Nasenbrillen aus anderem Material zum Ausprobieren schicken will. Gruß -- [[Benutzer:Dr.cueppers|Dr.cueppers]] - [[Benutzer Diskussion:Dr.cueppers|Disk.]] 20:00, 3. Mär. 2017 (CET)

== Tote Hose auf Mittelwelle im Autoradio ==

Früher war die Mittelwelle im Grundigradio voll mit Sendern. Jetzt, neues Auto, hightech Radio, nix mit MW. Warum? Und warum unterstützen die noch MW und LW, wenns doch nix bringt?
--[[Spezial:Beiträge/93.212.207.119|93.212.207.119]] 23:51, 2. Mär. 2017 (CET)
:In Deutschland wurden die Mittelwellensender ersatzlos abgeschaltet. Dank Internetlivestream ist niemand mehr auf den Mittelwellenfernempfang angewiesen. Der Mittelwellenortsempfang wurde in Deutschland bereits ab 1949 mit dem ersten UKW-Sender ganz allmählich getötet. Als Konsequenz des Zweiten Weltkriegs verlor Deutschland sehr viele Mittelwellenfrequenzen. Deswegen wurde ab 1949 das UKW-Sendernetz mehrfach flächendeckend aufgebaut. Dadurch, dass überall drei bis vier UKW-Sender zu empfangen waren, musste niemand mehr Mittelwelle im Nahbereich hören. Lediglich der Mittelwellenfernempfang hatte noch seine Berechtigung. Mit dem Siegeszug des Internets in den 1990er-Jahren verlor der Mittelwellenfernempfang mehr und mehr an Bedeutung, weswegen die Mittelwellensenderbetreiber ihre Sender nach und nach abschalteten. Aus demselben Grund erlangte der AM-Nachfolger DRM nie Bedeutung. Außerhalb Deutschlands ist die Situation anders. In den USA wurden sogar Frequenzen oberhalb 1600 kHz neu dem Mittelwellenrundfunk zugewiesen. Die Langwelle hat nie wirklich Bedeutung erlangt, weil die zur Verfügung stehende Bandbreite nicht im Verhältnis zur Reichweite der Sender steht. --[[user:Rotkaeppchen68|R<span style="color:red">ô</span>tkæppchen₆₈]] 00:06, 3. Mär. 2017 (CET)
::Kleiner Tipp: Das Lesen des Artikels [[Mittelwelle]] hätte die Frage auch beantwortet. --[[Spezial:Beiträge/2003:76:E12:952A:EC00:6C66:CA30:F78F|2003:76:E12:952A:EC00:6C66:CA30:F78F]] 00:51, 3. Mär. 2017 (CET)
:::Dann haben die Maschinen bereits gewonnen. Wie soll der Widerstand dann kommunizieren?--[[Benutzer:Wikiseidank|Wikiseidank]] ([[Benutzer Diskussion:Wikiseidank|Diskussion]]) 12:17, 3. Mär. 2017 (CET)
:Wie andere schon gesagt haben: Öffentlicher Hörfunk wird in Deutschland mittlerweile praktisch gar nicht mehr über AM gesendet (also LW/MW/KW). Das sieht in anderen Ländern anders aus, deshalb werden Radios auch weiterhin damit ausgestattet. Kann dein Hightech-Radio zufällig [[Digital Audio Broadcasting|DAB]] (das es auch schon seit etwa 20 Jahren gibt)? --[[Benutzer:Kreuzschnabel|Kreuz]][[Benutzer Diskussion:Kreuzschnabel|schnabel]] 15:32, 3. Mär. 2017 (CET)
::DAB gibt es zwar schon seit 20 Jahren, konnte sich aber wegen schlechter Tonqualität, schweineteurer Enpfänger und schlechter Senderabdeckung nicht durchsetzen. Der Durchbruch kam erst mit der flächendeckenden Einführung von DAB+. Mittlerweile sind so gut wie alle DAB-Classic-Programme abgeschaltet. --[[user:Rotkaeppchen68|R<span style="color:red">ô</span>tkæppchen₆₈]] 17:54, 3. Mär. 2017 (CET)
Vielen Dank für die Antworten. Ich meine mich aber zu erinnern, daß ich vor etwa 40 Jahren im Autoradio auf MW noch RTL und nebendran Radio Vatikan, z.B., gut empfangen konnte. Und mit dem hightech Radio jetzt krieg ich keinen Sender gut rein. Die alten Dinger hatten eine Ferritantenne. --[[Spezial:Beiträge/84.181.108.114|84.181.108.114]] 19:39, 3. Mär. 2017 (CET)

= 3. März 2017 =

== Ressourcenverknappung: Wie lange reichen unsere Metalle? ==

Alle reden von [[globale Erwärmung|globaler Erwärmung]] und [[Peak Phosphorus]]. Aber ich habe bisher noch keine halbwegs verlässliche Schätzung gesehen, wie lange unsere Metalle wie Kupfer noch reichen, wenn man im derzeitigen Tempo Mobiltelefone und andere Produkte produziert, ohne die Rohstoffe nach Gebrauch im Sinne einer [[Kreislaufwirtschaft]] weiterzunutzen. Kennt Ihr einen idealerweise aktuellen Artikel aus einer verlässlichen wissenschaftlichen Fachzeitschrift, der für verschiedene Materialien eine Schätzung abgibt, wie lange wir die Stoffe, etwa Kupfer, noch nutzen können? [[Spezial:Beiträge/130.226.41.18|130.226.41.18]] 08:54, 3. Mär. 2017 (CET)
:Beim Kupfer scheint die Lage nicht so dramatisch zu sein: [http://www.solarify.eu/2014/07/07/479-genug-kupfer-da/]. --[[Benutzer Diskussion:Snevern|Snevern]] 09:42, 3. Mär. 2017 (CET)
::Gerade Kupfer gibts genug, da droht keine Knappheit. --[[Benutzer:Ralf Roletschek|M@rcela]] [[Bild:Miniauge2.gif|27px]] 09:43, 3. Mär. 2017 (CET)

:::Die Anzahl der nutz- und förderbaren Metalle reicht für die gegenwärtige Erdbevölkerung auf unbegrenzte Zeit. Was begrenzt ist, ist die Zugänglichkeit. Aber wenn Du einen Bergmann des Mittelalters erzählt hättest, das wir heute komplette Bergspitzen abtragen und bereits bis 4000 Meter tiefe Bergwerke nutzen, hätte er es auch nicht geglaubt. Die Menschheitsgeschichte hat gezeigt, daß die unvorhersehbaren Märkte jede vermeintlich seriöse Prognose ad absurdum führen. Es ist nur eine Frage des Preises, ob der Mensch überhaupt ein Metall nutzen will. Bestes Beispiel sind die seltenen Erden, wo man sich lieber von den Tagebauerzen in China abhängig macht, als eigene, europäische Vorkommen zu erschließen.[[Benutzer:Oliver S.Y.|Oliver S.Y.]] ([[Benutzer Diskussion:Oliver S.Y.|Diskussion]]) 10:06, 3. Mär. 2017 (CET)

::::Das klingt ja alles optimistisch. Sieht das der wissenschaftliche Konsens auch so? [[Spezial:Beiträge/130.226.41.18|130.226.41.18]] 11:30, 3. Mär. 2017 (CET)
:::::Dazu braucht man keine Wissenschaft. 7 Mrd. Menschen auf der Erde haben 3 Mrd. Tonnen Kupfer. Pro Mensch also 500 Kilo. Selbst wenn so ein Telefon nahezu komplett aus Kupfer wäre, hätte also jeder Mensch 5000 Telefone. Da der Kupferanteil jedoch sehr viel geringer ist.... --[[Benutzer:Ralf Roletschek|M@rcela]] [[Bild:Miniauge2.gif|27px]] 11:51, 3. Mär. 2017 (CET)

:::::Naja, wie wäre es, mal in der Wikipedia nachzulesen? [[Kupfer/Tabellen und Grafiken#Förderung, Reserven und Vorratsbasis]]. Blei gleichbleibender Fördermenge reichen die aktuellen Reserven, die "wirtschaftlich" gefördert werden können noch 34 Jahre. Die "Förderfähigen" Vorkommen reichen 63 Jahre. Das betrifft aber nichtmal die gesamten 29% der Erdoberfläche an Land. Weite Gebiete wie die Sahara, Amazonasbecken, Grönland und die Antarktis sind noch nichtmal erkundet. Und es ist nicht "unwissenschaftlich", die 71% von Wasser bedeckten Gebiete für ähnlich zu halten. Du wirst keine "verlässliche" Fachzeitschrift finden, die da genau sagen kann, ob es letztendlich 3,5 oder 10 Mrd. Tonnen sind. Schon der Unterschied zwischen "reserves" und der "reserve base" ist wesentlich, wird häufig durch Journalisten aber gleichgesetzt.[[Benutzer:Oliver S.Y.|Oliver S.Y.]] ([[Benutzer Diskussion:Oliver S.Y.|Diskussion]]) 11:47, 3. Mär. 2017 (CET)

Eine weitere, in Zukunft wohl immer wichtiger werdende Quelle ist unser Abfall. Deponien, und Schrott und andere Abfälle enthalten einiges an Metallen, bislang sind sie aber in vielen Fällen nicht wirtschaftlich zurückzugewinnen. Bei einigen Sachen aber schon, so ist bei Kupfer mindestens die Hälfte des eingesetzten Kupfers mittlerweile Altmetall https://www.kupferinstitut.de/de/werkstoffe/system/recycling-kupfer.html. Bei anderen Metallen sieht es ähnlich aus und diese Quote dürfte in Zukunft steigen. Limitierend ist meist die Effektivität bzw. die Energiemenge zur Herstellung. --[[Benutzer:Elrond|Elrond]] ([[Benutzer Diskussion:Elrond|Diskussion]]) 12:30, 3. Mär. 2017 (CET)

:Dazu noch der Hinweis, daß in Sachsen (Helmholtzgesellschaft) und Polen bereits fortgeschrittene Experimente mit Mikroben stattfinden, welche eine Gewinnung von Kupfer und anderen Metallen aus vermeintlich minderwertigem Gestein und sogar aus mittelalterlichen Abraumhalden ermöglichen. Die kacken schlicht unverdauliches Kupfer aus :), das muss man nur noch zusammenfegen. [[Benutzer:Oliver S.Y.|Oliver S.Y.]] ([[Benutzer Diskussion:Oliver S.Y.|Diskussion]]) 12:37, 3. Mär. 2017 (CET)

:Klassischer Überlegungsfehler. Metalle werden in der Regel nicht verbraucht, sondern könnten Wiederverwertet werden. Kurzum in Zukunft wird man vermehrt Metalle aus Mülldeponien usw. gewinnen müssen. Wenn das mit dem erschließen von Sekundär-Rohstoffquellen gemacht würde, gibt es nicht so schnell einen Versorgungsengpass. Man wird nicht bei den klassischen Metallen ein Problem hab, die fast rein verwendet werden (Kupfer, Eisen, Aluminium). Sonden bei dennen, die nur zu einem kleinen Prozentsatz im Endprodukt stecken.--[[Benutzer:Bobo11|Bobo11]] ([[Benutzer Diskussion:Bobo11|Diskussion]]) 12:44, 3. Mär. 2017 (CET)

:: <small>Prognosen sind schwierig, besonders, wenn sie die Zukunt betreffen.</small>
:: Ich habe keine Ahnung, von welchen Voraussetzungen unsere Futorologen hier ausgehen. Wir haben auf der einen Seite einen zunehmenden Bedarf an Kupfer für Elektromotoren - zumindest, wenn die Prognosen der Regierungen zur Elektromobilität zutreffen. Um den dafür benötigten Strom zu erzeugen, werden wir auch noch etwas zusätzliches Kupfer für Generatoren benötigen. Dann bleibt die Frage, wie sich die Menschheit weiter vermehrt. Wenn ich einfach eine Gerade von 1800 bis heute lege, dann wächst die Menschheit unbeschränkt weiter, bis die Leute schliesslich auf den Schultern von anderen Leuten stehen oder nasse Füsse bekommen, weil an Land kein Platz mehr ist. Realistisch sollten wir aber vermuten, dass das karnickelartige Vermehren sich irgendwann verlangsamt (nebenbei bemerkt eine Katastrophe für unsere heutige Wirtschaft). Natürlich wird so eine kuschelige Nähe völlig andere Verhaltensweisen erfordern, beispielsweise müssen wir vermutlich auf den Köpfen Tabletts tragen, auf denen wir Grünzeug zur Ernährung anbauen, weil der Boden ja mit Menschen bedeckt ist.
:: Kurz: Du wirst hier keine seriöse Prognose bekommen. Aber jedem Menge von der anderen Sorte. [[Benutzer:Yotwen|Yotwen]] ([[Benutzer Diskussion:Yotwen|Diskussion]]) 13:21, 3. Mär. 2017 (CET)

:::Die Menschheitsvermehrung muss schon allein deshalb gestoppt werden, weil der technische Fortschritt die Menschen beschäftigungslos macht. Hierzulande kriegen die Menschen weniger Kinder und beschäftigen sich mit "schöngeistigen" Dingen. Woanders spielt die Jugend stattdessen Krieg. ("Spiel", weil total ineffektiv.) Ich wage mal eine Prognose, deren Eintreten ich vermutlich noch erlebe: Entwicklungshilfe, die ihren Namen tatsächlich verdient, wird in Zukunft an Bevölkerungsreduktion gekoppelt. Ob das in Afrika dann über Ein-Kind-Prämien oder über "Abschussprämien" realisiert wird, werden wir den afrikanischen Staaten sicherlich mal wieder selbst überlassen. Weil: Keine Einmischung. Bei den Leuten, die im Mittelmeer ersaufen, ist es uns ja heute auch schon egal. Es sei denn, irgendein Reporter hält auf einen in der Brandung rollenden Kinderleichnam drauf. Dann ist halt eine Woche Aufregung und danach wieder Normalbetrieb. -- [[Benutzer:Janka|Janka]] ([[Benutzer Diskussion:Janka|Diskussion]]) 15:16, 3. Mär. 2017 (CET)
::::Ein Wörtchen dürfte auch die natürliche Auslese mitspielen. Die Biologie reagiert , wenn auch sehr träge, auf veränderte Lebensbedingungen. --[[Benutzer:Ralf Roletschek|M@rcela]] [[Bild:Miniauge2.gif|27px]] 15:35, 3. Mär. 2017 (CET)
: Die en hat [[:en:Peak copper|Peak copper]] und sonst haben wir bald [[Meeresbodenbergbau]] oder besser [[Asteroidenbergbau]].--[[Benutzer:Asteroidenbergbauer|Asteroidenbergbauer]] ([[Benutzer Diskussion:Asteroidenbergbauer|Diskussion]]) 16:32, 3. Mär. 2017 (CET)
::Nun ja, bevor man zum Asteroiden Bergbau schreitet, werden sicher zuerst mal die bislang unrentabel Erzlagerstätte abgebaut. So finden sich im Skandinavien (und im übrigen Europa) einigen Kupfer-Minen die stillgelegt wurden bevor sie leer geräumt waren. Auch in den Abraumhalden ehemaliger Kupfermienen wird es noch Kupfer haben. Wenn die grossen Minen aus denen heute das meiste Kupfer stammt erschöpft sind, wird sich schlicht weg der Rohstoffpreis erhöhen, und plötzlich rentieren sich Lagerstätten die man heute gar nicht auf dem Radar hat. Und es rentiert dann plötzlich auch Kupfer besser aus dem Abfall herauszulösen. Aber generell wird es keine Frage danach sein ob wir noch Kupfer haben, sondern eher ein Frage ob wir es uns für gewisse Anwendungen noch leisten können Kupfer zu "verschwenden". Übrigens könnte man schon heute bei etliche Elektroanwendungen auch mit Aluminium arbeiten, ist halt nicht ganz ein so guter Leiter und bisschen alterungs-anfälliger als Kupfer. Aber schon bei der [[SBB Ae 4/6]] hat man wegen Kupfermangel im Zweiten Weltkrieg der Haupttransformator und die Fahrmotoren mit Aluminiumwicklungen ausgeführt. Denn grundsätzlich ist Alu statt Kupfer, bei Elektro-Anwendungen möglich. Das Kupfer besser geeignet ist streit ich nicht ab, aber es gibt Alternativen zu Kupfer. Denn Silber wären ja noch besser als Kupfer geeignet (ist der bessere Leiter), nur will das in der Regel niemand bezahlen.--[[Benutzer:Bobo11|Bobo11]] ([[Benutzer Diskussion:Bobo11|Diskussion]]) 19:21, 3. Mär. 2017 (CET)

Im Buch B. Ilschner, R. F. Singer: ''Werkstoffwissenschaften und Fertigungstechnik Eigenschaften, Vorgänge, Technologien'' 5., neu bearbeitete Auflage, gibt es zu genau diesem Thema ein paar Seiten. "Wie aus Abb. 1.2 hervorgeht, müssen wir damit rechnen, dass viele wichtige Rohstoffreserven in den nächsten 20 bis 50 Jahren zur Neige gehen. Bei einigen wenigen Stoffen ist die Situation sogar noch wesentlich dramatischer. Dazu gehört beispielsweise [[Indium]], für das nur noch Reichweiten von wenigen Jahren vorhergesagt werden"

Aus der erwähntn Abb. 1.2 entnehme ich mal die "Reservenreichweite" in Jahren. Zusätzlich ist noch eine höhere "Reserven- und Ressourcenreichweite" angegeben. ("Bei genauerer Betrachtung stellt sich die Situation für unsere meisten Rohstoffe allerdings weit weniger kritisch dar. [...] Reserven sind nachgewiesene, mit gegenwärtiger Technik zu gegenwärtigen Preisen wirtschaftlich gewinnbare Vorräte. Ressourcen sind dagegen geologisch nachgewiesene, aber gegenwärtig nicht wirtschaftlich abbaubare Vorräte.")
*Uran - 46
*Eisen - 95
*Aluminium - 141
*Titan - 122
*Nickel - 41
*Kupfer 31
*Zink 22
*Blei 20
Und zum Vergleich
*Braunkohle - 293
*Steinkohle - 137
Die Zahlen ergeben sich also nur mathematisch aus den derzeitigen Reserven dividiert durch den Jahresverbrauch. Technologischer Wandel, wie bspw. von Yotwen angesprochen und auch wirtschaftlicher Wandel sind da nicht berücksichtigt. --[[Benutzer:Der-Wir-Ing|DWI]] ([[Benutzer Diskussion:Der-Wir-Ing|Diskussion]]) 19:27, 3. Mär. 2017 (CET)
:Ja, mir machen auch die "seltenen" und "unedlen" Metalle mehr Sorgen, als der "Klassiker" Kupfer, was die Versorgungssicherheit anbelangt. Denn bei denen [[Erdalkalimetalle]]n [[Borgruppe|Erdmetallen]] und den [[Metalle der Seltenen Erden|Seltenerdmetalle]] funktioniert das Recycling noch nicht so gut. Schlichtweg auch aus dem Grund das unedle Metalle zu gern "unpraktische" Verbindungen eingehen. Und wie oben schon angesprochen, je weniger von etwas in der endgültigen Werkstoff ist, desto schwerer und aufwendiger ist es als halbwegs reinen Wertstoff wieder heraus zu kriegen.--[[Benutzer:Bobo11|Bobo11]] ([[Benutzer Diskussion:Bobo11|Diskussion]]) 19:53, 3. Mär. 2017 (CET)

== Prozess gegen Anton Schlecker, Ausschluß der Öffentlichkeit? ==

Kann Anton Schlecker beantragen dass die Öffentlichkeit ausgeschlossen wird zum Prozess gegen ihn? Ich meine er wollte nie in der Öffentlichkeit stehen und wollte nicht das Bilder von ihm gemacht werden, darf er dann das auch vor Gericht geltend machen?
--[[Benutzer:Fragesteller 2018|Fragesteller 2018]] ([[Benutzer Diskussion:Fragesteller 2018|Diskussion]]) 09:23, 3. Mär. 2017 (CET)
:Grundsätzlich: Nein. Ich sehe auf den ersten Blick auch keinen einschlägigen Ausnahmetatbestand, auf den er sich berufen könnte.
:Angeklagte, die keine Bilder von sich haben wollen, verstecken sich regelmäßig hinter Aktendeckeln. Müsste er wohl auch tun. --[[Benutzer Diskussion:Snevern|Snevern]] 09:36, 3. Mär. 2017 (CET)
::Aufs [[Recht am eigenen Bild]] kann er sich gem. §23 KUG als absolute Person der Zeitgeschichte nicht berufen. --[[Benutzer:Ralf Roletschek|M@rcela]] [[Bild:Miniauge2.gif|27px]] 09:42, 3. Mär. 2017 (CET)
:::Wenn man aber die Unschuldsvermutung beachtet, sollte man doch hier klarstellen, dass keine Bilder gemacht werden dürften bzw. veröffentlich bis ein Urteil steht?--[[Benutzer:Fragesteller 2018|Fragesteller 2018]] ([[Benutzer Diskussion:Fragesteller 2018|Diskussion]]) 10:35, 3. Mär. 2017 (CET)
::::Bezogen aufs Recht am eigenen Bild ist das irrelevant. Egal ob schuldig oder unschuldig ist er bekannt genug. --[[Benutzer:Ralf Roletschek|M@rcela]] [[Bild:Miniauge2.gif|27px]] 10:37, 3. Mär. 2017 (CET)
::::Es besteht gar kein Anlass, die Öffentlichkeit auszuschließen. Weder ist Anton Schlecker minderjährig, noch werden in diesem Prozess Staatsgeheimnisse berührt. --[[user:Rotkaeppchen68|R<span style="color:red">ô</span>tkæppchen₆₈]] 11:39, 3. Mär. 2017 (CET)
:Ich wüsste auch nicht, wieso die Öffentlichkeit ausgeschlossen werden können sollte: Schon ein Unbekannter muss mit der Berichterstattung über seinen Prozess leben - Gerichtsverfahren sind und sollen nunmal öffentlich sein. Dass ein Angeklagter in der Öffentlichkeit steht, er also grundsätzlich schon ''mehr'' Berichterstattung über sich hinnehmen muss, kann nicht auf einmal dazu führen, dass ''weniger'' berichtet werden dürfte. Eher das Gegenteil. --[[Spezial:Beiträge/83.135.224.214|83.135.224.214]] 15:24, 3. Mär. 2017 (CET)

:Prozessöffentlichkeit und journalistische Bilder sind zwei unterschiedliche Dinge. Bilder kann er als Person der Zeitgeschichte (Ralf verweist oben zurecht auf [https://www.gesetze-im-internet.de/kunsturhg/__23.html § 23 Kunstzrhg]) bei einem öffentlichen Informationsinteresse (23.000 Arbeitsplätze vernichtet und ''„[https://www.welt.de/wirtschaft/article114581561/Nichts-mehr-da-steht-fuer-zehn-Millionen-Euro.html ...es ist nichts mehr da...])“'' nicht verhindern. Für Prozesse gilt der [[Öffentlichkeitsgrundsatz]] mit seinen Ausnahmeregeln. --[[Spezial:Beiträge/80.130.231.152|80.130.231.152]] 19:17, 3. Mär. 2017 (CET)

== Woher kommen die alternativen Pluralformen im Bairischen ==

"enk" und "es"? In den übrigen deutschen Dialekten und in den heutigen germanischen Sprachen gibt es das meines Wissens nur selten. Es wird immer behauptet, dass es einen Zusammenhang mit dem Gotischen gebe. Stimmt das wirklich? Und wieso existieren eigentlich solche Formen?

<!-- Lass die nachfolgende Zeile am ENDE deiner Frage stehen. Sie wird in deine Signatur umgewandelt. -->
--[[Spezial:Beiträge/217.238.133.50|217.238.133.50]] 10:39, 3. Mär. 2017 (CET)


:Das sind alte [[Dual (Grammatik)|Dualformen]]. Die Frage ist also weniger, warum sie existieren, sondern warum sie im Hochdeutschen nicht mehr existieren (wo doch eigentlich jede vollwertige Sprache über einen Dual verfügt – letzteres eine persönliche Anmerkung ;) --[[Benutzer:J budissin|j.budissin]]<sup>[[Benutzer Diskussion:J budissin|+/-]]</sup> 10:45, 3. Mär. 2017 (CET)
Ich habe gestern in einem Artikel über eine Pflanzenart im englischen einen Begriff gelesen, mit dem ich nichts anfangen konnte und habe diesen bei Wikipedia auf den englischen Seiten nachgeschlagen. Der Begriff lautet "Shale" und wurde in Verbindung mit dem Begriff Sandstone, welcher für mich als Sandstein zu bezeichnen wäre, genannt. Der Begriff Sandstone und Sandstein sind in der englischen auf die deutsche Wikipedia verlinkt. Allerdings als ich mir die Fotos auf der Seite des Artikels “Shale“ anschaute und mal nachlas, um was es sich handelt, war mir sofort klar, dass es sich um Schiefer handelt. Sollte das dann vielleicht auch entsprechend verlinkt werden, denn ich finde, dass hier bei Wikipedia in der deutschen Form und bei Wikipedia in der englischen Form eigentlich viel zu viele Begriffe die eigentlich die gleiche Bedeutung haben, noch gar nicht miteinander verlinkt sind, obwohl sie miteinander verlinkt gehören. Finde ich halt ziemlich wichtig, denn sonst braucht man ja eigentlich die deutsche Wikipedia gar nicht mehr und kann dann eben auch gleich die deutsche Sprache in den Müll werfen, weil dann ja keiner sich mehr die Arbeit machen will, das miteinander zu verlinken. Ich mache das nicht, denn sonst kriege ich ja möglicherweise schon wieder eine auf die Schnauze und ich finde, dass da genügend andere Leute sind, die Jobs bei Wikipedia haben, die deutlich dümmer sind als ich es bin. --[[Spezial:Beiträge/2001:16B8:2830:9800:F081:CDF9:6C5E:1A4C|2001:16B8:2830:9800:F081:CDF9:6C5E:1A4C]] 10:16, 2. Okt. 2020 (CEST)
:::Wenn ich mir das [[Dual_(Grammatik)#Bairisch|ihm hiesigen Artikel]] anschaue, steht dort "Im Bairischen wird die ursprüngliche Dualform als allgemeine Pluralform verwendet. Einen eigenen Dual gibt es heute nicht mehr" --[[Benutzer:Elrond|Elrond]] ([[Benutzer Diskussion:Elrond|Diskussion]]) 12:19, 3. Mär. 2017 (CET)
:Der Artikel [[Schiefer]] ist verlinkt unter anderem mit [[:es:Esquisto]] und [[:fr:Schiste]]. Der Artikel [[:en:shale]] ist verlinkt unter anderem mit [[:es:Shale]] und [[:fr:Shale]]. Das Problem ist die Bedeutung des Wortes Schiefer: „Schiefer ist ein Sammelbegriff für unterschiedliche Gesteine“, „Der Begriff "Schiefer" ist ein Polysem, welcher seit grauer Vorzeit […] bis heute nicht scharf definiert und differenziert wurde. Allgemein und bergmännisch werden als Schiefer deutlich parallel angeordnete, in dünnen, ebenen Platten spaltbare Gesteine bezeichnet. Unter diesen Sammelbegriff fallen jedoch auch Tonschiefer, Schieferton, Tonstein, kristalline Schiefer, echte und unechte, kristalline und metamorphe Schiefer, sedimentärer Kupferschiefer, Plattenkalke und viele, viele synonyme Schiefer-Wortgebilde und lokale Bezeichnungen- was nicht zu einer eindeutigen Definition beiträgt.“[https://www.mineralienatlas.de/lexikon/index.php/RockData?lang=de&rock=Schiefer] Es gibt noch den Artikel [[:en:Schist]], der meines Erachtens besser zu [[Schiefer]] passt; das verlinkte Mineralienlexikon übersetzt das auch so. Der Artikel [[Schiefer]] nennt im Übrigen ''slate'' als Übersetzung von Tonschiefer, zu dem es keinen eigenen deutschen Artikel gibt. --[[Benutzer:BlackEyedLion|BlackEyedLion]] ([[Benutzer Diskussion:BlackEyedLion|Diskussion]]) 10:28, 2. Okt. 2020 (CEST)
::::<small>{{Ping|Elrond}} Ja. Und? --[[Benutzer:J budissin|j.budissin]]<sup>[[Benutzer Diskussion:J budissin|+/-]]</sup> 12:25, 3. Mär. 2017 (CET)</small>
::Meines Erachtens sollten also die Artikel in der Gruppe [[Schiefer]], [[:es:Esquisto]], [[:fr:Schiste]] usw. und die Artikel in der Gruppe [[:en:Schist]], [[:it:Scisto]] usw. zusammengeführt werden. --[[Benutzer:BlackEyedLion|BlackEyedLion]] ([[Benutzer Diskussion:BlackEyedLion|Diskussion]]) 11:21, 2. Okt. 2020 (CEST)
::Im Zusammenhang vielleicht noch interessant sind die [[Isländische_Sprache#Personalpronomen|isländischen Personalpronomen]]: Der alte Plural (entspr. dt. ''ihr'') ist eine spezielle Höflichkeitsform geworden, während der alte Dual (wie im Bayerischen) die normale Pluralform übernommen hat. --[[Benutzer:L47|L47]] ([[Benutzer Diskussion:L47|Diskussion]]) 10:51, 3. Mär. 2017 (CET)
:Siehe [[Bairische Kennwörter#és und enk]]. --[[Benutzer:Komischn|Komischn]] ([[Benutzer Diskussion:Komischn|Diskussion]]) 11:18, 3. Mär. 2017 (CET)
::Der Zusammenhang mit dem Gotischen ist dort leider nicht behandelt. Nach dem Beleg in [[:bar:Bairische Kennwörter#ees und enk]] reichen die bairischen Schriftbelege in das 15. Jh. zurück. Es gehen voraus althochdeutsch *ez (ihr beide), althochdeutsch *enker (euch beiden) und althochdeutsch *enker (euer), vergleichbar mit gotisch *it, igqis und igqar [https://books.google.de/books?id=w7c7m7XaMskC&lpg=PP1&hl=de&pg=PA39#v=onepage&q&f=false]. --[[Benutzer:Pp.paul.4|Pp.paul.4]] ([[Benutzer Diskussion:Pp.paul.4|Diskussion]]) 17:23, 3. Mär. 2017 (CET)


== TÜV wegen Undichtigkeit - morgen nass ==
== Senkschrauben in Dübel für Oberschränke? ==


Morgen habe ich einen Nachtermin beim TÜV wegen Öl am Motor. Voraussichtlich wird es aber regnen. Muss ich dann nochmal kommen, wenn man wegen der Nässe wenig sieht, und womöglich die Nachgebühr ein zweites Mal zahlen?--[[Spezial:Beiträge/82.113.121.63|82.113.121.63]] 14:50, 3. Mär. 2017 (CET)
Ich möchte zwei Besta-Hängeschränke von IKEA - beziehungsweise deren Befestigungsschiene - an der Wand befestigen. Ich nehme an, dass 6er Dübel reichen sollten (vier Stück pro Schrank). Dazu habe ich ein Universal-Dübelset gekauft. Die beiliegenden Schrauben haben allerdings einen Senkkopf. Ist das problematisch? Ich hätte angenommen, dass ein flacher Kopf mehr Halt bietet. --[[Spezial:Beiträge/95.90.219.121|95.90.219.121]] 10:27, 2. Okt. 2020 (CEST)
:Ruf an und frag. Oder mach gleich einen Ersatztermin aus. --[[user:Rotkaeppchen68|R<span style="color:red">ô</span>tkæppchen₆₈]] 15:16, 3. Mär. 2017 (CET)
: Die erforderliche Dübelgröße bzw. allgemeiner das erforderliche Befestigungsmittel hängt ganz erheblich von der Wandbeschaffenheit ab, das läßt sich nicht verallgemeinern. --[[Spezial:Beiträge/77.6.54.254|77.6.54.254]] 10:42, 2. Okt. 2020 (CEST)
{{erledigt|Hier falsch. --[[user:Rotkaeppchen68|R<span style="color:red">ô</span>tkæppchen₆₈]] 15:16, 3. Mär. 2017 (CET)}}
::Die Art der Dübel hängt von der Wandbeschaffenheit ab, die Größe der Dübel davon, wie stark das Regal belastet wird: wenn Du 30 Bücher in das Regal stellst, muss die Befestigung mehr aushalten als wenn Du nur ein paar Souveniers aus dem letzten Urlaub hineingibst. Und vor allme: die Größe der Dübel muss zur Dimension der Schrauben passen. Und zur eigentlichen Frage: der Schraubenkopf sollte da keine Rolle spielen, das Regal muss doch nur an der Wand gehalten werden, aber nicht gegen ein Verschieben gesichert werden, wo es auf die Größe der Kontaktfläche ankommt. --[[Benutzer Diskussion:TheRunnerUp|TheRunnerUp]] 11:10, 2. Okt. 2020 (CEST)
::Hab' ich ein so besonderes Auto, dass der Motor auch bei Regen trocken bleibt? --[[Benutzer Diskussion:Snevern|Snevern]] 17:55, 3. Mär. 2017 (CET)
:::ok, danke für die Antwort :) --11:17, 2. Okt. 2020 (CEST)
:Der Prüfer kann Wasser und Öl schon unterscheiden. Hat er beides schonmal gesehen. --[[Spezial:Beiträge/2003:76:E32:57FC:9C7C:DCB4:94AC:734F|2003:76:E32:57FC:9C7C:DCB4:94AC:734F]] 19:38, 3. Mär. 2017 (CET)
Wie schon geschrieben, es kommt darauf an. In eine solide gemauerte Ziegelwand, oder eine gegossene Betonwand bist Du mit den 6er Universaldübeln bestens bedient. Hast Du eine Rigipswand, gar einfach geplankt, dann brauchst Du spezielle dübel und je nach Wandaufbau mehr oder weniger. Fachwerk ist wieder was anderes und es gibt noch etliches mehr. Die Belastung spielt, wie auch schon gesagt, auch eine erhebliche Rolle, aber da sollte mit der Maximalvariante gerechnet werden, denn wer weiß in 10 Jahren noch, was Du Dir gedacht hast, selbst Du wahrscheinlich nicht mehr. --[[Benutzer:Elrond|Elrond]] ([[Benutzer Diskussion:Elrond|Diskussion]]) 11:25, 2. Okt. 2020 (CEST)
:Die eigentliche Frage war doch, ob der Senkkopf in Ordnung geht, richtig? Das hängt von der Art der Befestigungschiene ab, die ja von der Schraube gehalten werden muss. Sollte diese eine versenkte Schraubenaufnahme haben, könnte es passen. Aber ohne Details ist das nur geraten. --[[Benutzer:Drahkrub|Burkhard]] ([[Benutzer Diskussion:Drahkrub|Diskussion]]) 11:36, 2. Okt. 2020 (CEST)
::Die haltende Kraft eines Dübels kommmt durch die Spreizung im Bohrloch. Die Senkkopfschrauben dürften mehr oder weniger gleich gut sein wie andere. Die Schraubenform mag noch einen Einflus haben, also ob gleiche Dicke über die gesamten Windungen, oder vorne spitz mit steigendem Durchmesser. Da würde ich letztetes bevorzugen, aber auch das dürfte keinen sehr großen Einfluss haben. Wie geschrieben, auf die Wand kommt es drauf an. --[[Benutzer:Elrond|Elrond]] ([[Benutzer Diskussion:Elrond|Diskussion]]) 12:01, 2. Okt. 2020 (CEST)
::Das stimmt, Drahkrub. Es geht um [https://www.ikea.com/de/de/assembly_instructions/besta-suspension-rail-silver-colour__AA-1272162-10_pub.pdf diese Schiene]. Sie hat keine Senklöcher und ist, was die Schrauben betrifft, im Wesentlichen ein flaches Stück Metallblech mit Löchern. Ach ja, und die Wand ist kein Trockenbau, sondern eine Massivbau-Außenwand. Anscheinend relativ viel Putz, ansonsten Ziegel (rotes Bohrmehl). --[[Spezial:Beiträge/95.90.219.121|95.90.219.121]] 12:54, 2. Okt. 2020 (CEST)


== Essen und Atmen ==
== Rücktritt und Schadensersatz bei Kaufverträgen ==


Warum gibt es eigentlich 2 unterschiedliche Varianten einen misslungenen Kauf abzuwickeln, und diese eigentlich auf das selbe hinauslaufen? Warum nicht nur eine?
Die Konstruktion dass man durch den Mund sowohl atmen als auch essen kann ist ja ziemlich raffiniert und erfordert eine perfekte Umschaltung zw. diesen beiden Modi. Mir ist es allerdings ein Rätsel, wie die Evolutionsschritte dorthin geführt haben. Denn entweder die Umschaltung ist perfekt oder es wird nicht funktionieren. Gibt es Landtiere, die Luftröhre und Speiseröhre getrennt haben und gibt es Tiere, wo man Mischformen sieht? Edit: es geht mir nicht um den Mund, sondern um die Splittung in Luftröhre und Speiseröhre. --[[Spezial:Beiträge/85.212.34.166|85.212.34.166]] 10:58, 2. Okt. 2020 (CEST)
--[[Spezial:Beiträge/91.66.137.253|91.66.137.253]] 18:35, 3. Mär. 2017 (CET)
: Ich habe gelernt, dass man beim Essen den Mund schließt. Daher atme ich vorzugsweise durch die Nase. --[[Benutzer:Gadacz|Klaus-Peter ''<small>(<sup>auf</sup><small>und</small><sub>davon</sub>)</small>'']] 11:05, 2. Okt. 2020 (CEST)
:Es gibt sogar ''noch mehr'' Varianten, siehe {{§|437|bgb|juris}} BGB. Wo genau diese Varianten herkommen, ist eine gute Frage. Vieles, was wir heute haben, stammt ursprünglich aus dem [[römisches Recht|römischen Recht]]. Wo diese Varianten deiner Meinung nach auf dasselbe hinauslaufen, müsstest du noch etwas weiter erläutern, damit wir den entsprechenden Unterschied darstellen können; es gibt z.B. mehrere Arten des Schadenersatzes. --[[Spezial:Beiträge/83.135.224.214|83.135.224.214]] 19:20, 3. Mär. 2017 (CET)
::Ich würde sagen, das Gegenteil ist der Fall. Atmen hat sich aus schlucken entwickelt, siehe [[Schluckatmung]]. --[[Benutzer:BlackEyedLion|BlackEyedLion]] ([[Benutzer Diskussion:BlackEyedLion|Diskussion]]) 11:17, 2. Okt. 2020 (CEST)
:::Sehr alte Geschichte. Vor den Landtieren mussten schon die Fische beim ''atmen'' über die Kiemen das Maul öffnen, genau so wie vor dem Runterschlucken einer Nahrung ;-) Die Frage, ob es zuerst das Schlucken gab oder das Atmen, ist wie die Frage, ob es zuerst die Henne gab, oder das Ei. Tierische Lebewesen haben vor der Ausbildung des [[Urdarm]]s und des [[Urmund]]es den benötigten Sauerstoff uber die Haut aufgenommen.
:::Zitat aus [[Gastrulation]]: "''Die weitere Entwicklung des Urmunds teilt die bilateralsymmetrischen Tiere ([[Bilateria]]) in zwei Stammgruppen ein: Bei den [[Urmünder]]n (Protostomier) bildet der Urmund auch tatsächlich den späteren Mund, während bei den [[Neumünder]]n (Deuterostomier), zu denen die [[Wirbeltiere]] und auch der Mensch zählen, sich der Urmund mehr oder weniger deutlich zum After entwickelt, während der spätere Mund bei vollendeter Gastrulation auf der anderen Seite der Blastula „durchbricht“.''"
:::Also das Einverleiben von Nahrung durch einen Mund und das ''Schlucken'' vor dem ''Atmen'' durch einen geöffneten Mund. -- [[Benutzer:Muck|Muck]] ([[Benutzer Diskussion:Muck|Diskussion]]) 11:48, 2. Okt. 2020 (CEST)
Du möchtest also die Evolution widerlegen, weil „Mir ist es allerdings ein Rätsel, wie die Evolutionsschritte dorthin geführt haben“. Wie Muck oben schon ausgeführt hat ist das sauber rückführbar. Auch das Auge wurde nicht an einem Tag entwickelt, da ist es auch so, dass es zig Entwicklungsschritte gab, die man auch heute noch an diversen Organismen, auch noch lebenden, sehen kann. {{unsigniert|Elrond|12:59, 2. Okt. 2020 (CEST)}}

Version vom 2. Oktober 2020, 13:32 Uhr

Abkürzung: WP:AU, WP:AUS

Du konntest eine Information in Wikipedia trotz Benutzung der Suchfunktion der Wikipedia, einer Suchmaschine und des Archivs dieser Seite (Suchfeld unten) nicht finden? Dann beantworten Wikipedianer auf dieser Seite allgemeine Wissensfragen.

Bedenke dabei bitte:

  • So manche Antwort auf eine Frage ist im Internet per Suchmaschine schneller gefunden, als die Frage hier gestellt und beantwortet werden kann.
  • Die Auskunft ist kein Diskussionsforum. Daher ist auch nicht die Ausbreitung von Meinungen oder eigenen Theorien das Ziel, sondern die Verbreitung von belegbarem Wissen.

Für viele Anliegen gibt es spezielle Seiten:

Wie stelle ich meine Frage am besten?
  • Die Kurzanleitung erklärt, wie du eine Frage stellst.
  • Die Fragen werden ausschließlich auf dieser Seite beantwortet, nicht per E-Mail usw. Daher bitte keine persönlichen Kontaktdaten (Anschrift, Telefon, Mail) hinterlassen.
  • Wenn deine Frage ausreichend beantwortet wurde oder du eine Lösung gefunden hast, lass es uns wissen.
  • Für alle entsprechenden Fragen gelten die Hinweise zu Gesundheitsthemen, Rechtsthemen und Rechtsauskunft sowie zum Pilzesammeln.
Fragen beantworten
  • Du weißt die Antwort oder kennst wenigstens Hinweise darauf? Dann antworte so kurz wie möglich, so lang wie nötig, mit Links auf Wikipedia-Artikel oder andere Quellen, die zum Verständnis beitragen.
  • Wenn die Antwort noch nicht in der Wikipedia steht und relevant ist, vervollständige bitte die Artikel zum Thema und verlinke hier auf die entsprechenden Passagen. Sollte eine Ergänzung nicht ohne Weiteres möglich sein (z. B. weil entsprechende Belege fehlen oder es sich bei dem Geäußerten hauptsächlich um persönliche Ansichten der Autoren handelt), setze bitte einen entsprechenden Hinweis auf die Diskussionsseite der betreffenden Artikel. Die Auskunft soll nämlich auch helfen, die Artikel der Wikipedia zu verbessern.
  • Bitte rücke deine Antwort mit Doppelpunkt(en) am Zeilenanfang passend ein!

Abschnitte, die älter als 3 Tage oder seit einem Tag mit dem Baustein {{Erledigt|1=~~~~}} gekennzeichnet sind, werden automatisch archiviert. Möglicherweise findest du auch im Archiv die Antwort auf deine Frage. (Gesamtarchivletzte Woche). Eine Sammlung von häufig gestellten Fragen findest du auf der FAQ-Unterseite.

17. Februar 2017

Wer war evtl. der 50000. Schütze?

Laut Kicker gibt es vier Varianten zur Zählung der Gesamt-Bundesligatore. Variante 1 liefert "erst" 49950, Variante 2 49991, Variante 3 nun 50003 Tore. Bei der letzteren Variante heißt der Schütze des 50000. Bundesligatores Karim Bellarabi. Variante 4 - mit den Toren von abgebrochenen Spielen - weist 50017(?) Tore aus. Wer war denn hier der Schütze des 50000. Bundesligatores (es muss am letzten Spieltag gefallen sein)? --ObersterGenosse (Diskussion) 22:05, 17. Feb. 2017 (CET)Beantworten

Guck dir die halt Spielberichte vom letzten (20.) Spieltag an (findest du allesamt bei kicker.de) und zähl es selbst aus. Nach der 4. Zählvariante hat Bellarabi ja das 50013. 50014. Tor erzielt. Das 50000ste fiel dann wahrscheinlich letzte Woche Samstag, denn am Sonntag fielen nur 6 Treffer, am Samstag hingegen 15 14. Guck dir die Spiele an, und erstelle eine Tabelle, in der du auflistest, wer in welcher Spielminute ein Tor erzielt hat. Allerdings musst du dabei voraussetzen, dass bei jedem der Spiele die Spielzeit simultan lief, was spätestens ab der 2. Halbzeit wegen unterschiedlicher Nachspielzeiten am Ende der 1. Halbzeit unrealistisch ist. Fiel der rein von der Spielminute in Frage kommende Treffer also in der 2. Halbzeit und liegt er dort eng mit anderen Treffern beieinander (< 5 min), müsste man sich anschauen, um welche Uhrzeit die jeweils tatsächlich gefallen sind... --Gretarsson (Diskussion) 14:01, 19. Feb. 2017 (CET); zuletzt. geänd. 11:25, 22. Feb. 2017 (CET)Beantworten
Wobei die Methode nicht ganz unproblematisch ist, wenn z.B. bei einem Spiel mit Beginnzeit 15:30 ein Tor in der 73. Minute fällt und bei einem anderen Spiel am selben Tag mit der selben Ankickzeit in der 74. Minute, muss das Tof in der 73. Minute nicht unbedingt vor dem in def 74 Minute gefallen sein, da die Uhrheit vom Ankick vor Allem bei der zweitdn Halbzeit einige Minuten auseinanderliegen kann. --MrBurns (Diskussion) 17:29, 19. Feb. 2017 (CET)Beantworten
Hab ich das oben nicht geschrieben? ;-) --Gretarsson (Diskussion) 18:29, 19. Feb. 2017 (CET)Beantworten
P.S. Am Sonntag und beim 18.30-Uhr-Spiel am Samstag fielen insgesamt nur 8 Treffer, das fragliche Tor muss also eines der 13 12 sein, die während der fünf Nachmittagspartien fielen (davon war ich oben sowieso ausgegangen) und zwar tatsächlich eines der 6 7, die während der 2. Halbzeit fielen (damit muss man dann also nur noch 4 Spielberichte angucken, denn bei Bremen-M’Gladbach war der Halbzeitstand auch der Endstand)... --Gretarsson (Diskussion) 19:03, 19. Feb. 2017 (CET); zuletzt. geänd. 11:25, 22. Feb. 2017 (CET)Beantworten
Es scheint dich ja brennend interessiert zu haben, ObersterGenosse, wer denn nun der Rekordtorschütze nach inoffizieller Zählung war, wenn du hier zwar die Frage einstellst, dich dann aber anscheinend nicht weiter drum scherst, wie die Antwort lautet. Ist dir die Recherche selbst dann noch zu anstrengend, wenn dir das Ergebnis schon fast auf dem Silbertablett präsentiert wird?
Da es mich letztlich selbst interessiert hat, und für evtl. interessierte Mitlesende hier die Auflösung:
Die Partien Leipzig–HSV und Ingolstadt–Bayern scheiden aus, weil dort die in der 2. Halbzeit erzielten Tore (Numero 50003 bis 50005) alle in der 90. min oder später fielen. Das entscheidende Tor fiel also entweder bei Darmstadt–Dortmund oder L’kusen–Frankfurt, und, was soll ich sagen, auch nach dieser Zählung war der Rekordtorschütze ein „Pillendreher“: Es war Chicharito, der in der 63. Spielminute gegen Frankfurt das 2:0 für sein Team erzielte. Der 2:1 Siegtreffer der Darmstädter durch Colak fiel zwar in der 67. Minute und damit relativ dicht dran, doch den Uhrzeitangeben in den jeweiligen Livetickern ist zu entnehmen, dass Chicharito um 16:50 Uhr traf, Colak hingegen erst 16:54, die 2. Halbzeit bei beiden Spielen also ungefähr gleichzeitig angepfiffen wurde. Colak erzielte demnach also nur Tor Nummer 50001. Das 50002., der Vollständigkeit halber, schoss Kevin Volland (78.), der damit den 3:0-Endstand gegen die Eintracht herstellte. --Gretarsson (Diskussion) 11:14, 22. Feb. 2017 (CET)Beantworten

Danke für die Antworten! Hier hatte sich einige Tage lang niemand gemeldet, also dachte ich, die Frage sei unbeantwortet ins Archiv gewandert oder sowas. Tschuldigung an alle!

Und danke an Gretarsson für die Recherche! Beide Jubiläumstore gehen also auf das Konto von Vizekusen! Und wenn sie jetzt noch die beiden anderen (mehr oder weniger) offiziellen Jubiläumstore schießen... wäre doch cool, oder? --ObersterGenosse (Diskussion) 14:08, 22. Feb. 2017 (CET)Beantworten

OK, dann nichts für ungut. Ja, wäre schon ein großer Zufall, wenn tatsächlich „alle“ 50.000sten Tore ein Leverkusener schießen täte. Ob das cool wäre, weiß ich nicht. Davon kaufen könnten sie sich nichts. Viele fänden das wahrscheinlich gar nicht so cool, denn m.W. hat Leverkusen jetzt nicht sooo wahnsinnig viele Anhänger und der finanzielle Rückhalt, den sie durch die Bayer AG haben, wird ihnen auch gerne geneidet, wobei ich persönlich das schlicht als Standortvorteil werten würde... --Gretarsson (Diskussion) 14:37, 22. Feb. 2017 (CET)Beantworten
Nachtrag: Das 50.000ste nach Variante 2 fiel übrigens auch noch am vergangenen Spieltag und zwar wieder am Samstag und wieder in der 2. Halbzeit (damit ist klar, dass kein Bayer-Spieler „alle vier“ Jubiläumstreffer erzielen wird). Ausgehend von 49.991 Toren nach dem Spiel L’kusen–Augsburg fielen am Samstag Nachmittag in der 1. Halbzeit die Treffer 49.992 bis 49.996. Das Jubiläumstor nach dieser Zählung war demnach, wenn ich richtig gezählt hab, das 2:0 der Ingolstädter im Spiel gegen Frankfurt. Torschütze war Pascal Groß. Das Tor fiel in der 69. Minute, laut Kicker-Liveticker um 16:57, d.h. 7 min nach dem Treffer des Hoffenheimers Kramaric zum 1:0 gegen Darmstadt (64.) und deutlich vor den beiden letzten Treffern des Nachmittages, die beide erst in der Nachspielzeit erzielt wurden. Das letzte der vier Jubiläumstore dürfte dann am übernächsten Spieltag fallen (Stand derzeit, wenn ich richtig gezählt hab, 49.970). --Gretarsson (Diskussion) 21:29, 22. Feb. 2017 (CET)Beantworten

Wir haben jetzt also schon: Karim Bellarabi Chicharito Pascal Groß

und werden bei einem torreichen Spieltag am nächsten, sonst am übernächsten Spieltag wissen, wer das "vierte" Jubiläumstor geschossen hat. Daher:

Dieser Baustein verhindert die automatische Archivierung dieses Abschnitts und seiner Unterabschnitte. Wir warten noch auf das vierte 50000te Tor!

So, wenn 49.970 Tore bis zum letzten Sonntag gefallen sind - so wie es Gretarsson vermutet hat - und da ja gestern und heute schon 25 Tore fielen (es war also ein "torreicher Spieltag"), fehlen nur noch fünf. Dafür hätten wir noch Ingolstadt - Gladbach und Schalke - Hoffenheim. Also könnten durchaus morgen fünf Tore fallen und wir hätten den vierten 5000er schon morgen. Ansonsten am nächsten Spieltag... --ObersterGenosse (Diskussion) 20:51, 25. Feb. 2017 (CET)Beantworten

Heute fielen nochmal vier Tore, d.h., der Schütze zum 1:0 am nächsten Freitag beim Spiel Augsburg-Leipzig wird der Schütze des 50.000sten Bundesligatores nach Zählvariante 1. --Gretarsson (Diskussion) 00:45, 27. Feb. 2017 (CET)Beantworten

23. Februar 2017

Trappist-1 - wie funktioniert das mit der Gravitation?

Möglicherweise ganz einfach, aber meine 3-D-Vorstellungskraft ist gerade überfordert: Wie schaffen es die sieben Planeten ähnlicher Größe auf so nahen Orbits, dass sie nicht kollidieren, bzw. wie muss ich mir ihre Bewegungen zueinander vorstellen? 89.12.251.193 16:36, 23. Feb. 2017 (CET)Beantworten

Planetenbahnen
Man muss sich gar nicht viel 3D vorstellen, da die Bewegung der Planeten im Wesentlichen in einer Ebene abläuft. Und im Wesentlichen laufen sie auf konzentrischen Kreisen (im Zentrum ist die Sonne; in Wirklichkeit sind es Ellipsen, aber das macht keinen großen Unterschied, siehe Grafik rechts). Sie können sich also nicht in die Quere kommen. Die Orbits sind auch auch nicht nah. Auf vielen Darstellungen sind die Abstände im Vergleich zu den Größen viel zu klein dargestellt (bzw. die Größen zu groß), da man sonst die Größen oder die Abstände nicht wirklich erkennen könnte. Die Erde hat z.B. einen Durchmesser von ca. 13 000 km; der Abstand zu den Nachbarplaneten beträgt aber mindestens 30 Mio. km. Die Planeten sind von der Erde alle soweit entfernt, dass sie mit bloßem Auge nur als Punkt zu sehen sind. Im Gegensatz etwa zum Mond, der kleiner als alle Planeten ist, aber deutlich größer zu sehen ist, weil wesentlich näher (ca. 400 000 km), was immer noch 60mal soviel ist wie der Erdradius. --Digamma (Diskussion) 16:46, 23. Feb. 2017 (CET)Beantworten
Ergänzung: Planetenwege helfen, sich die Größenverhältnisse vorzustellen. Im häufig verwendeten Maßstab von 1 : 1 Milliarde hat die Erde einen Durchmesser von 1,3 cm. Der Abstand zur Venus beträgt mindestens 38 m, der zum Mars mindestens 55 m. --Digamma (Diskussion) 17:04, 23. Feb. 2017 (CET)Beantworten
Vielen Dank; allerdings ging es mir um das Trappist-1-System, und dort sollen die sieben Planeten alle innerhalb der Merkur-Bahn passen. Aber wahrscheinlich ist trotzdem mehr Platz, als ich mir vorgestellt hatte, auch wenn es heißt, man könnte dort die Nachbarplaneten in Mondgröße am Himmel bewundern. 89.12.251.193 17:09, 23. Feb. 2017 (CET)Beantworten
Die kleinen Bahnen ergeben sich, weil die Zentralsonne ein Roter Zwerg ist, der weit weniger Masse hat als unsere Sonne. Laut Wiki-Artikel rund 8 %. Die Oberflächentemperatur ist weit niedriger und die Strahlungsdichte ebenfalls, daher ist die habitable Zone weit näher am Stern. --Elrond (Diskussion) 18:24, 23. Feb. 2017 (CET)Beantworten
Missverständnis. (Schreib doch ganze Sätze.) --Digamma (Diskussion) 17:12, 23. Feb. 2017 (CET)Beantworten
Linkservice: TRAPPIST-1 --Neitram  17:28, 23. Feb. 2017 (CET)Beantworten
Sorry, das "Trappist 1" in der Überschrift habe ich überlesen. Die Sache ist vermutlich wirklich komplizierter, denn der gegenseitige Gravitationseinfluss der Planeten untereinander ist hier wahrscheinlich nicht zu vernachlässigen. Im Artikel zu en:Trappist 1 in der englischen Wikipedia steht, dass die Umlaufdauern der Planeten in Resonanz stehen. Wenn ich das richtig verstehe, bedeutet dies u.a. dass die Bahnstörungen durch die Gravitation der andern Planeten sich rausmittelt und die Bahnen nicht instabil macht. --Digamma (Diskussion) 17:30, 23. Feb. 2017 (CET)Beantworten
Vielleicht hilft dem Fragesteller ja auch schon der Satz aus dem Artikel: "Der Aufbau dieses Planetensystems ähnelt somit eher dem System der Jupitermonde als unserem Sonnensystem"? --Neitram  17:32, 23. Feb. 2017 (CET)Beantworten
Wie das mit der Gravitation funktioniert, weiß ich auch nicht. Aber wenn der Merkur 0,3 bis 0,5 Astronomische Einheiten weit von der Sonne entfernt ist, und die sieben Planeten zwischen 0,01 und 0,06 Astronomischen Einheiten um ihren Stern kreisen, dann ist dazu wohl genug Platz. Jedenfalls kann ich mir das so grad noch vorstellen. fz JaHn 00:48, 24. Feb. 2017 (CET)Beantworten
(Gedächtnisfragment) Es gibt auch in userem Sonnensystem ein Mondpaar deren Bahnen so nah beieinander sind das sie regelmäßig kollidieren müssten. Irgendwie wird das dadurch "gelöst" das der Impuls des von hinten aufholenden auf den vorderen übertragen worauf der vordere schneller wird und irgendwann selber von hinten kommt. Weis jemand welche Monde/Planet das ist? --Dreifachaxel (Diskussion) 13:31, 24. Feb. 2017 (CET)Beantworten
Das sind die beiden Saturnmonde Janus und Epimetheus. --Gretarsson (Diskussion) 14:09, 24. Feb. 2017 (CET)Beantworten
Es kann höchstens der hintere schneller werden. Abstoßende Impulsübertragung ohne Berührung wie bei elastischem Stoß oder wie bei gleich gepolten elektrischen oder magnetischen Feldern gibt es nicht und die Gravitation, die Ursache des Ganzen, wirkt immer nur anziehend. --Geri, ✉  18:37, 25. Feb. 2017 (CET)Beantworten
Hufeisenumlaufbahn, Janus (Mond)#Bahnverhalten von Janus und Epimetheus. --Neitram  10:56, 28. Feb. 2017 (CET)Beantworten
Es handelt sich um eine gravitative Übertragung von Bahnernergie und -impuls, ohne dass die Monde sich berühren. Der Hintere wird bei Annäherung an den Vorderen beschleunigt und steigt daher auf eine etwas höhere Umlaufbahn. Der vordere wird hingegen abgebremst und sinkt auf eine etwas tiefere Umlaufbahn. Da die Bahngeschwindigkeit auf der tieferen ULB größer ist als auf der höheren, entfernt sich der Vordere wieder und wird irgendwann selbst zum Hinteren, weil er den nun auf der höheren, langsameren ULB befindlichen Mond einholt. Dann beginnt das Spiel von neuem. --Gretarsson (Diskussion) 21:12, 1. Mär. 2017 (CET)Beantworten

25. Februar 2017

Wieso überschätzen wir seltene Ereignisse (Terroranschläge, Flugzeugabstürze) dermaßen?

Derzeit diskutiert die ganze Welt ja mal wieder über das Thema Terrorismus. Den US-Präsidenten hat das sogar dazu veranlasst, ganze Nationen von der Einreise in die USA auszuschliessen. Bei genauem Hinsehen zeigt sich, dass es in Deutschland 4000 mal wahrscheinlicher ist an einer Grippe zu sterben und immerhin 1,13 mal wahrscheinlicher an einem Blitzschlag zu sterben als an einem Terroranschlag (s. Berechnungen). Auch Flugangst ist so ein Fänomen: Wenn immer ich einen Flug antrete, ist meine Freundin sehr besorgt. Dabei steht dies in gar keiner Relation mit der im Vergleich dazu deutlich höheren Wahrscheinlichkeit bei einer Autofahrt zu sterben. Was aber führt zu solchen Fehleinschätzungen? Ist es der Medien-Hype der das Terrorthema gerne aufgreift? Ich erinnere mich an beinahe wochenlange Medienberichterstattungen nach dem Absturz des Germanwings-Flugzeuges. Problematisch ist dies ja, weil dadurch knappe Gelder auf die falschen Prioritäten verteilt werden: Ganz rational betrachtet müssten die Zeitungen ja voll von Nachrichten über Grippetote sein und die Politik müsste nichts anderes mehr tun als Gelder auf Grippeforschung zu verteilen, um das Risiko daran zu sterben zu reduzieren. Auch der Aufwind von Rechtspopulisten lässt sich wohl auf diese Fehleinschätzung zurückführen. Trump erfindet ja sogar Terroranschläge in Schweden, weil die extreme Friedfertigkeit in Schweden trotz der Aufnahme mehrerer 100.000 Flüchtlinge einfach so gar nicht in sein Vorurteil des kriminellen Zuwanderers passen will. Gibt es wissenschaftliche Arbeiten, die die Ursache für diese Fehleinschätzungen bei der Risikobewertung untersucht haben? Ich könnte mir vorstellen, dass die Kognitionsforschung sich doch bestimmt einmal mit Risikowahrnehmung befasst hat. Wo finde ich mehr darüber? Ich bin durchaus an wissenschaftlichen Artikeln darüber interessiert. 62.44.135.249 07:38, 25. Feb. 2017 (CET)Beantworten

Ich würde mal mit Wikipedia anfangen: Risikowahrnehmung. Dieser Artikel stützt sich allerdings wesentlich auf nur eine Arbeit, die zwar einmal (Fußnote 1) als Quelle angegeben wird, aus der aber in Tat und Wahrheit das Meiste abgeschrieben übernommen ist. Umfassender und mit sehr viel mehr Literaturangaben versehen ist der englische Schwesterartikel en:Risk perception. Grüße Dumbox (Diskussion) 08:00, 25. Feb. 2017 (CET)Beantworten
Für eine Naturkatastrophe hätten wir Verständnis. Menschengemachtes halten wir zurecht für Vermeidbares, da es politisch ist, sei es auch nur Organisations- oder Firmenpolitik. --Hans Haase (有问题吗) 08:45, 25. Feb. 2017 (CET)Beantworten
Es könnte auch andersherum sein, dass wir wahrscheinliche Risiken viel schneller ausblenden. Die Beeinflussbarkeit ist der andere Faktor der zu Verzerrungen führt. Die Wahrscheinlichkeit einen Autounfall zu vermeiden, meinen wir durch unser Verhalten beeinflussen zu können. Beim Flugzeug sind wir dem Piloten usw. ausgeliefert (wir haben nur die Wahl zwischen fliegen oder nicht fliegen). Diese beeinflusst und verzehrt die Wahrnehmung ungemein.--Bobo11 (Diskussion) 08:57, 25. Feb. 2017 (CET)Beantworten
Ja. Das Schlüsselwort ist (oben) Risikowahrnehmung.
Hier ein wissenschaftlicher Artikel von 2015 (den man da auch komplett herunterladen kann), der die heute gängigen Ansätze abarbeitet. Wichtig der Hinweis auf das soziokulturelle Umfeld der Risikowahrnehmung (das Beispiel unten mit den white males und der Verbreitung von Waffen!). Auch die Medien kriegen ihr Fett.
Ich persönlich würde eine Unterscheidung machen zwischen
  • "Was sehe ich in meinem täglichen Leben als Risiko" [Ich habe beispielsweise die gefährliche Angewohnheit, dass ich mir morgens das T-Shirt anziehe, während ich die Treppe hinunter gehe...] und
  • "Was verklickern mir die Medien VON ALLEN SEITEN als Risiko".
Da sieht alles schon mal gaaanz anders aus. GEEZER … nil nisi bene 10:31, 25. Feb. 2017 (CET)Beantworten
Das ist Thema der Kognitionspsychologie, insbesondere Wahrnehmungsfehler. Auch, wenn es eigentlich nicht zu dem Themengebiet gehört, findest du unter Prospect Theory eine ganze Reihe von Wahrnehmungsfehlern, die zur Erklärung von Fehleinschätzungen herangezogen werden. Yotwen (Diskussion) 11:13, 25. Feb. 2017 (CET)Beantworten
Lies auch mal alles über den Thalamus. Das ist das Tor zum Bewusstsein im Hirn, hier wird entschieden, was beachtenswert oder nicht ist. Das gleiche Bild an der Wand wird mehr oder minder ausgeblendet, es wird keine Info an das Gehirn geschickt. Erst wenn ein anderes da hängt, bekommt das Bewusstsein eine Meldung. Damit wird eine Überlastung des Hirns durch zu viele Eindrücke unterbunden. So ist es auch mit den "Unglücken", alles was da auf die Schiene des gewohnten/täglichen geschoben wird, wird weniger beachtet, erst wenn was außerordentliches passiert, schiebt es der Thalamus ins Bewusstsein.--2003:75:AF0C:6900:D5AB:114F:5900:6B6E 15:10, 25. Feb. 2017 (CET)Beantworten

Ich glaube man orientiert sich zu sehr an Medien. Früher hat man als Bub mitbekommen, dass der Karl von der Leiter gefallen ist und sich das Genick gebrochen hat und 20 Jahre später ist ähnliches passiert und daraus hat man sich eine Gefahr errechnet. Heute steht in der Zeitung, dass in Spanien 2 Leute an der Todesgrütze gestorben sind und nächste Woche sind es 3 in Österreich und dann glauben die Leute schon, bald sind sie dran. Ausserdem ist man in D ziemlich neurotisch geworden und glaubt, mann müsse jedes Risiko in den Griff bekommen. Bei uns hat ein ganzes Freibad seinen Betrieb eingestellt weil an verschiedenen Stellen die Geländer nicht mehr den aktuellen Sicherheitsanforderungen genügt haben. In Russland oder Vietnam würde man die Deutschen dafür auslachen. Das gleiche auch z.B. mit Rauchmeldern. Das ist auf einmal total wichtig und relevant. --2.246.78.200 15:16, 25. Feb. 2017 (CET)Beantworten

Naja, Rauchmelder sind durchaus sinnvoll. Es hat sich gezeigt, dass pro Tag im Schnitt etwa ein Mensch durch ein Feuer stirb (95% dabei aufgrund der dabei entstehenden giftigen Dämpfe). Vor der Einführung von Rauchmeldern waren es noch doppelt so viele. Für mich ist das eher ein Beispiel dafür wie ein unterschätztes Risiko mit einer einfachen Massnahme verringert werden kann. Das ist also so ziemlich das Gegenteil von den reisserischen Themen wie Terrorismus oder Flugzeugabstürzen. 130.226.41.19 15:58, 25. Feb. 2017 (CET)Beantworten
Der niederländische Soziologe Geert Hofstede hat eine Kulturbeschreibung auf der Basis einer empirischen Befragung von ca. 140000 Leuten aufgestellt. Demnach gibt es eine Dimension kulturellen Verhaltens, inwieweit eine Gesellschaft Risiken vermeidet oder nicht. Ein Beispiel für Gesellschaften mit niedriger Risikovermeidung ist Indien. Hohe Werte findet man in den sogenannten reichen Ländern, Deutschland, USA usw.
Das Modell ist statisch, aber es gibt keinen Grund anzunehmen, dass sich Kulturen nicht verändern. Es kann also durchaus sein, dass die Risikovermeidung mit der Zeit zunimmt. Ich fände es normal, dass in einer hysterischen Gesellschaft (und das sind wir spätestens seit die modernen Massenmedien auf uns eintrommeln) Veränderungen stattfinden. Was früher nur etwas für Paranoiker war, ist heute schon Mainstream. Yotwen (Diskussion) 17:34, 25. Feb. 2017 (CET)Beantworten
"eine Dimension kulturellen Verhaltens" Na absolut! In einem Land, wo Gottgewolltes passiert, ist es halt so. Punkt. In einem Land, in dem jede Sendung unterbrochen wird, weil ein Idiot mit einer Waffe(l) 'rumweldelt, werden Leute permanent auf den hormonellen Zehenspitzen gehalten.
Die "Gnade des Nichtwissens" (oder man entwickelt halt Mechanismens, direkte Realität und indirekte Realität einordnen zu können). GEEZER … nil nisi bene 14:29, 26. Feb. 2017 (CET)Beantworten
Das hat sicherlich auch was mit den Medien zu tun. Wenn etwa, sagen wir mal, ein japanischer Kernreaktor nach einem Erdbeben und einer Flutwelle havariiert, dann schalten manche Nationen auf Basis eines Moratorium alle Kernkraftwerke ab. Dabei ist die Wahrscheinlich, dass es zu einer Reaktorkatastrophe in diesen Nationen kommt, von den Ereignissen in Japan doch völlig unbeeinflusst. Sprich: Die rationalen Gründe für oder gegen Atomkraft haben sich nicht geändert, nur die (Risiko-)Wahrnehmung. (nicht signierter Beitrag von 188.101.73.75 (Diskussion) 23:20, 26. Feb. 2017 (CET))Beantworten
Der Unfall in Japan zeigt ziemlich eindrucksvoll, dass die Abschätzungen der Techniker und Ingenieure ziemlich gut sind. Die haben nämlich vorausgesagt, dass es alle ca. 20000 Jahre zu einer Kernschmelze kommt. Bei ca. 500 Kernkraftwerken weltweit kommen wir also auf 20000/500=40; d.h. so alle ca. 40 Jahre müssen wir mit einer Kernschmelze rechnen. Und wenn wir uns dann die Geschichte ansehen, dann haben wir mit Three Mile Island (1979); Tschernobyl (1986); Fukushima Daiichi (2011) drei Kernschmelzen in den rund 80 Jahren Kernkraftgeschichte. Das ist so dicht an der Vorhersage der Techies, dass ich dazu tendiere, die Risikoabschätzung für realistisch zu halten.
Irreal sind die Folgerungen, die daraus gezogen wurden. Denn das Risiko verändert sich nicht erheblich, wenn ein Kernkraftwerk "abgeschaltet" wird. Wir sparen, so scheint es mir, ca. 20-30 Jahre von einer Million Jahre, also 0.03 %. Unsere Nachfahren werden uns auf ewig dankbar sein. Yotwen (Diskussion) 09:12, 27. Feb. 2017 (CET)Beantworten

In diesem Buch wird das ganz eindrücklich als das „Problem des großen Nenners“ beschrieben. Wenn man ein Risiko als Bruch betrachtet, dann bildet die Schwere des Ereignisses den Zähler des Bruches, während die Eintrittswahrscheinlichkeit im Nenner ist. Und leider neigen Menschen dazu, denn Nenner zu übersehen und den Zähler extrem zu überschätzen.

Wenn man z.B. die Gefahren von verschiedenen Verkehrsmitteln vergleicht, dann ist der Zähler (also die Schwere des potentiellen Ereignisses) beim Flugzeugabsturz deutlich höher (viele Tote) als bei einem Autonunfall (relativ wenige Tote, höhere Übelebenschance). Beim Nenner (also der Eintrittswahrscheinlichkeit) verhält es sich aber mehr als andersherum: Hier ist es um ein vielfaches wahrscheinlicher in einem normalen Verkehrsunfall verwickelt zu werden, als jemals von einem Flugzeugabsturz betroffen zu sein. Zusammengenommen ist das Risiko bei einem Flugzeugabsturz zu sterben deutlich geringer, als das, bei einem Autounfall ums Leben zu kommen. Trotzdem fürchten sich die meisten Menschen vor ersterem mehr (eben weil sie den Zähler zu Ungunsten des Nenners überbewerten). // Martin K. (Diskussion) 09:30, 27. Feb. 2017 (CET)Beantworten

Martin das ist der eine Punkt. Der andere ist eben der, dass man sich einredet das man mit vorsichtiger Fahrweise ja die Unwahrscheinlichkeit senken kann. Was ja Statistisch auch stimmt, leider reicht das aber nicht um unter die Wahrscheinlichkeit eines Flugzeugabsturzes zu kommen. Der Punkt, kann aber unser Verständnis ganz gut ausblenden. Wir werden unbeeinflussbare Risiken immer höher bewerten als die Risiken die wir beeinflussen, weil wir ja das Risiko um 50% senken können. Dazu kommt eben die Menge der Tote auf einmal. Ein Unfall mit einem Toten bleibt nun mal nicht so hängen wie einer mit 100 Toten. Wenn jetzt statistisch beide Unfallarten wirklich vergleichbar wären weil sie genau gleich sind. Wir werden das Risiko das alle 100 Tage ein Unfall mit 100 Toten gibt höher gewichten, als der täglich Tote. Obwohl es bei beiden Risiken so ist, dass es in 100 Tagen 100 Tote gibt. Interessant ist ja auch der Bericht hier, da wird erklärt warum die Leute bei den Schätzungen der Toten pro Jahr und Unfallart so daneben liegen können (geschätzt jährliche 36 Tote durch Blitzschlag in der Schweiz, effektiv wären es aber 0,5 Tote pro Jahr).---Bobo11 (Diskussion) 23:20, 27. Feb. 2017 (CET)Beantworten
Noch ein Stichwort: Verfügbarkeitsheuristik / availability heuristic. Auch (Nicht-Not)Ärzte leiden darunter - werden sie gefragt, ob Verkehrsunfälle oder Herzinfarkte häufiger sind, schätzen sie die Häufigkeit von Unfällen höher ein, obwohl es tatsächlich das andere ist. Weil Berichte über Unfälle im Bewusstsein schlicht präsenter sind als Berichte über Herzinfarkte. Zum Beispiel kennen viele Leute jemanden, der nach Paris gereist ist und dort die massiven Sicherheitsmassnahmen gesehen hat - aber kaum jemand kennt einen, der einen Blitzschlag überlebt hatte (oder daran gestorben ist).--Keimzelle talk 09:53, 2. Mär. 2017 (CET)Beantworten

26. Februar 2017

Lernstrategien im Studium: Was führt zum Erfolg?

Gibt es Studien oder sogar Metastudien darüber, welche Lernstrategien man im Studium idealerweise anwenden sollte, um erfolgreich zu sein? Sollte man beispielsweise eher alleine lernen oder mit Kommilitonen? Sollte man Texte lieber vor oder nach der Vorlesung lesen, damit sie hängen bleiben? Solche oder ähnlich Fragen interessieren mich. Am liebsten wären mir Studien aus hochrangigen Fachzeitschriften. 62.44.135.249 17:00, 26. Feb. 2017 (CET)Beantworten

Es interessiert DICH als Einzelperson? Dann bringen dir Statistiken über den Erfolg von Strategien nichts. Du folgst einfach deinem Bauch/Kopf ... und was funktioniert, machst du weiter.
Nur einen Hinweis zu "Fachzeitschriften": Ich habe am Anfang des Studiums aus den drei besten Lehrbüchern gelernt, die es damals gab. Die Klausurfragen kamen aber aus Lehrbuch O. Ich habe 3 nicht so gute Freitagsklausuren gebraucht, bis ich das Aha!-Erlebnis hatte und dann im Durchschnitt bei 95 % der möglichen Punktzahl lag. Hehehe! GEEZER … nil nisi bene 17:31, 26. Feb. 2017 (CET)Beantworten
Nein, es interessiert mich nicht als Einzelperson. Ich bin Dozent und möchte meinen "Schäfchen" am Anfang des Semesters Lerntipps mit auf den Weg geben und da ich "research-based teaching" betreibe, d.h. jede PPT-Folie mit einer Quelle versehe, möchte ich nicht einfach behaupten "lernt in Gruppen!", wie das damals meine Dozenten gemacht hatten, sondern wirklich abgesichertes Wissen weitergeben. Das erscheint mir nicht nur im AfD-/Trump-Zeitalter besonders angebracht, sondern generell. 62.44.135.249 17:42, 26. Feb. 2017 (CET)Beantworten
PPT-Folien waren für mich persönlich immer ein triftiger Grund, ein Seminar nicht weiter zu besuchen, denn verblöden kann ich mich auch selbst. Aber auch da sind die Geschmäcker verschieden...--Edith Wahr (Diskussion) 17:47, 26. Feb. 2017 (CET)Beantworten
Für mich auch. Aber darum mache ich es jetzt ja besser. Ich habe sehr lange an einem Konzept für "gute" PPT-Folien gearbeitet. Geholfen hat mir auch, dass ich parallel zum Studium als Webdesigner gearbeitet habe, was beim guten Foliendesign sehr hilft. Ausserdem, das ist glaube ich in Deutschland noch immer nicht so, wird man hier in Dänemark didaktisch sehr gefördert, auch parallel zur Arbeit als Professor. Ich kriege in jeder Lehrevaluation gesagt, dass dies auch so ist. Liebe Edith, das musst Du mir jetzt glauben, aber meine Vorlesung würdest Du zumindest nicht wegen der Folien verlassen. 62.44.135.249 19:17, 26. Feb. 2017 (CET)Beantworten
Interesse/Eigenmotivation fördern, von Anfang! an Lernziele (Leistungsnachweisaufgaben) deutlich darstellen, (mehrere) Wege zeigen, individuelle Lernweisen ermöglichen.--Wikiseidank (Diskussion) 18:32, 26. Feb. 2017 (CET)Beantworten
Kapiert!
Lerntechniken: Hier, Hier. Aber ich bleibe dabei: Wie und wann man am besten lernt, ist sehr individuell (z.B. Morgenmenschen, Abendmenschen, mag andere Menschen, lernt lieber allein...)! Ein Student sollte in der Lage sein, seine eigene Strategie zu entwickeln, sonst hat er eigentlich an der Uni nichts verloren. Darauf hinzuweisen, würde dir Zeit sparen. GEEZER … nil nisi bene 18:44, 26. Feb. 2017 (CET)Beantworten
+1, jeder muss das selbst wissen. Ich hatte folgende Strategie und bin gut damit gefahren. Ich habe alles versucht zu verstehen, was der da vorne erzählte. Was ich nicht verstanden habe, habe ich mir notiert und möglichst noch am gleichen Tag nachgelesen bis ich es verstanden hatte. So war ich vor der nächsten Vorlesung auf dem Laufenden. Ich musste vor Klausuren relativ wenig nachbüffeln.--2003:75:AF07:FF00:4913:10E3:C308:CB52 19:28, 26. Feb. 2017 (CET)Beantworten
Ich haue auch noch mal in Wikiseidanks Kerbe: Das Fundament sind Interesse und Motivation! Methoden mögen zwar besser oder schlechter funktionieren, aber das spielt dem gegenüber eine weit geringere Rolle. Wo dieses Fundament nicht mitgebracht wird (was es bereichsweise überall gibt), macht es einen guten Lehrer aus, das so gut wie irgend möglich zu vermitteln.
Was dann Lernmethoden angeht: Da sollte man alternative Möglichkeiten vorstellen und zum Ausprobieren anregen. Je nach Aufgabe und Persönlichkeit kann individuell mal dies, mal das besser funktionieren.
Rainer Z ... 19:47, 26. Feb. 2017 (CET)Beantworten

https://www.kikidan.com/news/repraesentationssystem-visuell-auditiv-kinaesthetisch.html --2.246.98.233 23:26, 26. Feb. 2017 (CET)Beantworten

Es gibt nicht die Lernstrategie, sondern höchstens Deine Lernstrategie. Welche das ist, musst Du selber rauskriegen. Es gibt Gruppenlerner, die am besten mir zweidrei anderen lernen können, aber die anderen muss man erst mal finden. Dann gibt es die Einzelkämpfer, die nur alleine oder bevorzugt alleine lernen können/wollen. Dann gibt es die Permanentnachbereiter, die stets in Häppchen die Veranstaltungen rekapitulieren, auf der anderen Seite die Frontallerner, die binnen kürzester Zeit den Stoff von mehreren Semestern in die Birne knallen können (meist aber genauso schnell wieder vergessen) und vieles mehr. Versuch diverse Methoden aus, wenn Du nicht jetzt schon eine Ahnung hast, was Dir wahrscheinlich am besten liegt. Reine Fakten kann man auch mit der Karteikastenmethode erarbeiten. Viele Hochschulen bieten Seminare über Lerntechniken an, wo stliche Methosen vorgestellt werden, da solltest Du ggf. hingehen. --Elrond (Diskussion) 23:33, 26. Feb. 2017 (CET)Beantworten

Lies den Thread einfach mal genauer. Der Fragesteller ist Dozent und sucht fundierte Lerntipps für seine Studenten. Rainer Z ... 23:52, 26. Feb. 2017 (CET)Beantworten
Warum sollte für den Herrn Dozenten etwas anderes gelten. Es gibt nun mal nicht den Königsweg, sondern jeder Studierende muss seinen Weg finden. Was nutzt eine ach so erfolgreiche, mit zig Studien validierte Methode, wenn der einzelne Studi nicht damit klar kommt! Ich arbeite selber an einer Hochschule als Lehrender und habe nun mal diese Erfahrung gemacht. In meinem Studium dereinst aber auch schon. Wenn der Frager als Dozent die diversen Hochschuldidaktischen Zentren nicht kennt, oder nutzt, wird er hier sicher auch nichts besseres erfahren. --Elrond (Diskussion) 00:04, 27. Feb. 2017 (CET)Beantworten
Hm. Ich bin etwas verwirrt (und das mag auch an meinem Alter liegen...). Als ich – vor langer Zeit – mein Abitur bekam hatten wir am Gymnasium bereits weitgehend gelernt, wie man lernt. Neu waren die vielfältige Infrastruktur der Universität und der Fachbereiche, die (über die UB hinaus) zum Abrufen von Informationen zur Verfügung stand. Dafür gab es Einführungskurse und Führungen für die Erstsemester. Die meisten Vorlesungen und Seminare kamen auch ohne medialen Schnickschnack klar. Eine Tafel reichte zur Not aus. (Das mag in den naturwissenschaftlichen Fächern anders gewesen sein als in den Sozialwissenschaften.) Wir waren – wenn der kleine Scherz erlaubt ist – Hörerinnen und Hörer und nicht Seherinnen und Seher. Meines Erachtens gehört die Fähigkeit, selbstorganisiert zu lernen, zu dem Fundament einer „Hochschulreife“. Warum ist man davon abgekommen? Was machen die Kids dann eigentlich auf dem Gymnasium, wenn nicht (auch) die Vorbereitung auf ein (mögliches) Studium absolvieren? Damit es nicht mißverstanden wird: Zweifellos hat sich die Didaktik fortentwickelt und auch Medien können hilfreich sein. Das Anliegen des Fragestellers ist also durchaus ehrenwert. Aber das sind doch die Sahnehäubchen. Erfolgreich im Lernen kann man auch ohne das sein. Die Voraussetzungen für einen Erfolg liegen imho überwiegend anderswo und nicht (provokant zugespitzt) im mundgerechten, die eigene Konzentrationsleistung und Denkanstrengung auf ein Minimum reduzierenden Spektakel. --2003:46:A0C:6E00:E22A:82FF:FEA0:3113 05:41, 27. Feb. 2017 (CET)Beantworten
Ich habe "Studium" und "Powerpoint" gelesen. Jedem, der so etwas tut, möchte ich Yiannis Gabriel empfehlen und den folgenden Link Against the Tyrrany of Powerpoint.
Demnach verarmt Powerpoint Studenten und Dozenten. Es gibt sicher gute Gründe es zu verwenden, aber es gibt auch gute Gründe, hin und wieder darauf zu verzichten. Yotwen (Diskussion) 09:23, 27. Feb. 2017 (CET)Beantworten
@Hass at Powerpoint: Ich (aus dem Naturwissenschaften-Bereich: harte Fakten und Zahlen) will euch was zeigen. Wie soll ich das machen? Mein Zeuch mit Kreide an die Tafel malen? Eh ich fertig bin, ist der Zuhoerer schon "verbloeded" (Zitat von oben). Ich muesste also mein Diagramm irgendwie schnell hinkratzen und die wichtigen Dinge einkringeln, um es dann 5 Minuten spaeter wieder abzuwischen (weil die anderen 3 Tafel-Abschnitte auch schon mit schiefen Diagrammen voll sind). Ich koennte aber auch Overhead-Projektor-Folien nehmen und die ueber den Projektor schmeissen. Und weil ich faul bin, werden die alten Folien (vom letzten Jahr) nicht voellig neu gemalt, sondern ich korrigier / male neues Zeug dort dazwischen. Na wenn euch das lieber ist... ;) Oder ich nehm Powerpoint, kann direkt die Grafiken aussm Lehrbuch zeigen, und einfach neues Zeugs einfuegen und altes anpassen. Meine Folien haben nie mehr als 15 Woerter auf einer Folie (Titel, Beschriftungen, alles mitgezaehlt; selbst auf der Zusammenfassungsfolie am Schluss max. 15). Oder isses euch am liebsten, wenn ich garnix zeige und einfach nur "vorlese", wies oben auch schon vorgeschlagen wurde ("Hoerer statt seher"): "Also, wat is en Dampfmaschin? Da stelle mehr uns janz dumm. Und da sage mer so: En Dampfmaschin, dat is en grosset schwarzet Loch..." Nee, wem das so lieber ist, der ist bei mir verkehrt!--Nurmalschnell (Diskussion) 12:04, 27. Feb. 2017 (CET)Beantworten
Gabriel sagt nirgends, dass man vollständig auf PP verzichten soll. Er sagt, dass überall da, wo man es einsetzt, weniger Fragen gestellt werden, weniger hinterfragt wird und weniger Auslassungen erkannt werden. Kurz: mit Powerpoint scheinen Studenten zu verdummen. Dem kannst du nur entgegenwirken, indem du ihnen hin und wieder Powerpoint entziehst. Das mag durchaus mehr in den humanistischen Fächern erforderlich sein, aber auch in ganz nicht-humanistischen Fächern wie Ingenieurstudiengängen muss man irgendwann über Qualität, Sicherheit und Recht und Gesetz sprechen. Und da kann man vielleicht gut auf PP verzichten. Yotwen (Diskussion) 12:15, 27. Feb. 2017 (CET)Beantworten
Meiner Erfahrung nach eignet sich PP zwar recht gut für Vorträge, aber schlecht um damit zu lernen. Die wenigen Worte pro Folie eignen sich gut als Gedankenstütze für den Vortragenden und als Merkhilfe für die Zuhöhrer, aber es fehlt häufig an Erklärungen und Zusammenhängen für das spätere Lernen, die typischerweise mündlich dargeboten werden. Und Mitschreiben fällt schwerer, weil das Vortragstempo höher ist als wenn der Dozent nur an der Tafel arbeitet und selber schreibt.
Wie sinnvoll PP ist scheint mir durchaus vom Fach abhängig zu sein. In Mathe und Physik sind wohl Vorträge mit Tafel / ohne PP üblich, was sicher Gründe hat. In den ingenieurwissenschaftlichen Fächern die ich hatte, waren PP-Vorträge üblich, aber man möchte man ja schließlich ein Bild von der Maschine sehen um die es geht und nicht nur eine verbale Beschreibung oder Skizze an der Tafel. Die Vorlesungen die ich zu juristischen Themen hatte, kamen fast ganz ohne PP oder Tafel aus: Rein mündliche Vorträge und die Aufmerksamkeit war deutlich höher als sonst. --DWI (Diskussion) 15:14, 27. Feb. 2017 (CET)Beantworten
Folienpräsentationen sind vor allem ermüdend. Kennt man von Diavorträgen. Der Raum wird abgedunkelt und auf wundersame Weise entschlummert das Publikum. Heutzutage dank lichtstarker Projektoren nicht mehr ganz so ein Problem. Weiterhin neigen die Vortragenden dazu, viel zu viele Folien zu machen. Wurden an der Tafel pro Vorlesung zuvor vielleicht drei Tafelbilder (zu 2-4 Tafeln) gezeigt, sind es in der Folienform gleich 20 Folien oder gar mehr. Ich weiß nicht, wer den Lehrenden diesen Quatsch beigebracht hat, aber es gibt kaum etwas entnervenderes als wenn man als "Seher" durch zu schnelles Umblättern abgehängt wird. Da hilft es dann auch nicht, wenn nur ein einzelner Punkt pro Folie behandelt wird "den dann ja jeder verstanden haben müsste". Der Zusammenhang kann im Kopf nicht so schnell gebildet werden, wie der Vortragende ihn abspult. Man ist gefangen in einem Actionfilm, in dem man nicht zur Besinnung kommt. -- Janka (Diskussion) 15:49, 28. Feb. 2017 (CET)Beantworten


Auch wenn das heutzutage so gesehen wird: Das eigentliche Ziel der Uni ist nicht der "Job", sondern die Ausbildung zum Spezialisten. Darum gibt es "die" Strategie nicht. Lernen muss man, aber "lernen" ist auch nicht das einzige Ziel des Studiums. Du bist hier nicht in einer Fachhochschule. Es geht nicht so sehr darum, dass Du "gute" Noten hast (Was wichtig ist) als vielmehr darum, dass Du ein Wissensfeld und Kenntnisfeld abdeckst. Da musst Du Dich dann wirklich gut auskennen, bundesweit oder international. Geistiges Arbeiten erfordert nicht nur lesen (das meiste wirst Du exzerpieren), v.a. aber die Fähigkeit Standpunkte zu überdenken, zu recherchieren und intensiv geistig zu arbeiten (wie ein Mönch). --217.238.148.36 12:38, 27. Feb. 2017 (CET)Beantworten

Hier fehlt etwas: Der Überblick über das gesamte Fachgebiet und die Verzahnung ihrer Teile! Ich habe Chemie studiert und erst bei den Vorbereitungen zum Diplom so richtig "gelernt", wie alles zusammenhängt. Das erwartet z. B. ein auf Forschung ausgerichteter Arbeitgeber oder Doktorvater nach Abschluss des Studiums.
Außerdem "Was man von einem Studierten erwartet (oder verlangt), was aber kein 'Lehrstoff' irgendeiner Uni ist": Lösung einer Aufgabe, die gar nicht zum studierten Fach gehört. Mich hat mal eine Germanistik-Studentin in ihren 5. Semester Folgendes gefragt: "Wir machen ein Praktikum, das besteht darin, dass wir die Kloster-Dalheimer Musikfestspiele organisieren - ich bin für die Sicherheit verantwortlich und habe keine Ahnung, was ich dafür tun muss". Meine Auskunft: "Wenn ein Arbeitgeber einen Akadamiker mit so etwas konfrontiert, so erwartet er nach ein paar Tagen ein Konzept mit "Was muss erfüllt werden, was ist schon erfüllt, woran fehlt es, wer könnte das machen, was kostet das und wo nehmen wir das Geld dafür her". Am Anfang gibt es also nur ein leeres, weißes Papier und am Ende eine vortragsreife Antwort. Und dieser Vortrag sollte auch dann stotterfrei sein, wenn es 1000 Zuhörer gibt oder nur 3: Den Oberbürgermeister, den Kultusminister und den Hochschulrektor.
Lasst Euch nach einem langen Berufsleben sagen: Wer das am besten kann, der prägt sich seinem Abteilungsleiter als sein Nachfolger ein, denn als Vorgesetzer hat man jeden Tag so ein weißes Papier und das - nach wie vor wichtige - Fachwissen tritt immer mehr in den Hintergrund! Gruß -- Dr.cueppers - Disk. 14:18, 27. Feb. 2017 (CET)Beantworten

Als Dozent wird der Fragesteller nie allen Bedürfnissen und Lernertypen gleichzeitig gerecht werden. Variiere also die Didaktisierung deiner Lehrveranstaltungen; mach aber nichts, das du für Unfug hältst. Halte deine Studenten dazu an, verschiedene Strategien und Techniken auszuprobieren; wenn sie das eh nicht schon bis zum Abwinken in der Schule gemacht und ihr Ding gefunden haben. -- MonsieurRoi (Diskussion) 14:50, 27. Feb. 2017 (CET)Beantworten

Puh, oben wurde geschrieben: "Das eigentliche Ziel der Uni ist nicht der "Job", sondern die Ausbildung zum Spezialisten." Da muss ich aber heftig widersprechen. Das Ziel der Universität war noch nie Ausbildung, sondern stehts Bildung. Zum Glück haben wir im Deutschen dafür zwei Wörter. Der Philosoph Nida-Rümelin hat das einmal die Unterscheidung zwischen Humboldt und McKinsey genannt. Wilhelm von Humboldt schrieb selbst mal in einem Brief an den preussischen König: „Es gibt schlechterdings gewisse Kenntnisse, die allgemein sein müssen, und noch mehr eine gewisse Bildung der Gesinnungen und des Charakters, die keinem fehlen darf. Jeder ist offenbar nur dann ein guter Handwerker, Kaufmann, Soldat und Geschäftsmann, wenn er an sich und ohne Hinsicht auf seinen besonderen Beruf ein guter, anständiger, seinem Stande nach aufgeklärter Mensch und Bürger ist. Gibt ihm der Schulunterricht, was hierfür erforderlich ist, so erwirbt er die besondere Fähigkeit seines Berufs nachher so leicht und behält immer die Freiheit, wie im Leben so oft geschieht, von einem zum andern überzugehen.“ Ich glaube dies macht es sehr deutlich. Ähnlich äusserte sich John Stuart Mill: „Universitäten sind nicht da, um ein Wissen zu lehren, welches erforderlich ist, um zu einer bestimmten Art des Broderwerbs zu befähigen. Ihre Aufgabe ist es nicht, geschickte Rechtsgelehrte oder Aerzte oder Ingenieure zu bilden, sondern tüchtige und veredelte menschliche Wesen.“ Das bringt das Ziel der Universität sehr schön auf den Punkt. Für die Jobausbildung sind Universitäten jedenfalls nicht da. Das ist ein fundamentales Missverständnis und übersieht die Anstrengungen der aufgeklärten Bildung des Humanismus. 62.44.135.249 17:30, 27. Feb. 2017 (CET)Beantworten

Ich kenne den Namen des Dünkels nicht, den du da beschreibst, aber ich kann erkennen, dass es einer ist. Auch in einer Universität gibt es wirtschaftliche Zwänge. Die Forschung ist die Kirsche auf dem Sahnehäubchen der Bildung und die findet ihre Grundlage auf dem weichen Fundament einer möglichst breiten, nicht unmittelbar zweckbestimmten Ausbildung. Jede Universität muss diese Ausbildung anbieten, denn sonst kommt sie gar nicht auf den erforderlichen Umsatz, um die eigene Existenz zu rechtfertigen. Zudem kann keine Universität es sich wirtschaftlich leisten, allen Studenten die dazu befähigt wären die Weiterbeschäftigung im Wissenschaftsbetrieb zu ermöglichen. Das wären einfach zu viele, gesetzt, dass die Ausbildung an dieser Einrichtung sehr gut ist. Und umgekehrt kann die Masse der Studenten es sich auch nicht leisten, auf lange Sicht auf Einkommen zu verzichten. Über ein den eigenen Fähigkeiten entsprechendes Einkommen rede ich dabei noch nicht einmal.
Der wirklich schlimme Zustand ist ja eigentlich der, dass die mühsam erworbene Ausbildung (oder gar Bildung) inhaltlich später gar nicht abgerufen wird. Der spätere Arbeitgeber verknüpft mit einem bestimmten Abschluss die Fähigkeit, eigenständig Probleme zu lösen. Schön, gut und wichtig. Hier stellt sich mir aber die Frage, ob derjenige, der statt einer universitären Ausbildung eine zum Hochstapler genossen hat, nicht wesentlich besser fährt. Die Problemlösung sieht dann anders aus, dass das lange genug gut geht, haben wir in jüngster Vergangenheit immer wieder erfahren. -- Janka (Diskussion) 16:16, 28. Feb. 2017 (CET)Beantworten
Zum "eigentliche[n] Ziel der Uni" haben viele Leute eine eigene Meinung. Im Artikel Universität werden diese definiert als Hochschulen "die der Pflege und Entwicklung der Wissenschaften durch Forschung, Lehre und Studium dienen". Es ist also eine Mischung aus Forschungsinstitut und (Aus-)Bildungsstätte. Ob man am Ende des Studiums eher Spezialist oder Generalist ist, hängt vom Studiengang und den gewählten Schwerpunkten ab.
Janka, den "den Namen des Dünkels" angeht: Humboldtsches Bildungsideal. In der Praxis legen Leute die es vertreten werte auf (Schul-)fächer wie Deutsch, Fremdsprachen, Geschichte, Kunst und Musik wie man im humanistischen Gymnasium sieht, das von diesem Bildungsideal geprägt wurde. Ich seh das aber eher wie du. --DWI (Diskussion) 20:07, 28. Feb. 2017 (CET)Beantworten

Zurück zur Eingangsfrage: Was funktioniert, ist das Vor- und Nachbereiten von Veranstaltungen. Dabei geht es um die Optimierung der Wissensaufnahme und Speicherung. Optimale Wissensaufnahme ist bspw. gegeben, wenn der Student ~ 70 % des vorgestellten Inhaltes bereits kennt und ~ 30 % neue Informationen präsentiert bekommt. Zu viele neue Infos überfordern die Aufnahmekapazitäten, zu wenige führen zu abschweifenden Gedanken. Die Studenten sollten sich also grob ins Thema einlesen, um optimal von Studienveranstaltungen profitieren zu können. Damit sie das neu aufgenommene auch behalten, ist die Nachbereitung notwendig, sonst können sie später nicht auf das gespeicherte zugreifen. Ziel der Nachbereitung ist, Verständnislücken zu schließen und das neu aufgenommene möglichst intensiv mit dem schon bekannten zu verknüpfen. Hilfreich dabei ist, das neu aufgenommene zu hinterfragen, denn so kann man sich eine kritische Meinung bilden, was automatisch die selektive Bahnung im Gehirn fördert. Komplett abraten würde ich von auswendig lernen, denn das ist Lebenszeitverschwendung. Inhalte, die man auswendig lernen kann, kann man auch bei Bedarf nachschlagen, das ist wertlos. Im Zentrum des Lernens sollte immer das Verständnis, das gedankliche durchdringen der Materie stehen - zumindest an Hochschulen. Wem das zuviel ist, der sollte lieber eine Ausbildung machen. P.S.: Studien zu Deiner Frage findest Du, wenn Du nach der Neurobiologie des Lernens suchst. Das ist übrigens auch so ein Ding: Selber recherchieren macht schlau. Oder Du machst zu dem Thema ein Seminar und läßt das Deine Studenten als Hausaufgabe erledigen ...;) --88.69.253.77 01:28, 28. Feb. 2017 (CET)Beantworten

Internetseiten möglichst schnell aufbauen und Videos abspielen

Hallo, so ganz habe ich es noch nicht durchschaut. Ich möchte ein Notebook, welches zusammen mit einem angeschlossenen Monitor, Internetseiten möglichst schnell aufbaut und Videos im Browser möglichst schnell und gut anzeigt, auch bei mehreren Tabs. Auf was kommt es da hardwareseitig vor allem an? Ein schneller Prozessor? Viel Arbeitsspeicher? Welchen Einfluss hat eine Grafikkarte darauf? --Eibbloff (Diskussion) 19:26, 26. Feb. 2017 (CET)Beantworten

.....es kommt auf ein schnelles Internet an.--2003:75:AF07:FF00:4913:10E3:C308:CB52 19:38, 26. Feb. 2017 (CET)Beantworten
Es kommt immer auf das schwächste Glied in der Kette an. Dies kann auch die Internetanbindung sein. --FriedhelmW (Diskussion) 19:41, 26. Feb. 2017 (CET)Beantworten
Serverseitig: Schau Dir diese Links an. --Hans Haase (有问题吗) 19:52, 26. Feb. 2017 (CET)Beantworten
Ram ist hier sehr wichtig. Browser schlucken pro Tab erstaunliche viel davon. Wenn du was neues kaufst fang nicht unter 8 GB an. Vier sind jetzt schon knapp, bei fortschreitender Entwicklung werden Sie in zwei Jahren zu knapp sein. --2003:76:4E5B:795A:B115:CDF5:8A85:7198 13:01, 27. Feb. 2017 (CET)Beantworten
Zu Notebooks und kompakten Computern: Der einzige, der im Moment 4K-Videos in Basisausstattung abspielen kann, ist die intel-System-on-a-Chip-CPU J3455 und ihr großer Bruder J4205. Diese wurden klammheimlich auf den Markt geworfen und sind im Moment tendenziell im Preis am steigen. Denselben Trend machen Arbeitsspeicher derzeit mit. Von der CPU sind die nichts besonderes, außer, dass sie wenig strom benötigen und passive Kühlung üblich sein sollte. 8 GB RAM und eine SSD dazu machen den mechanikfreien Computer. --Hans Haase (有问题吗) 15:11, 27. Feb. 2017 (CET)Beantworten
So ziemlich jeder neue Laptop mit diskreter Grafikkarte sollte 4K-Videos flüssig abspielen können. Das ist auch zukunftssicherer als jede APU- oder System-on-a-Chip-basierende Lösung, es steht ja z.B. schon 5K vor der Tür. Zwar kann kaum ein Laptop-Display 4K anzeigen (allerdings ist mindestens eines der wenigen Modelle mit 4K und dedizierter Grafikkarte gar nicht mal so extrem teuer), jedoch schauen 4K-videos meist auch runterskaliert besser aus, weil sie auch eine höhere Datenrate haben und die durch Kompression erzeugten Blöcke auch kleiner sind.
Was den Aufbau von Webseiten betrifft: am wichtigsten ist natürlich die Internetverbindung, aber die CPU-Geschwindiglkeit kann durchaus einen unterschied machen, vor Allem wenn man mehrere Browser-Tabs geöffnet hat, besonders schwachbrüstige CPUs wie z.B. alles, was auf Intel Atom basiert, machen sich aber bei entsprechend komplexen Webseiten schon bei einem einzigen Tab bemerkbar.
Arbeitsspeichermäßig reichen jedenfalls 8 GB, falls nur der Browser läuft, sonst sollte man sicherheitshalber 16GB nehmen, ev. per Upgrade (Arbeitsspeicher ist heute ja nicht mehr teuer). --MrBurns (Diskussion) 14:11, 1. Mär. 2017 (CET)Beantworten

Nobilitierung

Wie ist es korrekter, oder üblicher? Wenn ein Mensch, z.B. ein Wissenschaftler, im fortgeschrittenen Alter für seine Verdienste nobilitiert wurde, und man in Textstellen über Ereignisse auf ihn Bezug nimmt, die zeitlich (noch lange) vor seiner Nobilitierung liegen, sollte man dann besser "Max von Mustermann" schreiben (d.h. sein Name, wie er auf dem Grabstein steht), oder besser "Max Mustermann", weil er zum betreffenden Zeitpunkt eben so hieß und noch nicht nobilitiert war? (Es geht nicht um Wikipedia, sondern um einen wissenschaftlichen Text, allerdings keine Biografie über Mustermann, sondern einen beliebigen Textabschnitt, der Mustermann erwähnt.) Ich finde Argumente für beide Varianten und suche eine Entscheidungshilfe, welche ich verwenden soll. --Neitram  19:43, 26. Feb. 2017 (CET)Beantworten

Hm, ich würde behaupten wollen, dass der Werther von Goethe verfasst wurde, und nicht etwa nicht von von Goethe, aber Wikipedia sieht das anders...--Edith Wahr (Diskussion) 20:48, 26. Feb. 2017 (CET)Beantworten
ich glaube, dass in deiner Antwort ein 'nicht' zuviel ist! 79.224.220.217 21:43, 26. Feb. 2017 (CET)Beantworten
WP vertritt an vielen Stellen den "Was er/sie am Ende war, wird genommen."-Ansatz (Heute: Der schwedische Komponist und Musikkritiker Wilhelm Peterson-Berger kommt zur Welt. ... und fängt gleich an zu komponieren?). Auch bei später verheirateten Frauen machen sie ihr Abitur als Verheiratete. Ich fände Differenzierung besser (in Marie Curie ganz gut durchexerziert), aber der Grabstein-Ansatz scheint für die meisten einfacher zu sein (man muss weniger aufpassen...). GEEZER … nil nisi bene 08:13, 27. Feb. 2017 (CET)Beantworten
Ist durchaus auch Kontext abhängig. Beispiel im Artikel von X.Y. Dann ist sinnvoll zu schrieben. „Maria Müller war die Jugendfreundin von X.Y.“. auch wenn sie später geheiratet hat und als Maria Bäcker begraben wurde (gerade wenn die Verbindung zu X.Y. mit der Heirat gekappt wurde). Ist Maria selber relevant, ist die Frage eher unter welchem Namen man sie kennt. Wenn sie weiter hin unter ihrem Jugendnahmen Bücher verfasst hat, dann Maria Müller. Schrieb sie aber ihre Bücher nach der Heirat unter Maria Bäcker, dann Maria Bäcker selbst dann wenn ihr Meisterwerk unter Maria Müller veröffentlicht wurde. Das jedenfals meine Meinung dazu. --Bobo11 (Diskussion) 08:24, 27. Feb. 2017 (CET)Beantworten
Guter Punkt, genau die gleiche Frage habe ich nämlich ebenfalls bei Frauen und Ereignissen, die vor ihrer Ehe (und der damit verbundenen Namensänderung) liegen. Es widerstrebt mir ehrlich gesagt innerlich, den Ehenamen der Frau zu verwenden, wenn es um ein Ereignis in ihrer Kindheit geht, oder z.B. um ihre Verlobung. An sich ist es ganz analog mit Nobilitierungen. --Neitram  15:39, 27. Feb. 2017 (CET)Beantworten

Das Problem haben wir ständig: Gebäude, die umbenannt werden, Stadien inzwischen z.B. gefühlt jährlich, Menschen, die ihr Geschlecht ändern, Städte, die auf einmal zu anderen Ländern gehören und sich anders schreiben. Eine wirklich stringente Linie haben wir aber in keinem der Fälle. --Studmult (Diskussion) 12:31, 27. Feb. 2017 (CET)Beantworten

Bei Orten und Straßen schreibe ich in meinem Text gerne Klammerzusätze, etwa "Er wurde am 1. Mai 1844 in Oberdietzbach geboren (der Ort wurde 1935 nach München eingemeindet) oder "Er zog in die Gartenstraße 20 (heute Schlossallee 44)". Bei Frauen verwende ich, wo immer ich es im Kontext korrekt empfinde, ihren Mädchennamen, und bei später Nobilitierten würde ich es eigentlich vorzugsweise auch so wie bei den Frauen halten wollen. --Neitram  17:07, 27. Feb. 2017 (CET)Beantworten

Nobilitierung? Ist in D eigentlich nicht üblich. Nobilitierung erinnert schwer an den Lateinuntericht. ... :-) Und sie fällt auf! Und zwar negativ. --80.187.119.44 21:59, 27. Feb. 2017 (CET)Beantworten

Jaja - wer seinen Artikel selbst erstellt oder dahintersteckt: Ich schrieb schon, sie fällt auf, nicht positiv und eben eigentlich nur bei der "Zielgruppe" :-) (Hier macht aber eh jeder was er will) --80.187.119.44 22:01, 27. Feb. 2017 (CET)Beantworten
Hier ist die WP:Auskunft, bitte weitergehen zum WP:Café. --Neitram  10:59, 28. Feb. 2017 (CET)Beantworten

Ich tendiere auch dazu, den Nobilitierungszeitpunkt zu beachten. Meine „gefühlte“ Beobachtung: Je „berühmter“ jemand ist, umso eher wird auf das „von“ oder den „Freiherrn“ verzichtet. Beispiel: Müller wurde kurz vor seinem Tode geadelt, also heißt die Straße später nach ihm todsicher „Von-Müller-Straße“; gibt es auch eine „Von-Schiller“-oder „Von-Goethe“-Straße? --Dioskorides (Diskussion) 14:52, 28. Feb. 2017 (CET)Beantworten

E-Mail-Testsystem ohne echte Funktion

Ich bin auf der Suche nach einem quasi E-Mail-Testsystem zum Erlernen des Umgangs mit E-Mail. Quasi mit Oberfläche zum E-Mails senden und E-Mails empfangen, wobei das aber nur pseudomäßig erfolgen soll und E-Mails quasi nur intern als Nachricht weitergeleitet werden soll. Da ich hierfür weder Webserver noch Domain zur Verfügung habe, sollte das also irgendwie lokal im eigenen Netz erfolgen. Hat da jemand eine Idee? (nicht signierter Beitrag von Quedel (Diskussion | Beiträge) 21:13, 26. Feb. 2017 (CET))Beantworten

In einem Linux-System kannst du dir deinen eigenen Mail-Server einrichten. --FriedhelmW (Diskussion) 21:22, 26. Feb. 2017 (CET)Beantworten
Du kannst Dir dazu einfach ein paar Adressen bei einem Kostenlosen (werbefinanzierten) Anbieter einrichten. oder Dein Provider erlaubt dir die Einrichtung mehrerer E-Mail-Adressen (bei mir sind 25 Adressen in 5 verschiedenen Postfächern möglich). --212.183.52.150 21:55, 26. Feb. 2017 (CET)Beantworten
Postfix oder Sendmail --Hans Haase (有问题吗) 22:03, 26. Feb. 2017 (CET)Beantworten
In einem Windows-System geht das genauso gut, beispielsweise mit Hamster oder Jana-Server. --Kreuzschnabel 10:49, 1. Mär. 2017 (CET)Beantworten
Warum alles so kompliziert? Ein kostenloser Account bei GMX und ein weiterer kostenloser Account bei web.de und bei Bedarf ein kostenloses Thunderbird auf dem eigenen Computer einrichten und ab geht die Post. --80.130.225.92 06:21, 27. Feb. 2017 (CET)Beantworten
Da es zum Austesten ist, sollte das System nicht wirklich aktiv sein, außerdem sprechen datenschutzrechtliche Gründe gegen Postfächer bei Web.de/GMX u.ä. Das System soll ja durch mehrere Kinder aktiv genutzt werden, halt aber nur lokal bzw. auf eigenem Webspace. --Quedel Disk 10:51, 27. Feb. 2017 (CET)Beantworten
Unter en:Comparison of mail servers findest Du eine Vergleichstabelle verschiedener Mailserver, auch Opensource. Vielleicht ist da etwas passendes dabei. Es gibt einen Windows IIS SMTP-Server, der aber nur bei den Serverversionen von Windows dabei ist, nicht beim abgespeckten IIS von Windows Professional/Pro/Ultimate. --Rôtkæppchen₆₈ 11:11, 27. Feb. 2017 (CET)Beantworten
Die Frage war: „ohne echte Funktion“, also ein Mailer im Intranet, der nicht nach außen austauscht. Das bedeutet aufs reale Leben bezogen: funktionsfähige Briefkästen bei streikenden Zustellern. Es geht wohl um das testen von Funktionen, die einen Mailserver bedienen. --Hans Haase (有问题吗) 15:03, 27. Feb. 2017 (CET)Beantworten
minimehl.de --Rôtkæppchen₆₈ 00:18, 28. Feb. 2017 (CET)Beantworten

27. Februar 2017

Alternativprogramm in der Provence

Wir sind gerade in Saint Vincent les Forts und morgen geht es micht zum vol libre. Gibt es hier Schlechtwetterprogramm wie Schwimmbäder etc. im Umkreis? --109.40.1.42 17:11, 27. Feb. 2017 (CET)Beantworten

Voi­là. --Komischn (Diskussion) 18:08, 27. Feb. 2017 (CET)Beantworten
Im Umkreis von dort wo du jetzt gerade bist? Sicher, schau unter https://www.dortmund-tourismus.de/home.html --King Rk (Diskussion) 18:44, 27. Feb. 2017 (CET)Beantworten
Wow, bist du cool. Was weiß ich, wo meine Mobil IP hinroamt? Gruß aus mittlerweile Saint André les Alps. 109.40.3.25 00:56, 28. Feb. 2017 (CET)Beantworten
Ok, sorry, von IP-Roaming wusste ich bisher noch nichts. Ich hätte auch nett fragen können. --King Rk (Diskussion) 05:48, 28. Feb. 2017 (CET)Beantworten
Es gibt in der Provence an einigen Stellen Höhlen, die man besichtigen kann, die auch bei Regenwetter schön sind. --Sr. F (Diskussion) 16:59, 28. Feb. 2017 (CET)Beantworten

Was ist Bossing ?

Wenn angeblich alle Mitarbeiter leiden, ist das dann Bossing und was ist, wenn der auch leidet, vielleicht unter den Mitarbeitern oder anderen Vorgesetzten. Kann man da was machen ausser dicke Haut zulegen ?Urlaub nehmen, doppelt so viel arbeiten, die Kollegen zu einem Eis einladen, trösten finde ich zu persönlich (oft sind die Leute ja selbst beteiligt ), krank melden, an was anderes denken ?

--Nina Eger (Diskussion) 18:48, 27. Feb. 2017 (CET)Beantworten

Der Duden bezeichnet Bossing als Schikanieren (nur) einzelner Mitarbeiter zum Zweck aus der Firma zu ekeln. Was du da umschreibst ist eher (übermäßiger) Leistungsdruck in der ganzen Firma. Ich nehme mal an, es handelt sich um einen Kleinbetrieb, sonst wäre der Betriebsrat die richtige Anlaufstelle. Miteinander darüber reden ist immer gut, wenn aber der Druck "von oben" kommt, muss mit diesem oben geredet werden. Alles andere, wie dickes Fell, krank melden ist ungeeignet--2003:75:AF0D:2800:22:1927:5543:1FCD 19:23, 27. Feb. 2017 (CET)Beantworten
Vereinfacht gesagt: Der mobbende Chef/Vorgesetzte. Bossing wird benutzt, um Mitarbeiter zur Kündigung zu bewegen. Der häufigste Grund ist das Vermeiden von Abfindungen, die ab einer gewissen Unternehmenszugehörigkeit fällig werden. --Hans Haase (有问题吗) 20:09, 27. Feb. 2017 (CET)Beantworten
+1. Mobbing = Quälerei auf der Horizontalen (durch Kollegen = Konkurrenten), Bossing = Quälerei in der Vertikalen (von oben nach unten); jeweils mit Schwerpunkt gegen eine einzelne Person, die weggeekelt werden soll. Dazu die alte Weisheit: Der wahre Charakter eines Mensch zeigt sich daran, wie er/sie Untergebene behandelt. -- Zerolevel (Diskussion) 16:03, 1. Mär. 2017 (CET)Beantworten

Krieg zwischen Demokratien

Ziemlich oft stößt man auf die Behauptung, zwei demokratische Staaten hätten noch niemals die Waffen gegeneinander aufgenommen (so ich gestern in einem Philosophie-Kalender). Ohne geringste Verzögerung ist mir sofort der Krieg zwischen den demokratischen Republiken USA und CSA eingefallen – aber dann wurde es schon dünner. Südamerika rund um 1900 drängt sich irgendwie rein, aber mir fällt es schwer, sofort die Demokratien von den Nicht-ganz-so-lupenrein-Demokratien zu unterscheiden … Wer kann die Liste erweitern? Kann man sie vielleicht sogar vervollständigen, kann man alle zwischen demokratischen Staaten geführten Kriege listen? --85.178.126.152 19:07, 27. Feb. 2017 (CET)Beantworten

Krieg der USA gegen Mexiko, Irakkrieg, Koreakrieg, Vietnamkrieg, 1. und 2. WK. - kommt auf den eigenen politischen Standpunkt an, da ja alle diese kriegsbeteiligten je nach Standpunkt als "demokratisch" oder "undemokratisch" bezeichnet werden. Selbst bei den USA ist diese Frage letztlich offen, da letztich nicht der "demos" die Macht "krateia" hat, sondern eine Oligarchie vorliegt. andy_king50 (Diskussion) 19:12, 27. Feb. 2017 (CET)Beantworten
Die "lateinischen Demokratien Amerikas" "um 1900" waren doch keine Demokratien! Republiken ja, einige hatten sogar schon allgemeines Wahlrecht, aber es herrschte stets eine kleine aristokratische Elite. Auch bezweifle ich das die CSA als "Demokratie" betrachtet werden können, allein schon wegen ihrer Sklaverei, und inwieweit das Wahlrecht in Südstaaten schon demokratisiert war müsste ich mal recherchieren. Also ich bin noch auf keinen Fall gekommen... Manche sehen die Grenzzusammenstöße zwischen Peru und Ecuador um 1980 oder die ersten indisch-pakistanischen Krieg als Gegenbeispiele. Peru, Ecuador und Pakistan würde ich aber jeweils den Status als Demokratie nicht zusprechen. Wirklich schon länger demokratische Staaten gibt es nur wenige, da fällt eine Einzelbetrachtung nicht zu schwer.--Antemister (Diskussion) 21:01, 27. Feb. 2017 (CET)Beantworten
genau das ist der Knackpunkt. "Demokratie" ist ein sehr vager Begriff. Die Attische Demokratie, vermeintlich das plakative "Vorbild" für diese, würde heute allenfalls als Oligarchie/Plutokratie durchgehen, da das "demos" eine geringe Zahl wohlhabender Männer war (Fremde, Sklaven, Frauen, Habenichtse, nicht wehrfähige und so ein Gesox: nö danke = nix demos = nix Mitsprache) - andy_king50 (Diskussion) 22:22, 27. Feb. 2017 (CET)Beantworten

Lese mal hier [[1]] und sehe beim ZDF hier über YT [[2]]. Zu dieser Planung (Operation oder Planung RED) gabs noch in diesem Jahr Einträge in en und dt Wiki. Heute finde ich sie nicht mehr. :-) --80.187.104.249 21:28, 27. Feb. 2017 (CET)Beantworten

Die Vorstellung geht auf die Schrift Zum ewigen Frieden von Immanuel Kant zurück. In der Tat fällt mir kein (tatsächlich militärisch geführter) Krieg zwischen zwei Staaten ein, die sich als „demokratisch“ im Sinne des Ideals der philosophischen Aufklärung verstanden haben. Der Sezessionskrieg („civil war“!) fällt sicherlich nicht hierunter, ebensowenig Kriege zwischen demokratischen Staaten in diesem Sinne und Staaten mit anderen Regierungskonzepten (Volksdemokratien) oder Wirtschaftskriege (die es reichlich auch zwischen „westlichen“ Demokratien gibt − und geben wird). --Melekeok (Diskussion) 23:33, 27. Feb. 2017 (CET)Beantworten
Das Demokratien aggressiv Kriege gegen Diktaturen führen ist als Nachsatz zu obiger Behauptung Allgemeingut. Wirtschaftskriege sind keine Kriege i. eig. Sinn. Die sog. Kabeljaukriege fallen in dieses Muster.--Antemister (Diskussion) 23:40, 27. Feb. 2017 (CET)Beantworten

Die englische Wikipedia hat sogar einen Artikel dazu: https://en.wikipedia.org/wiki/List_of_wars_between_democracies .--Alexmagnus Fragen? 00:44, 28. Feb. 2017 (CET)Beantworten

Man kann es natürlich immer so hindrehen, dass einer der Kontrahenten keine Demokratie ist. Sind die ganzen Monarchien in Europa wirklich Demokratien? Im Vereinigten Königreich z.B. könnte der Monarch das Parlament einfach auflösen und nach Hause schicken. Macht er nur nicht, weil das Volk ziemlich sicher dagegen ist und das im Endeffekt zur Abschaffung der Monarchie führen würde (und weil er keinen Grund dazu hat). In jeder Monarchie kann das Staatsoberhaupt nicht vom Volke bestimmt werden, nichtmal indirekt. Das wird immer der Zausel, der altersmäßig gerade dran ist.
Die südamerikanischen Staaten waren zeitweise immer mal wieder Millitärdiktaturen. Ebenso die afrikanischen Staaten. Die osteuropäischen Staaten sind ja erst seit kurzer Zeit Demokratien. Es gibt also nur relativ wenige Demokratien, sie bestehen meistens erst seit kurzer Zeit, die Chance, das zwei davon Krieg führen ist also relativ gering. --Optimum (Diskussion) 02:02, 28. Feb. 2017 (CET)Beantworten
Gott! Trump ist erst seit fünf Wochen im Amt. Gebt ihm doch ein bisschen Zeit. Er wird die Theorie auch noch kippen. Yotwen (Diskussion) 07:29, 28. Feb. 2017 (CET)Beantworten
Woher kommt denn die Behauptung aus der Ausgangsfrage?--Wikiseidank (Diskussion) 10:15, 28. Feb. 2017 (CET)Beantworten
Dean Babst, 1963, siehe Demokratischer Frieden. --Neitram  11:13, 28. Feb. 2017 (CET)Beantworten
Der englische Artikel legt den Begriff dabei aber sehr weit aus, in dem er alle Länder mit gewählter Regierung (sind heute die meisten, aber das ist noch nicht lange so) als "Demokratien" betrachtet. Dem würden viele nicht zustimmen. Interessant finde ich in Liste aber: Die Burenkriege!--Antemister (Diskussion) 18:58, 28. Feb. 2017 (CET)Beantworten
Dem könnte ich folgen, solange es eine wirksame Regelung (=|= Putsch / Revolte etc) zur Ablösung der jeweils abgelaufenen Regierung gibt. Manche Quasi-Diktatoren argumentieren: Einmal demokratisch gewählt ist mir genug Demokratie bis an mein Lebensende. -- Zerolevel (Diskussion) 15:58, 1. Mär. 2017 (CET)Beantworten

Begriff „Flüchtlingskrise“

Flüchtlingskrise ist das Anfangswort mehrerer einschlägiger Lemmata; jedoch halte ich diesen Begriff, auch wenn er in aller Munde ist, für nicht unproblematisch, da er impliziert, dass die Flüchtlinge selbst, d. h. als Personen, eine Krise darstellen. Auf den Diskussionsseiten der Artikel Flüchtlingskrise in Europa ab 2015 und Flüchtlingskrise in Deutschland ab 2015 habe ich ebenfalls bereits darauf hingewiesen, allerdings bislang nur eine (ablehnende) Rückmeldung zu meinem dort geäußerten Vorschlag erhalten, die Lemmata jeweils mit der abstrakteren Bezeichnung Migrationskrise statt Flüchtlingskrise beginnen zu lassen und die alten Lemmata als Weiterleitungen beizubehalten. Zur Begründung: Man denke zum Vergleich bspw. an einen analogen Ausdruck wie Studentenkrise, mit das Problem der Überlastung der Universitäten bezeichnet werden könnte. Dabei sind ja nicht die Studierenden selbst daran schuld, sondern in Wahrheit ein Versagen der Gesellschaft, da nicht-akademische Berufe aufgrund vielfältiger Faktoren offenbar immer mehr an Attraktivität verlieren. Ich hoffe, mein Anliegen ist durch diesen zugegeben nur behelfsmäßigen Vergleich womöglich etwas verständlicher geworden. Wie seht ihr das?--Hubon (Diskussion) 19:08, 27. Feb. 2017 (CET)Beantworten

willkommen in der Euphemismus-Tretmühle - eine euphemisierende Namensänderung bei fortbestehendem Problem hat dieses noch nie gelöst. - andy_king50 (Diskussion) 19:18, 27. Feb. 2017 (CET)Beantworten
Hubon hat durchaus recht, dass »Flüchtlingskrise« kein neutraler Begriff ist. Allerdings hat er sich etabliert. So etwas kommt häufiger vor und färbt natürlich die Wahrnehmung der Situation. Wer erfolgreich einen Begriff prägt, setzt auch die Agenda. Rainer Z ... 19:28, 27. Feb. 2017 (CET)Beantworten
Dankeschön! @andy_king50: Es geht mir keineswegs, wie von dir unterstellt, um Euphemismus. Und ich bin auch gewiss nicht der Meinung, dass durch ein anderes Wort ein politisches Problem gelöst werden kann! Vielmehr geht es mir einfach darum, dem Kind einen inhaltlich halbwegs passenden Namen zu geben. Und ich finde übrigens nicht unbedingt, dass wir da gezwungenermaßen einfach das nachplappern müssen, was Politik und Medien aufgrund ihrer systembedingt ohnehin leider allzu oft verkürzten Sprache kommunizieren – für eine Enzyklopädie, so glaube ich, sollten da doch etwas höhere Standards gelten, auch was eine möglichst objektive bzw. politisch neutrale Begriffswahl anbelangt... Kollegial--Hubon (Diskussion) 21:05, 27. Feb. 2017 (CET)Beantworten
genau das habe ich gemeint: egal welche "Umbenennungsversuche" aus der eigenen offensichtlich einseitiger polischer Sicht aus vermeintlich "einzige neutraler" Sicht erfolgen, sie sind vergeblich. sie werdeb stets aufs Neue vom Sprachbrauch verworfen. Das was da abläuft ist allenfalls eine brutale,geradezu kriminelle Ausnutzung der Schwäche des deutschen politischen Sytems gegen alle Deutschen, Europäer und im Übrigen auch der integrierten Türken, Italiener etc. Da kommt eine Masse illegal und rechtswidrig ins Land und soll dann noch Rechte genießen ???? andy_king50 (Diskussion) 22:10, 27. Feb. 2017 (CET)Beantworten

Hallo! Also es ist nicht hilfreich, wenn mit einer bereits feststehenden Meinung an ein Problem heranzugehen. Denn Deine Logik ist schlicht Unfug Hubon! Denn genausowenig ist der Speisenwert an der Nahrungsmittelpreiskrise 2007–2008 Schuld, oder das Öl an der Ölkrise/Ölpreiskrise. Es sind immer Qualifikatoren, welche umgangsprachlich wie fachsprachlich am besten ein Problem bezeichnen. Und ohne das ich AFD-Anhänger bin, stimmt es eben nicht, daß die Flüchtlinge nicht die Krise darstellen. Deren mehrfachen illegale Grenzübertritte haben doch erst eine Sitution herbeigeführt, welche zu den Merkmalen der Krise führte, welche eben nicht aus deren Zahl weltweit oder den Gründen für die Flucht beruhte, sondern der unregulierten Einreise ohne staatliche Kontrolle, die das reine Versorgungsproblem mit einem Verwaltungs und Justizproblem verbanden. Darum ist der Begriff doch für die meisten Deutschsprachigen sehr wohl zutreffend, wenn es um die Ereignisse 2015/16 geht. Und es ist eben keine "Migration" gewesen, weder illegal noch legal! Zur Migration wird es erst durch die staatlichen Genehmigung des Aufenthalts, wobei manche davon auch strikt die Einwanderung zum Daueraufenthalt unterscheiden wollen. Wenn man vermeintlich neutrale Begriffe will, sollte man zumindest die juristische Fachsprache als Grundlage akzeptieren. Und da ist Migration in der Deutschen Sprache wie Amtssprache unbestimmter als Flüchtling.Oliver S.Y. (Diskussion) 22:20, 27. Feb. 2017 (CET)Beantworten

Die Frage hatten "wir" hier genau so, als das Lemma 2015 immer öfter hochkam (Archiv), auch im Bezug zur Bankenkrise und den zuvor genannten "Krisen".--Wikiseidank (Diskussion) 10:14, 28. Feb. 2017 (CET)Beantworten

Nach WK II gab es in Deutschland Flüchlinge und Vertriebene, sowohl gemeinsprachlich als auch amtlich definiert. Gemäß dermn individuellem Schicksal passte das auch nicht immer: Manche „Vertriebene“ waren eigentlich geflohen und für die andere Kombination gibt's wahrscheinlich auch passende Beispiele. --Dioskorides (Diskussion) 14:46, 28. Feb. 2017 (CET)Beantworten

Es gibt auch einen prominenten Fall, wo sich eine von einer Dienstreise in die damaligen deutsch besetzten Gebiete heimgekehrte Person als vertrieben bezeichnet. --Rôtkæppchen₆₈ 16:05, 28. Feb. 2017 (CET)Beantworten

Kletterrosen

Meine Kletterrosen am Rosenbogen sind die ersten zwei Jahre normal gewachsen und geblüht. Plötzlich haben sie nur mehr kleine weiße Blüten bekommen und nach dem Abblühen rote so ovale Beeren bekommen. Was muss ich tun das sie wieder blühen wie am Anfang ? (nicht signierter Beitrag von 62.93.107.245 (Diskussion) 19:23, 27. Feb. 2017 (CET))Beantworten

Die roten Beeren sind wohl Hagebutten. Wegen der Blüten - Hast du die Rosen beschnitten? ev. sogar sehr stark? Dann kann es passiert sein, dass Du die veredelte Rose abgeschnitten hast und dann die Unterlage ausgetrieben und geblüht hat. --Elrond (Diskussion) 19:43, 27. Feb. 2017 (CET)Beantworten
Frage: nur kleinere Blüten aber sonst wie vorher gestaltet, oder aber einfache, ungefüllte wie bei Wildrosen? Ggf hast Du beim Schnitt die "gesunden, wüchsigen Triebe" = Wildtriebe belassen und weniger attraktiven Triebe der Veredelung rausgeschnitten ? - andy_king50 (Diskussion) 19:51, 27. Feb. 2017 (CET)Beantworten

Das wäre eigentlich dann ganz gut, denn diese Form von Rosen wäre "natürlicher". Oder? --ObersterGenosse (Diskussion) 22:27, 27. Feb. 2017 (CET)Beantworten

Falsch beschnitten, die Rose ist verwildert. --M@rcela 05:52, 28. Feb. 2017 (CET)Beantworten
Die Frage ist doch: "Was muss ich tun das sie wieder blühen wie am Anfang ?". Das ist einfach: Finde alle Triebe die noch so blühen wie gewünscht und schneide alle anderen weg. Falls keine mehr zu finden sind: Neue Rose kaufen (oder sich an die aktuelle Variante gewöhnen). Ggf. noch eine Anleitung zum Schnitt von Kletterrosen besorgen und verstehen. Grüße --RalfDA (Diskussion) 18:52, 28. Feb. 2017 (CET)Beantworten
Sehr stark vereinfacht: Triebe, die aus dem Boden kommen, sind wild. Sie haben meist 7 Blätter, die Triebe der Zuchtrose 5 Blätter. Allerdings ist das nicht dogmatisch, es gibt seit einigen Jahrzehnten auch Sorten mit 7 oder mehr Blättern. Wilde Triebe wachsen sehr schnell, sind oft gerade und damit sehr lang. --M@rcela 18:58, 28. Feb. 2017 (CET)Beantworten

Edelrosen werden meist auf Hundsrosen okuliert. Sehen die Blüten so oder so ähnlich aus? Oder so? --Elrond (Diskussion) 19:42, 28. Feb. 2017 (CET)Beantworten

Ist die Teilnahme an Russisch Roulette legal?

Frage bezieht sich auf Deutschland, aber Antorten für andere Länder sind auch interessant. Zur Sicherheit: Nein, ich habe nichts dergleichen vor :) --89.246.169.72 20:03, 27. Feb. 2017 (CET)Beantworten

Eigentlich eine interessante Frage. Nur: Falls nicht legal. Mit welcher Strafandrohung willst Du dies sanktionieren, bei jemand, der bereit ist, dieses Spiel zu spielen? :-) Du liest: Die Frage fällt in sich selbst zusammen. --80.187.118.11 20:45, 27. Feb. 2017 (CET)Beantworten
Regelt das Waffengesetz (Deutschland) Kurz: Nur mit Wasserpistole oder auf dem Schießstand. Stichwort "Führen einer Waffe" Graf Umarov (Diskussion) 20:59, 27. Feb. 2017 (CET)Beantworten
Hm. Es war nicht danach gefragt, ob und wo die Waffe rechtmäßig geführt wird. Das ist also Unsinn. Mal sehn: Gegen Tote wird ja ohnehin nicht ermittelt. Es geht also nur um den oder die Gewinner und/oder um die mögliche Strafbarkeit des Versuchs. Und ob da irgendwo ein Delikt vorliegt. Offenbar ist Russisches Roulette – bei allen hier angebrachten Vorbehalten (keine Rechtsberatung!) – nicht strafbar. Ich lese in: Robert Esser, Volker Krey: Deutsches Strafrecht, Allgemeiner Teil, Kohlhammer Verlag, Stuttgart, 6, Aufl. 2016, ISBN 978317029882 unter: „§ 11 Objektiver Tatbestand, IV. Lehre von der objektiven Zurechnung, 4. Handeln des Opfers auf eigene Gefahr “ als Spezialfall des Ausschlusses der objektiven Zurechnung, a) Teilnahme an einer freiverantwortlichen Selbstgefährdung des Opfers, (2) Straflosigkeit der Beteiligung an der freiverantwortlichen Selbstgefährdung eines anderen, Fall 56 – Russisches Roulette Nr.1 –“ ein Zitat aus einem BGH-Urteil, das ich auf die Schnelle nicht recherchieren kann (und vielleicht angesichts der seriösen Quelle auch nicht muss): „Eigenverantwortlich gewollte und verwirklichte Selbstgefährdung unterfallen nicht dem Tatbestand eines Körperverletzungs- oder Tötungsdelikts, wenn das mit der Gefährdung bewusst eingegangene Risiko sich realisiert. Wer lediglich eine solche Selbstgefährdung veranlasst, ermöglicht oder fördert, macht sich nicht wegen eines Körperverletzungs- oder Tötungsdelikts strafbar.“ --80.130.225.92 21:22, 27. Feb. 2017 (CET)Beantworten
in den U.S.A wurde 1946 jemand verurteilt, weil er bei einer Variante („Russisch Poker“) einen Freund tötete: en:Russian roulette... --Heimschützenzentrum (?) 21:19, 27. Feb. 2017 (CET)Beantworten

Also entweder bist Du nach der Teilnahme tot oder du wirst so in die Mangel genommen, dass Du über aller juristischen Theoriefuxereien und (zumindest bei Nichtjuristen) allfälliger moralischer Probleme hinaus sowas sicher sowas nie wieder machst. Die Nachweislast, dass es sich nicht um einen Mord handelt, liegt dann wohl allein bei Dir. Viel Spaß beim juristisch wasserdichten Beweis. 22:16, 27. Feb. 2017 (CET) (ohne Name signierter Beitrag von Andy king50 (Diskussion | Beiträge))

Aus moralischen oder ethischen Gründen kann viel passieren. Aber der Staat muss euch erstmal beweisen, dass du ein Mörder bist! Siehe Unschuldsvermutung oder in dubio pro reo. Auch Eigenverantwortliche Selbstgefährdung (auf die im Rahmen eines AT-Lehrbuchs oben schon verwiesen wurde), aber auch Disponibilität oder Rechtsgut könnten, ebenso wie Tötung auf Verlangen interessant sein! --ObersterGenosse (Diskussion) 22:30, 27. Feb. 2017 (CET)Beantworten

dann wäre es besser, es wären immer alle Kammern geladen, und das Gesox würde uns damit solche theoretischen Eventualtitäten von vorn herein ersparen. - andy_king50 (Diskussion) 22:35, 27. Feb. 2017 (CET)Beantworten
@Andy king50, (Beitrag 22:16, 27. Feb. 2017): Das ist gequirlter Unsinn. Mord setzt Vorsatz und mindestens ein sogenanntes Mordmerkmal voraus. (§ 211 Abs. 2 StGB: „Mörder ist, wer aus Mordlust, zur Befriedigung des Geschlechtstriebs, aus Habgier oder sonst aus niedrigen Beweggründen, heimtückisch oder grausam oder mit gemeingefährlichen Mitteln oder um eine andere Straftat zu ermöglichen oder zu verdecken, einen Menschen tötet.“) In der Regel wird man, wenn der Tatort nicht verändert wird, bei einem Opfer durch Russisches Roulette die freiverantwortlichen Selbstgefährdung problemlos nachweisen können. --2003:46:A0C:6E00:E22A:82FF:FEA0:3113 22:37, 27. Feb. 2017 (CET) Nachtrag zur der inkompetenten und irreführenden Bemerkung über die Nachweislast: Die Staatsanwaltschaft (Deutschland) hat nach § 160 StPO die „Pflicht zur Sachverhaltsaufklärung“. Ich lese: „(2) Die Staatsanwaltschaft hat nicht nur die zur Belastung, sondern auch die zur Entlastung dienenden Umstände zu ermitteln und für die Erhebung der Beweise Sorge zu tragen, deren Verlust zu besorgen ist.“ Bei einem Toten durch Gewalteinwirkung sind Spurensicherung, KTU und Obduktion obligatorisch. --2003:46:A0C:6E00:E22A:82FF:FEA0:3113 22:47, 27. Feb. 2017 (CET) (hier auch als 80.130.225.92 unterwegs)Beantworten
ich finde diese ständige lästige Beanspruchung der Auskunft durch virtuelle juristische Fragen stellende IP (offensichtlich immer die gleiche Person) definitiv zum Kotzen: Alle diese IP exekutieren = sperren. 23:11, 27. Feb. 2017 (CET) (ohne Name signierter Beitrag von Andy king50 (Diskussion | Beiträge))
Ich hab hier noch nie so eine Frage gestellt, reg dich mal ab. Danke für alle Antworten. --89.246.169.72 23:28, 27. Feb. 2017 (CET)Beantworten
Du namenloses Datum mit Uhrzeit, diese Frage ist immer wieder interessant und leicht zu beanworten: Selbstgefährdendes Verhalten inklusive Selbstverletzendes Verhalten ist nicht strafbar. --Rôtkæppchen₆₈ 23:34, 27. Feb. 2017 (CET)Beantworten
Lasst einfach mal die Waffen weg und macht Bunjee-Jumping mit uralten Seilen. Solange jeder Springer sich das Seil nimmt, dass er erwürfelt... Yotwen (Diskussion) 07:34, 28. Feb. 2017 (CET)Beantworten
Eine Verurteilung wegen Mord ist unwahrscheinlich (Anklage nicht!), eine Verurteilung Totschlag kann aber durchaus drin sein. Setz aber eben voraus, dass ein Vorsatz nachgewiesen werden kann. Aber eben im Bereich der Selbsttötung gibt es noch einige andere Punkte die zu Verurteilung führen können. Einmal im Bereich des Waffenrechts, gerade wenn es nicht der Revolver des Verlierers war (Ohne Revolver, ist kein Russisch Roulette möglich). Dann eben im Bereich der Unterlassung und Anstiftung, gibt es durchaus Möglichkeiten die der Staatsanwalt ausprobieren könnte. Gerade wenn der Verlieren betrunken (oder sonst wie benebelt) war, andere Teilnehmer aber nicht. Wenn die Gefahr also für den Verliere nicht mehr klar ersichtlich war, für die umstehenden schon. Das kann eben durchaus so ausgelegt werden, dass man bewusst diese Situation herbeigeführt hat. Da der Verlieren nur in dem Zustand überhaupt beim russisch Roulette mit gemacht hat. Kurzum es würde darauf hinauslaufen zu beweisen, dass ein Vorsatz da war. Und dann -mit Vorsatz-, sind ziemlich deftige Strafmasse möglich. Wenn es nur auf ein Urteil wegen nur Unterlassung und Fahrlässigkeit, hinausläuft. Nun ja, dann könnte man Glück haben und mit einer bedingte Strafe davonkommen. Aber Ärger wird man so oder so kriegen, wenn man dabei war und nichts unternommen hat das russisch Roulette zu verhindern.--Bobo11 (Diskussion) 08:16, 28. Feb. 2017 (CET)Beantworten
Wie meinen? Hast du die anderen Antworten gelesen? Und schreibst dennoch solchen Unsinn? Und wenn du schon abwandelst... Bring doch mal Aliens ins Spiel. Oder schwule Fußballer, die mal Akne hatten. Wäre sicher mal interessant zu erfahren, wie die Rechtslage dann ist... also nach deiner Einschätzung, die von der Realität ja allzu oft abweicht. -- Ian Dury Hit me  09:58, 28. Feb. 2017 (CET) Beantworten
Keine Ahnung, wie das in der Schweiz so ist. Ich kann mir aber nicht vorstellen, dass es erheblich von der deutschen Regelung abweicht. Es ist eher liberaler als dort. Und in Deutschland...
„Beihilfe“ zur Selbsttötung bedeutet die Selbsttötung mit Hilfe einer Person, die ein Mittel (meist ein Medikament) [oder Pistole] zur Selbsttötung bereitstellt. Eine Selbsttötung liegt aber nur dann vor, wenn der Suizident den letzten Schritt noch selbst beherrscht, also die sogenannte Tatherrschaft über das Geschehen hat. Sofern die andere Person die letzte todbringende Handlung vornimmt, ist kein Suizid mehr gegeben, sondern es liegt eine Tötungshandlung vor. Die Beihilfe zum Suizid, beispielsweise das Besorgen oder Bereitstellen tödlich wirkender Medikamente, ist in Deutschland mangels Vorliegens einer fremden, rechtswidrigen Haupttat nicht strafbar (Prinzip der limitierten Akzessorietät). Der Suizid richtet sich nicht gegen eine „andere“ Person und ist mithin kein Tötungsdelikt im Sinne der §§ 211 ff. StGB, sodass auch die Hilfe hierzu keine strafbare Tat darstellt. Dies bezieht sich jedoch auf die Beihilfe im juristischen Sinne und ist zu unterscheiden von einer aktiven Hilfestellung bei der Selbsttötung, die als Beteiligung nach Täterschaftsgrundsätzen strafbar sein kann.
Viel klarer gehts nicht. Yotwen (Diskussion) 12:18, 28. Feb. 2017 (CET)Beantworten

Die Frage ist, ob die Teilnehmer tatsächlich noch eigenständige Entscheidungen treffen. Zumindest wenn der erste Teilnehmer seinen Versuch unbeschadet überstanden hat, dürfte eine Situation eintreten, die sich nicht mehr völlig rational voraussehen und daher auch nicht pauschal ex ante beurteilen lässt. Ganz abgesehen davon, dass solche Spiele nicht selten mit Erblindung, aber nicht mit dem Tod enden. --Zxmt Nutze Dein Stimmrecht! 13:03, 28. Feb. 2017 (CET)Beantworten

Es gibt halt Leute, die ihr Hirn deutlich grösser einschätzen, als es tatsächlich ist. Und wenn sie an solchen Happenings teilnehmen, dann wundert mich das nicht mal. Yotwen (Diskussion) 13:40, 28. Feb. 2017 (CET)Beantworten
Nochmal deutlich: Es geht nicht um Selbsttötung. Die Teilnehmer an dem Spiel haben nicht das Ziel, sich auf diesem Weg das Leben zu nehmen. Sie spielen in der Hoffnung, zu gewinnen und zu überleben und die Teilnahme an dem Spiel ist Ausdruck einer freien Entscheidung. Es geht, wie oben bereits ausgeführt, um eine freiverantwortliche Selbstgefährdung und bei dem bzw. den Überlebenden um Teilnahme an einer freiverantwortlichen Selbstgefährdung des Opfers. Das oben verlinkte, bei Esser/Krey 2016 gefundene Zitat stammt aus einem Urteil des BGH. BGH ist hier nicht die Abkürzung für „bin gleich hier“ sondern für Bundesgerichtshof. Der BGH entscheidet als einer der fünf obersten Gerichtshöfe der Bundesrepublik zwar letztinstanzlich, aber angesichts der geballten juristischen Kompetenz in diesem Thread wird man dort sicher nochmals in sich gehen und prüfen, ob man Russisches Roulette statt als freiverantwortliche Selbstgefährdung nicht doch lieber als Suizid und Ausdruck des Selbstbestimmungsrechts (straffrei) oder als Teilnahme an einem Suizid (ebenfalls straffrei) versteht. Es wäre ja nicht das erste Mal, dass auf der Wikipedia-Auskunft von anonymen Accounts einem Obersten Gericht kräftig der Marsch geblasen wurde, weil wir es einfach besser wissen. Die werden sich also noch ganz schön umgucken, wenn wir erstmal richtig loslegen... --91.40.196.119 14:39, 28. Feb. 2017 (CET) (hier auch als 80.130.225.92 und 2003:46:A0C:6E00:E22A:82FF:FEA0:3113 unterwegs)Beantworten
Wie bereits richtig festgestellt, der BGH urteilt letztinstanzlich – aber eben mehrheitlich im Einzelfall und eher selten als Grundsatzentscheidung. Dementsprechend halte ich es für verwegen, Entscheidungen, die einen ganz anderen Sachverhalt betreffen einfach umzusetzen. Daher halte ich die Thesen über mir für zumindest nicht stichhaltig.
Eine Teilnahme egal, ob an einem Suizid oder einer anderen Tat ist ausgeschlossen – hier hat der BGH mehrfach Klartext gesprochen: Da beide Beteiligten abwechselnd den Revolver betätigen, liegt auf jedem Fall für beide Tatherrschaft vor. Es kann also gar keinen Teilnehmer unter den Spielern geben (BGH 4 Str 328/08). Ein Suizid liegt ebenfalls nicht vor, da jeder Teilnehmer die Erwartung hegt, das Spiel zu gewinnen, also gerade nicht zu sterben. Aber auch eine eigenverantwortliche Selbstgefährdung dürfte vor dem BGH kaum Zuspruch ernten. Diese Einwilligung ist nämlich höchstrichterlich eingeschränkt: Sie „verliert ihre (insoweit) rechtfertigende Wirkung dort, wo die Grenze zur Sittenwidrigkeit überschritten ist, also bei konkreter Todesgefahr, unabhängig von der tatsächlich eingetretenen Rechtsgutverletzung“ (aus oben zitiertem Urteil, mit weiteren Nachweisen).
Ohne mich zu weit aus dem Fenster lehnen zu wollen (wer hätte vor zwei Tagen daran zu denken gewagt, dass ein Gericht ein illegales Straßenrennen als Mord werten könnte), würde ich mich auf den Standpunkt stellen, dass die Rechtsprechung zu tödlich verlaufenen Straßenrennen hier anwendbar sein könnte, sich die überlebenden Teilnehmer damit zumindest der fahrlässigen Tötung schuldig machen, da sie als rechtstreue Bürger gar nicht erst ein Russisches Roulette hätten veranstalten oder daran hätten teilnehmen dürfen und sich somit nicht auf eigenverantwortliche Selbstgefährdung berufen dürfen. Auch derjenige, der den Revolver zur Verfügung stellt macht sich strafbar, die geht jedoch schon aus waffentechnischen Spezialgesetzen hervor, die einen Waffenträger einige besondere Sorgfaltspflichten aufbürden. Eine Verurteilung wegen Totschlags würde ich nur anhand der aktuellen („Berliner“) Rechtsprechung nicht vollständig ausschließen wollen – siehe aktuelle Berichterstattung, einen Mord könnte man unter dem Merkmal „niedriger Beweggrund“ an den Haaren herbeiziehen. Aber ich bevorzuge die „ältere“ Rechtsprechung hinsichtlich fahrlässiger Tötung.--OnlyMe (Diskussion) 22:31, 28. Feb. 2017 (CET)Beantworten
Ok, es wird mir jetzt langsam zu blöd und deshalb habe ich nochmal nach dem von Esser/Krey 2016 zitierten BGH-Urteil gesucht. Ich komme nicht ran, weil die Vorschau begrenzt ist und finde es leider auch nicht über Websuche. Aber so etwas landet ja nicht ohne Grund in einem Lehrbuch. Wer möchte kann sich auch in den Artikeln über Robert Esser und Volker Krey zu deren Vita informieren. Immerhin finde ich ein Urteil des 1. Strafsenates des Bundesgerichtshofs aus dem Jahr 1984: BGH, 14.02.1984 - 1 StR 808/83. Ich lese dort: „Amtlicher Leitsatz: Eigenverantwortlich gewollte und verwirklichte Selbstgefährdungen unterfallen nicht dem Tatbestand eines Körperverletzungs- oder Tötungsdelikts, wenn das mit der Gefährdung bewußt eingegangene Risiko sich realisiert. Wer lediglich eine solche Selbstgefährdung veranlaßt, ermöglicht oder fördert, macht sich nicht wegen eines Körperverletzungs- oder Tötungsdelikts strafbar.“ Der Fall (von Esser/Krey als „weiteres Beispiel“ angeführt) behandelt eine erfolgreiche Sachbeschwerde gegen die Verurteilung wegen fahrlässiger Tötung. Ein Drogenkranker hatte zusammen mit seinem Freund Heroin gespritzt, der Freund war dadurch verstorben. Das verlinkte Urteil stellt sowohl die rechtliche Würdigung der Jugendkammer dar, die den Überlebenden verurteilte als auch die Erwägungen, die das BGH einen Erfolg der Beschwerde erkennen lassen (wegen einer Gesamtfreiheitsstrafe wird allerdings nicht freigesprochen sondern zurückverwiesen, damit eine neue Gesamtfreiheitsstrafe gebildet werden kann). Wen es interessiert mag die recht komplexen Überlegungen dort nachvollziehen. Sie sind grundlegender Natur und auf die Frage des Russischen Roulettes durchaus ebenfalls anwendbar. Es mag richtig sein, dass fallbezogen gedacht werden muss. Und dass der „Amtliche Leitsatz“ dieses Urteils bei Esser/Krey 2016 auch auf einen Fall von Russisch Roulette mit Todesfolge angewandt wird liegt dann wohl sicherlich an der evidenten Inkompetenz der beiden Autoren. Zum Thema Fallbezogenheit daher auch: Im Kontext gehen Esser/Krey 2016 an gleicher Stelle nach dem „Fall 56 – Russisches Roulette Nr.1 –“ auch auf gemeinschaftlichen Drogenkonsum und Autorennen mit Todesfolge ein. Das kann ja dann, wer will, mit ihrer Sicht auf das sogenannte „Konstanzer Urteil“ (BGH 4 Str 328/08) abgleichen, das OnlyMe hier einbringt und dessen Sichtweise er übernimmt. Verrate ich zuviel, wenn ich die auf den „Fall 56 – Russisches Roulette Nr.1 –“ bezogene Einordnung bei Esser/Krey 2016 dazu zitiere? Ich lese (fallbezogen...): „Anders dagegen lag es beim sog. »Konstanzer Wagenrennen«: Der Tod des Beifahrers war den Fahrern auch objektiv zurechenbar, weil sie im Gegensatz zum Beifahrer die Herrschaft über das Geschehen innehatten (Fremdgefährdung)“. Das von OnlyMe angeführte Urteil BGH 4 Str 328/08 ist für die Fragestellung also untauglich. --2003:46:A19:1200:E22A:82FF:FEA0:3113 02:25, 1. Mär. 2017 (CET)Beantworten
In der deutschen Rechtstheorie weiss ich keinen Titel, aber in Großbritannien haben H.L.A. Hart und Tony Honoré mit Causation in Law einen sehr großen Einfluss auf was schuldhaft zurechenbar ist und was nicht. Und bei den Verbindungen von Honoré nach Freiburg kann ich mir nicht vorstellen, dass das auf die deutsche Rechtstheorie keinen Einfluss gehabt haben soll.
Denn das ist die zugrundeliegende Frage: Ist ein Teilnehmer an dem "Spiel" kausal für die Handlungen der anderen (mit)verantwortlich? Yotwen (Diskussion) 04:27, 1. Mär. 2017 (CET)Beantworten
1. mir fällt da dann noch ein, dass die Einwilligung des Opfers manchmal nicht ernst genommen werden darf... etwa beim Arzt, wenn der Pat unbedingt n Alpenveilchen im Knie möchte (dann muss man erstmal ganz oft zum Psychiater...)... oder beim Meiwes (bei solchen Wünschen hilft auch kein Psychiater mehr... also die werden nie ernst genommen...)... oder beim Kneipenwirt, wenn der Gast sagt, dass er nun aber mit Vollgas nach Hause fahren muss, damit er ob seines Vollrausches durch mitgebrachte Spirituosen nich das ganze Auto vollspuckt... 2. das passt auch zu user:Bobo11's Überlegungen mit betrunkenen und nüchternen Teilnehmern/Zuschauern... --Heimschützenzentrum (?) 05:37, 1. Mär. 2017 (CET)Beantworten
Bobo11s 'Überlegungen' haben aber nichts mit der Fragestellung zu tun. Wer am RR teilnimmt und dabei zu Tode kommt ist übrigens allenfalls 'Opfer' seines Übermuts. -- Ian Dury Hit me  11:26, 1. Mär. 2017 (CET)Beantworten
Die Frage war ja "Ist die Teilnahme legal?" Und damit sind die Implikationen in Zusammenhang mit dem Waffenrecht eben nicht trivial. Selbst mit Waffenschein darf ich nicht just for fun in der Gegend rumballern. Und ich darf nach §34 WaffG die Waffe niemandem überlassen, der nicht die nötige Berechtigung hat. Und ohne Rechte darf mein Gegenüber die Waffe auch gar nicht benutzen. Selbst, wenn der Waffenbesitzer nicht an einem "Körperverletzungs- oder Tötungsdelikt" schuldig ist, ist doch die Frage, ob ein sachgemäßer Umgang mit der Waffe vorliegt, wenn jemand damit zu Tode kommt. Legal wäre somit wahrscheinlich nur die merkwürdige Situation, in der zwei Besitzer von Waffenscheinen mit ihren eigenen Revolvern auf einem dafür geeigneten Schießplatz Russisches Roulette spielen. --Optimum (Diskussion) 23:15, 1. Mär. 2017 (CET)Beantworten
ehm? 1. aber der Gedanke an Alkohol-Rausch oder sonstige Geisteskrankheit oder sogar Zwangslage drängt sich einem bei Russisch Roulette doch gerade auf, denn: Warum sonst sollte man an sowas teilnehmen? normal isst man doch wohl möglichst sogar Bio-Gemüse... 2. zusehen und nicht helfen ist normal verboten... 3. Manipulationen an der Waffe sind auch ein beliebtes Szenario im Fernsehen... dann wird es aber doch auf jeden Fall kriminell? --Heimschützenzentrum (?) 05:47, 2. Mär. 2017 (CET)Beantworten
Etwas muss nicht ausdrücklich verboten sein, wenn das Resultat strafbar ist, bzw. es nur mit Übertrettung von Gesetzen möglich ist. Beim russisch Roulette haben wird durchaus Punkte die nicht legal sind. Denn die von Optimum angesprochene Fremdgefährdung ist da durchaus so ein Punkt, wenn man in einem Raum eine Waffe abfeuert, selbst wenn das nur ein „vielleicht“ ist. Der Waffenbesitzer möchte ich jedenfalls nie sein, wenn die Waffe freiwillig ausgehändigt wird, dies im Wissen das damit russisch Roulette gespielt werden soll. Aber lassen wir das ganze mal ausser vor, wer in welcher Position welche Tatbestände erfüllen könnte. Denn Prinzipiell wird es für alle Beteiligte unangenehm wenn es zu einem Unfall mit Schusswaffen kommt. Weil spätestens dann wird sich die Polizei und die Staatsanwaltschaft der Sache annehmen und anfangen Fragen zu stellen. Die Fragen werden sicher mehr, wenn es sich herauskristallisiere, dass es um einen vermeidbaren Unfall handelt (es muss niemand russisch Roulette spielen, ergo vermeidbar). Richtig unangenehm werden die Fragen wenn der Staatsanwalt findet „vermeidbarer Unfall“ passt irgend wie nicht so recht. Und findet da habe wer vorsätzlich gehandelt damit es zu dem "Unfall" kam. Die Wahrscheinlichkeit das für die aktiven Teilnehmer auch Untersuchungshaft angeordnet wird, halte ich nicht für ausgeschlossen. --Bobo11 (Diskussion) 07:53, 2. Mär. 2017 (CET)Beantworten
Wie so oft sind deine Antworten nebulös und entziehen sich jedem Beweis. Wenn du die Antwort weißt oder wenigstens Hinweise darauf kennst, dann antworte so kurz wie möglich, so lang wie nötig, mit Links auf Wikipedia-Artikel oder andere Quellen, die zum Verständnis beitragen - so steht es im Intro. Natürlich kann eine unerlaubte Überlassung einer Waffe vorliegen. Die Frage bietet dafür aber keinen Anhaltspunkt. Abschließend: Wegen was würden die Teilnehmer verurteilt werden? Wegen illegaler Teilnahme am RR jedenfalls nicht, die Teilnahme daran ist legal. Manipuliert jemand die Waffe in der Absicht, dass der andere zu Tode kommt, dann ist das ggfls. Totschlag oder auch Mord. Hat er die Waffe so manipuliert, dass sie nach auslösen alle Atomwaffenlager der Welt zur Explosion und den Bundestag zum Einsturz bringt, dann erwartet ihn alles mögliche, in D aber keine Veurteilung wg. Teilnahme am RR. -- Ian Dury Hit me  14:34, 2. Mär. 2017 (CET)Beantworten
+1, Ich verstehe auch nicht die hartnäckige Ignoranz einiger Beiträge. Die Frage war „Ist die Teilnahme an Russisch Roulette legal?“ Es geht also um die strafrechtliche Verantwortlichkeit der Teilnehmenden bei einem möglichen oder eingetretenen Todesfall. Die waffenrechtliche Diskussion ist aufgesetzt und konstruiert. Wenn z.B. die Waffe dem Opfer gehört und der überlebende Teilnehmer sie niemals in der Hand hatte, spielt das Waffenrecht für den Fall keine Rolle. Ein waffenrechtliches Delikt kann also hinzukommen, muss aber nicht. Ich denke auch, dass es dem Fragesteller um diesen Aspekt nicht ging. Eigentlich ist die Frage schon längst beantwortet.
Ich lese deshalb nochmal vor: „Fall 56: – »Russisches Roulette Nr. 1« – A und O gehören zur »Russen-Mafia«. Aus Imponiergehabe spielen sie »Russisches Roulette«: A hält sich einen mit nur einer Patrone geladenen Revolver an die Schläfe, ohne zu wissen, in welcher der acht Kammern der Revolvertrommel sich die Patrone befindet, und drückt ab; er hat Glück. Dann gibt er die Waffe dem O, der sich, nachdem er die Revolvertrommel hat rotieren lassen, ebenfalls den Revolver an die Schläfe setzt und abdrückt. O hat weniger Glück und stirbt. Strafbarkeit des A wegen Fahrlässiger Tötung, wenn beide Gangster nicht mit der Möglichkeit eines tödlichen Ausgangs gerechnet hatten? Die Frage ist zu verneinen: O war das Opfer einer freiveranwortlichen Selbstgefährdung. A war lediglich an dieser Gefährdung beteiligt. Für solche Konstellationen hat der BGH zutreffend entschieden: »Eigenverantwortlich gewollte und verwirklichte Selbstgefährdung unterfallen nicht dem Tatbestand eines Körperverletzungs- oder Tötungsdelikts, wenn das mit der Gefährdung bewusst eingegangene Risiko sich realisiert. Wer lediglich eine solche Selbstgefährdung veranlasst, ermöglicht oder fördert, macht sich nicht wegen eines Körperverletzungs- oder Tötungsdelikts strafbar.« Die Begründung hierfür liegt auf der Hand: Aus der Straflosigkeit der vorsätzlichen Mitverursachung des freiverantwortlichen Suizids und der Selbstverletzung anderer folgt mittels eines Erst-Recht-Arguments die Straflosigkeit der vorsätzlichen Mitverursachung der freiverantwortlichen Selbstgefährdung des Opfers. Soweit die Teilnahme am Suizid oder an der Selbstverletzung straflos ist, kann die Teilnahme an der wissentlichen Selbstgefährdung, auch wenn sich die Gefahr realisiert, nicht als Fahrlässige Tötung oder Fahrlässige Körperverletzung strafbar sein. In casu ist von einer freiverantwortlichen Selbstgefährdung des O auszugehen. Somit ist der Tod des O dem A nicht als sein Werk objektiv zuzurechnen und A hat sich nicht der Fahrlässigen Tötung schuldig gemacht. Als weitere Beispiele für den Ausschluss der objektiven Zurechnung wegen freiverantwortlicher Selbstgefährdung des Opfers werden in Rechtsprechung und Lehre diskutiert: (...) Tödlich ausgehendes Motorrad-Wettrennen zweier Mitglieder einer Bande (...) Abgabe von Heroin an einen Drogenkonsumenten, der sich das Rauschgift injiziert und daran stirbt (...) Beischlaf eines HIV-positiven Mannes mit seiner Freundin, die diese Infektion und das Übertragungsrisiko kennt (...)“ (Robert Esser, Volker Krey: Deutsches Strafrecht, Allgemeiner Teil, Kohlhammer Verlag, Stuttgart, 6, Aufl. 2016, ISBN 978317029882).
Den jeweiligen Personenartikeln kann man entnehmen: Robert Esser ist „seit Februar 2007 (...) Inhaber des Lehrstuhls für Deutsches, Europäisches und Internationales Strafrecht und Strafprozessrecht sowie Wirtschaftsstrafrecht an der Universität Passau. Er ist zudem Stellvertretender geschäftsführender Direktor des Instituts für internationales und ausländisches Recht – Abteilung für europäisches und internationales Straf- und Strafverfahrensrecht. Im Jahr 2010 wurde zudem die Forschungsstelle "Menschenrechte im Strafverfahren – Human Rights in Criminal Proceedings (HCRP)" gegründet, deren Direktor Robert Esser seit Gründung ist. HRCP ist eine Forschungs-, Fortbildungs- und Beratungsstelle für alle Fragen des internationalen Menschenrechtsschutzes im Strafverfahren. Zu den Aufgaben von HRCP zählt die Auswertung und Systematisierung der Rechtsprechung des Europäischen Gerichtshofs für Menschenrechte (EGMR) in Straßburg sowie der Spruchpraxis des Human Rights Committee und des Committee Against Torture der Vereinten Nationen (UN-Menschenrechtsausschuss / UN-Antifolterausschuss).“ Volker Krey war „1974 C3-Professor und Wissenschaftlicher Rat an der Universität Bielefeld. 1975 wurde er als einer der Gründungsprofessoren des Fachbereichs Rechtswissenschaft an der Universität Trier zum C4-Professor für Strafrecht, Strafprozessrecht und Methodenlehre der Rechtswissenschaft ernannt. Von 1978 bis 1998 war Krey zudem Richter am 1. Strafsenat des Oberlandesgerichts Koblenz. (...) Seine Lehrbücher zum Allgemeinen und Besonderen Teil des Strafrechts und zum Strafprozessrecht sind weit verbreitet.“ Der Bundesgerichtshof erkennt, wie weiter oben bereits diskutiert, zwar fallbezogen, aber letztinstanzlich. Zwei hochkarätige, allgemein anerkannte Juristen. Ein oberster Gerichtshof. Keine einzige Literaturstelle, die sich im Widerspruch oder in der Kontroverse befindet. So what? --91.40.202.96 23:50, 2. Mär. 2017 (CET) (hier auch als 80.130.225.92 und 2003:46:xxx unterwegs)Beantworten
RA Ingo Heinemann ist auch irgendwie ganz oben und findet schon gewisse Kredit-Verträge blöd, auch wenn die ohne Waffen-Einsatz unterschrieben wurden... --Heimschützenzentrum (?) 05:19, 3. Mär. 2017 (CET)Beantworten
Aber nein, Homer, der hat zwar das zweite Staatsexamen, aber der ist ganz sicher nicht „irgendwie ganz oben“. Und dahin ist es für ihn noch weit. Kein Prägen der aktuellen Rechtssprechung über Jahrzehnte an einem Oberlandesgericht. Kein Lehrstuhl. Kein bedeutender, nichtmal ein nennenswerter Beitrag zur Rechtwissenschaft. Kein Mitherausgeber eines Kommentars. Nach seinen Publikationen lernen nicht über Jahrzehnte hinweg tausende von Jurastudenten. Keine Mitwirkung in bedeutenden Gremien. Aus meiner Sicht also nicht hochkarätig. Und als Konsequenz hat er nichtmal einen Wikipedia-Artikel... Ganz oben ist woanders. --80.130.231.152 18:58, 3. Mär. 2017 (CET) (hier auch als 91.40.202.96 und 2003:46:xxx unterwegs)Beantworten

Kölnische Profanität

Ich glaube mich an eine kölnische Profanität zu erinnern welche ein Schiff (ggf. Boot) und ein ziehende bzw. schubsende Bewegung beinhaltet. Ungefähr "Deu mer doch den Kaan". Trügt meine Erinnerung und ist etwas über die Herkunft dieser Redewendung bekannt? Ich kann die Frage auch gerne nächste Woche wiederholen. Danke, --2003:70:EF74:FBD:4986:8EBD:EE03:FA4D 20:19, 27. Feb. 2017 (CET)Beantworten

Kölsch (Sprache) kennt „Deu mer doch der Naache!“ (Schiebe doch meinen Kahn an!) mit der standarddeutschen Entsprechung „Leck mich am Arsch!“
Hier findet man demgegenüber „Däu mer doch der/dr/d'r Naache“ von Marie-Luise Nikuta als „Lass mich doch in Ruhe“. Die Zeile kommt auch in Immer wigger von den Bläck Fööss vor. --Vsop (Diskussion) 20:50, 27. Feb. 2017 (CET)Beantworten
Adam Wrede, Neuer Kölnischer Sprachschatz (1958), Bd. 2, S. 220, schreibt: „Däu mer der Naache! Do kanns mer der Naache däue, ablehnende Antwort: nein, nichts da!“ Der Grad der Derbheit liegt irgendwo zwischen "Lass mich in Ruhe" und "Leck mich am Arsch". Kommt wie immer auf die Situation, den Tonfall, das Verhältnis der Personen zueinander usw. an. --Jossi (Diskussion) 13:38, 28. Feb. 2017 (CET)Beantworten
In Will Hermanns Aachener Wortschatz steht eher neutral 'Du kanns mich ens der Naache döie (abschlägige Antwort)' --Elrond (Diskussion) 16:17, 28. Feb. 2017 (CET)Beantworten

Käpt´n Ohnefurcht

Hallo. Ich hab kürzlich eine kurze Szene der Ermordung von Jürgen Kanter durch die Terrorgruppe Abu Sajaf; geschockt & ohne Worte hab ich das Video abgebrochen. Unfassbar! Nachdem habe ich anschließend nach ihm (J.K.) googlete, bin darauf gestossen dass beide, Kanter und seine Partnerin 2008 schon einmal 2008 vor Somalia das gleiche Schicksal (jedoch mit einem positiven Ausgang) ereilte, wobei die Bundesregierung (der Staat) ein gewisses Lösegeld zahlte. Nun, nachdem ich heut morgen in den Nachrichten mitbekam dass Kanter exekutiert wurde, erwähnte kein Nachrichten(-sender)programm dieses wichtige Detail. Nachdem Abu Sajaf dieses mal ein Lösegeld in Höhe von 500- bis 600tausend Euro (Dollar) mit Ultimatum forderte, und diesem nicht nachgekommen wurde, lautet meine Frage: Wollte der Staat dieses Mal kein Lösegeld zahlen weil die Zuständigen für einen solchen Fall sich sagten (oder dachten), „wir hatten doch schon mal bezahlt, wenn Herr Kanter nichts daraus gelernt hat, selbst Schuld; nochmal holen wir ihm die Kohle nicht aus dem Feuer“...!?? . lG und Danke für eine weiterdenkende Antwort. --2001:7E8:C086:6801:A07C:760F:76AD:9714 22:47, 27. Feb. 2017 (CET)Beantworten

Erstens würde ich die Meldungen von morgen abwarten, Zweitens könntes Du richtig liegen und Drittens: Du verwechselst die Auskunft mit einem Bloog! --80.187.114.147 23:10, 27. Feb. 2017 (CET)Beantworten

1. Stimmt. 2. Ok, aber ich habe nicht nach deiner POV gefragt. 3. Die Frage hab ich deshalb HIER gestellt, in der Annahme, dass jemand vielleicht tiefergehende Kenntnisse besitzt, durch irgendwelchen Link das weiterführt bezüglich dieses Themas bzw. jemand zu diesem Thema schon detaillierter nachgeforscht hat. 4. Ich weiss sehr wohl was ein Blog (oder Bloog, oder Blooooooog) ist, und die Auskunft ist keiner. Trotzdem ein Dankeschön. --88.207.220.156 00:32, 28. Feb. 2017 (CET)Beantworten
Die Frage hast Du hier gestellt in der Hoffnung auf „tiefergehende Erkenntnisse“? Diese Auskunftsseite handelt von "Allgemeinen Wissen", nicht von tiefergehenden Erkenntnissen zum Verhalten des Staates in einer aktuellen Extremsituation. Selbst wenn hier alles und jedes diskutiert werden sollte und dürfte, so müsste sich hier schon ein eingeweihter Staatsbediensteter äußern, um Dir die Frage da oben zu beantworten. Dass das nicht geschehen wird, weißt Du sicher. Erwarte also bitte nicht zu viel und sei nicht enttäuscht darüber, dass nicht jeder solche Fragen hier als in der Auskunft passend ansieht. Es führt hier zu einem Gespräch über einen schrecklichen Vorfall, aber sicherlich niemals zu einer Antwort auf Deine Frage zu diesem speziellen Vorfall, denn die kann keiner wissen. Spekulationen und Plaudereien sind dann aber eher eine Sache für's WP:Café. Nix für ungut. :-) VG --Apraphul Disk WP:SNZ 10:34, 28. Feb. 2017 (CET)Beantworten
Das Auswärtige Amt warnt seit Jahren vor Reisen nach Basilan, Mindanao und in den Sulu-Archipel. Das Opfer ist dem Anschein nach wider besseres Wissen in diese Region gereist und hat somit seine Entführung und seinen Tod selbst verschuldet. --Rôtkæppchen₆₈ 00:57, 28. Feb. 2017 (CET)Beantworten
Das nennt man Täter-Opfer-Umkehr.--Hinnerk11 (Diskussion) 01:58, 28. Feb. 2017 (CET)Beantworten
Wenn ich, wissend um die Situation in dieser Gegend selbige bereise, darf ich mich nicht beschweren, wenn der Nervenkitzel böse Realität wird. Ein Bekannter hat vor einigen Jahren Iran, Irak und Afghanistan mit seinem Geländewagen erkundigt. Auf die Gefahr angesprochen, meinte er lapidar: " War schon spannend". Wie auch immer, das Mitleid bei diesen Menschen hält sich bei mir in Grenzen, genauso wie bei anderen Adrenalinjunkies. Wer sich in Gefahr begibt, kommt darin um. --Elrond (Diskussion) 10:54, 28. Feb. 2017 (CET)Beantworten

Es ist ja ganz offensichtlich, dass dem Opfer keine zweite Chance gegeben wurde. Es ist ja ein offenes Geheimnis, dass der Staat in solchen Situationen "Entwicklungshilfe" an diese Gruppen zahlt, damit diese aus lauter Dankbarkeit ihre "Gäste" bewirten und mit Abschiedsgeschenken (ihrem Leben) wieder ziehen lassen. Nur um das böse Wort Lösegeld zu vermeiden, was ja nicht sein kann, da der Staat ja sonst als erpressbar und deutsche Staatsbürger als "sichere Bank" gelten würde. Diese Hilfestellung hat also offenbar Grenzen: Ein Freischuss, dann hat der Unbelehrbare halt Pech gehabt. Benutzerkennung: 43067 08:43, 28. Feb. 2017 (CET) Mir wird hier zu wenig die Rolle des philippinischen Staates beleuchtet. Wer sagt denn, das die Deutschen nicht auf nem guten Weg waren? Laut Nachrichten wurden aber von Regierungstruppen Stellungen der Abu Sajaf kurz vorher bombardiert, sicher ein Vertrauensbeweis.--scif (Diskussion) 14:50, 28. Feb. 2017 (CET)Beantworten

Die Frage geht von falschen Prämissen aus. Die Entscheidungsträger im Staatsdienst sind nicht so einfältig wie viele sie gern sehen. Herrn Kantor wurde nach seiner ersten Freilassung von den zuständigen staatlichen Stellen mit Sicherheit zu verstehen gegeben, daß er sich im Falle des Falles nicht darauf verlassen kann auch ein zweites Mal freigekauft zu werden. --DJ 20:49, 1. Mär. 2017 (CET)Beantworten

28. Februar 2017

Ton-/Datenträger für Langzeitarchivierung

Hallo, was eignet sich bei

  1. Tonträgern (digital oder analog) sowie
  2. Datenträgern (digital)

am besten für Langzeitarchivierung? Die Daten sollen bei dieser theoretischen Überlegung einmalig gespeichert werden, ein wiederbeschreibbares Medium ist also nicht unbedingt nötig. --Morten Haan 🛣 Wikipedia ist für Leser daSkin-Entwurf 00:37, 28. Feb. 2017 (CET)Beantworten

Hast Du unseren Artikel Langzeitarchivierung schon entdeckt? --Rôtkæppchen₆₈ 01:11, 28. Feb. 2017 (CET)Beantworten
Klar, nur werde ich da nicht so ganz schlau aus der Tabelle, zumal ich nicht weiß, wie aktuell die Daten sind, und analoge Tonträger wie Schallplatten ganz fehlen. --Morten Haan 🛣 Wikipedia ist für Leser daSkin-Entwurf 14:13, 28. Feb. 2017 (CET)Beantworten
Es geht um Medien, die selbst bespielt werden können. Bei Schallplatten war das die Ausnahme und betraf Exoten. Der Graveur benötigte etwas weicheres Material. Selbst für 8-Spur-Kassetten gab es Rekorder, da sich um (Endlos)-Tonband handelt, was aber trotz Graphitbeschichtung höheren Verschleiß hat als Rollentonband. Unser Archiv der Auskunft weiß auch nicht mehr als der Artikel. --Hans Haase (有问题吗) 16:51, 28. Feb. 2017 (CET)Beantworten

Zu selbst gebrannten CDs, danach DVDs. Gebrannt zw. 1997 und 2009. Anzahl weit über 100. Gebrannt wurden jeweils Duplikate. Mit Rohlingen und Geräten der Zeit. Kopieren auf Festplatte ab 2011. Bei über 20 war eines der beiden Duplikate mit mehr oder weniger Lesefehlern behaftet. Bei ca. zehn waren beide Kopien nicht mehr sinnvoll lesbar. Hilfreich war bei der Auslesung der Rückgriff auf alte Brenner bzw. alte oder ausgewählte CD/DVD Laufwerke. Ergo: Ich gebe einer CD/DVD acht Jahre, so man zwei davon hat. Grüße --80.187.97.115 20:51, 28. Feb. 2017 (CET)Beantworten

Bei selbstgebrannten CDs und DVDs kommt es extrem stark auf die Qualität der Rohlinge und den Umgang damit an. Ich hatte schon CD-Rs, die nach zwei Jahren unleserlich wurden, weil minderwertiger Kleber verwendet wurde und die Schicht deswegen Blasen warf. Andere CD-Rs wurden durch minderwertigen Lack und Fingerabdrücke oder säurehaltige Filzstifte unbrauchbar. Seitdem fasse ich CD-Rs nur noch am Mittelloch an und beschrifte sie nur noch im durchsichtigen Bereich rings um das Mittelloch. DVD±Rs haben eine deutlich geschütztere Schicht, sodass sie problemlos angefasst und flächig beschriftet werden können, auch mit oleinhaltigem 1970er-Jahre-Edding aus dem Altbestand meines Arbeitgebers. --Rôtkæppchen₆₈ 23:36, 28. Feb. 2017 (CET)Beantworten
Und wie verhalten sich CDs, insbesondere gepresste, im Vergleich zu Schallplatten? --Morten Haan 🛣 Wikipedia ist für Leser daSkin-Entwurf 00:16, 1. Mär. 2017 (CET)Beantworten
Übliche CDs enthalten Aluminium in der Reflektionsschicht. Durch durch die Lackschicht diffundierenden Sauerstoff oxidiert diese allmählich und die Reflektivität nimmt dadurch ab. Damit wird die CD allmählich unleserlich. Weiterhin droht Gefahr durch säurehaltigen oder sonstwie nicht dauerhaften Lack. Die en-wp hat hierzu den Artikel en:Disc rot. Gepresste CDs lassen sich aber theoretisch restaurieren, indem die Lack- und Aluschichten chemisch entfernt und der Polycarbonatpressling danach neu vergoldet oder aluminiumbeschichtet wird. Magnetbänder halten auch nicht ewig. Manche Hersteller sparen am Lack, andere an der Trägerfolie. Ton- oder Videobänder aus Celluloseacetat mussten schon als Totalverlust abgeschrieben werden. Bei anderen Bändern hat zwar die biaxial orientierte Polyester-Folie durchgehalten, aber der Lack wurde zur Pampe. Außerdem hängt es davon ab, ob Bandmaschine und Bandmaterial für Profi- oder für Heim- und Amateurzwecke vorgesehen sind. Profiband ist hart, Profimagnetköpfe sind weich. Damit verschleißt im Profibetrieb der billig wiederzubeschaffende Magnetkopf und nicht das Magnetbandunikat. Für den Einsatz zuhause wurden Tonband-, Cassetten- und Videogeräte mit harten, verschleißarmen Köpfen und dafür weiches Bandmaterial verkauft. War das Band ausgenudelt, sollte der Heimnutzer bitte ein neues kaufen. Es wäre dem Heimnutzer ebenso unzumutbar, sein Magnetbandgerät regelmäßig zum Kopftausch in die Werkstatt zu geben. Bei Schallplatten kommt es auf das Trägermaterial an und wie oft die Platte mit welchem Tonabnehmer gespielt wurde. Schellackplatten waren mit Steinmehl gefüllt und die Nadeln waren relativ weich und Verbrauchsmaterial. Bei häufigem Abspielen verschliss also nicht die Platte, sondern die Nadel. Vinylplatten sind mit Ruß gefüllt und relativ weich, die Abspielnadeln sind aus Diamant oder Saphir, also hart. Beim Abspielen verschleißt also die Platte und die Nadel überlebt. Schallplattenunikate bestanden aus Lackfolie. Damit besteht prinzipiell die gleiche Zersetzungsproblematik wie bei Magnetbändern. --Rôtkæppchen₆₈ 00:50, 1. Mär. 2017 (CET)Beantworten
Die PVC-Schallplatte an sich hält bei nicht ganz falscher Lagerung ewig. Wie Rotkaeppchen68 aber schon schrieb, verschleißt sie bei jedem Abspielvorgang. Aber eine Platte, die du heute einlagerst, können deine Ururururenkel noch abspielen, sofern sie geeignete Abspielhardware haben (und da gibt es bekanntlich sehr einfache Lösungen, da bei der Abtastung bereits die mechanische Schwingung entsteht und nichts mehr gewandelt werden muss – es reicht ja, eine Ecke einer Postkarte in die rotierende Platte zu hängen). Eine Untersuchung des Instituts für Rundfunktechnik (Nürnberg) aus den 90ern kam auch zu dem Schluss, dass sich das Nadeltonverfahren mit weitem Abstand am besten zur Langzeitarchivierung eignet – erstens wegen der Langlebigkeit des Trägers, zweitens wegen des technisch einfachen Abspielens. --Kreuzschnabel 11:12, 1. Mär. 2017 (CET)Beantworten
Ihr erinnert euch doch bestimmt an Scherben im CD/DVD-Laufwerk? Eine CD/DVD enthält auch Weichmacher. Taugen die nichts mehr oder sind verdampft, war es das. Da kann das Alu ewig halten, wenn das Trägermaterial nichts mehr taugt. Ist der Bruch da, ist man auf Mikroskope und Bildauswertung und Prüfsummen angewiesen, falls das jemand schon je so gebaut hat. --Hans Haase (有问题吗) 14:07, 1. Mär. 2017 (CET)Beantworten
Bei Festplatten altern Bauteile und Schnittstellen werden obsolet. --Hans Haase (有问题吗) 14:08, 1. Mär. 2017 (CET)Beantworten
@„Eine CD/DVD enthält auch Weichmacher.“ Dafür hätte ich gerne entweder eine Entschuldigung für die Falschaussage oder einen validen Beleg. --Rôtkæppchen₆₈ 15:19, 1. Mär. 2017 (CET)Beantworten
--80.187.96.246 15:26, 1. Mär. 2017 (CET)Scherben im CD/DVD-Laufwerk? Eine CD/DVD enthält auch Weichmacher. Obwohl jahrzehnte Erfahrung: 1. Da war vielleicht mal was? Hilfts Du mir auf die Sprünge? 2. Keine Ahnung - nie gelesen. --80.187.96.246 15:26, 1. Mär. 2017 (CET)Beantworten
Also ist es nicht nur aus nostalgischen Gründen sinnvoll, Alben als Schallplatten herauszubringen sowie zu kaufen. Wie sieht es bei Solid-State-Drives aus, gibt es dazu schon Daten? --Morten Haan 🛣 Wikipedia ist für Leser daSkin-Entwurf 20:48, 1. Mär. 2017 (CET)Beantworten
SSDs sind auf Geschwindigkeit und Speicherplatzdichte optimiert, nicht auf Dauerhaftigkeit. EEPROMs, um die es sich bei SSDs schlussendlich handelt, halten ihre Ladung bzw Daten nicht ewig. Irgendwann vermindert sich die Ladung auf dem floating gate so stark, dass die darauf gespeicherte Information hin ist. Das hängt von der Einwirkung von Wärme und ionisierender Strahlung ab, wobei auch die Hintergrundstrahlung/Nulleffekt etwas ausmacht. Bei MLC-Flash geht das schneller als bei SLC-Flash. Magnetische Festplatten sind da nicht empfindlich. Magnetische Festplatten können allerdings anderweitig altern, beispielsweise durch degradierendes Schmiermittel, Schutzgasverlust, Oxidation, versprödende Gummi- und Kunststoffteile. Besser wäre es, nur die eigentlichen Magnetplatten und dazu einen Bauplan für das Laufwerk aufzubewahren. Da die langzeitarchivierten Daten aber ohnehin nur gelesen und nicht modifiziert oder gelöscht werden sollen, sind wiederbeschreibbare Speicher nicht notwendig. WORM-Medien wie DVD±R, CD-R, BD-R, M-Disc sind da besser geeignet. Bei diesen Medien ist eine Trennung von Laufwerk und eigentlichem Medium üblich, sodass hier die Gefahr der Unlesbarkeit wegen Ausfalls der alten Laufwerkstechnik unwahrscheinlich ist. Ich habe hier über 25 Jahre alte VHS-Bänder, die noch spielbar sind, aber nicht mehr den ursprünglichen VHS-Recorder. --Rôtkæppchen₆₈ 23:37, 1. Mär. 2017 (CET)Beantworten
Das heißt also bei Tonträgern ist die PVC-Schallplatte am besten, bei Datenträgern die gepresste Blu-ray Disc (höhere Kapazität als CD und DVD). Ist das so richtig zusammengefasst? --Morten Haan 💿 Wikipedia ist für Leser daSkin-Entwurf 16:03, 2. Mär. 2017 (CET)Beantworten
Wie es mit gepressten DVDs, HD-DVDs und BDs ausschaut, weiß ich nicht. Dort befindet sich die Datenschicht im Innern der zusammengeklebten Schichten. Bei der gepressten CD ist nichts zusammengeklebt, sondern der Pressling wird mit Aluminium (meistens) oder Gold (selten) beschichtet und dann lackiert. Den Lack kriegt man wieder ab, wenn er die Metallschicht versaut. Zusammengeklebte Schichten bekommt man wahrscheinlich nicht mehr getrennt. Wenn DVD oder BD, dann bitte nur einschichtig und mit Gold als Reflektionsschicht, also DVD-5 oder BD-25. --Rôtkæppchen₆₈ 23:37, 2. Mär. 2017 (CET)Beantworten
Eine BD mit 25GB ist ja schon mal nicht wenig, darauf kann man Video-, Text-, Bild- und Musikdateien sowie beliebige andere Formate speichern. --Morten Haan 💿 Wikipedia ist für Leser daSkin-Entwurf 17:18, 3. Mär. 2017 (CET)Beantworten

Nasser Grund ... veräppelt Wikipedia den Leser ?

Guten Tag. Ich hab den Artikel Nasser Grund gelesen. Was inhaltlich zu Gewässerschichtungen beschrieben wird kommt mir plausibel vor. Von dem Lemma hab ich allerdings noch nie etwas gehört. "Nasser Grund" dort wo sowieso schon alles übernässt ist? Will da jemand den Leser veräppeln? Kennt jemand diesen Begriff oder einen Besseren für diese Dichteanomalie die auch in Thermische Schichten beschrieben wird?--Tom (Diskussion) 08:24, 28. Feb. 2017 (CET)Beantworten

Wenn ich das richtig lese, geht es um die Bezeichnung, die die Marine (angeblich) für das Phänomen benutzt, deswegen auch der Zusammenhang mit U-Booten. Die Quelle muss man wohl in Papierform einsehen. --Ailura (Diskussion) 08:32, 28. Feb. 2017 (CET)Beantworten
Habe ich mitmal das Suchen im Internet verlernt oder ist darüber tatsächlich rein gar nichts zu finden? VG --Apraphul Disk WP:SNZ 08:43, 28. Feb. 2017 (CET)Beantworten
Vielleicht wäre es besser, einen Löschantrag zu stellen. --Schlesinger schreib! 08:46, 28. Feb. 2017 (CET)Beantworten
Bis auf den Begriff "Nachbar"schichten stimmt es. Es geht um verschiedene Tiefen. --M@rcela 08:48, 28. Feb. 2017 (CET)Beantworten
Na, dann wirst du uns gleich einen Beleg dafür liefern und in den Artikel einfügen. Dir liegt diese"Marine-Enzyklopädie A bis Z 2003" aus dem Tosa-Verlag, eher bekannt für Karl May, vor? --Schlesinger schreib! 08:52, 28. Feb. 2017 (CET)Beantworten
Wenn nicht, gibt es diesen Schmöker ISBN 3854927576 gebraucht für 5 euro bei ZVAB.com.--Rôtkæppchen₆₈ 09:05, 28. Feb. 2017 (CET)Beantworten
Ich habe Löschantrag gestellt.--Meloe (Diskussion) 09:08, 28. Feb. 2017 (CET)Beantworten
Na dann hat sich ja die Quellensuche erledigt. --M@rcela 09:18, 28. Feb. 2017 (CET)Beantworten
Merkwürdige Auffassung. Hindert dich jemand daran, mit neuen, dann aber seriösen Quellen, einen besseren Text zu verfassen, der wenigstens ansatzweise die Zusammenhänge erklärt? --Schlesinger schreib! 09:22, 28. Feb. 2017 (CET)Beantworten
Ja, destruktives Verhalten. --M@rcela 10:16, 28. Feb. 2017 (CET)Beantworten
Destruktiv wäre sofortiges Löschen gewesen, du kannst dich konstruktiv in der LD beteiligen. --84.129.180.227 10:40, 28. Feb. 2017 (CET)Beantworten
(BK) Naja, den LA hätte ich jetzt auch nicht gestellt und auch nicht schon jetzt erwartet. So wirklich schlecht begründet ist er allerdings nicht, so dass ich das nicht als destruktives Verhalten bezeichnen würde. Wenn einem der LA nicht gefällt, könnte man dann ja erst recht Belege suchen, um dem LA-Steller den LA ggf. um die Ohren zu hauen. ;-) Zurück zum Thema: Ich würde, selbst bei völligem Fehlen des Begriffs im Internet, nicht behaupten wollen und schon gar nicht beweisen können, dass es den Begriff nie gegeben hat. Aber auch Begriffe und Ausdrücke sollten eine gewisse Relevanz besitzen, um einen Wiki-Artikel zu haben; wir sind ja kein Wörterbuch. Eine Relevanz/Gebräuchlichkeit/Bekanntheit müsste sich aber doch irgendwo im Internet widerspiegeln. Das scheint nicht der Fall zu sein. VG --Apraphul Disk WP:SNZ 10:47, 28. Feb. 2017 (CET)Beantworten
Als LA-Steller: Wer mag, möge einen Artikel akustische Schattenzone anlegen, der wäre belegbar. Mag sein, mag nicht sein, dass das im Seemannsjargon auf Ubooten mal "Nasser Grund" genannt wurde. Sofern die genannte Quelle der einzige Beleg dafür wäre, wäre dass auch dann nicht relevant, wenn es zutreffen würde. Dazu müsste der Ausdruck schon eine gewisse Verbreitung erlangt haben, was zu beweisen wäre. Da das Phänomen existiert, kann jeder, der es haben will, dazu, unter neuem Lemma, einen neuen Artikel anlegen. Ich wüsste nicht, was der existierende Artikel außer dem möglichen Beleg einer Wortverwendung in einem Soldatenjargon, dazu beizutragen hätte.--Meloe (Diskussion) 11:05, 28. Feb. 2017 (CET)Beantworten
Im Lehrbuch der "Navigation auf Seewasserstraßen" von 1974 existiert kein Seemannsjargon. Aber ist ja erfolgreich eliminiert. Erstmal Löschen, so kommen wir sicher weiter. --M@rcela 12:22, 28. Feb. 2017 (CET)Beantworten
Es ist gar nichts gelöscht worden. Das betreffende Artikelchen ist in eine BKS umgewandelt worden und dessen ursprünglicher Inhalt befindet sich jetzt in der Liste seemännischer Fachwörter (N bis Z), für den Leser auffindbar über die BKS. Du kannst dich also wieder entspannen und dir ein trockenes Höschen anziehen... --Gretarsson (Diskussion) 13:18, 28. Feb. 2017 (CET)Beantworten
@all: Herzliches Dankeschön für die Aufklärung dieser nassen Angelegenheit. Grüße --Tom (Diskussion) 15:44, 1. Mär. 2017 (CET)Beantworten

Gottfried Wilhelm Leibniz

Ich will ein paar Werke von Leibniz lesen. Hat hier jemand Vorschläge, womit ich am besten anfange? Zum Hintergrund: Ich studiere Mathematik und bin über Gödel auf Leibniz gestoßen, und möchte nun wissen, was ihn an Leibniz so fasziniert hat. --132.230.195.232 09:40, 28. Feb. 2017 (CET)Beantworten

Als Mathematiker könnten Dich wohl die Monaden interessieren. --Elrond (Diskussion) 11:06, 28. Feb. 2017 (CET)Beantworten
Ach ja, weil Du Gödel ansprachst, dann sind vielleicht auch seine Arbeiten zur Logik von Interesse. --Elrond (Diskussion) 11:08, 28. Feb. 2017 (CET)Beantworten
Auch Russell interessierte sich für Leibniz. Interessant ist wohl seine Darstellung der Logik, über die ich mal gelesen habe, indem die Prämissen Primzahlen waren und die Schlussfolgerung die Multiplikation aus diesen. (nicht signierter Beitrag von 188.101.73.75 (Diskussion) 22:02, 28. Feb. 2017 (CET))Beantworten

Speicherdauer von mehr oder minder öffentlichen Überwachungskamera-Aufnahmen

Wie lange kann man damit rechnen, dass die Bilder von mehr oder minder öffentlichen Überwachungskameras in Österreich gespeichert werden? Im konkreten Falle geht es um eine Kamera bei der Österreichischen Post, die möglicherweise vor ca. zwei Wochen ein Vergehen gefilmt hat, und um Zeitdruck für das Einleiten von Schritten (das müsste über den Filialleiter beim Sicherheitsdienst der Österreichischen Post beantragt werden). Das "mehr oder minder" soll einerseits zum Ausdruck bringen, dass die entsprechende Kamera zwar in den Räumichkeiten der Postfiliale ist, aber in einem 24 Stunden lang zugänglichen Bereich; andererseits, dass die Post mittlerweile ein privatisiertes Unternehmen ist, aber zweifelsohne doch noch geprägt von ihren früheren staatlichen Strukturen und weiterhin mit großer öffentlicher Bedeutung. --AF (Diskussion) 10:48, 28. Feb. 2017 (CET)Beantworten

Aufnahmen müssen normalerweise nach 72 Stunden gelöscht werden. --Schaffnerlos (Diskussion) 10:53, 28. Feb. 2017 (CET)Beantworten
Im Ernst? Was ist die entsprechende Vorschrift? --AF (Diskussion) 12:39, 28. Feb. 2017 (CET)Beantworten
DSG-Novelle 2010 § 50b (2). Eine längere Aufbewahrungsdauer ist möglich, aber zu begründen. Sicherheitsbehörden dürfen überhaupt nur 48 Stunden speichern, die Wiener Linien zum Beispiel halten es genauso. --Schaffnerlos (Diskussion) 12:54, 28. Feb. 2017 (CET)Beantworten
Ok, danke schön. --AF (Diskussion) 15:58, 28. Feb. 2017 (CET)Beantworten

Nutzerdaten in Bibliotheken archiviert

Hallo allerseits,

werden in Universitäts- und Stadtbibliotheken Daten darüber archiviert, wer wann welches Buch ausgeliehen hat? Wenn ja, gibt es diese Praxis schon lange? Kann man so beispielsweise (in Einzelfällen) irgendwie herausfinden, welche Bücher (willkürliches Beispiel) Gottfried Benn mal aus der Universitätsbibliothek ausgeliehen hatte? Danke im Voraus, besten Gruß :) --80.132.174.44 12:09, 28. Feb. 2017 (CET)Beantworten

Früher gab es in Büchern die Leihkarten, wo Entleiher, Leihdatum und Rückgabedatum vermerkt waren (die Karten wurde bei der Entleihe aus dem Buch genommen und in einen Karteikasten gesteckt). Die EDV-Speicherung wird die Sache vereinfachen und bestimmt werden die Daten nicht gleich bei Rückgabe gelöscht, allein aus dem Grund, dass man eine erst später entdeckte Beschädigung ggf. nachvollziehen kann. Über die Speicherdauer ist mir aber nichts bekannt, irgendwo wird der Datenschutz da Grenzen setzen. --Magnus (Diskussion) 12:16, 28. Feb. 2017 (CET)Beantworten
Man müsste dann irgend eine Einverständniserklärung unterzeichnet haben, auf der man darauf hingewiesen wird, dass die Daten elektronisch blablablabla... Du musst also nur einmal hingehen und die die Bedingungen ansehen. Yotwen (Diskussion) 12:28, 28. Feb. 2017 (CET)Beantworten
Das passiert üblicherweise beim Einschreiben, dass man sich mit irgendwelchen Sachen einverstanden erklärt. Wird wie immer beim Kleingeschriebenen nicht gelesen und somit nicht wahrgenommen. --Elrond (Diskussion) 13:33, 28. Feb. 2017 (CET)Beantworten
Natürlich, die guten alten Leihkarten! Ich danke euch. --80.132.174.44 12:30, 28. Feb. 2017 (CET)Beantworten

In Deutschland könnte Gottfried Benn eine Anfrage nach dem Informationsfreiheitsgesetz des jeweiligen Bundeslandes stellen, dann sollte er Auskunft über die zu seiner Person im System der Bibliothek gespeicherten Daten erhalten. --Jaroslaw Jablonski (Diskussion) 13:51, 28. Feb. 2017 (CET)Beantworten

Ich nehme mal an, es geht um diesen Gottfried Benn. Der kann keine Anfragen mehr stellen. --Digamma (Diskussion) 14:27, 28. Feb. 2017 (CET)Beantworten
Nun gut, der sicher nicht, falls er gemeint gewesen sein sollte. --Jaroslaw Jablonski (Diskussion) 14:35, 28. Feb. 2017 (CET)Beantworten
Uiuiui. Wieder mal viel Unwissen unterwegs. Wir reden zunächst um die Zeit zwischen sagen wir 1905 und 1956. Da wäre zunächst zu klären, welche Bücherei es war. Zweitens, wie war der Verleihmodus. Irgendwelcher elektronischer Kram scheidet da aus. Ob Bibs Nutzerkarteikarten aus dieser Zeit mittlerweile eingescannt haben, wäre zu klären. Fraglich aber ist , ob solche Karteikarten überhaupt archiviert werden. Warum sollten Bibs das tun? Ob man zu Benns Zeiten schon Nutzererklärungen im datenschutzrechtlichen Sinne unterschrieb, wage ich überdies zu bezweifeln. Vielleicht vorher mal mit dem Fragegegenstand befassen und das Problem im zeitlichen Kontext sehen, da klärt sich vieles von allein.--scif (Diskussion) 14:44, 28. Feb. 2017 (CET)Beantworten
Kein Grund, hier rumzugiften. --Jaroslaw Jablonski (Diskussion) 14:51, 28. Feb. 2017 (CET)Beantworten

Jeder hat so seine Wahrnehmung. Oder möchtest du mir erzählen, das Benn was mit dem Informationsfreiheitsgesetz anfangen kann?--scif (Diskussion) 15:14, 28. Feb. 2017 (CET)Beantworten

Benn war mir nicht bekannt, sonst hätte ich auf das Informationsfreiheitsgesetz nicht hingwiesen. Den Oberlehrer kannst du stecken lassen, dafür bin ich mittlerweile zu alt. --Jaroslaw Jablonski (Diskussion) 15:20, 28. Feb. 2017 (CET)Beantworten
Ich schätze, dass man von Anfang an bis in die erste Hälfte des 20. Jahrhunderts Ausleihbücher führte, die man natürlich aufhob, da sich darin ja auch die Einträge über nicht zurückerhaltene Bücher fanden. Danach dürfte man – zu Benns Lebzeiten – zum System der Ausleihe mittels Leih- und Buchkarten übergegangen sein, das bis zum Ende des 20. Jahrhunderts üblich war. --Pp.paul.4 (Diskussion) 15:33, 28. Feb. 2017 (CET)Beantworten
Frage aber ist doch, ob diese archiviert werden. Kann ich mir ehrlich gesagt nicht vorstellen.--scif (Diskussion) 15:58, 28. Feb. 2017 (CET)Beantworten
Vertu Dich da mal nicht, Bibliothekare lassen sich lieber die linke Hand abhacken, als solche Schätze zu vernichten. --Elrond (Diskussion) 16:10, 28. Feb. 2017 (CET)Beantworten
Vielleicht sind aber auch Bibliothekare weisungsgebunden von höherer Stelle. Ich habe jedenfalls mal in eigener Sache bei der Stadtbücherei nachgefragt (um meiner Erinnerung etwas auf die Sprünge zu helfen), in der ich als junger Mensch viel ausgeliehen hatte, und die hatten die Leihkarten schon vor 20 Jahren, oder so, vernichtet. Geoz (Diskussion) 18:17, 28. Feb. 2017 (CET)Beantworten
Viele UBs haben ask a librarian service, einen virtuellen helpdesk, die oft sehr hilfsbereit und auskunftsfreudig sind. Sonst https://fragdenstaat.de ? --Asteroidenbergbauer (Diskussion) 18:28, 28. Feb. 2017 (CET)Beantworten
Ich habe mal ein Werk konsultiert und mich auf der dazugehörenden Karte eingetragen; einige Einträge über meinem hatte Goethe (ja, der Goethe) unterzeichnet, der den Band auch dort benutzt hatte. Ich hab's nicht rumerzählt, wusste aber schon, dass Goethes Beschäftigung mit dem Werk bereits bekannt war. Bink22 (Diskussion) 20:05, 28. Feb. 2017 (CET)Beantworten

Nun ja! Gehe mal in eine Landesbibel, suche im Katalog unter: Spengfallen, Rohrbomben, Schmutziges, Fernsteuerung etc. und lasse Dir das alles in den Lesesaal bringen. - Natürlich ist Gottfried Benn die andere Seite. Grüße --80.187.96.246 16:14, 1. Mär. 2017 (CET)Beantworten

Jausenbesteck

An die Österreicher: Was ist der Unterschied zwischen einem Jausenbesteck und einem normalen Besteck? --92.107.205.150 15:10, 28. Feb. 2017 (CET)Beantworten

Ich kenne seit meiner Kindheit nur Jausenmesser. Die haben eine Klinge mit Wellenschliff und eine zweizinkige Spitze, so dass man man sie auch als Gabel verwenden kann. Rainer Z ... 15:26, 28. Feb. 2017 (CET)Beantworten
Glaskugel als Nicht-Österreicher: Die Google-Bildersuche zeigt in den ersten Treffern jeweils nur Messer und Gabel (mit rustikalem Holzgriff). Ein Essbesteck hat mehr Teile. Gruss --Nightflyer (Diskussion) 15:36, 28. Feb. 2017 (CET)Beantworten
Ich habe auch gerade nachgeschaut: Die von mir beschriebenen Messer scheinen ausgestorben zu sein. Wundert mich sehr. Rainer Z ... 15:47, 28. Feb. 2017 (CET)Beantworten
solche Messer vermeine ich auf einem Weihnachtsmarkt bei einem Messerstand gesehen zu haben. Der Preis lag deutlich noch im einstelligen €-Bereich. --Elrond (Diskussion) 16:04, 28. Feb. 2017 (CET)Beantworten
kein Jausenmesser
Außerhalb Österreichs wäre das eher ein Käsemesser. --Rôtkæppchen₆₈ 15:58, 28. Feb. 2017 (CET)Beantworten
Jausebesteck ist eher etwas Einfaches, Rustikales. Gestannztes Blech mit Griffen aus Holz oder Horn. [3], [4], neuerdings auch Plaste [5]. --M@rcela 16:02, 28. Feb. 2017 (CET)Beantworten

Ich dachte, es wäre vielleicht eher der Knauf am Ende. Hier ein Link eines Össi-Herstellers: https://www.hack.at/deutsch-1/online-shop/ --92.107.205.150 16:13, 28. Feb. 2017 (CET)Beantworten

Ha! Mein uraltes Jausenmesser mit zwei Zinken ist sogar von Hack (das kann man gerade noch erkennen). Die Form stellen sie aber wohl nicht mehr her. Dabei hätte ich gerne einen Nachfolger, weil die Klinge nix mehr ist. Rainer Z ... 18:51, 28. Feb. 2017 (CET)Beantworten
Der Unterschied scheint vor allem in der Klinge zu liegen. Ein "normales" Besteckmesser ist stumpfer, das Jausenmesser scheint mehr in Richtung Steakmesser zu gehen, ist also wohl stärker auf die Schnittleistung ausgelegt, so dass man harte Brotrinde, epökeltes und geräuchertes Fleisch und Hartkäse mit Rinde damit besser schneiden kann. Also kann das meiste damit schneiden, wozu man sonst vorbereitend in der Küche die typischen Küchenmesser verwenden würde, also Brotmesser, Tranchiermesser etc. Die Gemeinsamkeit zum normalen Essbesteck ist die kurze Klinge, die Form ist abweichend spitzer, so dass man z. B. in ein Brot einstechen kann, um eine Scheibe abzuschneiden, was mit einem normalen Besteckmesser praktisch nicht geht.--Giftzwerg 88 (Diskussion) 22:09, 28. Feb. 2017 (CET)Beantworten
Bei der Jause muss man selber eine Scheibe vom Laib Brot abschneiden und außerdem den Spreck in kleine Streifen schneiden. Diese zwei Aufgaben muss ein Jausenmesser erfüllen können. Gurkerl, bzw. Pfefferoni aus dem Glasl fischen eventuell auch noch und den Butter verstreichen ebenso. Also das sind die Anforderungen. --109.100.148.154 22:28, 28. Feb. 2017 (CET)Beantworten
Und was machst Du mit den geschnittenen Speckstreifen. (Übrigens werden die nicht in Streifen sondern in Teile geschnitten) Dies müssen aufgespiest und zum Mund geführt werden. Also Spitze, so oder doppelso. :-) --80.187.96.246 15:19, 1. Mär. 2017 (CET)Beantworten
Teile? What the hack meinst du mit Teile? Bist du ein Piefke, ein Preiß? Wenn ja, warum schreibst du zu dem hier Thema mit? Ja ehrlich, warum? --109.100.148.154 16:29, 1. Mär. 2017 (CET)Beantworten
Ich muss mich korrigieren. Was ich bisher für ein Jausenmesser gehalten habe, findet man als Tomatenmesser im Handel. Die gibt es weiterhin mit solchen zwei Spitzen. Da war ich, was die Bezeichnung angeht, wohl lange auf dem Holzweg. Rainer Z ... 13:06, 2. Mär. 2017 (CET)Beantworten

Online-Plattform für Büchereiexemplare

Kennt jemand eine Online-Plattform, die auch Bücher von Bibliotheken aufkauft? Momox hat mir das verneint.--scif (Diskussion) 15:16, 28. Feb. 2017 (CET)Beantworten

Das ZVAB könnte ein Ansprechpartner sein. Dort kann man den einzelnen Antiquariaten die dem Netz angeschlossen sind Angebote machen. --Elrond (Diskussion) 15:57, 28. Feb. 2017 (CET)Beantworten
Bei abebooks und anderen finden sich oft ehemalige Bibliotheksexemplare mit Ausscheidestempeln, siehe auch Deakzession.Steht dann auch in der Buchbeschreibung.--Asteroidenbergbauer (Diskussion) 18:49, 28. Feb. 2017 (CET)Beantworten
Warum sollten das die üblichen Ankäufer nicht annehmen? Es geht doch nur um die Frage ISBN oder nicht. Ansonsten gewöhnliche Antiquare. Abebooks, ZVAB, Booklooker sind keine Händler die ankaufen sondern nur Marktplätze, ein erheblicher Unterschied.--Antemister (Diskussion) 19:34, 28. Feb. 2017 (CET)Beantworten
Es kommt sehr stark darauf an, was für eine Art Bibliothek die Bücher anbietet. Wenn es eine öffentliche Bibo ist, sind die Bücher zumeist Altpapier, auch bei Fachbibliotheken nehmen meist nur spezialisierte Antiquare diese Bücher an, wenn überhaupt. Daher ist das Schwarze Brett von ZVAB & Co die beste Möglichkeit. Große Reichtümer sind sowieso mit großer Wahrscheinlichkeit keine zu erwarten, es sei denn, es handelt sich um gesuchte Raitäten. --Elrond (Diskussion) 19:50, 28. Feb. 2017 (CET)Beantworten
Die Antiquariate haben ein Netzwerk, mit dessen Hilfe man an ausgemusterte Bücher oder Bestände aus aufgelösten oder zusammengelegten Bibliotheken kommt. Ich habe sogar schon mal ein Buch aus einer englischen Unibibliothek über Amazon gekriegt. Es hat immer noch die Signatur, die Leihkarte und eine Scancode drin. Wer auf die "gebrauchten" bei Amazon geht kommt so manchmal auf solche Fundstücke. Es wird aber nicht gesagt, wo das Exemplar jeweils herkommt.--Giftzwerg 88 (Diskussion) 00:02, 1. Mär. 2017 (CET)Beantworten

Es geht um Gesetzeskommentare, wie z.B. den Fischer StGB in der Vorauflage (63.) in größerer Stückzahl. Die Schwarte kostete was an die 90 Euro, nach einem Jahr noch sehr gut erhalten. Momox weigert sich, weil nach eigener Aussage zuviel Bib-Exemplare von Endkunden "retourniert" werden. Ich weigere mich aber, solche Vermögenswerte einfach ins Altpapier zu schmeißen. Vorsorglich: es gibt auch keine Massen an Referendaren, die händeringend danach suchen, zumal deren Ansprüche auch arg gestiegen sind.--scif (Diskussion) 13:50, 1. Mär. 2017 (CET)Beantworten

Lange Haare von Männern in der Antike (Naher Osten)

War es eigentlich in der Antike im Nahen Osten üblich, dass die Männer meistens lange Haare trugen? Jesus wird ja fast immer so dargestellt. --2A01:C23:C014:8B00:F432:9081:855E:7770 18:26, 28. Feb. 2017 (CET)Beantworten

Nicht ganz dein Thema, aber kürzlich lief im DLF ein haariger Beitrag. --Stefan »Στέφανος«⸘…‽ 18:55, 28. Feb. 2017 (CET)Beantworten
Wir haben tatsächlich einen Artikel Langes Haar. Dort vermerkt auch die berüchtigte Stelle aus 1 Kor. 11, 14, nach der es für Männer "von Natur aus" eine Schande ist, ihre Haare lang wie Frauenhaar wachsen zu lassen. Für den Nahen Osten bin ich ziemlich ahnungslos, aber die männliche Haartracht war in der Antike insgesamt recht häufig wechselnder Mode unterworfen: Archaisches Griechenland lang, kurz markierte den Sklaven; später eher kurz; Rom in der alten Republik (sehr) kurz, später länger und gestylt, oft nach der Fasson des aktuellen Herrschers. Grüße Dumbox (Diskussion) 19:11, 28. Feb. 2017 (CET)Beantworten
Ferner haben wir Dreadlocks#Rastafari mit noch mehr einschlägigen bzw. einschlägig gemachten Bibelstellen. --Edith Wahr (Diskussion) 19:15, 28. Feb. 2017 (CET)Beantworten
Wobei ausgerechnet Paulus selbst einmal das Nasiräer-Gelübde geleistet und (zumindest zeitweise) gehalten haben soll. Geoz (Diskussion) 19:37, 28. Feb. 2017 (CET)Beantworten
Bildliche Darstellungen Jesu (und seiner Zeitgenossen) gibt es erst ab dem 4ten Jahrhundert. Nach generellem Konsensus, gestützt durch einige Bibelzitate, den kontemporären Haarstilen der Römer und Hellenen aber auch Untersuchungen skelettaler Funde trugen Juden dieser Epoche kurzes Haar. Da die Verfasser der Evangelien Jesus wahrscheinlich nie sahen gibt es auch im Neuen Testament keine Beschreibungen der Erscheinung Jesu. --Cookatoo.ergo.ZooM (Diskussion) 20:27, 28. Feb. 2017 (CET)Beantworten
Hier eine Büste eines “fast” Zeitgenossen Jesu, Flavius Josephus.
Römische Büste, die auf Flavius Josephus gedeutet wurde.
--Cookatoo.ergo.ZooM (Diskussion) 20:31, 28. Feb. 2017 (CET)Beantworten

Haarlänge in der Antike, als Athen und Rom: Wo wären die brauchbaren Quellen? --80.187.97.115 21:12, 28. Feb. 2017 (CET)Beantworten

Fürs erste Nachschlagen z. B. der Kleine oder Neue Pauly. Dort die literarischen Belegstellen, dazu natürlich die zeitgenössischen Plastiken, Vasenmalereien etc. Grüße Dumbox (Diskussion) 21:15, 28. Feb. 2017 (CET)Beantworten
Danke! Mir ist der alte, neue und KLP (hat man sich gekauft) geläufig. DU zu kurz, sehr kurz - zu Rom - ist mir zu kurz! :-) Grüße --80.187.97.115 21:22, 28. Feb. 2017 (CET)Beantworten
War ja auch streng genommen nicht zum Thema. Grüße Dumbox (Diskussion) 21:26, 28. Feb. 2017 (CET)Beantworten
OK! Nochmal gelesen - wenn Du meinst dass dies auf Jesus bezogen war, so muss ich Dir recht geben. Grüße --80.187.97.115 21:36, 28. Feb. 2017 (CET)Beantworten
Die Haarlänge und Haarfarbe des J. v. N. war kein Thema, über das irgendeiner der Autoren des 1. Jh. geschrieben hat. Alles was wir heute dazu haben beruht in irgendeiner Form auf der künstlerischen Freiheit. Zum historischen Background: Es ist natürlich ziemlich eindeutig, dass die Römer dieser Zeit eher auf kurze Haare und Rasur standen, was aber nicht auf die unterdrückten Nationen zutreffen muss. Denkbar ist also, dass aus diesen sich manche als politische "Statements" Haartrachten und Bartmoden einfallen ließen, die dazu kontrastierten. Es ist wie in den Kontaktanzeigen: Alles ist möglich, nichts muss.--Giftzwerg 88 (Diskussion) 21:51, 28. Feb. 2017 (CET)Beantworten
S. auch: Brian Palmer: The Jesus Hairstyle: Did a lot of men have flowing locks in ancient Judea?, in slate, Nov. 2011. --Edith Wahr (Diskussion) 22:19, 28. Feb. 2017 (CET)Beantworten
Wassjez?!? Und blaue Augen hatte er wohl auch nicht? GEEZER … nil nisi bene 08:26, 1. Mär. 2017 (CET) Beantworten

Die Antike war ein Zeitraum von mindestens 1000 Jahren (500 vor und 500 nach Christus). Bei den Römern waren lange Haare und Bart lange Zeit ein Zeichen für zurückgebliebene Barbaren, deshalb rasierte sich der römische Bürger und auch der römische Legionär täglich und trug die Haare kurz. Das war auch ein Statussymbol, man konnte es sich leisten, hatte scharfe Klingen und Zeit genug für diese Form der Körperflege. Im ersten Jahrhundert nach Christus, in der frühen Kaiserzeit, ändert sich diese Mode aber, etwa bei den bärtigen Kaisern Hadrian und Marc Aurel, mit Unterbrechung des immer glatt rasiert dargestellten Trajan (98 bis 117). Die dann in Mode gekommenen Bärte waren jedoch immer noch gepflegt und eher kurz, auch wenn Vollbart. Also setzte sich eher der klassisch griechische Einfluss bei den Römern durch, als der barbarisch-germanische oder orientalische. Es gibt natürlich detaillierte kulturgeschichtliche Studie zu der Frage, aber so grob kann man sagen, ab dem ersten Jahrhundert nach Chr. werden Vollbart und etwas längere Haare in Rom modern. Diese kulturelle Prägung der Römer war aber so stark, dass sich in der Westkirche nie die alttestamentalische Vorstellung vom Samson-Mythos durchsetzte, also dass man sich die Haare nicht abschneiden dürfe, während ostkirchliche Popen bis heute lange Haare und lange Bärte haben, ebenso wie islamische Imame. Der gute römische westkirchliche Abendländer ist hingegen kurzhaarig und rasiert, nicht wegen dem Christentum, sondern weil die römische Tradition hier stärker war als die hebräische, griechische oder nahöstliche. --109.100.148.154 22:52, 28. Feb. 2017 (CET)Beantworten

Zur Erinnerung: die Ursprungsfrage lautete War es eigentlich in der Antike im Nahen Osten üblich, dass die Männer meistens lange Haare trugen?. Was Rom und die Römer betrifft ist sicher interessant, aber hier nur bedingt hilfreich. --Elrond (Diskussion) 23:00, 28. Feb. 2017 (CET)Beantworten

Durch Kaiser Konstantin und die spätere Geschichte sind wir aber in Westeuropa extrem stark römisch geprägt, egal ob katholisch, evangelisch oder anglikanisch, und nur sehr wenig griechisch, jüdisch oder sonstwie östlich. Und wie gesagt, es gibt keine künstlerische Jesus-Darstellung vor Kaiser Konstantin, die sich erhalten hätte. Die langen Haare Jesu sind also samt und sonders künstlerische Imagination, bzw. ein Topos um ihn eben nicht als Römer darzustellen. Einen Römer erkennt man im Bild (Ikon) eben durch die kurzen Haare und den rasierten Bart. Jesus sollte aber eben nicht als Römer dargestellt werden und damit die Beobachter das gleich erkennen, verpassten ihm die Künstler lange Haare. Ob er diese wirklich hatte, wissen wir nicht. Das Turiner Grabtuch ist ja auch eine mittelalterliche Reliquien-Fälschung und somit keine valide Quelle. --109.100.148.154 23:22, 28. Feb. 2017 (CET)Beantworten
ei guck/ecce homo: nochne imitatio christi
Das Grabtuch ist vielleicht eine Fälschung, aber dafür ein echter Dürer, ein Selbstbildnis natürlich, wie ich unlängst in diesem schönen Schmöker las, den hiermit & unter uns Pfarrerstöchtern als Strandlektüre empfehlen mag. --Edith Wahr (Diskussion) 00:03, 1. Mär. 2017 (CET) Beantworten
Wenn mich nicht alles täuscht, dann war unter germanischen Stämmen das Haar ein bürgerliches Statussymbol und der rasierte Kopf ein Zeichen von Leibeigenschaft. Reste davon haben sich bis ins frühe Mittelalter erhalten, als beispielsweise Karl der Kahle jahrelang rasiert wurde, um zu verhindern, dass er Kaiser werden konnte. Und damals hatte die Kirche noch jede Menge Konkurrenten auf dem Kontinent und vom nächsten Schisma war man noch weit entfernt. Es kann durchaus sein, dass die künstlerische Darstellung Anleihen aus dem germanischen Alltag zog. Daher haben unsere Jesus-Bilder ja auch europäische Züge und Hautfarbe. Yotwen (Diskussion) 09:12, 1. Mär. 2017 (CET)Beantworten
Die Frage der Haartracht von Jesus hat Hans Belting in Bild und Kult ausführlicher thematisiert. Er weist (ich zitiere jetzt nur aus dem Gedächtnis) auf zahlreiche unterschiedliche Darstellungsweisen in den ältesten Bildern hin, aus denen der "klassische" langhaarige Jesus mit Mittelscheitel sich erst nach und nach entwickelte. Wichtigste Anregung dafür seien die Bildnisse des Jupiter gewesen.--Meloe (Diskussion) 10:30, 1. Mär. 2017 (CET)Beantworten
Wie die beiden Vorschreiber. Die Evangelisten wie auch Paulus hatten womöglich schlichtweg kein Interesse an einer Beschreibung des genauen Aussehens Jesu, hervorhebenswert war allenfalls die Annahme einer Gestalt als Mann bzw. Sohn. Interessanterweise stützen sich einige Darstellungen oder Verkörperungen Jesu, die wir heute nicht als gutaussehend bezeichnen würden, auf Jesajas Prophezeiungen: „Er hatte keine schöne und edle Gestalt, sodaß wir ihn anschauen mochten. Er sah nicht so aus, dass wir Gefallen fanden an ihm. Er wurde verachtet und von den Menschen gemieden, ein Mann voller Schmerzen, mit Krankheit vertraut. Wie einer, vor dem man das Gesicht verhüllt, war er verachtet; wir schätzten ihn nicht.“ Das verläßt aber den Boden des Themas Haarlänge. --Turris Davidica (Diskussion) 12:48, 1. Mär. 2017 (CET)Beantworten
In der Offenbarung sind seine Haare weiß (wie Wolle) - nach all dem Stress - aber auch keine Längenangabe. GEEZER … nil nisi bene 14:36, 1. Mär. 2017 (CET) Beantworten
Wissenschaftlich betrachtet ist festzustellen, dass man hierzu nichts aussagen kann. Belastbare Quellen fehlen schlicht. --80.187.96.246 16:22, 1. Mär. 2017 (CET)Beantworten
Es lässt sich feststellen, dass DU keine Ahnung hast, liebe(r) 80.187.96.246. --109.100.148.154 16:35, 1. Mär. 2017 (CET)Beantworten
Na dann bau mal Deine neueren belastbaren Quellen ein. Verate aber welche und wo. --80.187.96.246 17:47, 1. Mär. 2017 (CET)Beantworten
Im Gegensatz zu anderen Zeitepochen und anderen Kulturräumen ist die Quellenlage zur Römischen Antike wahrlich nicht mager, siehe etwa dem Corpus Inscriptionum Latinarum, die über das Mittelalter überlieferte lateinische Literatur, tausende archäologische Funde (zB Mosaike), etc. Wir wissen über die Völker des römischen Reiches so ziemlich alles, was sie zum Mittagessen gegessen haben und welche Unterhose sie getragen haben, wohingegen wir über die Völker außerhalb des Reiches (Skythen, Pikten, Berber, Sudanesen, Germanen, Hunnen, etc.) nur fragmentarische Informationen haben, die wenn vorhanden meist auch über römische Filiation überliefert sind. Von Attila dem Hunnenkönig haben wir zB kein einziges zeitgenössisches Portrait, keine Texte, das wenige was wir über die Hunnen wissen, wissen wir von den Römern, nicht von den Hunnen selber. --109.100.148.154 21:42, 1. Mär. 2017 (CET)Beantworten
Alles gut und schön, aber wissenschaftliche Quellen, zudem belastbar, über Haarlängen gibt es nicht. Und darum geht es hier. Übrigens: Ich habe auch den ordentlichen Hauptschulabschluß. --80.187.123.130 22:30, 1. Mär. 2017 (CET)Beantworten
Gratuliere! --109.100.148.154 22:57, 1. Mär. 2017 (CET)Beantworten

Phosphorsäure und Eisen

Wenn ich auf eine nicht verrostete Stelle aus Eisen Phosphorsäure streiche, gibt das dann eine Reaktion? Geht um verrostste Stellen an einer Heizung, die ich großflächig abgeschliffen habe. Zur Prophylaxe für noch ev. verbliebene Rostporen möchte ich mit Phosphorsäure arbeiten. Die Frage ist nun, ob das eher kontraproduktiv ist, bevor ich die Grundierung auftrage. --91.14.11.96 23:14, 28. Feb. 2017 (CET)Beantworten

Ja, es gibt eine Reaktion. Das Eisen reagiert mit der Phosphorsäure zu Eisenphosphaten. Dabei wird das Eisen passiviert. Das ist nicht kontraproduktiv, sondern nützlich, weil korrosionsschützend. --Rôtkæppchen₆₈ 23:27, 28. Feb. 2017 (CET)Beantworten
Zur weiteren Information. Phosphorsäure ist stark ätzend (Arbeitsschutz!) und andere Metalle reagieren auch damit, meist auf eine Art, die nicht gewünscht ist. Dann müssen alle Säurereste peinlich genau vom Werkstück entfernt werden, denn Säurereste können anschließend genau das Gegenteil von dem bewirken, was es soll, nämlich dass das Werkstück rostet. Reine Phosphorsäure ist nebenbei nicht so wirksam wie kommerzielle Rostumwandler, weil die noch etliche Hilfsmittel enthält wie Tenside, Verdickungsmittel (damit die Säure länger einwirken kann), Reaktionsbeschleuniger, Korrosionsschutzmittel etcpp. --Elrond (Diskussion) 23:45, 28. Feb. 2017 (CET)Beantworten
Abwaschen steht aber in der Bedienungsanleitung jedes Rostumwandlers. Seinerzeit hat die Stiftung Warentest übrigens Coca-Cola als Rostumwandler mit „gut“ bewertet. --Rôtkæppchen₆₈ 23:52, 28. Feb. 2017 (CET)Beantworten
Wenn der Frager aber keinen konfektionierten Rostumwandler hat, sondern ne Pulle mit Phosphorsäure ohne weitere Informationen, dann ist dieser Hinweis m.E. aber wichtig. --Elrond (Diskussion) 10:37, 1. Mär. 2017 (CET)Beantworten
Sollte man nicht eher Coca-Cola Light (oder Zero) verwenden? Enthält auch Phosphorsäure und Citronensäure, ist aber wegen dem fehlenden Zucker nicht so klebrig. --MrBurns (Diskussion) 09:58, 1. Mär. 2017 (CET)Beantworten
Die Klebrigkeit ist doch gerade erwünscht. Sonst läuft die Flüssigkeit ab, ohne Gelegenheit zu haben, einzuwirken. --Rôtkæppchen₆₈ 11:04, 1. Mär. 2017 (CET)Beantworten

Maine-Coon-Katze

Expertenfrage: Sind in diesem Artikel die Bildunterschriften in der Form korrekt (inhaltlich, sprachlich, typografisch), insbesondere was die jeweiligen Farbbezeichnungen angeht?--Hubon (Diskussion) 23:28, 28. Feb. 2017 (CET)Beantworten

In der Box würde ich ändern:
  • epistatisches weiß, schwarz (brown in US-Sprachgebrauch), blau, rot, creme, jeweils mit oder ohne Silber und mit oder ohne Scheckungs-Weiß
in
  • epistatisches Weiß, Schwarz (brown in US-Sprachgebrauch), Blau, Rot, Creme, jeweils mit oder ohne Silber und mit oder ohne Scheckungs-Weiß
Es geht hier um die Farben an sich und wird nicht adjektivisch verwendet (rotes Was, blaues Wer). GEEZER … nil nisi bene 09:21, 1. Mär. 2017 (CET)Beantworten
Das denke ich auch. Aber auch bei den Bildern ist wohl das Meiste auf Englisch und noch dazu kursiv gesetzt. Bin aber kein Fachmann – vielleicht soll das so? Ich lasse da, glaube ich, lieber mal die Finger von, bevor ich noch was falsch mache...--Hubon (Diskussion) 15:42, 1. Mär. 2017 (CET)Beantworten

Eindeutig groß. Dem Autor dieser Bildunterschrift kamen spätestens bei "Scheckungs-Weiß" ja offenbar selber Zweifel. Warum sonst sollte das dann großgeschrieben werden?
Die in Kursiv gesetzten englischsprachigen Bezeichnungen kann man m.E. so lassen. --93.212.230.216 16:56, 1. Mär. 2017 (CET)Beantworten

Habe es angepasst. --Digamma (Diskussion) 20:30, 1. Mär. 2017 (CET)Beantworten
Danke dir. Aber dann müsste doch bei den Bildunterschriften wie bspw. beim 1. Bild unter der Infobox auch großgeschrieben werden („Typvoller Maine-Coon-Kater mit auffälligen Lynxtips in black-mackarel-tabby-white“), oder?--Hubon (Diskussion) 03:55, 2. Mär. 2017 (CET)Beantworten
Mit den engl. Bezeichnungen ist es hoffnungslos (aber nicht alternativlos). In GoogleBooks (sehr viel BoD überr Coons) mal groß, mal klein.
Entweder findet man ein Standardwerk zur Orientierung - oder man fragt sich in jedem Fall: "Ist die Farbe gemeint oder ist es adjektivisch." GEEZER … nil nisi bene 08:03, 2. Mär. 2017 (CET)Beantworten
Ich denke, die englischen Bezeichnungen können als Zitatwörter nach englischer Rechtschreibung klein geschrieben werden. --Digamma (Diskussion) 10:22, 2. Mär. 2017 (CET)Beantworten

1. März 2017

Kleinsche Flasche

Weiß jemand, oder kann jemand herausfinden, wann und von wem die Bezeichung "Kleinsche Flasche" oder englisch "Klein bottle" zum ersten Mal verwendet wurde? Vielen Dank. --Digamma (Diskussion) 11:23, 1. Mär. 2017 (CET)Beantworten

Verstehe nicht. Ist denn der Satz im Artikel falsch? → „[...] wurde erstmals 1882 von dem deutschen Mathematiker Felix Klein beschrieben“ VG --Apraphul Disk WP:SNZ 11:55, 1. Mär. 2017 (CET)Beantworten
Mir geht es um die Bezeichnung. Klein hat meines Wissens nicht die Bezeichung "Flasche" verwendet, schon gar nicht "Kleinsche Flasche". Bei Hilbert/Cohn-Vossen wird die Fläche "Kleinsche Fläche" genannt. --Digamma (Diskussion) 12:32, 1. Mär. 2017 (CET)Beantworten
Obere Abschaetzung: 1893, siehe Annual Report, S. 29 MfG -- Iwesb (Diskussion) 12:38, 1. Mär. 2017 (CET) gestrichen, s.u. -- Iwesb (Diskussion) 01:42, 2. Mär. 2017 (CET)Beantworten
Wow. Danke. --Digamma (Diskussion) 14:32, 1. Mär. 2017 (CET)Beantworten
Das ist keine geeignete Abschätzung. Die verlinkten Annual reports of the Dayton Public Library and Museum., v.32-94 1891/1892-1954/1957 incompl. stammen aus den Jahren 1891 bis 1957. --Pp.paul.4 (Diskussion) 16:47, 1. Mär. 2017 (CET)Beantworten
Weiterhin interessiert mich aber, welcher Autor die Bezeichnung zuerst verwendet hat. Und außerdem die erstmalige Verwendung der deutschen Bezeichnung "Kleinsche Flasche". --Digamma (Diskussion) 16:07, 1. Mär. 2017 (CET)Beantworten
Stimmt, Pp.paul.4 hat recht, das Erscheinungsdatum bei Google Books ist irrefuehrend. Das liefert uns "lediglich" <=1957. In diesem Buch, S. 161f. ist ein Scan eines Briefes von E. Witt an G. Herglotz von Januar 1943, dort heisst es "Kleinscher Schlauch" (und wird uebersetzt als "Klein bottle"). Das ist eine Superfrage. Beim Versuch, diese "Flasche" zu fuellen, wird keiner gluecklich :-) MfG -- Iwesb (Diskussion) 01:42, 2. Mär. 2017 (CET)Beantworten
Google-Suche liefert Ergebnisse aus Zeitschriftenbänden, die regelmäßig nicht unter dem Erscheinungsjahr des Aufsatzes, sondern unter dem der Reihe verhaftet sind. Bei den Schnipseln sind Erscheinungsjahr, Titel oder Autor des Aufsatzes häufig kaum zu bestimmen. Dass, wie die englische Wikipedia unter Berufung auf diese Quelle andeutet, Klein bottle eine Fehlübersetzung von Kleinsche Fläche sein könnte, glaube ich nicht. Jeder Leser und Übersetzer von Arbeiten aus diesem Gebiet ist mit dem Wort „Fläche“ vertraut und weiß, was es bedeutet. Ich halte das englische "Klein bottle" entweder für eine schiefe Übersetzung von Kleinscher Schlauch (Neuer Wein in alten Schläuchen) oder für eine kreative Neuschöpfung. Kleinsche Flasche (1958) halte ich, da der englische Ausdruck "Klein bottle" (1947?) zeitlich voranzugehen scheint, für eine Übersetzung des englischen Ausdrucks. --Pp.paul.4 (Diskussion) 15:59, 3. Mär. 2017 (CET)Beantworten
Die in en:Klein bottle angegebene Quelle steht dort seit September 2010. Die Quelle ist von 2009. Die Behauptung mit der Falschübersetzung steht aber schon seit März 2005 im Artikel. Es ist somit nicht auszuschließen, dass der Buchautor bei Wikipedia abgeschrieben hat. --Digamma (Diskussion) 16:43, 3. Mär. 2017 (CET)Beantworten
In Hilbert, Cohn-Vossen: Anschauliche Geometrie von 1932 steht "Kleinsche Fläche" und mindestens an einer Stelle auch "Kleinscher Schlauch". In der englischen Übersetzung Geometry and the Imagination von 1952 steht "Klein bottle". --Digamma (Diskussion) 17:04, 3. Mär. 2017 (CET)Beantworten
Ein weiteres verstörendes Merkmal der Google-Buchsuche. Wenn ich von deinem Link (S. 272) zur Folgeseite (S. 273) mit dem Kleinschen Schlauch gehe, erhalte ich den Hinweis „Anzeigebeschränkung“, verlinke ich S. 273 direkt, erhalte ich die Seite 273. Was mich auch wundert, wieso merken du, Digamma, und andere, schon nach wenigen Minuten, wenn ich auf einem ein oder mehrere Tage alten Thread antworte? --Pp.paul.4 (Diskussion) 17:53, 3. Mär. 2017 (CET)Beantworten

Gedicht - aus welchem Jahr

Kann jemand feststellen, aus welchem Jahr das Gedicht "Sprichwörter" von Fred Endrikat stammt? Quelle wäre ggfs. nett. --85.178.195.31 12:56, 1. Mär. 2017 (CET)Beantworten

Angeblich aus „Liederliches und Lyrisches“ (Buchwarte Berlin 1942). Wann es geschrieben wurde, dürfte kaum verlässlich festzustellen sein. Siehe Nachwort bei https://www.lwl.org/literaturkommission-download/Bibliothek_Westfalica/Endrikat_Lesebuch.pdf. --Vsop (Diskussion) 14:01, 1. Mär. 2017 (CET)Beantworten
Vielen Dank für die Informationen. Ich bin sehr unsicher, ob die Angabe des WDR stimmt, denn in allen Inhaltsverzeichnissen, die ich online zu „Liederliches und Lyrisches“ finden kann, taucht "Sprichwörter" nicht auf beispielsweise hier --85.178.195.31 14:56, 1. Mär. 2017 (CET)Beantworten
Whow, "sehr" unsicher... Und was bringt dich zu der Annahme, dass der Christian Ritter aus Heidesee und die Familie Kindermann aus Radeberg sorgfältiger arbeiten als die professionellen Journalisten einer öffentlich-rechtlichen Sendeanstalt? Haben die beim WDR sich das ausgedacht oder gibt es im Web jede Menge Seites, die schnell und falsch voneinander abschreiben? --62.226.227.152 15:42, 1. Mär. 2017 (CET)Beantworten
Hmmmm - ob das Projekt Gutenberg des Spiegel-Verlags bei den guten Menschen aus Heidesee und Radeberg abschreibt oder all die Ritters und Kindermanns in Hamburg, weiß ich natürlich nicht - jedenfalls taucht das Gedicht auch nicht in der Gutenberg.de-Zusammenstellung auf. --85.178.195.31 16:45, 1. Mär. 2017 (CET)Beantworten
Na, dann schreib doch auch genau das als Begründung für deine Skepsis. Dann lohnt sich auch genaueres Hinsehen. Wenn ich es richtig sehe wird der Beitrag des WDR referenziert mit: Liederliches und Lyrisches/ Create Space. Create Space ist eine Self-Publishing-Plattform von Amazon. Die Quelle ist zwar korrekt angegeben, aber damit für unsere Frage nicht weniger unseriös wie Familie Kindermann. Über eurobuch.com sehe ich, dass das "wirkliche" Buch in diversen Ausgaben der 1940er Jahre antiquarisch erhältlich ist. Wenn es ein Antiquar in deiner Nähe hat lässt er dich vielleicht reingucken oder es sagt dir am Telefon ein netter Antiquar, was er findet und nicht findet (die Adressen der Antiquariate findet man sehr gut bei zvab.com). Weiterhin ist als Taschenbuch 2015 eine Neuausgabe erschienen (Endrikat, Fred: Liederliches und Lyrisches - Verse vom vergnüglichen Leben - , ISBN: 9783958220683), die sich von der ursprünglichen Ausgabe unterscheiden könnte. Das käme als Ursache für den Widerspruch auch in Frage. Gruß --2003:46:A0F:5600:E22A:82FF:FEA0:3113 19:20, 1. Mär. 2017 (CET) (hier auch als 62.226.227.152 unterwegs) Nachtrag: GBS kennt auch eine Ausgabe: Leda Verlag, 1967 (sic!), ISBN 3764521023, 9783764521028. --2003:46:A0F:5600:E22A:82FF:FEA0:3113 19:32, 1. Mär. 2017 (CET) Korrektur / Ergänzung: Die Berliner Blanvalet-Ausgabe reicht bis mindestens 1951 zurück ("Neuausg., 180. - 185. Tsd.", bei eurobuch.com sehe ich sogar 174.-185. Tsd. 1950). Und neben der Omnium-Ausgabe von 2015 gibt es noch eine Renovamen-Ausgabe von 2016. Die DNB datiert die erste Buchwarte-Ausgabe, 1.-20. Tsd. auf 1940 und nicht wie oben Vsop herausfand, 1942. 1940 deckt sich auch mit den Angaben im Projekt Gutenberg. Nach Joachim Wittkowsk, S. 129, den Vsop oben anführt, erreicht das Bändchen noch 1940 das 227. Tausend. --62.226.227.152 22:50, 1. Mär. 2017 (CET)Beantworten

lotte gegen dortmund

Was kam gestern in der ARD um 20.15 anstelle des abgesagten Spiels Lotte gegen Dortmund? Gruß

--217.251.201.249 14:19, 1. Mär. 2017 (CET)Beantworten

Ein anderes Pokalspiel. --MrBurns (Diskussion) 14:20, 1. Mär. 2017 (CET)Beantworten
Danke!217.251.201.249 14:22, 1. Mär. 2017 (CET)Beantworten
Die ARD behauptet: ein alter Tatort. --Wrongfilter ... 14:22, 1. Mär. 2017 (CET)Beantworten
Na erstmal haben sie 45 Min. (LIVE !) vor Ort über das Nichtspiel gesprochen. Zur besten Sendezeit! GEEZER … nil nisi bene 14:28, 1. Mär. 2017 (CET)Beantworten
Das war doch zum Schutz der Angestellten des Senders. Was glaubst Du was losgewesen wäre, wenn ein zu spät einschaltender erst mal einen Tatort gesehen hätte, oder sonstewas? So viele Todesdrohungen kann man keinem Menschen an einem Servicetelefon zumuten. So sahen die erst mal - nichts, gespickt mit irgendwelchen Kommentaren, aber den Zuschauern war klar, dass da irgendwas mit dem Spiel nicht so läuft wie erwartet. Den Stress in den vielen Wohnzimmern möchte ich allerdings trotzdem nicht mitgekriegt haben. --Elrond (Diskussion) 14:37, 1. Mär. 2017 (CET)Beantworten
Dafür hätte wohl ein Insert gereicht. Also unten im Bild ein Text, wo sowas steht wie "Das Pokalspiel Lotte - Dortmund wurde wegen Schneefall abgesagt" (eventuell in Laufschrift, falls die Schrift sonst zu klein wäre). --MrBurns (Diskussion) 14:44, 1. Mär. 2017 (CET)Beantworten
Du glaubst gar nicht, was aufgeregte Fans zu übersehen im Stande sind! Kein Scherz, sondern erlebte Realität. --Elrond (Diskussion) 15:12, 1. Mär. 2017 (CET)Beantworten
(BK)Ich glaub eher, der Tatort wäre das Ersatzprogramm gewesen, wenn (z.B. wegen großflächigen Unwettern) alle Pokalspiele mit entsprechender Ankickzeit (also 28.02., 20:45 oder früher, waren nur 2) entfallen wären. Jedenfalls lief wie ich kurz bei ARD vorbeigezappt habe (ich glaub um ca. 21:00) dort definitiv ein Fußballspiel. War aber wohl nur eine Aufzeichnung oder Zusammenfassung, da kein anderes Match am Hauptabend war, jedenfalls laut DFB-Pokal_2016/17#Viertelfinale. Müsste demnach eine Aufzeichnung oder Zusammenfassung von Frankfurt - Bielefeld gewesen sein.
@Geezer: ich hab definitiv Fußballer herumrennen gesehen. --MrBurns (Diskussion) 14:33, 1. Mär. 2017 (CET)Beantworten
Das war die Zusammenfassung des Spiels Frankfurt gegen Bielefeld (das war schon vorher zu Ende, da der Anstoß um 18.30 Uhr war). Danach kam ein Tatort. Gruß --Mikered (Diskussion) 14:54, 1. Mär. 2017 (CET)Beantworten

Google hilft: [6]. --Ailura (Diskussion) 14:57, 1. Mär. 2017 (CET)Beantworten

Erst lief die Sportschau live aus Lotte mit Alexander Bommes und Mehmet Scholl, die über das nicht stattfindende Spiel sprachen. Dann kam die ausführliche Zusammenfassung des Spiels Frankfurt-Bielefeld und danach ein kurzer Ausblick auf die heutige Partie Bayern-Schalke. Dann kam dann der Berliner Tatort „Gefährlicher Glaube“ oder so ähnlich, während dem hin und wieder unten links die Einblendung kam, dass das Spiel „wegen Schneetreibens“ (oder so ähnlich) ausgefallen sei (tatsächlicher Grund war wohl eher eine allgemeine Unbespielbarkeit des Platzes, das Schneetreiben am Abend setzte dem Ganzen nur die Krone auf – als die Sportschau lief hatte es jedenfalls schon aufgehört zu schneien), danach dann die Tagesthemen, die eigentlich während der Halbzeit des Lotte-Dortmund-Spiels hätten ausgestrahlt werden sollen. Ich weiß es sicher, denn ich saß die ganze Zeit (moderat angepisst) vor der weiter nebenbei laufenden Glotze (frag mich aber nicht, wer bei dem Tatort die Leiche und wer der Täter war)... --Gretarsson (Diskussion) 15:27, 1. Mär. 2017 (CET)Beantworten

Danke für alle Antworten, besonders für die letzte von Gretarsson! 217.251.201.249 15:29, 1. Mär. 2017 (CET)Beantworten
Der Tatort hieß „Blinder Glaube“ (Nr. 703 vom 31. August 2008). --Rôtkæppchen₆₈ 15:46, 1. Mär. 2017 (CET)Beantworten
Kommt hin, es ging da auch um Blinde bzw. um Medizintechnik den Sehsinn betreffend...
@217.251.201.249: Keine Ursache. --Gretarsson (Diskussion) 15:51, 1. Mär. 2017 (CET)Beantworten
Zugegeben off topic: Was ich aber aber immer wieder bewundere, ist die Vielfalt der Live-Spiele, die das Erste überträgt! Beispiel: 2. Hauptrunde: Bayern-Augsburg; Achtelfinale Bayern-Wolfsburg und Lotte-BVB. 2015 sah es so aus: 2. Hauptrunde Wolfsburg-Bayern, Achtelfinale Bayern-Darmstadt und Augsburg-BVB; Viertelfinale: Bochum-Bayern und Stuttgart-BVB.--IP-Los (Diskussion) 16:18, 1. Mär. 2017 (CET)Beantworten
Da die Anhänger/Sympathisanten von Bayern und Dortmund wohl die größten Gruppen unter den deutschen Fußballfans stellen, kommt es mit dieser Spielauswahl sicher den Wünschen/Erwartungen der relativen Mehrheit der fußballinteressierten ARD-Gucker nach. Kann aber auch sein, dass es hier den Spielansetzungen des DFB „ausgeliefert“ ist, weil es nur die Übertragungsrechte für die 20.45-Uhr-Partien hat. Ist dann halt ’ne Frage, inwieweit die ARD bei den Ansetzungen Einfluss auf den DFB hat. Davon abgesehen kann ja die ARD nix dafür, dass Dortmund und die Bayern die letzten Jahre immer mindestens bis ins Viertelfinale gekommen sind. Darüber hinaus darf man sich dann noch für eine Erfindung namens Pay-TV im Speziellen und Privatfernsehen im Allgemeinen bedanken, mit denen der ÖRR um die Übertragungsrechte konkurriert, für die der Preis vom DFB somit entspechend hoch angesetzt werden kann. Wäre er niedriger und die Konkurrenz geringer, könnte man sich die Übertragungsrechte für mehr Spiele sichern, sodass man die eine oder andere Partie ohne Beteiligung der Bayern oder Dortmunds dann evtl. bei den Dritten oder auf einem der Digitalkanäle der beiden „Großen“ ÖRR verfogen könnte. Aber dann beschwerten sich wahrscheinlich wieder die Fußball-Nichtmöger, warum denn der ÖRR ständig Fußball überträgt... --Gretarsson (Diskussion) 18:23, 1. Mär. 2017 (CET)Beantworten
Gerade beim ÖRR, der sich ja hauptsächlich durch Zwangsgebühren und nicht quotenabhängige Werbung finanziert (erst recht nicht zur Primetime, weil da Werbepausen auf ARD/ZDF verboten sind), könnte man sich etwas mehr Ausgeglichenheit erwarten. Ansonsten weiß ich nicht wie es beim DFB-Pokal ist aber generell ist es im Fußball heute üblich, dass der Ausrichter nur die möglichen Beginnzeiten sowie eventuell die Zahl der Spiele pro Beginnzeit festlegt und der Fernsehsender dann bestimmt, welches Spiel zu welcher Beginnzeit läuft. --MrBurns (Diskussion) 19:00, 1. Mär. 2017 (CET)Beantworten
Ich wage zu bezweifeln, dass das so ist (woher beziehst du dieses Wissen?), zumindest dass hier die ARD das letzte Wort hat. Die Spiele werden auch auf Sky übertragen und die machen Werbung in der Halbzeitpause. Es ist also für Sky lukrativer als für die ÖR, wenn die Partien der Bayern und des BVB um 20.45 angepfiffen werden. Ist aber gut möglich, dass der DFB von sich aus die quotenträchtigeren Spiele auf die attraktiveren Anpfiffzeiten legt... --Gretarsson (Diskussion) 21:36, 1. Mär. 2017 (CET)Beantworten
Sky verdient hauptsächlich an den Abonnenten, die Werbung ist auch nur ein Nebengeschäft. Aber natürlich ist für Sky natürlich auch eine attraktivere Anpiffzeit besser, weil die Abonnenten werden ja nicht dazu gezwungen, Abonnenten zu werden und zu bleiben, vor allem ist es für Sky aber gut, wenn das Spiel nicht im sog. Free-TV läuft, weil was bringt das Abo wenn man das Match auch ohne Abo woanders schauen kann? Der einzige Grund warum es überhaupt noch spiele im Free-TV gibt ist dass die Vereine nicht nur an den TV-gGldern verdienen, sondern auch an Sponsorengeldern und für die Sponsoren sind Spiele im Free-TV wegen der höheren Reichweite interessante. --MrBurns (Diskussion) 07:32, 2. Mär. 2017 (CET)Beantworten
(BK) Die Entscheidung liegt bei der ARD. Das Problem ist folgendes: Das ist keine Vielfalt, sondern schlichtweg verkapptes Bayern- bzw. BVB-TV! Beispiel? Bayern-Augsburg. Es hätte aber auch die Partien Hoffenheim-Köln oder Nürnberg-Schalke gegeben. Diese wären deutlich spannender gewesen. Das hätte man nicht erahnen können? In diesem Falle schon. Es ist doch immer das Gleiche: der übermächtige Goliath Bayern gegen den David. Ab dem Halbfinale werden beide Spiele übertragen. Ab da sähe man Bayern also sowieso mit hoher Wahrscheinlichkeit (siehe Ergebnisse der letzten DFB-Pokale). Was ist mit Hamburg-Gladbach? Das hätte man so ansetzen können, daß es eben statt Lotte-BVB übertragen wird. Aber nein, es spielt der BVB gegen einen Drittligisten! Nichts gegen Lotte, aber Hamburg-Gladbach mutet ausgeglichener und damit interessanter an. Nun rate mal, wie oft Bayer Leverkusen und Gladbach zusammen im ZDF in der Champions League am Mittwoch zu sehen waren, wie oft Bayern und der BVB? (bei 6 Spieltagen 1x Gladbach, 2x Bayern und 3x Dortmund, also 1:5).
Also: Davon abgesehen kann ja die ARD nix dafür, dass Dortmund und die Bayern die letzten Jahre immer mindestens bis ins Viertelfinale gekommen sind. Nein, aber sie können etwas dafür, daß schon vorher fast ausschließlich Bayern gezeigt wird. Sie könnten für die Viertelfinalspiele wenigstens ein Spiel auswählen, an dem nicht Bayern oder der BVB beteiligt ist. Die ARD kann zwei Spiele übertragen. Eine echte Chance hätten andere Vereine nur noch, wenn durch die Auslosung Bayern und der BVB schon da gegeneinander spielten. Mir geht es hier als neutraler Beobachter schlichtweg um Ausgewogenheit, die aber von den öffentlich-rechtlichen Sendern eben nicht mehr geleistet wird.
Wäre er niedriger und die Konkurrenz geringer, könnte man sich die Übertragungsrechte für mehr Spiele sichern, sodass man die eine oder andere Partie ohne Beteiligung der Bayern oder Dortmunds dann evtl. bei den Dritten oder auf einem der Digitalkanäle der beiden „Großen“ ÖRR verfogen könnte. Man könnte aber auch einfach mal andere Vereine berücksichtigen! Es zahlen alle Rundfunkgebühren, nicht nur Bayern- und BVB-Fans, sondern auch Fans anderer Vereine, einfache Fußballinteressierte und Fußballhasser. Da sollte man dann doch schon mal auf etwas mehr Vielfalt achten, denn "vielfältige Inhalte" kann ich da beim besten Willen nicht mehr erkennen.--IP-Los (Diskussion) 19:35, 1. Mär. 2017 (CET)Beantworten
Zumal gerade das Phänomen, dass der BVB eindeutig der zweitpopulärste Verein in Deutschland ist, m.E. ein relativ neues Phänomen ist. Ich denke vor 10 Jahren war der BVB noch nicht viel populärer als z.B. Schalke. Da frag ich mich schon auch, ob der BVB diesen Popularitätssprung nicht auch dem TV zu verdanken hat... --MrBurns (Diskussion) 19:49, 1. Mär. 2017 (CET)Beantworten
@MrBurns: Viele heutige Dortmund-Fans sind es wahrscheinlich schon seit der Hitzfeld-Ära in den 90ern, unter Sammer wurde Dortmund dann auch 2002 nochmal Meister. Da ist also schon eine gewisse Kontinuität vorhanden. Der aktuelle Popularitätsschub der Dormunder geht eindeutig auf die Klopp-Ära zurück, der aus einer Truppe voller junger Nobodys ein Meisterteam formte. Sowas macht Eindruck auch bei Anhängern anderer Vereine. Hinzugehen und zu behaupten, dass an deren aktueller Popularität auch das Fernsehen seinen Anteil hätte, schmälert m.E. die Leistungen, die man von dieser Mannschaft in den letzten Jahren geboten bekommen hat. Welcher Bundesligaverein kann denn von sich behaupten, schon mal Real Madrid in einem Pflichtspiel mit 3:0 besiegt zu haben? Ich bin kein erklärter BVB-Fan, schaue mir aber aktuell, so ich die Zeit habe, jedes Live-Spiel dieses Clubs gerne an, was ich so über den HSV, Gladbach, Leverkusen, Wolfsburg, Mainz oder Schalke sicher nicht sagen kann. --Gretarsson (Diskussion) 22:10, 1. Mär. 2017 (CET)Beantworten
Ich denke, ich habe oben im ersten Satz den wohl gewichtigsten Grund genannt, warum Bayern und der BVB präsenter bei den Live-Übertragungen der ÖR sind als andere Vereine. Dass mag denen nicht schmecken, die ebenjene anderen Vereine favorisieren, indes ist dies hier der falsche Ort, sich darüber zu echauffieren. Schreib halt ’ne Hassmail an die Programmverantwortlichen. Ich kann’s nicht ändern und ich hab auch nichts entschuldigt, nur zu erklären versucht. --Gretarsson (Diskussion) 22:10, 1. Mär. 2017 (CET)Beantworten
Ich wollte lediglich aufzeigen, daß die ÖR hier eben nicht ausgewogen auswählen, mehr nicht. Der Grund, warum man hier auf Eintönigkeit setzt, ist klar. Deshalb habe ich auch versucht, Deine weiteren Vermutungen richtigzustellen. Aber dieses extreme Ungleichgewicht fällt nicht nur mir als wie gesagt neutraler Beobachter auf, sondern durchaus auch anderen.
Ach ja: Schalke hat Real 4:3 im Bernabeau-Stadion geschlagen und wäre damit fast weitergekommen (womit niemand gerechnet hätte) - das war auch eine reife Leistung (ohne die des BVB schmälern zu wollen). Da zeigt sich aber: nicht nur der BVB spielt tollen Fußball, und manchmal wäre es doch auch mal ganz angenehm, andere Mannschaften sehen zu können (so wie eben das Hinspiel zwischen Schalke und Real damals).--IP-Los (Diskussion) 00:47, 2. Mär. 2017 (CET)Beantworten

Tod von Kirsten Erl

Wie ich gelesen habe, ist die TV Richterin Kirten Erl am 10.02.1017 verstorben. Wird das eingetragen auf der Seite von Frau Erl? In Wikipedia! Hier der Link der Traueranzeige https://waz.trauer.de/Traueranzeige/Kirsten-Erl

VIELEN Dank --5.146.169.43 14:32, 1. Mär. 2017 (CET)Beantworten

Reicht das als Quelle? Es könnte ja zufällig eine nicht prominente Kirsten Erl mit dem selben Geburtsdatum geben. Weder Kirsten noch Erl sind seltene Namen. --MrBurns (Diskussion) 14:38, 1. Mär. 2017 (CET)Beantworten
Und zu Dülmen haben beide dann auch irgendwie eine Beziehung?! --Elrond (Diskussion) 14:41, 1. Mär. 2017 (CET)Beantworten
Trag es doch selber ein Kirsten Erl --Elrond (Diskussion) 14:39, 1. Mär. 2017 (CET)Beantworten
Ich hab das mal erledigt. Name + Geburtsdaten + Ort (Dülmen/Essen) sollten als Identitätsmerkmale ausreichen. Ansonsten ist von Fr. Erl aus der Zeit nach ihrer TV-Präsenz auch nicht viel zu finden. --Magnus (Diskussion) 15:46, 1. Mär. 2017 (CET)Beantworten
Ich sehe es bis heute nicht redaktionell bestätigt. Wer kann eine Anzeige aufgeben? Die Seite gehört einem Medienunternehmen aus Bremen. Google blendet einige Suchtreffer aus. --Hans Haase (有问题吗) 17:23, 1. Mär. 2017 (CET)Beantworten
wp:Q fordert keine redaktionelle Bestätigung, sondern nur „überprüfbare Informationen aus zuverlässiger Literatur“. Insofern ist eine auf einem Trauerportal wiedergegebene Todesanzeige einer renommierten Zeitung erstens überprüfbar und zweitens zuverlässig, zumal diese Zeitung aufgrund ihrer Geschichte selbst von Rübenkresseanhängern konkludent als zuverlässig eingestuft wird. --Rôtkæppchen₆₈ 02:03, 2. Mär. 2017 (CET)Beantworten

Ich weiss aus eigener Erfahrung, das seriöse Zeitungen das überprüfen. Traueranzeige des Unternehmens ging nur nach Angabe des Bestattungsunternehmens, das dann die Tatsachen bestätigt hat. --Eingangskontrolle (Diskussion) 10:05, 2. Mär. 2017 (CET)Beantworten

Google Instant

Die Autovervollständigung beim Eintippen in das kleine Suchfeld meines Browsers (aktueller Firefox) oben rechts funktioniert nicht mehr, obwohl ich nichts geändert habe und "Suchen mit Google" eingestellt ist. Auch bei deaktiviertem Adblock tut sich nichts. Woran könnte das liegen?--Hubon (Diskussion) 15:39, 1. Mär. 2017 (CET)Beantworten

Hast du das schon durchgearbeitet: [7]? --Nurmalschnell (Diskussion) 15:58, 1. Mär. 2017 (CET)Beantworten
Ja, danke. Bei mir steht "aktiviert". Und bei der Google-Seite selbst funktioniert es auch, aber eben nicht (wie zuvor) bei dem kleinen Suchfeld oben rechts. ???--Hubon (Diskussion) 16:22, 1. Mär. 2017 (CET)Beantworten
Hast Du Autocomplete in den Firefox-Einstellungen aktiviert?[8] --Rôtkæppchen₆₈ 18:29, 1. Mär. 2017 (CET)Beantworten
Jetzt geht's wieder! Die Lösung steht auf dieser Seite. Bloß weiß ich jetzt nicht wirklich, warum sich das offenbar von alleine umgestellt hat. Kann so etwas denn auch ohne eigenes Zutun passieren oder übernimmt hier etwa langsam die Netz-KI die Weltherrschaft...?--Hubon (Diskussion) 04:01, 2. Mär. 2017 (CET)Beantworten

Hochschulregeln der Birzait Universität

Im Artikel zur Birzait-Universität wird dargestellt, dass die Universität "Judenrein" zu sein hat, und das "aufgrund einer entsprechenden Rechtsnorm der Hochschule", so dass kein Jude die Universiät betreten darf. Dazu habe ich keine Regelungen auf den Seiten der Universität gefunden. Weiß einer von Ihnen, wo ich diese Hochschulregel finden kann? Über eine Auskunft wäre ich sehr dankbar. --88.76.146.115 16:32, 1. Mär. 2017 (CET)Beantworten

Kannst Du mal einen Link reinstellen! Birzait-Universität finde ich nicht. --80.187.96.246 16:42, 1. Mär. 2017 (CET)Beantworten

Über Trottel-Institutionen, die sich fälschlich "Universität" nennen, diskutieren wir hier nicht. So einfach ist das. --109.100.148.154 16:43, 1. Mär. 2017 (CET)Beantworten
Universität Bir Zait ... GEEZER … nil nisi bene 16:46, 1. Mär. 2017 (CET)Beantworten
(BK)Linkservice: Universität Bir Zait. Von "judenrein" steht dort nichts. In dem als Einzelnachweis angeführten Artikel von Amira Hass steht nicht, dass Juden im Allgemeinen die Universiät nicht betreten dürften, sondern jüdische Israelis. --Digamma (Diskussion) 16:48, 1. Mär. 2017 (CET)Beantworten

...dass Juden im Allgemeinen die Universiät nicht betreten dürften, sondern jüdische Israelis Wo ist denn der Unterschied? Übrigens ist die HP schwer erreichbar, Seitenwechsel fast unmöglich. Also bislang kein eigener Eindruck. --80.187.96.246 17:00, 1. Mär. 2017 (CET)Beantworten

Unterschied: Angehörige der Besatzungsmacht. --Digamma (Diskussion) 17:12, 1. Mär. 2017 (CET)Beantworten
Naja. 14,3 Millionen Juden weltweit, davon 6,2 Millionen Israelis. Mathe? Marcus Cyron Reden 18:17, 1. Mär. 2017 (CET)Beantworten

Oh, meine Wortwahl war wohl nicht so ordentlich, Ich muß also eine Unterscheidung zwischen Juden aus Israel und normalen Juden machen? Das ändert jedoch nichts an meiner Frage: Wo finde ich die Quelle, dass Juden aus Israel die Bir Zait - Universiätät nicht betreten dürfen?--88.76.146.115 17:06, 1. Mär. 2017 (CET)Beantworten

Ich vermute mal, dass die das nicht auf die Homepage schreiben werden. --Digamma (Diskussion) 17:15, 1. Mär. 2017 (CET)aBeantworten
...Unterscheidung zwischen Juden aus Israel und normalen Juden machen? Was sind denn "normale Juden" Hallo! --80.187.96.246 17:35, 1. Mär. 2017 (CET)Beantworten
Die HP der Uni ist inzwischen bekannt und bei mir eof hierzu!! --80.187.96.246 17:40, 1. Mär. 2017 (CET)Beantworten

Bevor das hier noch ausartet, erlaube ich mir, (auch) nochmal auf die Frage zu verweisen. Kann also jemand eine solche Regelung bestätigen und weiß auch, wo sie steht? Fall ja, dann bitte kurz erläutern. Falls nein, ist das ja nicht schlimm. :-) VG --Apraphul Disk WP:SNZ 17:43, 1. Mär. 2017 (CET)Beantworten

Damit wir nicht alle zugleich suchen: Die Universität hat zahlreiche ihrer Satzungen auf der Website veröffentlicht. Bei oberflächlicher Durchsicht (per Google-Übersetzung) der vielversprechendsten Dokumente habe ich eine entsprechende Regelung nicht gefunden. Die Grundordnung der Universität ist allerdings nur in der Fassung von 2015 bereitgestellt [9] - nachdem der Vorfall 2014 war und die Regelung laut einer der Quellen "seit zwei Dekaden" bestand, kann es natürlich sein, dass sie nach dem Vorfall (bzw. wegen des Vorfalls) geändert wurde. Die Grundordnung stammt wohl von 1979 und wurde erstmals 1989 geändert - mein heißester Kandidat wäre also die zweite Fassung. --Rudolph Buch (Diskussion) 18:46, 1. Mär. 2017 (CET)Beantworten
Eben, der entsprechende Satz ist im Konjunktiv (indirekte Rede) geschrieben und macht damit deutlich, dass er keine absoluten Wahrheiten verkündet. Dass Frau Hass von der Konferenz ausgeschlossen wurde, ist wohl ein unbestrittenes Faktum, und dass es deshalb war, weil sie Israelin und Jüdin ist, ist nun nicht allzu abwegig... --Gretarsson (Diskussion) 18:39, 1. Mär. 2017 (CET)Beantworten
Nur eben nicht belegt und wenn man dies noch eine Ecke weitertreiben lässt, ist Wiki der Beleg dafür!! --80.187.96.12 20:56, 1. Mär. 2017 (CET)Beantworten
Der Nebensatz "da sie sich als israelische Jüdin aufgrund einer entsprechenden Rechtsnorm der Hochschule nicht dort aufhalten dürfe" ist seit meiner heutigen Änderung im Konjunktiv. Davor stand da "darf". Ich habe "darf" durch "dürfe" ersetzt, weil die einzige Quelle der Zeitungsartikel ist, in dem Amira Hass auch nur die Aussagen von Mitarbeitern der Uni zitiert.
Also: Beleg ist der Haaretz-Artikel von Amira Hass, aber der gibt auch nur wieder, dass andere behaupten, es gäbe so eine Bestimmung. Belegt ist aber, dass Amira Hass unter Berufung auf so eine Bestimmung vom Campus verwiesen wurde. --Digamma (Diskussion) 21:10, 1. Mär. 2017 (CET)Beantworten
Hättest du es nicht geändert, hätte ich es getan. So wie der Satz jetzt dort steht, kann er stehen bleiben. Wenn sich Frau Hass auf Mitarbeiter der Uni bezieht, dann ist das erst mal so hinzunehmen. Die werden ihr ja hoffentlich keinen Unsinn erzählt haben. Zudem ist Hass offensichtlich Links und als Mitarbeiterin von Haaretz nun nicht als zionistisch und antipalästinensisch einzustufen, im Gegenteil, dürfte sie zu den israelischen Juden gehören, die von zionistischen Hardlinern gerne als „jüdische Selbsthasser“ tituliert werden. Im Übrigen kann man hier nachlesen, dass Ilan Pappe für einen Vortrag an die Bir-Zait-Uni eingeladen wurde und aufgrund der betreffenden Vorschrift diesen „off campus“ halten musste. Hass war also nicht die einzige Betroffene, sondern leider „nur“ im Vorfeld schlecht informiert oder ein „Opfer“ des Umstandes, dass man wegen ihr allein nicht die Konferenz einfach woanders abhalten konnte. --Gretarsson (Diskussion) 22:52, 1. Mär. 2017 (CET); nachträgl. erg. --Gretarsson (Diskussion) 00:12, 2. Mär. 2017 (CET)Beantworten

Ich merke mal an, dass Ramallah in der Area A liegt, jüdischen Israelis dort der Zutritt also normalerweise von israelischer Seite ohnehin untersagt ist. Insofern wird es nicht gerade viele zufällige Besucher geben, für die man die Regelung öffentlich machen müsste. --Chricho ¹ ³ 14:23, 2. Mär. 2017 (CET)Beantworten

Grenzschutz in Österreich

Ich habe jetzt eine längere Zeit gesucht, finde es aber nicht. Welche Institution entspricht in Österreich der Bundespolizei in Deutschland, wem obliegt die Grenzsicherung? --Marcus Cyron Reden 18:10, 1. Mär. 2017 (CET)Beantworten

en:Template:Border guards weiß es. --Rôtkæppchen₆₈ 18:22, 1. Mär. 2017 (CET)Beantworten
Vielleicht kann man das noch bei Österreichische Grenzübergänge in die Nachbarstaaten eintragen. --Stefan »Στέφανος«⸘…‽ 18:24, 1. Mär. 2017 (CET)Beantworten
Leider steht es nicht wirklich im Artikel. Da wird einmal ein Gesetz erwähnt, das war es aber auch schon. Bin echt überrascht, daß sich das in unserer Wikipedia bislang nicht wirklich als Aussage findet. Marcus Cyron Reden 18:28, 1. Mär. 2017 (CET)Beantworten
Das macht die Fremdenpolizei. --M@rcela 18:30, 1. Mär. 2017 (CET)Beantworten
(BK) Hoffentlich war meine Änderung jetzt nicht völlig falsch. Plani bitte auf den Plan, Plani bitte. --Stefan »Στέφανος«⸘…‽ 18:31, 1. Mär. 2017 (CET)Beantworten
Das ist freilich alles nicht ganz hundertprozentig korrekt. Der Schutz der österreichischen Außengrenzen obliegt seit der Zusammenführung von Zollwache und Polizei dem Wachkörper Bundespolizei, also der "ganz normalen" Polizei. Das ist freilich nur die halbe Wahrheit, weil spezialisierte AGM (Schengen-Ausgleichsmaßnahmen-)Dienststellen gegründet wurden, deren Beamte sich großteils aus ehemaligen Mitgliedern der Zollwache rekrutierten. Diese sind schwerpunktmäßig mit der Grenzüberwachung betraut. Nachdem Österreich aber ausschließlich von anderen Schengen-Mitgliedsstaaten umgeben ist, sind tatsächliche Grenzkontrollen an Grenzübergängen eine Ausnahme. Wie der Name "Schengen-Ausgleichsmaßnahmen" schon erahnen lässt, finden Kontrollen eher im Grenzhinterland durch die sog. Schleierfahndung statt. Die einzigen tatsächlichen Grenzkontrollen in diesem Sinne gibt es noch an den österreichischen Flughäfen, wobei diese eben von Beamten der Bundespolizei besetzt sind. Ach ja, die Angelegenheiten des Zollwesens an Flughäfen und im Grenzhinterland übernimmt übrigens nicht die Bundespolizei, sondern die Operative Zollaufsicht. Ich hoffe, mit dieser Antwort geholfen zu haben. Beste Grüße, Plani (Diskussion) 11:10, 2. Mär. 2017 (CET)Beantworten
+ der Einsatz des ÖBH [10], [11], oder ist der vorbei? --213.147.162.52 13:02, 2. Mär. 2017 (CET)Beantworten
Das Österreichische Bundesheer steht fallweise (Stichwort: Flüchtlingskrise) an der Grenze im Assistenzeinsatz für die Bundespolizei. Grundsätzlich zum Schutz der Grenze zuständig ist das Bundesheer aber nicht. Beste Grüße, Plani (Diskussion) 13:31, 2. Mär. 2017 (CET)Beantworten

Schraubglas öffnen

Wenn man ein Schraubglas, z.Bsp. saure Gurken gerade ebends, beim ersten Versuch nicht aufgedreht bekommt, und dann mit der Hand gegen den Glasboden schlägt, und es geht dann auf, was passiert bei diesem Vorgang, daß das Glas dann aufgeht?--Sigismund Sülzheimer (Diskussion) 18:19, 1. Mär. 2017 (CET)Beantworten

Dazu gibts anscheinend unterschiedliche Theorien.[12] --MrBurns (Diskussion) 18:39, 1. Mär. 2017 (CET)Beantworten
Es liegt an der Veränderung des Luftdrucks im Glas. Wenn man dagegen schlägt wird der Deckel entweder von oben oder von unten ganz leicht gewölbt. Dadurch kann ein bisschen Luft eindringen und der Unterdruck, der im Glas herrscht und das Aufmachen vorher erschwert hat wird ausgleichen. Hier ist eine Quellenangabe Das Buch der 100 Fragen und Antworten S.310,311 Verlag :LOEWE ISBN 3-7855-2381-5 1993 Dort steht das auch drin ,bloß hat das Glas dort noch eine Folie zwischen der Marmelade und deem Deckel. --Fiver, der Hellseher (Diskussion) 18:48, 1. Mär. 2017 (CET)Beantworten
Danke für die Infos. --Sigismund Sülzheimer (Diskussion) 19:05, 1. Mär. 2017 (CET)Beantworten
Es hilft auf den Schraubdeckel zuschlagen. Die Dichtung verharzt und verklebt. Der Druck wird von den in das Glasgewinde ragenden Blechnasen im Deckel gehalten. Durch den Schlag geben diese nach, da das Metall gespannt ist. Ein Druckunterschied kann durch den Schlag auf die Dichtung spätestens durch Verformung verringert werden. --Hans Haase (有问题吗) 21:08, 1. Mär. 2017 (CET)Beantworten

Protipp: Mit einem Buttermesses oder gleichfalls geeigneten Gegenstand an einer Stelle den Deckel seitlich leicht anhebeln. Überlicherweise macht es dann "pfflluuuupppfff" und der Unterdruck ist ausgeglichen, der Deckel kinderleicht abschraubbar. Hat eine extrem bessere Erfolgsquote als das Glas zu prügeln und weniger schmerzhaft an den Händen ist es auch (wenn man nicht mit dem Messer abrutscht). Benutzerkennung: 43067 15:01, 2. Mär. 2017 (CET)Beantworten

Das ist zwar auch meine Methode der Wahl, speziell bei Sachen die kleben können wie Konfitüre & Co, allerdings hab ich schon erlebt, daß das Glas der Klügere war und nachgab, dann ist das Glas und der Inhalt meist hin. --Elrond (Diskussion) 15:41, 2. Mär. 2017 (CET)Beantworten
Man muss da nicht wie blöde auf den Glasboden einprügeln. Zumindest bei 720-ml-Gläsern reicht ein einfacher beherzter Schlag mit dem Handballen völlig aus. Das tut nicht weh und geht drei mal schneller als erstmal einen geeigneten Gegenstand rauszusuchen, dann mit diesem am Deckel rumzuhantieren um sich am Ende vielleicht noch ein Auge auszustechen... --Gretarsson (Diskussion) 15:24, 2. Mär. 2017 (CET)Beantworten
Früher stach man ein Loch in den Deckel. Die Ahle dazu hatte man sowieso in der Küchenschublade, wegen der Dosenmilch für den Kaffee. --Rôtkæppchen₆₈ 15:28, 2. Mär. 2017 (CET)Beantworten
Womit dann aber der Deckel hin ist... Und heutzutage wissen viele wahrscheinlich nicht mal, wie man „Ahle“ schreibt oder wie eine aussieht, geschweige denn, dass sie eine besitzen... --Gretarsson (Diskussion) 15:31, 2. Mär. 2017 (CET); nachträgl. erg. 15:34, 2. Mär. 2017 (CET)Beantworten
Tesa drauf, tschüs. --Rôtkæppchen₆₈ 15:53, 2. Mär. 2017 (CET)Beantworten
Ja, genau, erst fummel ich die Ahle raus um ein Loch in den Deckel zu machen, dass ich anschließend dann mit Tesafilm wieder zukleben darf. Kann ich mir mit der „Schlag-Methode“ beides schenken ;-) --Gretarsson (Diskussion) 16:02, 2. Mär. 2017 (CET)Beantworten

kurze werbeeinblendung: Vacumex! --JD {æ} 15:58, 2. Mär. 2017 (CET)Beantworten

Stimmt, so ein ähnliches Ding haben wir bei der Wohnungsauflösung irgend einer Großmutter auch gefunden. Es ruht in den Tiefen einer Küchenschublade, genauso wie ein professioneller Deckelöffner (Abb. ähnlich). --Elrond (Diskussion) 16:27, 2. Mär. 2017 (CET)Beantworten
Nochn Trick: heißes Wasser über den Deckel laufen lassen. Deckel dehnt sich aus, die Luft drunter auch. Rainer Z ... 19:16, 2. Mär. 2017 (CET)Beantworten
Wenn Luft unter dem Deckel ist, hat der Hersteller irgendetwas falsch gemacht. Normalerweise sollte Vakuum bzw sehr dünner Wasserdampf (7–11 hPa bei Kühlschranktemperatur, 23–32 hPa bei Raumtemperatur) unter dem Deckel sein. --Rôtkæppchen₆₈ 22:14, 2. Mär. 2017 (CET)Beantworten

Alternativen zur Mondlandung

Vor Jahren habe ich mal in einem Vortrag von Jesco von Puttkamer gehört das die NASA Kennedy eine Liste von 10 (evtl. auch nur sechs) Projekten vorgelegt hat wie man auf den Sputnikschock reagieren könne um die "Schande" nur zweiter zu sein auszugleichen.
Neben der dann tatsächlich realisierten Mondlandung weis ich noch als Alternativen eine Raumstation und eine Nuklearexplosion auf dem Mond (Projekt A119). Weis jemand was die anderen waren? --Dreifachaxel (Diskussion) 18:43, 1. Mär. 2017 (CET)Beantworten

Das passt alles nicht zusammen. Sputnick-Schock 1957, NASA-Gründung 1958 und Kennedy erst 1962 Präsident.--2.240.16.75 19:39, 1. Mär. 2017 (CET)Beantworten
Die erste Reaktion auf den Sputnik-Schock war dass die Amis versucht haben, einen Mann in den Weltraum zu schicken. Erst wie ihnen da auch die Russen zuvorgekommen sind (siehe Juri Gagarin), hat man sich Alternativen überlegt und sich dann bald für die Mondlandung entschieden, dann kam Kennedys berühmte Rede. Fun-Fact: es gibt die Vermutung, dass die Russen bei ihrem Mondlandungsprojekt nur gescheitert sind, weil Baikonur nicht so nahe am Äquator liegt wie das Kennedy Space Center und die Russen deshalb wegen dem höheren Energieaufwand für die Bahnkorrektur eine größere Rakete brauchten als die Amis hätten bauen müssen und diese Rakete haben sie dann nicht mehr so richtig hinbekommen. Andererseits war die Saturn V so überdimensioniert, dass sie sogar noch größer war als die N1, die Amerikaner hätten ja auch nicht unbedingt mit mit 3 Mann in den Mondorbit fliegen müssen. --MrBurns (Diskussion) 20:01, 1. Mär. 2017 (CET)Beantworten
Das sowjetische bemannte Mondprogramm ist eher deswegen gescheitert, weil die sowjetischen Raumfahrtingenieure der ihnen durch die Konzeption der N1-Rakete gestellte Herausforderung nicht gewachsen waren. Der modifizierte Entwurf für die erste Stufe sah 30 (in Worten dreißig) gleichzeitig brennende Haupttriebwerke vor. Das Konkurrenzprodukt Saturn V hatte insgesamt in allen drei Stufen nur insgesamt elf Triebwerke. --Rôtkæppchen₆₈ 00:14, 2. Mär. 2017 (CET)Beantworten
Ja, das war wohl das Hauptproblem, da die Amerikaner es geschafft haben, eine Rakete zu bauen, die auch für eine Mondmission von Baikonur stark genug gewesen wäre. Allerdings ist nicht klar, ob die Russen es nicht doch geschafft hätten, wenn ihr Weltraumbahnhof auch so weit südlich gewesen wäre und sie daher weniger Triebwerke gebraucht hätten (auch wenns noch immer viel mehr gewesen wären als die Saturn V hat). --MrBurns (Diskussion) 04:23, 2. Mär. 2017 (CET)Beantworten
PS: Laut der Formel von [13], Kapitel "Orbit Plane Changes", entspricht eine Inklinationsänderung von 19° (entspricht ungefähr dem Unterschied zwischen Baikonur und dem Kenndy Space Center) eine Geschwindigkeitsänderung, die ca. 33% der aktuellen Geschwindigkeit entspricht. Da man das nicht beim Start macht sondern möglichst weit oben, wo die Geschwindigkeit geringer ist, ist der Unterschied insgesamt aber nicht so groß. --MrBurns (Diskussion) 05:14, 2. Mär. 2017 (CET)Beantworten
Für einen Mondflug braucht man doch gar keine Bahn, die in der Äquatorebene liegt, sondern eine, die in der Mondbahnebene liegt, was ungefähr die Ekliptikebene ist. --Digamma (Diskussion) 10:33, 2. Mär. 2017 (CET)Beantworten
Das ist im Prinzip richtig, aber mit den damaligen Steuerungsmöglichkeiten war wohl nur ein Äquatorkurs praktikabel. --MrBurns (Diskussion) 10:36, 2. Mär. 2017 (CET)Beantworten
Auf Wettlauf ins All#Vom „Wettlauf ins All“ zum „Wettlauf zum Mond“ (und dem Rest des Artikels) steht ein bisschen was zum Thema. Am 20. April 1961 schickte Kennedy an seinen Vize Lyndon B. Johnson die Notiz: „Haben wir eine Chance, die Sowjets zu schlagen, indem wir ein Labor im Weltraum errichten oder durch einen Flug um den Mond oder durch eine Rakete zur Landung auf dem Mond, die einen Menschen hin- und zurückbringt?“ Johnson, gleichzeitig auch Leiter des National Aeronautics and Space Council, bejahte diese Fragen. --Neitram  09:54, 2. Mär. 2017 (CET)Beantworten
Der Artikel ist mir natürlich bekannt :-) Meine Frage war aber welche anderen Alternativen man in Erwägung gezogen hatte.
An die IP ganz oben ("passt alles nicht zusammen"): Es war keine Rede davon das die Liste einen Tag nach Sputnik vorgelegen habe... --Dreifachaxel (Diskussion) 14:13, 2. Mär. 2017 (CET)Beantworten

An die Lateiner unter uns

Habe heute Mittag den Artikel Conditio Jacobaea gelesen. Seitdem beschäftigen mich zwei Fragen:

  1. Müsste Jacobaea nicht kleingeschrieben werden?
  2. Woher leitet sich das ae in Jacobaea ab? Müsste es nicht einfach Jacoba oder Jacobia heißen?

Dank + Gruß, 2A02:8109:2C0:B54:10D0:631F:28B2:E93F 19:43, 1. Mär. 2017 (CET)Beantworten

Groß oder klein gibt sich nichts. Ae stellt den Genitiv dar. Nach so um 400 nach gelten andere Spielregeln. :-) --80.187.96.12 20:10, 1. Mär. 2017 (CET)Beantworten
"Hammer" doch alles: [[14]] --80.187.96.12 20:18, 1. Mär. 2017 (CET)Beantworten
Also hört mal! Es geht um das Suffix -ae; das ist eine Adjektivbildung "zu ... gehörig" nach griechischem Vorbild. Und die Endung lautet -a, für Mom. Sg. fem. Grüße Dumbox (Diskussion) 20:20, 1. Mär. 2017 (CET)Beantworten
wir reden über Conditio Jacobaea? Oder? --80.187.96.12 20:23, 1. Mär. 2017 (CET)Beantworten
Ich jedenfalls schon. Name Jacobus, abgeleitetes Adjektiv Jacob-ae-us, Femininum dazu Jacobaea. Groß übrigens wg. Eigenname, nach Konvention auch bei Adjektiven, wie im Englischen. Grüße Dumbox (Diskussion) 20:55, 1. Mär. 2017 (CET)Beantworten

[BK] - Genau: über Conditio Jacobaea. Jacobaea ist das vom Namen Jacobus abgeleitete Adjektiv und hat, was das ae angeht, nach Dumbox ein griechisches Vorbild. Kann es damit zusammenhängen, dass der ursprünglich hebräische Name (Jaákov) über den griechischen "Zwischenwirt" (Jakobos) ins Lateinische (Jakobus) gekommen ist und von daher die Adjektivbildung sich am Griechischen orientiert? Im Lateinischen finden sich doch auch andere von Namen abgeleitete Adjektivformen, zB Gregoriana. Andererseits: Caesarea Philippi ... *denk, *kratz, *schweig ... 2A02:8109:2C0:B54:10D0:631F:28B2:E93F 21:00, 1. Mär. 2017 (CET)Beantworten

Im Griechischen sind häufige Zugehörigkeitssuffixe -ios und -aios; -aios aber meist bei a-Stämmen. Mir scheint aber -aios auch besonders bei nichtgriechischen Namen vorzukommen. Vielleicht hat Aramäisch (man beachte die Wortbildung!) ein entsprechendes Suffix (Nisbe), das man einfach übernehmen konnte. Lateinisch wäre -icus, -ius, -anus, inus, -ensis und einige mehr. Grüße Dumbox (Diskussion) 21:26, 1. Mär. 2017 (CET)Beantworten
Dann beantwortet mal dem Fragesteller seine Fragen hinreichend. Grins + Grüße --80.187.96.12 21:03, 1. Mär. 2017 (CET)Beantworten
@Dumbox: Gratias ago! 2A02:8109:2C0:B54:F15E:767A:45B6:6350 08:37, 2. Mär. 2017 (CET)Beantworten
Mein gesammeltes Wissen zur Groß- und Kleinschreibung in neulateinischen Texten (aus einsichtigen Ursachen hatten die klassisch-lateinischen Texte kein Problem damit) hatte ich vor einiger Zeit im Artikel „Bipontum“ (mit Absicht hier nicht verlinkt) eingebracht. Vielleicht beginnt ja mal jemand einen Artikel zur neulateinischen Orthographie? --Pp.paul.4 (Diskussion) 09:13, 2. Mär. 2017 (CET)Beantworten

Warum befindet sich der CVR im Heck des Flugzeugs?

Warum befindet sich der Cockpit Voice Recorder im Heck des Flugzeuges? In der Vergangenheit ist es mehrfach vorgekommen, dass der CVR vorzeitig ausfiel, weil die Kabel, die vom Cockpit zu ihm führen, z.B. durch Feuer beschädigt wurden. Wenn er sich im Cockpit befände, könnte so etwas nicht passieren.

--188.118.248.227 20:56, 1. Mär. 2017 (CET)Beantworten

Bei einem Absturz könnte sich der Vorderteil des Flugzeugs in den Boden bohren. Damit würde der CVR erstens besonders stark beschädigt und zweitens besonders unzugänglich. Deswegen macht man das nicht. --Rôtkæppchen₆₈ 21:01, 1. Mär. 2017 (CET)Beantworten
Es wurden schon 12 Meter Tiefe in den Boden geschafft. Da bleibt nichts mehr übrig. --Hans Haase (有问题吗) 21:04, 1. Mär. 2017 (CET)Beantworten
Flugzeuge schlagen nicht immer mit der Nase voran auf. Sie können auch zuerst mit dem Heck aufschlagen. Gibt es Statistiken die zeigen, dass die Wahrscheinlichkeit, dass das Cockpit zu hart aufschlägt größer ist als dass das mit dem Heck passiert? --MrBurns (Diskussion) 21:17, 1. Mär. 2017 (CET)Beantworten
(BK) Weil das Heck das Teil des Flugzeuges ist, wo auf bezogen auf zig Unfällen einfach die geringste Wahrscheinlichkeit an eines Totalschaden des CVR zu erwarten ist. Und auch reisst das Heck beim Aufprall gerne ab, erst recht wenn es als erste Teil aufkommt, und schafft es dadurch meist nicht im Hautfeuer zu landen. Auch bei Abstürzen über Gewässer ist meist das Heck, das Teil das als ganzes geborgen werden kann (Da kriegt man relativ einfach ein Seil darum das nicht abrutscht). Schau dir einfach auch mal die Flugzeuge an die nach der Notladung usw. ausgebrannt sind, da bei sieht das Heck sehr oft noch "gut" aus, im Vergleich vom Rest vom Flugzeug jedenfalls (Asiana-Airlines-Flug 214, Emirates-Flug 521). Das CVR muss ja einiges aushalten können. Kurzum das CVR ist auch darauf ausgelegt alle Unfall Szenarien, die dem Heck zustossen könnten, zu überleben. An anderer Stelle eingebaut, müsste es noch massiver gebaut sein. --Bobo11 (Diskussion) 21:20, 1. Mär. 2017 (CET)Beantworten
Die FAR-Vorschrift besagt, dass das Flugzeug je ein CVR und FDR (Flight Data Recorder) haben muss. Einer muss dann möglichst nah am Cockpit und einer muss möglichst weit weg vom Cockpit eingebaut werden. Es ist den Konstrukteuren überlassen, welcher wo ist. Alles andere ist phantasiereiche Spekulation unserer beiden ausgewiesenen Laien der Auskunft aus A und CH.--2003:75:AF59:2800:A5B2:B441:10F4:20A3 22:33, 1. Mär. 2017 (CET)Beantworten
Dann erklär mal, wieso das dann mit dem Zusammenfassen im CVDR geht? Das wäre dann ja nicht Zulassfähigkeit. --Bobo11 (Diskussion) 23:07, 1. Mär. 2017 (CET)Beantworten
Bei den kombinierten Geräten besagt die Vorschrift, dass sie möglichst weit vom Cockpit eingebaut werden müssen, bzw. mindestens hinter den Tragflächen. Das Kriterium ist das Feuer durch den Sprit in den Tragflächen und nicht irgendwelche Crash-/ Überlebensbetrachtungen. Diese Beschleunigungen haben die Geräte, egal wo sie eingebaut sind auszuhalten. --2003:75:AF59:2800:F07D:E864:5F8:F7DF 23:29, 1. Mär. 2017 (CET)Beantworten
Stimmt leider nicht, der Einbauart hat sehr starken Einfluss darauf welche Beschleunigung überhaupt auf das Gerät einwirken kann. Und je mehr Blech es zwischen Felswand und Rekorder hat desto länger ist der Bremsweg. Und das heisst nicht anderes, als das die kinetische Energie wird über einen längerer Zeitraum abgebaut werden kann. Und Flugzeuge werden äussert selten von hinten gerammt. Und selbst wenn ist es die Geschwindigkeitsdifferenz die für Geräte-Beschleunigung Ausschlag gebend ist. Mir wäre gerade kein Unfall bekannt wo das Heck mit mehr als Lande/Start-Geschwindigkeit zuerst aufgekommen wäre. Mit Reiseflug Geschwindigkeit war es immer Nase voran. Und da bei Berechnung der Kinetische Energie die Geschwindigkeit im Quadrat Einfluss nimmt, ist diese durchaus ein nicht zu vernachlässigende Faktor. Also welches Unfallszenerio sich mit welcher Geschwindigkeit abspielt. --Bobo11 (Diskussion) 07:56, 3. Mär. 2017 (CET)Beantworten
Die aufgezeichneten Daten müssen 3400g aushalten und auslesbar bleiben, da ist es egal, wo sie im Rumpf eingebaut sind. Hör auf dich auch als Flugzeugbauer zu produzieren, das gibt einen Absturz.--2003:75:AF44:4D00:5D27:EEB3:CBAE:4858 13:06, 3. Mär. 2017 (CET)Beantworten
Dafür, dass du so vollmundige Behauptungen aufstellst (davon mindestens eine mit Sicherheit falsche, falls du dieselbe Person wie der Kommentierer von vorgestern 22:33 bist), wäre es schon mal an der Zeit, dass du Belege (Links) beibringst. Zufällig scheinen nämlich die Spekulationen der alpinen und sonstigen Bevölkerung in diesem Fall durchaus zutreffend zu sein, vgl. auch [15]. --91.89.146.92 15:44, 3. Mär. 2017 (CET)Beantworten
Deine Aussage stimmt so nicht. In den FAR wird im Normalfall für den CVR die Position hinten ("as far aft as practicable") vorgeschrieben, siehe FAR Sec. 23.1457 — Cockpit voice recorders (e)(1). --91.89.146.92 15:44, 3. Mär. 2017 (CET)Beantworten
Das habe ich nicht infrage gestellt. Der Flugzeugkonstrukteur, weiß aber, dass der Grund alleinig aus der Gefahr vor dem brennenden Sprit in der Tragfläche resultiert und nicht der Schutz vor dem crash. Da gibt es für die Recorder genauere und begründete Vorschriften bei RTCA/ EUROCAE. Die sucht euch selbst raus, es reicht, wenn ich es weiß.--2003:75:AF44:4D00:2806:45E6:AD03:CBA5 16:34, 3. Mär. 2017 (CET)Beantworten
Achso, das hast du "nicht infrage gestellt". Dann warst Du wohl doch nicht die IP, die bzgl. CVR und FDR "Es ist den Konstrukteuren überlassen, welcher wo ist" geschrieben hatte. ;) --91.89.146.92 18:43, 3. Mär. 2017 (CET)Beantworten
Noch etwas zum Fliegerischen: Ein Aufschlag am Heck weist oft darauf hin, dass die Piloten eine Landung versucht hatten. Dafür sind Landeklappen und Fahrwerk ausgefahren, und die Geschwindigkeit erheblich reduziert. Schlägt das Flugzeug mit der Nase ein, geschieht die Kollision mit einer sehr hohen Geschwindigkeit. Das ganze Flugzeug dient dann als Knautschzone für die Recorder.--Keimzelle talk 09:48, 2. Mär. 2017 (CET)Beantworten
Oder anders ausgedrückt: Ein Flugzeug bewegt sich grundsätzlich vorwärts. Somit ist das Heck am weitesten entfernt vom theoretisch Punkt des Aufschlags. Schlägt das Flugzeug nicht mit dem Bug auf, hat es zuvor die Richtung gewechselt und dabei fast zwangsläufig die Geschwindigkeit verringert. --Nobody Perfect (Diskussion) 16:21, 2. Mär. 2017 (CET)Beantworten
Richtig. Ein Flugzeug das vom Himmel fällt, ist in der Regel "nur" mit Fallgeschwindigkeit unterwegs (Dynamische Komponente von oben nach unten). Der klassische CFIT-Unfall (Controlled Flight Into Terrain = kontrollierter Flug in den Boden) ist Nase voran (jedenfalls im Bezug auf die (Haupt-)Richtung der (beschleunigte) Massenbewegung). --Bobo11 (Diskussion) 07:40, 3. Mär. 2017 (CET)Beantworten
Er kann es nicht lassen, sich als Laie weiter was anzulesen und die falschen Schlüsse als Obergutachter (Richtig!) herauszudrücken. "Klassische" CFIT gibt es auf der Welt gerade mal 1-2 Unfälle pro Jahr, also verschwindend klein.--2003:75:AF44:4D00:5D27:EEB3:CBAE:4858 11:59, 3. Mär. 2017 (CET)Beantworten

Welche Zeitung kann das gewesen sein?

Ich bin hinter Informationen zur Familie Mahlberg her. Hier, auf S. 55, gibt es einen Abschnitt zu Stella Mahlberg, in dem die Lebensdaten von Paul Mahlberg handschriftlich eingetragen sind. Findet jemand heraus, welches "Communist newspaper in Stuttgart" er damals herausgegeben haben soll? --Xocolatl (Diskussion) 21:11, 1. Mär. 2017 (CET)Beantworten

Der Link ist keine Zeitung HSB Rote Kapelle? --80.187.123.130 21:30, 1. Mär. 2017 (CET)Beantworten
Die verlinkte pdf hat weder as it noch nach eigener Seitenzahl den Verweis zu Stuttgart auf 55 im Text. Seiten wurden gelesen. Hat Zeit gekostet. Was willst Du? --80.187.123.130 21:38, 1. Mär. 2017 (CET)Beantworten
Ich habe irgendwie das Gefühl, dass du weder meine Frage noch den zugehörigen Text gelesen hast. Was ich will, ist eine Antwort auf die Frage, wie die angeblich kommunistische Zeitung geheißen hat, die angeblich Paul Mahlberg angeblich in Stuttgart herausgegeben hat. Schrieb ich aber schon oben. --Xocolatl (Diskussion) 21:42, 1. Mär. 2017 (CET)Beantworten
Ja, auf Seite 55 der pdf. Da gibts aber sowas nicht! --80.187.123.130 21:49, 1. Mär. 2017 (CET)Beantworten
Schau genauer hin. --Magnus (Diskussion) 21:51, 1. Mär. 2017 (CET)Beantworten
Ja habs verstandem --80.187.123.130 21:56, 1. Mär. 2017 (CET)Beantworten
So eine Zeitung gabs in S nicht. Wahrscheinlich ist was anderes gemeint. Allerdings lese ich noch. Grüße
Englischer Text nach 45. Handschriftliche Einträge kennst Du wohl. PM etc. Also S war zur Nazizeit die Stadt der Auslandsdeutschen. Wenn Dir die handschriftlichen Einträge zu Namen was sagen, waren sie dem Randgebiet. um D herum anzusiedeln?--80.187.123.130 22:09, 1. Mär. 2017 (CET)Beantworten
Rote Kapelle ist Dir klar? Grüße --80.187.123.130 22:13, 1. Mär. 2017 (CET)Beantworten
Ohne Antwort lass ich es still as bevor verweit auf dieser Seite weg von S - R: O --80.187.123.130 22:23, 1. Mär. 2017 (CET)Beantworten
Es wird behauptet, dass die Information zur Herausgabe der Zeitung falsch ist.[16] --BlackEyedLion (Diskussion) 23:59, 1. Mär. 2017 (CET)Beantworten
Was heißt damals? Von wann stammt der Bericht? --BlackEyedLion (Diskussion) 00:12, 2. Mär. 2017 (CET)Beantworten
Dass es in Stuttgart keine kommunistische Zeitung gegeben hätte, ist falsch. Unsere Stimme wurde in Stuttgart herausgegeben, siehe en:Paula Acker. --BlackEyedLion (Diskussion) 00:18, 2. Mär. 2017 (CET)Beantworten
@ BlackEyedLion, von wann genau der Bericht stammt, weiß ich nicht, aber irgendwo habe ich gelesen, dass Stella Mahlberg sich 1947 umgebracht haben soll, das wäre dann also ein Terminus post. Und Terminus ante wäre 1955, weil da Paul Mahlberg eine andere Beschäftigung hatte. Die Beiträge im forum.ahnenforschung habe ich natürlich gelesen, ich will ja eben darüber hinaus kommen. Aber erstmal danke für deine Bemühungen, ich werde mir mal den Acker-Artikel anschauen. Gruß --Xocolatl (Diskussion) 00:23, 2. Mär. 2017 (CET)Beantworten
In Kurt Koszyk, Geschichte der deutschen Presse. 4. Pressepolitik für Deutsche 1945–1949 könnte dazu etwas stehen. Zu Paula Acker: [17]. --BlackEyedLion (Diskussion) 00:25, 2. Mär. 2017 (CET)Beantworten
Mhm... da muss ich wohl mal einen Bibliotheksurlaub machen. Unsere Stimme scheint mir so gefühlsmäßig nicht recht zu passen, aber wer weiß. Ist ja eine illustre Familie gewesen. --Xocolatl (Diskussion) 01:53, 2. Mär. 2017 (CET)Beantworten
Ist zwar nicht gefragt, aber zur Info, falls es jemanden wundert: DOB steht für date of birth (Geburtsdatum). --Pp.paul.4 (Diskussion) 09:31, 2. Mär. 2017 (CET)Beantworten
Das Archiv der Partei ist übrigens im Stiftung Archiv der Parteien und Massenorganisationen der DDR im Bundesarchiv erhalten. --BlackEyedLion (Diskussion) 09:50, 2. Mär. 2017 (CET)Beantworten

<linksrück>Es gab auch noch die Volksstimme ZDB-ID 126019-4, Tages-Ausgabe ZDB-ID 126566-0, Wochen-Ausgabe ZDB-ID 126567-2. --HHill (Diskussion) 11:19, 2. Mär. 2017 (CET)Beantworten

Danke, ja, die klingt von der Beschreibung her für mich irgendwie passender, ich habe aber zumindest online auch keine Zusammenhänge mit Mahlberg finden können. Wäre der Wendelin Niedlich damals schon in Stuttgart gewesen... war er aber nicht. --Xocolatl (Diskussion) 14:40, 2. Mär. 2017 (CET)Beantworten
Zumindest hat er 1946 ein Buch bei einem Stuttgarter Verlag herausgebracht. --HHill (Diskussion) 14:48, 2. Mär. 2017 (CET)Beantworten

2. März 2017

Standleitung ins Netz vs. SDSL

Laienfrage an die Netzwerker: Wie unterscheidet sich eine Standleitung ins Internet von SDSL? Danke für die Antwort. (nicht signierter Beitrag von 188.101.73.75 (Diskussion) 00:20, 2. Mär. 2017 (CET))Beantworten

„Standleitung“ ist ein veralteter Begriff aus dem Zeitalter der Leitungsvermittlung. Das Internet und die Zugänge dahin sind heutzutage ausschließlich paketvermittelt. SDSL ist nur eine von vielen Techniken, Internetdaten vom Hauptverteiler zum Kunden zu bringen. Allerdings vergeben viele Fernmeldedienstleister für SDSL-Anschlüsse feste IP-Adressen. Bei ADSL-, VDSL- oder DOCSIS-Anschlüssen ohne Zwangstrennung gibt es das mit Einschränkungen auch, ohne dass diese Verbindungen als „Standleitung“ bezeichnet werden. --Rôtkæppchen₆₈ 00:31, 2. Mär. 2017 (CET)Beantworten
Man kann durchaus auch heute noch eine Standleitung z.B. zwischen zwei Firmenstandorten im selben Ort bekommen. Da rennen dann Telekomiker rum und klemmen tatsächlich Kabel um. Überregional kann man sich auch Glasfasern mieten usw. Ist alles eher was für den dickeren Geldbeutel. -- Janka (Diskussion) 01:02, 2. Mär. 2017 (CET)Beantworten
Dickeren Geldbeutel trifft es genau. Mein Arbeitgeber hatte vor ein paar Jahren die Aufgabe, zwei 240 Meter Luftlinie entfernte Standorte des Unternehmens telefon- und LAN-mäßig zu verbinden. Ich schlug als allererstes Richtfunk vor, der eine Gebäudeeigentümer schlug Telekom-Standleitung oder alternativ selbstverlegtes Erdkabel vor. Das Erdkabel hätte einen fünf- bis sechsstelligen Betrag gekostet, die Telekommietleitung erwies sich als unbezahlbar. Da wurde dann doch eine 5-GHz-WLAN-Bridge mit Richtantennen und VoIP für Telefon und Fax installiert. Nach wenigen Wochen stellte sich heraus, dass der zweite Unternehmensstandort wenige hundert Meter entfernt den Arbeitsablauf erheblich beeinträchtigte und der zweite Standort wurde wieder aufgegeben. Die Richtfunkinstallation existiert heute noch, allerdings ungenutzt – es fühlte sich niemand dazu berufen, die Hardware zu demontieren und bei Ebay zu veräußern. So war das ganze reine Geldverschwendung. --Rôtkæppchen₆₈ 01:37, 2. Mär. 2017 (CET)Beantworten

Spanische Strafzumessung

Den Amis ist ja so einiges zuzutrauen (aber auch da könnte man die Frage mal stellen). Aber warum sind in Spanien - obwohl Menschen legal nur 40(?) Jahre ihres Lebens im Knast verbringen dürfen - Strafzumessungen möglich, die (rückwärts gerechnet) teilweise bis ins Mesolithikum oder Paläolithikum reichen würden? Zum Beispiel hat Jamal Zougam genau 42.922 Jahre Haftstrafe bekommen. Eine weitere Frage wäre: Wie kommen bei diesen hohen - rein symbolischen - Zahlen die dennoch auf das Jahr genau präzise Strafzumessung zustande, so z.B. dass Otman el-Gnaoui (obwohl wir im mittleren fünfstelligen Bereich sind!) genau zwei Jahre Haft mehr bekam als Jamal Zougam? --ObersterGenosse (Diskussion) 01:45, 2. Mär. 2017 (CET)Beantworten

Das hat meines Wissens nach damit zu tun, dass diese Rechtssysteme die weitere Vollstreckung der Strafe nach dem Tod und einer dann folgenden Wiedergeburt oder andersartigen Wiederkehr sichern sollen, dass letztendlich die "Persönlichkeit" oder die "Seele" und nicht nur ein von diesen Entitäten bewohnter Körper bestraft wird. Ich weiß jetzt allerdings nicht, ob (in den USA) "Life without parole" (Lebenslang ohne Bewährung) die längere oder kürzere Haftstrafe ist als bsp. "3500 Jahre Gefängis", ich nehme mal an, dass "Lebenslang" formaljuristisch als kürzer gilt (da es ja auch bsp. 5-fach Lebenslang gibt - Ariel Castro war so ein Verurteilter). Letztendlich ist das eine Verquickung von Moral- und Glaubensprinzipen, die nicht in jedem Weltbild rational erklärbar sind. Ganz praktisch könnte das noch den Hintergrund haben, dass die verhängte Strafdauer Einfluss auf Bewährungsmöglichkeiten und Strafstatistiken (die nach verhängten Haftzeiten ausgegeben werden können) nimmt. Grüße, Grand-Duc (Diskussion) 02:42, 2. Mär. 2017 (CET)Beantworten
Wo steht, dass ein Mensch (außerhalb des Geltungsbereichs unseres Grundgesetzes und des Einflussbereichs unseres Bundesverfassungsgerichts) legal nur 40 Jahre im Gefängnis verbringen darf?
Die Strafzumessung über die erwartete Lebenswahrscheinlichkeit hinaus ergibt sich in den USA und andernorts in der Regel durch bloße Multiplikation und hat mit der gesetzlich vorgesehenen Strafe zu tun, nichts mit einer Strafe über den Tod hinaus. Sinnvoll ist das natürlich nicht - es sei denn, man will eine Strafe auch für den Fall einer teilweisen Begnadigung sicherstellen (bei "zweimal lebenslänglich" kann ein mal erlassen werden, und der Verurteilte bleibt trotzdem bis zum Ableben inhaftiert, oder bei 40.000 Jahren kann die Strafe ohne praktische Folgen auf 10% reduziert werden). --Snevern 06:30, 2. Mär. 2017 (CET)Beantworten
Keine Multiplikation, sondern eine Addition, oder? Siehe Absorptionsprinzip (Recht) bzw. Konkurrenz (Strafrecht Deutschlands), nach meinem Verstädnis wird in Deutschland jemand der 30 mal das gleiche Verbrechen begeht dafür einmal bestraft (wenn auch i.d.R. eher am oberen Ende des möglichen Strafmaßes) während z.B. in den USA die 30 Einzelstrafen aufaddiert werden - oder? --Studmult (Diskussion) 07:41, 2. Mär. 2017 (CET)Beantworten
In den USA wird üblicherweise addiert, in Deutschland wird eine Gesamtstrafe berechnet, wirklich genaue Vorschriften dazu wie das gemacht wird, scheint es nicht zu geben, es wird nicht einfach addiert, aber die kann durchaus höher sein als die höchstmögliche Strafe für eine einzelne Tat, siehe §§ 53-55 StGB. --MrBurns (Diskussion) 07:50, 2. Mär. 2017 (CET)Beantworten
Klar ist es eine Addition, denn jede Multiplikation ist ja nichts anderes als eine wiederholte Addition. Wenn ich für 3 Mordopfer eines Anschlags jeweils eine Einzelstrafe von 100 Jahren auswerfe, dann komme ich zu 100 + 100 + 100 Jahren (Addition) oder zu 3 x 100 Jahren (Multiplikation) Gesamtstrafe.
Die Regeln für die Bildung einer Gesamtstrafe in den USA kenne ich nicht; in Deutschland ist das in §§ 52 ff. StGB geregelt. --Snevern 09:21, 2. Mär. 2017 (CET)Beantworten
Dad ist korrekt, aber die Regelungen in §§ 53 ff sind nicht sehr genau. Ske legen eigenntlich nur Ober- und Untergrenzen fest. Z.B. für 15 Straftaten die jeweils als einzuge Tat mit 1 Jahr bestraft werden würden, sind gemäß Wortlaut zwischen 1 Jahr und 15 Jahren und 11 Monaten möglich. Die tatsächliche Strafzumessung ist also wohl mehr auf Richtergewohnheiten aufgebaut als auf dem Gesetzestext.--MrBurns (Diskussion) 00:54, 3. Mär. 2017 (CET)Beantworten

Bisher war der Gedanke von Grand-Duc der Beste. Im Falle von Reinkarnationen muss man die (gesamte?) Haftstrafe absitzen!

Aber sonst bezog ich mich speziell auf spanisches Strafzumessungsrecht, und auch nur bezüglich dieses Rechtskreises stand in en:WP, dass man nur 40 Jahre im Knast sitzen darf. Wie ist also die (Gesamt-)Strafenbildung im spanischen Strafzumessungsrecht (im Codigo Penal, oder gibt's da was eigenes?) geregelt? --ObersterGenosse (Diskussion) 00:59, 3. Mär. 2017 (CET)Beantworten

Denk dir vorsichtshalber mal ein Citation needed an meine Aussage, ich weiß nämlich nicht mehr, wo ich das aufgeschnappt habe. Dafür habe ich aber einen interessanten BBC-Artikel zum Thema gefunden. Grüße, Grand-Duc (Diskussion) 03:10, 3. Mär. 2017 (CET)Beantworten

Tetris-Rätsel... (erl.)

Etwas für die Informatiker: Auf dieser Seite wird ein Tetris-Multiplayer-Modus beschrieben, bei dem man gegen einen bzw. fünf Gegner auf Zeit spielen kann. Was mich wundert: Ich kann das Spiel unterbrechen und später wieder fortsetzen, was jedoch bspw. im Arena-Modus nicht mehr möglich ist. Warum geht das aber im Zwei- bzw. Sechs-Spieler-Modus? Die Erklärung kann doch eigentlich nur die sein, dass man hier gegen eine KI spielt, oder? Für aufschlussreiches Feedback stets dankbar, euer--Hubon (Diskussion) 04:08, 2. Mär. 2017 (CET)Beantworten

PS: Gerade habe ich das nochmals getestet und gegen diesen „Nutzer“ gespielt, der aber nichts getan hat, sodass die Steine einfach ungeordnet aufeinander gefallen sind. Wenn es sich aber tatsächlich um eine KI handelte, warum würde man dann ein vergleichsweise so aufwändiges Profil mit [fiktiven] Freunden etc. erstellen? Wenn aber nicht, wie soll das mit der Pause dann bitte bei einem echten, menschlichen User funktionieren? Das Ganze erscheint mir zugegeben immer rätselhafter, je mehr ich darüber nachdenke...--Hubon (Diskussion) 04:29, 2. Mär. 2017 (CET)Beantworten

Wie das mit der Pause bei einem echten, menschlichen Gegner funktioniert? Die einfachste Möglichkeit wäre: einer drückt auf Pause, wenns weitergeht drückt einer auf Fortsetzen, was man natürlich normalerweise per Chat abspricht. --MrBurns (Diskussion) 05:33, 2. Mär. 2017 (CET)Beantworten
Siehe hier. --Rôtkæppchen₆₈ 11:10, 2. Mär. 2017 (CET)Beantworten
Ja, aber das Verrückte ist: In diesen Modi gibt es keinen Chat, bei dem man eine solche Unterbrechung absprechen könnte! Und wie gesagt: Man wird auf der Seite rund um die Uhr einen Spielpartner finden – so wie heute Nacht, wo aber einfach die Tetrimos meines Gegenübers aufeinander gefallen sind, ohne dass dieses aktiv eine Bewegung ausgeführt hat, sodass ich das Spiel im Nu gewann. Ist doch seltsam, nicht?--Hubon (Diskussion) 14:26, 2. Mär. 2017 (CET)Beantworten
Auf der verlinkten Seite steht der Grund dafür: „You're playing against replays of other players, it isn't true multiplayer.“ --Rôtkæppchen₆₈ 15:48, 2. Mär. 2017 (CET)Beantworten
Was heisst das jetzt genau? "replays of other players" verstehe ich nicht. gruß 79.224.193.227 17:16, 2. Mär. 2017 (CET)Beantworten
Jemand spielt Tetris und sein Spiel wird aufgezeichnet. Dieses aufgezeichnete Spiel wird dann eingeblendet, während du spielst. Was du siehst ist also die Aufzeichnung eines Spiels. Der Spieler ist gar nicht mehr anwesend. --Snevern 18:40, 2. Mär. 2017 (CET)Beantworten
Und so wie es aussieht, war der schon bei der Aufzeichnung nicht mehr anwesend, weil er entweder das Interesse verloren hatte oder weil seine Verbindung zur Website abgerissen ist (kommt öfter vor), weshalb sich die Blöcke einfach uneingepasst stapeln... --Gretarsson (Diskussion) 19:13, 2. Mär. 2017 (CET)Beantworten
Zum Dank eine Runde virtuelles Eis für alle! ;-)
Oh, wie ich euch danke! Das muss ich wohl übersehen haben. Dieses Rätsel hätte mich fast verrückt gemacht... Also nochmals ein großes Dankeschön für die Aufklärung – damit habt ihr mir die Nacht gerettet!!! (Zur Feier des Tages probiere ich gleich noch mal mein Können und denke dabei an euch!) ;-) Herzlichst, euer--Hubon (Diskussion) 01:15, 3. Mär. 2017 (CET)Beantworten
Dieser Abschnitt kann archiviert werden. --Hubon (Diskussion) 01:15, 3. Mär. 2017 (CET)

Unterführung und Personentunnel

Was ist der Unterschied zwischen beiden Sachen? Ist letzteres die spezifische Bezeichnung für Tunnel unter Bahnhöfen, selbst wenn sie in offener Bauweise gebaut und nicht länger als 80 Meter sind, was laut ersterem Artikel nicht als Tunnel gilt? 129.13.72.198 09:07, 2. Mär. 2017 (CET)Beantworten

Ein Tunnel muss nicht unter der Oberfläche liegen.--Wikiseidank (Diskussion) 09:22, 2. Mär. 2017 (CET)Beantworten
Da schreibt der Artikel Tunnel aber was anderes. Außerdem: Eine Unterführung auch nicht, siehe erstes Bild im Artikel. 129.13.72.198 10:11, 2. Mär. 2017 (CET)Beantworten
Intuitiv sehe ich das nur als Frage der Perspektive: Ein Tunnel führt "durch etwas hindurch", eine Unterführung "unter etwas hindurch". Im ersten Fall kann es ein relevantes Objekt über dem Tunnel geben, aber es ist nicht zwingend. Im zweiten Fall muss es ein relevantes Objekt geben, unter dem etwas geführt wird. Yotwen (Diskussion) 12:18, 2. Mär. 2017 (CET)Beantworten

Buch Hiob: Relation Gott - Satan

Leider wird auf auf der Disk.Seite des Artikels "Ijob" (Das Buch Hiob im Alten Testament) meine Frage nicht bemerkt. Deshalb versuche ich es hier: In dem Artikel über das geplagte und elende Leben des Hiob fällt der Satz "Daraufhin erlaubt Gott es Satan, Ijob zu prüfen." Für mich als Leser wird dabei die schlichte Relation Gut vs. Böse bzw. Gott vs. Satan außer Kraft gesetzt. Satan erscheint daher als Diener Gottes, der beim "Allmächtigen" um Erlaubnis bitten muss. Werden nicht Teile des christlichen Weltbildes und die Rolle des "Verführers" dadurch in Frage gestellt? Sprich: Ist der Satz nun falsch (oder falsch formuliert) oder ist dieser Teil der Interpretation des Verfassers (des Artikels) falsch? Ich bin Agnostiker, habe aber die Bibel gelesen. Und dies ist nur ein Teil meiner "Verwunderung" über das "Buch der Bücher". Aber mir erscheint dieser Satz falsch formuliert. Oder entspricht dies der allgemeinen Interpretation der Geschichte Hiobs? Was dann wiederum Fragen über die Relation Gott - Satan aufwirft. Danke schon mal für Meinungen.--87.184.129.106 10:17, 2. Mär. 2017 (CET)Beantworten

Im Kontext ist die Geschichte doch klar. Und das Verhältnis Gott-Satan geht aus dem Artikel auch hervor. Der eine sagt, Hiob sei nicht oder nur aus bestimmten Gründen fromm und der andere sagt, das Hiob unter allen Umständen fromm sei bzw. bliebe. Und dann prüfen sie Hiob (machen also ihre Wette). Ob ich den Satz mit dem Verb "erlauben" schreiben würde, weiß ich nicht. (Was steht denn in der Bibel?) Aber der Satz für sich alleine verändert doch nicht die Darstellung der damaligen Sicht bzw. Wahrnehmung von Gut und Böse. Ich, als jemand, der--87.184.129.106 13:27, 2. Mär. 2017 (CET) als Kind von Gott und dem Teufel auch immer nur im Sinne von Gut und Böse erzählt bekommen hat, staune sowieso, dass die da so locker miteinander sprechen, und dem armen Hiob die Pest aufs Auge drücken, nur um zu beweisen, wie fromm er doch ist/sein kann. Dieses Erstaunen meinerseits hängt aber nicht an diesem einen von Dir kritisierten Satz. VG --Apraphul Disk WP:SNZ 10:40, 2. Mär. 2017 (CET)Beantworten
Der Unterschied ist, dass du wohl gläubiger Christ bist und ich Agnostiker. Und es ist in der Tat für mich dieser eine Satz, der mich stutzig macht. Nicht in Bezug auf die Hiobsgeschichte an sich, sondern genau die Aussage des Satzes, dass Satan "um Erlaubnis bitten/fragen muss". Für mich bedeutet dies in der Form, in der der Satz existiert, eine Diskrepanz in der Beziehung Gott - Satan. Wahrscheinlich ist schlicht die Formulierung falsch. Aber wenn man nicht gläubig ist, fallen einem solche Kleinigkeiten auf, da man nicht "glaubt", sondern "nicht weiß" und daher Sätze mit Skepsis liest. Das Anzweifeln von religiösen Aussagen (und der Satz definiert für mich eine Aussage) ist ein Grundsatz des Agnostizismus. Daher reicht ein unverständlicher Satz, um wieder weitere Fragen auf zu werfen. Ich kann auch noch mal in die Bibel schauen. Abseits meiner philosophischen Auffassung, ist es aber auch die sprachliche Komponente, die auffällt, die für mich fragliche Aussage des einen Satzes. Daher auch meine Frage: Ist der Satz, der eine (vermeintliche) Tatsache darlegt, in der vorliegenden Form und dem Wort "erlauben", richtig oder falsch?--87.184.129.106 11:03, 2. Mär. 2017 (CET)Beantworten
Man sollte das Buch Hiob nicht mit christlichen Massstab messen, da es jüdischen Ursprungs ist. Das eine ist, das derSatan nicht wirklich entscheidend für die Aussage des Buches ist, wichtig ist die Treue Hiobs zu Gott und die Widersprüche seiner Freunden. Wie es zu den Unglücken kam spielt m.E keine so große Rolle. Das andere ist dass sich die Vorstellungen über den Satan/Diabolos sicher seit der Entstehung des Buche geändert hat, sowohl im Judentum als auch im Christentum. Siehe auch Teufel#Judentum 178.3.144.88 10:57, 2. Mär. 2017 (CET)Beantworten
@178.3..., einmal das; und, @87.184..., wäre ich wer weis wie gläubig, könnte ich Dir sicher eher sagen, wie der Satz gemeint ist und ob er passend ist oder nicht. Kann ich aber nicht. Während Du Dich an einen Satz hängst (nicht böse gemeint) und sich Dir damit und deswegen noch weitere Fragen stellen, sehe ich die Aussagen zu der Geschichte im Artikel im Kontext, also als Ganzes. Und da werfen sich für mich dann nicht wirklich viele Fragen auf. Einzelne Sätze ausserhalb ihres Zusammenhangs dürfen m. M. n. nicht zu streng gesehen werden. Wenn das Wort "erlauben" hier tatsächlich irreführend sein sollte und wenn man zudem bedenkt, dass Gott es nicht selbst tun will und es aber auch nicht verhindern will, so könnte man vielleicht mit "zulassen" anstelle von "erlauben" umschreiben. Das dürfte man sicher im Artikel entsprechend so korrigieren. VG --Apraphul Disk WP:SNZ 11:14, 2. Mär. 2017 (CET)Beantworten
Ich habe nur die King James Bibel vorliegen, werde aber noch einmal genau lesen. Ich hänge mich wirklich an den einen Satz, da er irrtierend wirkt und sehr viel in Frage stellt. Vor allem für "Nichtgläubige". Und zumindest in meiner Auslegung sagt der Satz sehr viel über die Beziehung Gott - Satan aus. Wenn der Satz falsch ist, werde ich ihn entfernen bzw. anpassen. PS: Ich wollte dich nicht als Gläubigen abstempeln, mitnichten. Ich dachte nur, da du als Kind damit "konfrontiert" wurdest... ;-) --87.184.129.106 13:28, 2. Mär. 2017 (CET)Beantworten
Kein Problem. :-) Ich bin mittlerweile in einem Alter, in dem man öfter schon mal die (geglaubte) bloße Möglichkeit, dass es mehr geben könnte zwischen Himmel und Erde als der Mensch erklären kann, sehr begrüßt. Es hilft manchmal, Dingen einen Sinn (oder mögliche Erklärungen) zu geben, an die man in der rein wissenschaftlich akzeptierten Welt ansonsten vielleicht verzweifeln würde. Von daher dürfte man mich auch gerne als gläubig "abstempeln". :-) VG --Apraphul Disk WP:SNZ 13:58, 2. Mär. 2017 (CET)Beantworten
Na, dann... ;-)--87.184.129.106 14:04, 2. Mär. 2017 (CET)Beantworten
Sorry, aber die Frage von OP ist schon berechtigt. Man kann biblische Texte selbstverständlich analysieren wie jeden anderen Text auch, und dabei spielt dann natürlich jedes Detail eine Rolle. Dass eine Figur "nicht wirklich entscheidend für die Aussage" ist, mag im Hinblick auf den Glauben richtig sein, aber einer Textanalyse steht das nicht im Weg. Ich bin mir sicher, dass speziell zu diesem "Detail" hier schon viel geschrieben worden ist, kann aber leider selbst nicht weiterhelfen. --89.246.198.251 11:53, 2. Mär. 2017 (CET)Beantworten
Bin ich OP? Und was heißt OP... :-)?--87.184.129.106 14:06, 2. Mär. 2017 (CET)Beantworten
Jawohl, Du bist der Fragensteller, der Abschnittsersteller oder Abschnittseröffner, der neudeutsch Threadopener (TO) oder neudeutsch kurz Opener (OP). Na, wie auch immer - Du bist gemeint. ;-) VG --Apraphul Disk WP:SNZ 14:11, 2. Mär. 2017 (CET)Beantworten
Ok. Ich bin ein OP, aber auch ein TO. Gut zu wissen. Man lernt nie aus, auch in unserem, sorry, meinem Alter nicht... ;-)--87.184.129.106 14:42, 2. Mär. 2017 (CET)Beantworten
"OP" heißt Original Poster, siehe Liste von Abkürzungen (Netzjargon)#O. --Neitram  17:15, 2. Mär. 2017 (CET)Beantworten
Aber dann muss man den Text lesen und nicht die Inhaltsangabe in der Wikipedia.
Hilfreich ist vielleicht der Artikel Satan. --Digamma (Diskussion) 12:27, 2. Mär. 2017 (CET)Beantworten
Dann ist aber der Satz im Artikel Ijob falsch und sollte geändert werden. Denn schließlich sollte Wikipedia das (paraphrasiert) wiedergeben, was textlich in der Bibel steht und dabei keine Eigeninterpretation liefern. Es ist nur ein Satz, aber er fällt einem, wenn man über einigermaßen kompetentes Textverständnis und linguistisches sowie literatur-analytisches Grundwissen verfügt, ins Auge. Schließlich sagt er in der jetzigen Form weitaus mehr als, als man meinen mag. --87.184.129.106 13:38, 2. Mär. 2017 (CET)Beantworten
In meiner Bibel (Einheitsübersetzung von 1980) steht
"Der Herr sprach zum Satan: Gut, all sein Besitz ist in deiner Hand, nur gegen ihn selbst streck deine Hand nicht aus! Darauf ging der Satan weg vom Angesicht des Herrn." (Ijob 1,12 EU)
Ist das mit dem Satz "Daraufhin erlaubt Gott es Satan, Ijob zu prüfen." richtig wiedergegeben? --Digamma (Diskussion) 16:30, 2. Mär. 2017 (CET)Beantworten
Im Artikel Ijob, Absatz "Inhalt: Handlung: Frömmigkeit:" steht der letzte Satz so. Und um diesen Satz geht es. Wohlgemerkt, es ist KEIN Bibelzitat, sondern die Inhaltsangabe (und leider auch Interpretation) des Artikelschreibers.--87.184.129.106 20:11, 2. Mär. 2017 (CET)Beantworten
Das ist mir auch klar. Deshalb ja meine Frage: Ist deiner Meinung nach die zitierte Aussage aus der Bibel ("Der Herr sprach ...") durch den Satz im Artikel ("Daraufhin erlaubt ...") richtig wiedergegeben? Beruht dein Unbehagen nur auf der Formulierung im Artikel, oder spürt du es auch bei der Originalformulierung der Bibel? --Digamma (Diskussion) 20:22, 2. Mär. 2017 (CET)Beantworten
Der interpretative Satz im Artikel ist mein Stein des Anstoßes, wenn man so will. Ich dachte, das wäre im Verlauf der Diskussion bereits durchgeklungen. Der Satz ist, nachdem ich noch mal in die Bibel (KJB) geschaut habe, falsch. Den gibt es, soweit ich es überblicken kann und ich lese jetzt nicht das ganze Buch Hiob noch einmal, so nicht. Also ist der Satz des Schreibers im Artikel "Ijob" überflüssig. Der philosophische Ansatz über die Relation Gott - Satan hat nur Berechtigung, wenn der Satz so auch irgendwo auftauchen würde. Soweit ich jetzt erkenne, sind wir hier zum Schluss gekommen (und diese Disk. läuft mehr oder weniger den ganzen Tag), dass es die falsche Interpretation des Schreibers ist, die nicht passt. Wobei das ein User (letzter Abschnitt) wieder anders sieht und die von mir angesprochene Frage zur Position Satans im hirarchischen Aufbau der auftretenden "Charaktere" wieder ins Spiel bringt. Und nein, ich spüre kein "Unbehagen" bei der biblischen Originalformulierung. Als Agnostiker spüre ich beim Lesen der Bibel relativ wenig.--87.184.159.68 21:47, 2. Mär. 2017 (CET)Beantworten
Im Buch Hiob steht: "Der Herr sprach zum Satan: Gut, all sein Besitz ist in deiner Hand, nur gegen ihn selbst streck deine Hand nicht aus! Darauf ging der Satan weg vom Angesicht des Herrn." Meiner Meinung nach kann man das schon mit "er erlaubte" wiedergeben. Was ist es denn deiner Meinung nach sonst?

--Digamma (Diskussion) 22:15, 2. Mär. 2017 (CET)Beantworten

Ein Rat, eine Abmachung oder ein Kompromiss. "Daraufhin erlaubt Gott es Satan, Ijob zu prüfen." ist jedoch die Behauptung, ein Höherstehender würde seinem Untergebenen etwas zugestehen, was er aber auch ablehnen kann. Was andeutet, dass es eine hirarchische Befehls- und Ordnungsstruktur gibt. Die jedoch in der Bibel, soweit ich mich erinnere, nach dem "Höllensturz", also dem "Rauswurf" der aufständischen Engel aus dem Himmel, so nicht gegeben ist. Ich finde aber auch Benutzer Meloes (s.u.) Einschätzung interessant, die genau diese Frage der Hirarchie aufwirft.--87.184.159.68 10:27, 3. Mär. 2017 (CET)Beantworten
Klingt für mich nach einer ausdrücklichen Erlaubnis. Aber vielleicht sollte man zur Sicherheit noch einen Rechtsanwalt hinzuziehen (SCNR). --Gretarsson (Diskussion) 17:10, 2. Mär. 2017 (CET)Beantworten
Luther verwendet da noch n Konjunktiv, weil er wohl noch Latein konnte... [18]... in Constantine (Film) mit Keanu Reeves *hechel* hat der Satan irgendwie auch noch n Wörtchen mitzureden, indem er Krankheiten lindert, die ansonsten mit dem Leben nicht länger vereinbar sind (also schon ziemlich mächstig...)... ansonsten sind Menschen-Experimente Gott ja nich fremd: Adam (mit der dicken langen Schlange), Kain (der wilde Brutalo), und mehr weiß ich nich (aber ob Gott nun ne Schlange bastelt oder irgendwie Zwietracht säht oder dem Satan suggeriert, er habe fast freie Hand, ist doch irgendwie immer das Gleiche...)... wir haben uns in der Schule immer gefragt, warum wir an irgendwas Schuld sein sollten, wenn Gott das alles so unnötig kompliziert gemacht hat... --Heimschützenzentrum (?) 18:04, 2. Mär. 2017 (CET)Beantworten
Die Frage ist doch nicht, ob der Übersetzer Latein kann, sondern was die bessere Übersetzung ist. --Digamma (Diskussion) 18:16, 2. Mär. 2017 (CET)Beantworten
In der Vulgata steht übrigens der Indikativ "sunt". --Digamma (Diskussion) 20:25, 2. Mär. 2017 (CET)Beantworten
dann ist das „sei“ wohl irgendwie kein Konjunktiv? bevor man sich fragt, ob ne Erlaubnis passt, sollte man erstmal rausfinden, ob es ne Erlaubnis ist... darum sollte man möglichst dicht am Original sein... --Heimschützenzentrum (?) 21:56, 2. Mär. 2017 (CET)Beantworten
Um diese Frage geht es ursprünglich auch gar nicht. Sondern darum, ob der Satz "Daraufhin erlaubt Gott es Satan, Ijob zu prüfen.", der so im Artikel Ijob steht (Kein Bibelzitat!), so Bestand hat. Der Schreiber hat seine eigene Interpretation in diesen einzigen Satz gepackt. Und da geht es jetzt darum, diesen zu löschen oder umzuformulieren.--87.184.129.106 20:17, 2. Mär. 2017 (CET)Beantworten
Als Agnostiker bin ich der Meinung, dass man sich bei der Inhaltsangabe nicht davon leiten lassen sollte, ob die Aussage zum christlichen Weltbild passt. --Digamma (Diskussion) 20:34, 2. Mär. 2017 (CET)Beantworten
Ändert für mich nicht wirklich etwas daran, dass es nach einer ausdrücklichen Erlaubnis klingt. „So sei es“ ist auch nur eine etwas hochtrabende Formulierung für „Deal!“ oder „Top, die Wette gilt!“ Der Konjunktiv ist da nicht überzubewerten... --Gretarsson (Diskussion) 19:20, 2. Mär. 2017 (CET)Beantworten
Im Gegenteil: Der Konjunktiv bedeutet eine ausdrückliche Aufforderung ("soll sein"). --Digamma (Diskussion) 19:23, 2. Mär. 2017 (CET)Beantworten
Ich verstehe die biblische Erzählung so: Gott ist allmächtig, Satan nicht. Daher diskutiert Satan mit Gott vorab - vor allen Engeln - damit Gott nicht die folgenden Handlungen Satans aus der Position des Stärkeren heraus einfach verhindert oder rückgängig macht. Das heißt aber für mich noch nicht, dass Satan ein Diener Gottes wäre; sein Handeln wird von Gott bloß geduldet, damit am Ende für die beobachtenden Engel klar wird, dass Gott nicht nur der Stärkere ist, sondern dass Satan mit seinen Behauptungen unrecht hatte. Und der Allmächtige schafft trotzdem, zumindest im Nachhinein. auch für den leidenden Menschen noch einen versöhnlichen Ausklang. --DFGza (Diskussion) 13:33, 2. Mär. 2017 (CET)Beantworten
Aber wenn Gott allmächtig ist, warum muss er dann dieses perfide Spiel spielen? Er weiß dann doch schon wie es ausgeht/ausgehen würde. Mir waren Sadisten immer schon suspekt, genauso wie Personen, die sich immer wieder beweisen lassen müssen, was für tolle Hechte sie sind. Vielen Dank! --Elrond (Diskussion) 15:19, 2. Mär. 2017 (CET)Beantworten
Aber wenn Gott allmächtig ist, warum duldet er Satan und vernichtet ihn nicht? Weil Satan zum christlichen Weltbild dazugehört? Weil Gott die Hölle und ihren Herrscher braucht, um den Menschen zu strafen und ihn die Furcht vor dem Bösen fühlen lässt? Insofern sehe ich Satan als Teil von Gottes Plan, was dann wiederum bedeuten könnte, das Satan kann reiner Gegenspieler, sondern trotz aller Gegensätze, unter der allmächtigen Gewalt Gottes steht. Und im Einzelfall das tun muss, was Gott sagt. Natürlich ist das alles rein hypothetisch gesprochen. Aber vielleicht ist es jetzt ein bißchen zu viel des Guten... ;-) --87.184.129.106 13:49, 2. Mär. 2017 (CET)Beantworten
"Satan erscheint daher als Diener Gottes, der beim Allmächtigen um Erlaubnis bitten muss." Diese Aussage ist völlig korekt, sie gilt nach amtskirchlich christlichen Verständnis sowieso für alle Handlungen Satans. Das Buch Hiob ist dazu noch ein Sonderfall, weil es die historisch älteste Darstellung der Materie ist. Er ist hier (noch) nicht der nahezu allmächtige Fürst der Finsternis, sondern eher sowas wie ein Ankläger-Engel an Gottes Hofstaat. Die Rolle des Teufels ist innerhalb der christlichen Tradition einschneidenden Änderungen unterworfen gewesen und auch innerhalb der Bücher der Bibel nicht widerspruchsfrei. Zur vertiefenden Lektüre empfohlen: Kurt Flasch: Der Teufel und seine Engel. Die neue Biographie. Beck Verlag, München 2015, ISBN 978-3-406-68412-8.--Meloe (Diskussion) 13:54, 2. Mär. 2017 (CET)Beantworten

Spätestens seit der Aufklärung wissen wir, dass wir unser Weltbild nicht auf der Basis eines einzigen Buches aufbauen sollten. Bibliotheken führen die Bibel, aber sie führen auch 1.000.000 andere Bücher. Aus der Bibel kann man spannende Dinge ziehen, aber aus all den anderen Büchern eben auch. Die Widersprüchlichkeit der Bibel ist analog zur Widerspruchlichkeit dessen was andere Bücher bëinhalten, etwa die Schriften von Naturwissenschaftlern und Philosophen. Da sie, wie alle Bücher dieser Welt, von Menschen geschrieben wurde, sollte Dich die Widersprüchlichkeit der Bibel doch nicht wirklich verwundern? 62.44.134.189 06:40, 3. Mär. 2017 (CET)Beantworten

Ich halte es nicht für realitätsfern. Die meisten Chefs (die ich kennenlernen durfte) haben doch einen Adlatus fürs Grobe. Das war sicherlich auch früher so - und da hat man es halt mit hineingeschrieben. Heisst ja nicht umsonst "der Versucher". GEEZER … nil nisi bene 07:44, 3. Mär. 2017 (CET) Beantworten
Du schreibst reichlich am Thema vorbei. Es geht darum, ob ein Satz, der die falsche Aussage hat, in einen Artikel gehört oder nicht. Als Agnostiker wundert mich nicht viel, aber vieles verstehe ich nicht, weil es rational eigentlich keinen Sinn macht. Deshalb bin ich wohl auch Agnostiker.--87.184.159.68 10:02, 3. Mär. 2017 (CET)Beantworten

Ich habe den Satz im Artikel Ijob jetzt umformuliert. Sollte es jemand anders sehen, dann bitte nicht einfach löschen oder zurück setzen, sondern hier argumentativ diskutieren. Danke.--87.184.159.68 12:07, 3. Mär. 2017 (CET)Beantworten

Zunächst geht es darum, ob der Inhalt von Ijob 1,12 EU mit "Gott erlaubt Satan" korrekt wiedergegeben ist. Ich denke dass ja. Der erste Satz hat kein Verb "alles was ihm [gehört] [ist] in deiner Hand". Den kann man als Feststellung "es ist" oder als Erlaubnis "es sei" auffassen, wobei ersteres wörtlicher ist. Der zweite Satz aber schränkt den ersten mit einem Jussiv ein ("du mögest deine Hand nicht gegen ihn ausstrecken"), was eben nicht eine Feststellung, sondern eine Erlaubnis einschränkt.
Generell ist das Verhältnis Gott–Satan in der Bibel nie als purer Dualismus gleicher Partner beschrieben, sondern geht immer von der Souveränität Gottes aus, die der Satan nicht hat, z.B. SachEU, wo der Satan in einer Vision als Ankläger erscheint, der jedoch nicht zu Wort kommt, weil sich Gott nicht für die Anklage interessiert. Auch im Buch Ijob interessiert sich niemand mehr für den Satan, wo der doch eine Wette verloren hat. Ansonsten ist das Bild Satans in der Bibel durchaus recht schillernd; das ist aber Thema des Artikels Satan und gehört nicht nach Ijob. --Lantani (Diskussion) 13:26, 3. Mär. 2017 (CET)Beantworten

Übersetzer/-ung des Coleridge Gedichts "Ballade vom Alten Seemann"

Vorweg: ich habe bereits eine Übersetzung dieses Gedichts, aber diese hier finde ich wesentlich besser, habe aber nur diese wenigen Zeilen:

  • "'..Gott schütze dich, alter Seefahrer,
  • Was war es, das da dich verdroß?
  • Was raubt dir das Heil?' - 'Mit meinem Pfeil
  • Schoß ich den Schicksalsvogel ALBATROS!'"

Google bringt mich nicht weiter. Wo könnte ich diese Übersetzung komplett finden oder nach dem Übersetzer suchen? Das beschäftigt mich schon länger. Danke. --95.208.26.144 11:30, 2. Mär. 2017 (CET)Beantworten

Hier sehe ich schon mal den Ausschnitt (allerdings ohne den "Schicksalsvogel"). --Wrongfilter ... 11:43, 2. Mär. 2017 (CET)Beantworten
Hast Du das hier (klick) gesehen? Besonders der zweite Post ... VG --Apraphul Disk WP:SNZ 11:53, 2. Mär. 2017 (CET)Beantworten
(BK und Kampf mit dem Spamfilter) Der von mir genannte Ausschnitt stammt aus dem Aufsatz "Der rassistische Albatros" von Immanuel Wallerstein (gefunden durch Suche nach "Albatros" in dem Buch/der Zeitschrift), dazu findet sich ein pdf hier , damit kann man in Anmerkung 2 nachschauen und findet die Angabe des Übersetzers: Heinz Pollitzer (Frankfurt am Main 1993). (An Erika Fuchs dachte ich übrigens auch schon...) --Wrongfilter ... 12:01, 2. Mär. 2017 (CET)Beantworten
Abschließend: Heinz Politzer mit einem l. Ich finde keine Ausgabe von 1993, nur von 1963 und 1968, die antiquarisch nicht schwer zu finden sind. --Wrongfilter ... 12:10, 2. Mär. 2017 (CET)Beantworten
Der Link von Aprahul hatte mich auch schon eher weitergebracht, weil es auch den Übersetzer enthält. Das pdf hatte ich gleich danach gefunden. Der antiquarische Link führte bei mir nicht zu dieser Website. Danke für die vielen Antworten. 95.208.26.144 12:38, 2. Mär. 2017 (CET)Beantworten
@wrongfilter: tschuldigung, ich hatte nicht den Link angeklickt, sondern die Adresse händisch kopiert und keine Seite angezeigt bekommen.95.208.26.144 12:41, 2. Mär. 2017 (CET)Beantworten

Ferdinand Freiligrath: s:Benutzer:A. Wagner/DER ALTE MATROSE (The Rime of the Ancient Mariner). --Vsop (Diskussion) 15:42, 2. Mär. 2017 (CET)Beantworten

Das ist nicht die gesuchte Übersetzung. Bei Freiligrath lautet die betreffende Strophe: "Vor bösen Geistern schütz’ dich Gott,//Du alter Schiffsgenoß!//Was stierst du? – mit der Armbrust mein//Schoß ich den Albatros!". --Wrongfilter ... 15:45, 2. Mär. 2017 (CET)Beantworten
Ich kenne da noch die Donald-Duck-Variante: "Weh mir Frevler, daß ich schoß den Schicksalsvogel Albatros! Dreimal wehe, daß ich traf, denn nun trifft mich des Schicksals Straf!" --Neitram  17:13, 2. Mär. 2017 (CET)Beantworten
Politzer 1963 https://books.google.de/books?id=kXk4AAAAIAAJAA. Dass man eine Eindeutschung, die nicht mal den Rhythmus halten kann, „wesentlich besser“ finden könnte als die von Freiligrath, die damit keine Schwierigkeiten hat, hatte ich mir bisher nicht vorstellen können. --Vsop (Diskussion) 17:16, 2. Mär. 2017 (CET)Beantworten
@wrongfilter: Die Übersetzung von Freiligrath kannte ich bisher gar nicht, danke für den Tipp. --95.208.26.144 19:59, 2. Mär. 2017 (CET)Beantworten
Der Dank muss Vsop gelten. --Wrongfilter ... 20:33, 2. Mär. 2017 (CET)Beantworten

Übersetzt wurde der Mariner in jüngerer Zeit u.a. von Wolfgang Breitwieser (1963), Siegfried Schmitz (1967), Edgar Mertner (1973), Günter Kunert (1983)...--Edith Wahr (Diskussion) 21:23, 2. Mär. 2017 (CET) @ Wrongfilter: Ja, natürlich, ich hatte es erst gesehen, als ich es bereits eingegeben hatte. Also @ Vsop: danke, ebenso an @ Edith Wahr. --95.208.26.144 23:52, 2. Mär. 2017 (CET)Beantworten

Post-sowjetische Staatsbürgerschaft

Gestern habe ich einen Foreign-Policy-Artikel gelesen (ist aktuell noch frei verfügbar, wird aber bald hinter einer Paywall verschwinden Sorry, nicht gesehen, ist schon weg…), in dem es um einen Mann geht, der kurz vor Auflösung der Sowjetunion in ein russisches Gefängnis kam und jetzt entlassen wurde. Er hat naturgemäß die damaligen Fristen verpasst, die russische Staatsbürgerschaft direkt im Anschluss zu erwerben, und da er ursprünglich aus Kasachstan kommt, wollen die russischen Behörden vor Ort ihn jetzt nicht die russische Staatsbürgerschaft geben. So weit, so gut. Mit Hilfe der englischen Wikipedia (und dabei auf sie vertrauend) konnte ich auch gut nachvollziehen, warum er 1991 nicht automatisch russischer Bürger geworden ist und wohl jetzt auch ziemlich schlechte Karten hat, die russische Staatsbürgerschaft zu bekommen. Der Artikel zu Kasachstan ist leider … wenig aussagekräftig. Zwei Fragen:

  • 1.) Kann jemand erklären, wie die Lage in Kasachstan aussieht, und ob der Mann genau so schlechte Chancen auf eine kasachische wie auf eine russische Staatsbürgerschaft hat?
  • 2.) Als spektakulär später Fall ist das sicherlich ein Einzelfall, aber ich kann mir vorstellen, dass es nach dem Zusammenbruch der Sowjetunion doch ziemlich oft zu „Entstaatlichungen“ von Ex-Sowjetbürger gekommen sein muss. Der englische WP-Artikel zur russischen Staatsbürgerschaft hat ein bisschen was dazu. Auf jeden Fall scheint es ja keine Automatismen gegeben haben. Gab es denn damals viele Fälle, in denen entweder Fristen versäumt worden sind oder aber die Betroffenen vielleicht auch absichtlich verzichtet haben, Anträge zu stellen? Wie groß war die Zahl der Ex-Sowjetbürger ohne neue Staatsbürgerschaft, und wie viele davon gibt es noch heute?

--92.225.69.252 12:00, 2. Mär. 2017 (CET)Beantworten

Tja. Die Lage der Nichtbürger ist besonders krass in Lettland, Estland und Litauen. Bleibt die Staatsangehörigkeit unklar, kann zuerst ein Fremdenpass beantragt werden.--ElmarG (Diskussion) 13:04, 2. Mär. 2017 (CET)Beantworten

Alexander Lion

Wann starb Alexander Lion? (Aktuell steht er auf der Hauptseite unter "Was geschah am 2. März?".) Siehe Diskussion:Alexander_Lion#Sterbeort_und_-datum -- ich mache für heute Feierabend, vielleicht möchten andere die Suche weiterführen. --Neitram  17:05, 2. Mär. 2017 (CET)Beantworten

ZFC-Axiome als normale Aussagesätze auffassen

Warum werden die Axiome von ZFC (Zermelo-Fraenkel-Mengenlehre mit Auswahlaxiom) als Axiome eingeführt? Macht es nicht mehr Sinn, sie als normale Aussagesätze einzuführen und zu beweisen? Den Satz des Pythagoras führt man ja auch nicht als Axiom ein, sondern beweist ihn. --Thomas Limberg 1986 (Diskussion) 17:29, 2. Mär. 2017 (CET)Beantworten

Es gibt keine Aussagen, auf die die Sätze zurückgeführt werden könnten. An irgend einem Punkt musst du ein Axiom aufstellen, auf welches sich spätere Aussagen berufen. So kannst du den Satz des Pythagoras auf die ZFC-Axiome zurückführen. Oder besser Axiom. Yotwen (Diskussion) 17:36, 2. Mär. 2017 (CET)Beantworten
Wow, echt schnelle Reaktion hier !!! Aber:
Zitat: "Es gibt keine Aussagen, auf die die Sätze zurückgeführt werden könnten.". Das sehe ich irgendwie anders. Nehmen wir mal das Leermengenaxiom. Warum nicht definieren als , dann beweisen, dass diese Definition konsistent ist, und schon hat man bewiesen, dass , hat also das Leermengenaxiom, als Aussagesatz aufgefasst, bewiesen. --Thomas Limberg 1986 (Diskussion) 17:47, 2. Mär. 2017 (CET)Beantworten
Wieso soll das ein Existenzbeweis für die leere Menge sein? --Digamma (Diskussion) 17:54, 2. Mär. 2017 (CET)Beantworten
Na, weil mit gibt es ein B, für das gilt .--Thomas Limberg 1986 (Diskussion) 18:00, 2. Mär. 2017 (CET)Beantworten
Dafür brauchst du erst einmal die Möglichkeit „Objekte zu definieren“. Das kannst du mit einer Klassenlogik. Wenn du da aber einfach beliebige Definitionen zulässt, bekommst du die Russel’sche Antinomie. Sich schlichtweg auf konsistente Definitionen einschränken geht nicht, weil es im Allgemeinen unentscheidbar ist, ob eine Definition konsistent ist. Also muss man schon angeben, was für Klassendefinitionen man zulassen möchte – und landet damit bei etwas ähnlichem wie den Axiomen von ZFC. --Chricho ¹ ³ 18:06, 2. Mär. 2017 (CET)Beantworten
Zitat: "...weil es im Allgemeinen unentscheidbar ist, ob eine Definition konsistent ist.". Dazu: Im konkreten Fall der Leermenge (und weiterer Axiome) geht das aber schon, würde ich mal behaupten. Also, warum es nicht tun? --Thomas Limberg 1986 (Diskussion) 18:12, 2. Mär. 2017 (CET)Beantworten
Nein, auch im konkreten Fall geht es nicht. Denn dafür müsstest du wissen, dass die Regel „jede definierbare Menge existiert, für die diese Existenz konsistent ist“ selber konsistent ist (Konsistenz muss ja auch Konsistenz mit allen Regeln heißen). Noch die kleinste Existenzaussage hinge dann von der Konsistenz des gesamten Systems ab. --Chricho ¹ ³ 18:28, 2. Mär. 2017 (CET)Beantworten
Ich denke, an dieser Stelle macht es wenig Sinn, weiterzudiskutieren. Am besten, ich formuliere erst einmal meinen Konsistenzbeweis, und dann kannst du mir ja die genaue Stelle zeigen, an der eine Inkonsistenz (oder mangelnder Beweis der Konsistenz) vorliegt. Aber danke erst einmal für die Hinweise! --Thomas Limberg 1986 (Diskussion) 18:37, 2. Mär. 2017 (CET)Beantworten
Ein Hinweis: Es ist doch davon auszugehen (vllt. hängt das von formalen Details ab, aber so wie ich dein System verstehe, wird das passieren, wenn wir davon ausgehen, dass dein System keine offensichtliche Inkonsistenz hat und es hinreichend stark ist, um klassische Arithmetik etc. zu betreiben), dass die Situation eintritt, dass es zwei Definitionen für Mengen gibt, sodass die Existenz jeder einzelnen annehmbar ist, jedoch die Existenz beider gleichzeitig zu einem offensichtlichen Widerspruch führt. Ich denke etwa an die Mengen „alle x, sodass x sich selbst nicht enthält oder dein φ“ und „alle x, sodass x nicht sich selbst enthält oder nicht φ“, wobei φ eine Aussage ist, die sich in deinem System nicht beweisen und nicht widerlegen lässt. Solch eine gibt es nach dem Unvollständigkeitssatz, wenn dein System zumindest etwas Arithmetik kann. Unter der Annahme, dass dein System konsistent ist, wäre auch die Annahme der Existenz je einer dieser beiden Mengen konsistent mit deinem System. Dann müssten ja aber beide in deinem System existieren. Eine von den beiden würde dann aber zur Russel’schen Antinomie führen. Also haben wir einen Widerspruch. Das heißt, wenn dein System nicht widersprüchlich ist, muss es sehr schwach sein, Induktionsbeweise über arithmetische Aussagen mit + und · dürften außer in wenigen Fällen nicht möglich sein. --Chricho ¹ ³ 18:58, 2. Mär. 2017 (CET)Beantworten

Nur noch ein Hinweis: Deine Regel „jede definierbare Menge existiert, für die diese Existenz konsistent ist“ ist selbst auch ein Axiom/Axiomenschema, das man annehmen kann oder eben auch nicht, aber nicht beweisen (wenn man denn nicht andere Axiome hat). Dieses Axiom anzunehmen hat den Nachteil, dass es sehr kompliziert ist (viel komplizierter als etwa das Leermengenaxiom). Auch den Satz des Pythagoras beweist man ausgehend von Axiomen. --Chricho ¹ ³ 19:03, 2. Mär. 2017 (CET)Beantworten

Axiome sollten Sätze sein, die man logisch nicht weiter zerlegen kann. Den Satz des Pythagoras kann man weiter logisch zerlegen. Dies nennt man dann Beweis. Das Leermengenaxiom kann man meines Erachtens logisch zerlegen, weshalb ich es nicht als Axiom einführen würde, sondern als zu beweisenden Satz. --Thomas Limberg 1986 (Diskussion) 19:20, 2. Mär. 2017 (CET)Beantworten
Weiterhin, Zitat: "Deine Regel ... hat den Nachteil, dass es sehr kompliziert ist". Dazu: Ich finde nicht, dass es sehr kompliziert ist. Und es kann ja auch für den Beweis der anderen Axiome (als normale Sätze aufgefasst) (Paarmengen-, Vereinigungs-, Unendlichkeitsaxiom u.s.w.) verwendet werden. In Summe ist es also viel einfacher! --Thomas Limberg 1986 (Diskussion) 19:34, 2. Mär. 2017 (CET)Beantworten

Wenn mein Gedächtnis mich nicht trügt, dann wurde im Ebbinghaus Axiome als "implizite Definitionen" bezeichnet. Sprich: Nachdem Cantors Mengenbegriff gescheitert ist, sagt man "Eine Menge ist das, was den folgnden Axiomen entspricht:" Die Axiome sind sozusagen ein Kriterienkatalog für die "Objekte", über die man reden will. (nicht signierter Beitrag von 188.101.64.217 (Diskussion) 21:55, 2. Mär. 2017 (CET))Beantworten

Namenszusatz "von"

Gibt es einen Grund, wieso man den Namenszusatz "von" bei einigen Personen weglässt, wenn man nur deren Nachnamen erwähnt, bei anderen aber nicht? Wieso heisst es "Von Graffenried wuchs in Bern auf", aber "Wegen des breiten Spektrums seiner Fähigkeiten galt Haller als Universalgelehrter."? --85.3.130.75 18:23, 2. Mär. 2017 (CET)Beantworten

Hinweis: Für Fragen speziell zur Wikipedia gibt es ein eigenes Diskussionsforum: Fragen zur Wikipedia.
Vielleicht gibt es keinen Grund, vielleicht will mancheiner aber auch zwischen Namensbestandteil (Vorsatzwort) und Adelsprädikat unterscheiden. --Magnus (Diskussion) 19:18, 2. Mär. 2017 (CET)Beantworten
Es betrifft nicht nur die Wikipedia, die beiden Personen werden immer so genannt, siehe z.B. "Von Graffenried gibt Einblick in Spendenliste" und "Hallers ungeheurer Wissensdrang". --2A02:1205:5038:24B0:514E:527D:E3A1:2E0C 21:59, 2. Mär. 2017 (CET)Beantworten
Je „berühmter“ jemand ist, umso eher wird auf das „von“ verzichtet. Beispiel: Müller wurde kurz vor seinem Tode geadelt, also heißt die Straße später nach ihm todsicher „Von-Müller-Straße“; aber wer kennt eine „Von-Schiller“-oder „Von-Goethe“-Straße? --Dioskorides (Diskussion) 00:25, 3. Mär. 2017 (CET)Beantworten
Danke! --85.3.130.34 11:38, 3. Mär. 2017 (CET)Beantworten

Bei heutigen Namensträgern handelt es sich ja lediglich um einen Namen. Da das "von" heute kein Adelsprädikat oder dergleich ist, kann man es auch nicht weglassen, denn Eigennamen sind unveränderlich. Das gilt für Buchtitel, Produktnamen, Nachnamen usw. usf. Da Eigennamen nicht den Rechtschreibregeln unterliegen, müsste man das "von" in einem heutigen Namen konsequent auch klein schreiben, so wie man ja auch "iPhone" nicht "Iphone" schreiben kann – auch am Satzanfang. Bei historischen Namen kann man anders argumentieren. Hier ist das "von" ein offizieller Titel, den man dann ggf. auch weglassen kann. "Humboldt unternahm umfangreiche Forschungsreisen". Damals war das "von" nicht Teil des Eigennamens "Humboldt", sondern wurde ihm vorangestellt. 62.44.134.189 06:34, 3. Mär. 2017 (CET)Beantworten

Vielen Dank für die ausführliche Erklärung! Müsste man von Graffenried folglich z.B. in einem Quellenverzeichnis unter V aufführen, d.h. von Graffenried, Alec anstatt Graffenried, Alec von? --85.3.130.34 11:38, 3. Mär. 2017 (CET)Beantworten
Es gibt für die Sortierung DIN-Normen und wahrscheinlich Schweizer Normen, an die sich große Sortierbetriebe wie Telefonbuchverlage oder Bibliotheken halten. Nach dem Vorbild von Karin Marti-Weissenbach: Graffenried, Christoph von. In: Historisches Lexikon der Schweiz. kämen die Graffenrieds vielleicht unter G. --Pp.paul.4 (Diskussion) 16:30, 3. Mär. 2017 (CET)Beantworten

Lateiner gesucht (mal wieder...)

Hallo, ich bitte um sinngemäße Übersetzung der folgenden Grabinschrift (Hinweis: es geht um Avo (Corvey)):

„Hoc rogo quisquis adis limina sacra,
Quem calcas pedibus noscito quis fim.
Abbas Avo sui nomine dictus:
Nunc cinis, in cinerem hic resolutus
Tempus judicii specto supremi:
Hinc Christi domini te per amorem
Pro me, quaeso, preces fundere crebras
Ob hoc me petii hic sepeliri,
Quinto Idus abii ipso Novembris.“

Danke. Textquelle, falls ich mich verguckt/verschrieben habe: Volltext in der Google-Buchsuche --Magnus (Diskussion) 19:08, 2. Mär. 2017 (CET)Beantworten

OT? Jedenfalls Info auf die Schnelle: mit dem Topos "Asche zu Asche" (nunc cinis...) auch die Epitaphia Corbeiensia II, in: MGH Poetae 4, hg. v. K. Strecker, S. 1041,3., von hier. zum Proxystöbern, weiß nicht, ob sie auf Übersetzungen hinweisen. --Aalfons (Diskussion) 19:21, 2. Mär. 2017 (CET)Beantworten
"Darum bitte ich, wer immer du bist, der an die heiligen Schwellen tritt/, du sollst erkennen, wer ich bin (sim), auf den du mit deinen Füßen trittst./ Ich war (fui) der Abt, Avo genannt:/ Jetzt bin ich Asche, und hier in Asche verfallen erwarte ich die Zeit des höchsten Gerichts:/ Drum bitte ich dreifach (i. S. v. sehr?), um Christi des Herrn Liebe willen,/ für mich häufige Gebete zu verrichten/Deswegen bat ich darum, hier begraben zu werden./ Ich starb am 9. November." Grüße Dumbox (Diskussion) 19:44, 2. Mär. 2017 (CET)Beantworten
Danke sehr! --Magnus (Diskussion) 19:48, 2. Mär. 2017 (CET)Beantworten
@Tsungam, Aalfons: Poetae Latini medii aevi 4,2.3: Poetae Latini aevi Carolini (IV). Teil 2: Rhythmi aevi Merovingici et Carolini. Teil 3: Supplementa. Herausgegeben von Karl Strecker. Berlin 1923, S. 1041–1042 (Monumenta Germaniae Historica, Digitalisat). --HHill (Diskussion) 22:21, 2. Mär. 2017 (CET)Beantworten
Ah prima, das bietet zwei bessere Lesarten: Haec am Anfang könnte sich auf limina sacra beziehen: "an diese heiligen Schwellen"; und das doofe ter wird zum schlichten te: "Ich bitte dich häufige Gebete zu verrichten". Grüße Dumbox (Diskussion) 22:27, 2. Mär. 2017 (CET)Beantworten
Wo hast du "ter" gelesen? --Digamma (Diskussion) 22:33, 2. Mär. 2017 (CET)Beantworten
In der verlinkten Textvorlage des OP. Grüße Dumbox (Diskussion) 22:34, 2. Mär. 2017 (CET)Beantworten
Danke. Ich hatte nur den getippten Text gesehen. --Digamma (Diskussion) 22:36, 2. Mär. 2017 (CET)Beantworten
Die MGH bieten den emendierten Text, sagen in den Anmerkungen noch manches darüber, vermerken dort die Lesarten und verweisen auf die Nachzeichnung der Inschrift und die Möglichkeit der nachmittelalterlichen Entstehung. --Pp.paul.4 (Diskussion) 08:05, 3. Mär. 2017 (CET)Beantworten
Es sei noch angemerkt, dass der Text öfters benutzt wurde. Im Kloster Liesborn bei Münster erhielt Abt Heinrich von Kleve 1490, also 600 Jahre nach Avo, eine fast identische Grabinschrift. Da steckt also noch eine eigene Geschichte hinter. --Aalfons (Diskussion) 14:04, 3. Mär. 2017 (CET)Beantworten
Den verlinkten Band gibt es übrigens auch direkt bei der Germania Sacra online. --HHill (Diskussion) 14:56, 3. Mär. 2017 (CET)Beantworten
Zur Frage nach dem Alter der Avo-Inschrift äußern sich die MGH ja verhalten. Mit den Buchstabenformen der oben verlinkten Nachzeichnung und dem verlinkten Textfund von 1490 halte ich die Avo-Inschrift für frühneuzeitlich, keinesfalls für karolingisch. Ob Gelehrtenfleiß (Matthias Borbonius widmete 1595 jedem Kaiser ein Gedicht [19], warum sollte nicht ein Gelehrter in gleicher Art jedem Abt ein Gedicht widmen) oder weniger fromme Gelehrtenfälschung dahintersteckt? --Pp.paul.4 (Diskussion) 16:44, 3. Mär. 2017 (CET)Beantworten
Und hier für Nichtmitglieder des MGH-Dechiffriersyndikats das Überlieferungsproblem mit dem Avo-Epitaph auf deutsch. Rein nach Beleg (1587) ist die Liesborner Inschrift die nachgewiesen ältere. --Aalfons (Diskussion) 17:43, 3. Mär. 2017 (CET)Beantworten

Resteverwertung Entenbrust

Wie kann man die verbliebene Hälfte einer übriggebliebenen, rosarot gelungenen Entenbrust am nächsten Tag noch anders verwerten als sie scheibchenweise aufs Butterbrot zu legen? --BN-PG 2263 (Diskussion) 20:02, 2. Mär. 2017 (CET)Beantworten

Klassisch: Das gute, alte Resteragout? Hoffentlich schimpft Oliver S. Y. nicht... ;) Dumbox (Diskussion) 20:07, 2. Mär. 2017 (CET)Beantworten
Danke. Schon mal eine gute Idee, mit Reis, Möhrchen und einer mit einem Müller-Th. abgelöschten und mit Eigelb abgebundenen Soße, fiele mir da so ein. Aber vielleicht hat Oliver S.Y. eine noch bessere Idee? -- BN-PG 2263 (Diskussion) 20:20, 2. Mär. 2017 (CET)Beantworten
BK. DAFÜR wird mich Oliver wohl sicher schimpfen ;-) ... Schlicht und einfach scheibchenweise in der Mikrowelle aufwärmen? Ente wird nach meiner Erfahrung dabei nicht besonders zäh, wenn man ein klein wenig Flüssigkeit drübergibt und nicht mit voller Leistung wärmt, sondern etwas langsamer. --Benutzer:Apierta 20:22, 2. Mär. 2017 (CET)Beantworten

Eine schöne kräftige Gemüsesuppe mit dem Fleisch anreichern? Für ein Rillettes dürfte die Menge zu klein sein. --Elrond (Diskussion) 20:59, 2. Mär. 2017 (CET)Beantworten

Nochmals vielen Dank an alle, die einen Radschlag für mich hatten. Das Bild zeigt, wie die Geschmacksfrage letztlich beantwortet wurde. Nach der Entdeckung einer Rotkohlkonserve knapp jenseits des Haltbarkeitsdatums im Vorratsschrank lag die Kombination mit Bratkartoffeln nahe. (Rotkohl kam ganz zum Schluss nur für's Foto in die Pfanne.) -- BN-PG 2263 (Diskussion) 19:50, 3. Mär. 2017 (CET)Beantworten

Hallo! Ne, wenn man sowas daheim macht schimpfe ich sicher nicht, nur bei solchem Pfusch in der Gastronomie oder Kochbüchern. Meistens ist dieses "rosarot" ja klar denaturiert, weshalb ein erneutes Garen nicht nötig ist. Wenn es um Kochtips geht, wie wäre es mal auf nem Enten-Club-Sandwich statt Hühnerbrust. Dafür nur der Länge nach halbieren oder dritteln, nicht in Scheiben schneiden, ist schließlich echtes Fleisch. Womit man beim nächsten typischen Gericht ist, Geflügelcocktail, Fleisch fein würfeln, und mit Gemüse und Dressing anmachen, kurz marinieren, aufs Brot oder als Vorspeise lecker. Dem Wetter entsprechend kann man dafür auch gut die ersten Frühlingskräuer nehmen, und statt Mayo eine Vinaigrette, Leute wie ich mögen Ente mit Senf ganz gern. Wenn es warm sein soll, dann lege man die Scheiben auf eine Pizza, nachdem sie aus dem Ofen kam. Die Restwärme reicht völlig. Und wenn Mikrowelle, bitte einfach vorher etwas Flüssigkeit durch eine Marinade oder einem leichten gewürzten Rotwein zugeben.Oliver S.Y. (Diskussion) 21:01, 2. Mär. 2017 (CET)Beantworten

Dünne Scheiben ganz kurz in heißer Butter erwärmen, dazu béarnaise. --M@rcela 21:11, 2. Mär. 2017 (CET)Beantworten
... womit die Entenbrust dann zum bloßen Butter- und Saucenträger degradiert worden wäre. Dann besser Kartoffeln in Butter ertränken und die Ente stattdessen der Katze überlassen. Scnr, --Zxmt Nutze Dein Stimmrecht! 20:00, 3. Mär. 2017 (CET)Beantworten

Hat Berlin eigentlich einen mittelalterlichen Stadtkern?

Im Vergleich zu München ist Berlin ja rein architektonisch betrachtet stocklangweilig. München bietet im Zentrum südländisches "Theater", während Berlin ein deutsches New York sein möchte, es aber nicht schafft. Irgendwie tragisch. Breite Straßen, da ein bisschen Klassizismus, da und dort ein paar moderne Bauten, bunt durcheinander gewürfelt, wie das in einem Stadtmilieu ist, das - schon rein historisch betrachtet - beziehungslos und bindungslos ist. Ein wirkliches Ensemble existiert nicht. Meine Frage: Gibt es eigentlich ein "altes" Berlin? Und wo ist das Zentrum? Am Gendarmenmarkt (???).

--217.238.153.230 20:21, 2. Mär. 2017 (CET)Beantworten

Ich glaube, Alt-Berlin beantwortet Deine Frage(n). Gruß, --Benutzer:Apierta 20:36, 2. Mär. 2017 (CET)Beantworten
(BK)Alt-Berlin und Cölln sind der eigentlich Siedlungskern der Stadt Berlin. Einfach mal Geschichte Berlins lesen. --Bobo11 (Diskussion) 20:38, 2. Mär. 2017 (CET)Beantworten
"mittellterlicher Stadtkern" Nicht wirklich. Berlin ist in dem Sinne keine "logisch gewachsene" Stadt, sondern eine Art "Sammlung". Zudem steht Berlin für den permanenten Wandel, Beispiel: Bei U-Bahnarbeiten vor dem Berliner Rathaus fand man unterirdisch die Reste eines alten Rathauses an fast gleicher Stelle. Berlin baute schon immer neues auf den Ruinen des alten - Schloss/Palast der Republik/Humboldtforum. Der Denkmalschutz versucht sein bestes, aber ein zusammengehöriges Stadtensemble war zu keiner Zeit erhaltbar. Wenn du durchs Nikolaiviertel gehst, vermutest du einen historischen Stadtkern. Allerdings ist es komplett zu DDR-Zeiten aufgebaut worden, da stand nur noch eine Ruine der Nikolaikirche. "Spießige Altstadthistorie" wirst du in Berlin nicht finden, auch wenn es Ansätze dafür in einigen Randbezirken gibt.--Wikiseidank (Diskussion) 20:44, 2. Mär. 2017 (CET)Beantworten

Also Berlin hat von 1848 bis 1914 und 1918 bis 1939 jeweils genausoviele Gebäude im Zentrum abgerissen, wie es dann im Krieg zerstört wurde. Der anschließende Wiederaufbau in Ost- und Westberlin taten ein Übrigens. ABER, es wurde kein mittelalterlicher Stadtkern zerstört, sondern Gebäude die meist erst am Ende des 17.Jahrhunderts entstanden. Bin mir nicht so sicher, ob Münchens Stadtkern wirklich so zutreffend ist. Erhaltende Stadtkerne und deren Strukturen findest Du eher in den beiden anderen historischen Städten Spandau und Köpenick, die im Mittelalter eine ebenso große Bedeutung hatten. Ansonsten will Berlin gar kein deutsches New York sein, das wollen nur paar Zugezogene Gernegrößen, egal ob sie nun aus Sachsen, Saarland, Hamburg oder aktuell der Schweiz kommen. Die haben feuchte Träume, weil sie denken, Investoren für ihre Legopläne zu finden. Berlin ist maßgeblich durch Schinkel und seine Schüler geprägt worden, genauso wie das Stadtbild bereits Mitte des 19.Jh. festgelegt wurde. Wenn Du Milleu willst, die Spandauer Vorstadt ist immer noch ziemlich authentisch, und mit Erklärungen die Stadtgeschichte nachzuvollziehen. Wenn man heute ein "Zentrum" definieren will, ist es das Dreieck Potsdamer Platz/Leipziger Platz/Pariser Platz. Das andere ist die City West und der Alexanderplatz (auch wenn er es nicht ist^^). Darum hat sich in Berlin ja gerade dieses Kiezgefühl herausgebildet, weil diese Straßenzüge Insellagen bildeten, wo die Bevölkerung einer deutschen Kleinstadt über- und nebeneinander wohnte. Einige von diesen Vierteln sind darum auch bis heute noch so empfindbar. Aber auch die findet man nur mit Führungen von Eingeweihten, nicht durch die Standardrouten von Reiseführern.Oliver S.Y. (Diskussion) 20:54, 2. Mär. 2017 (CET)Beantworten

Es kommt hinzu, daß das Mittelalter schon fast vorüber war, als Berlin eine nennenswerte Stadt wurde. --M@rcela 21:40, 2. Mär. 2017 (CET)Beantworten
Ist bei München übrigens auch nicht anders. Im Mittelalter waren in Bayern andere Städte vorne, Regensburg bspw. --Gretarsson (Diskussion) 01:01, 3. Mär. 2017 (CET)Beantworten
Mittelalter ist das nicht mehr, aber Stadtmauern / Stadtkern sind da abzulesen.
Map of Berlin in 1688
--Cookatoo.ergo.ZooM (Diskussion) 22:24, 2. Mär. 2017 (CET)Beantworten

Einen schicken mittelalterlichen Stadtkern - sollte der Frager darauf aus sein - gibt es glaube ich oft in kleine(re)n Städten, so in Memmingen... --ObersterGenosse (Diskussion) 23:04, 2. Mär. 2017 (CET)Beantworten

„Mittelalterlich“ war nur der Stadtkern im ummauerten Bereich auf der nebenstehenden Karte. Die eigentlich mittelalterliche Struktur ist dabei höchstens das Straßennetz, selbst die Mauer erst 17./18. Jh. Berlin entwickelte sich eigentlich erst ab dem beginnenden 18 Jh.: Dorotheenstadt, Friedrichstadt etc. Man sollte sich auch nicht täuschen: auch in den „mittelalterlich“ wirkenden Städten sind außer den Kirchen und wenigen Profanbauten die meisten anderen alten Gebäude erst ab dem 16.Jh. entstanden, als das MA schon vorbei war. --Dioskorides (Diskussion) 00:20, 3. Mär. 2017 (CET)Beantworten

Ach, die euroskeptische, „reaktionär-religiotische“ (nicht von mir) 217.238er IP aus Oberbayern (Tacherting) mal wieder, mit ihrem Markenzeichen: Ganz viel Meinung und ganz wenig Ahnung. Warum wusste ich das schon, bevor ich den OP zuende gelesen hatte? (Im Café übrigens ebenfalls mit einem Ich-hol-mir-einen-auf-Bayern-runter-Thread am Start)... --Gretarsson (Diskussion) 01:01, 3. Mär. 2017 (CET)Beantworten

@Dioskorides: Dazu kommt, dass Sichtfachwerk, was viele mit mittelalterlichen Stadtkernen assoziieren, auch lange nach Ende des Mittelalters gebaut wurde. Bei mir im Wohnort gibt es ein echtes Sichtfachwerkhaus, was erst nach 2012 gebaut wurde. Die Schlossbergschule in Neuenbürg wurde in den 1950er-Jahren als verputzter Fachwerkbau neu gebaut. --Rôtkæppchen₆₈ 01:16, 3. Mär. 2017 (CET)Beantworten

Wobei ich die Frage der IP aber berechtigt und interessant finde. Wenn man sich die Stadtpläne anderer großer deutscher Städte, wie etwa Köln oder Frankfurt, anschaut, erkennt man ja die ursprünglichen Stadtkerne noch sehr deutlich, auch wenn der Bombenkrieg und der anschließende „zeitgemäße“ Wiederaufbau von der eigentlichen Bausubstanz wenig übriggelassen haben. In Berlin ist das nicht so. Warum? Eine besonders krasse Folge des sozialistischen Städtebaus in der Hauptstadt der DDR, der etwa die Marienkirche als einsame gotische Insel in einem Meer von Hochhäusern und Grünanlagen beließ? Oder sind die Gründe schon früher zu suchen, v.a. im 18. Jahrhundert, als die vorherige Residenz eines Provinzfürsten innerhalb weniger Generationen zur Hauptstadt eines neuen europäischen Königreiches aufstieg (siehe Artikel Einwohnerentwicklung von Berlin: 6.000 Einwohner im Jahr 1648, 20.000 im Jahr 1688, 55.000 im Jahr 1709 und über 100.000 im Jahr 1747) und man womöglich auch damals schon bestehende Stadtstrukturen beseitigte? --slg (Diskussion) 02:49, 3. Mär. 2017 (CET)Beantworten

Die Berliner Innenstadt war 1945 total zerstört. Beim Wiederaufbau musste/wollte man im Osten wirklich nicht so große Rücksicht auf bestehende Grundstücksgrenzen nehmen wie im Westen, und deshalb war es wohl leichter, Veränderungen im Straßennetz vorzunehmen. Du hast Recht, in vielen Städten ist das Straßennetz das konservativste Strukturelement, nicht die Bausubstanz. --Dioskorides (Diskussion) 06:27, 3. Mär. 2017 (CET)Beantworten

Es gibt mit dem Nikolaiviertel in Berlin natürlich einen historischen Stadtkern. Später jedoch, zu Zeiten preußischer Größe, baute man sich gewissermaßen ein neues Zentrum. Alle vier preußischen Grundpfeiler waren vertreten: Die Kultur mit dem Alten Museum, das Zeughaus mit dem Militär, der Protestantismus mit dem Berliner Dom und das Schloss mit dem König. Heute wird das Schloss dort neu gebaut, allerdings missachtet man die Bedeutung des Platzes. Zum einen weil an dieser Stelle heute die Straße Unter den Linden viel befahrener ist und dadurch diese historische Stelle durchschneidet, zum anderen, weil das Schloss dort gebaut wurde und nicht etwa ein Symbol für die Demokratie wie etwa ein neues Parlament oder Kanzerlamt. Zum wird das Schloss eine Art Museum, was es auf der anderen Seite ja schon gibt. Eine vertane Chance jedenfalls, Berlin wieder einen passigen Stadtkern zu schenken. 62.44.134.189 06:29, 3. Mär. 2017 (CET)Beantworten

Sry, das ist Unfug! Das Nikolaiviertel war 1980 platt. Es wurde mit Plattenbauten und Einzelprojekten ohne historische Authenzität wieder errichtet. Ansonsten war es nur ein Teil des Stadtkern, nicht der Stadtkern. Dessen Ausmaße kann man wirklich sehr leicht durch die Spree, das Heilig-Geist-Spital (Berlin) am ehemals nördlichen Stadtrand, der S-Bahntrasse bzw. Dircksenstraße und der Ruine vom Grauen Kloster nachvollziehen. Die Fläche war also wesentlich größer. Und viele mittelalterliche Straßenzüge lassen sich ja bis heute auch noch nachvollziehen. Das alte Museum wird mitten in den Lustgarten des Schlosses auf die Insel platziert. Das hat nichts mit den westlichen Vorstädten aus dem 18. Jahrhundert zu tun. Genauso ist es eher Beweis für das neue Kulturverständnis von FW III gewesen, der seine kurz zuvor gekauften (nicht eroberten) Sammlungen unterbringen wollte. Außerdem krankt Deine These daran, daß zu diesem Ensemble die katholische Hedwigskathedrale gehört, denn das war der spezielle Kit Preußens, nicht der Protestantismus ansich, sondern die Gottefürchtigkeit und eine Form von bescheidenem "preußischen" Katholizismus, welcher schon wegen der Wiedererstehung im 18.Jahrhundert kaum etwas mit der barocken, ausladenen Form in Mittel- und Süddeutschland zu tun hatte. Das Schloss war mitten in der Stadt, der König sah über die Spree, und sah die Wäscherinnen Berlins. Nicht der Bezug zu den Linden ist das Problem, damals wie heute eigentlich nur eine Ausfallstraße ohne Nutzwert, sondern der Bezug zum Berliner Stadtkern. Einfach mal nachschauen, wo die preußische Geheimpolizeil und Hauptpost standen. Die Preußenkönige waren da mehr Berliner, als mancher heute glauben mag. Oliver S.Y. (Diskussion) 10:26, 3. Mär. 2017 (CET)Beantworten
Als Vergleich: Vom mittelalterlichen Stadtkern ist in London oder Paris auch nicht viel mehr als der Tower oder die Kathedralen erhalten. In Paris hat man im 19. Jh ganz gezielt „aufgeräumt“, die alte Bausubstanz entsorgt und das Straßennetz neu gestaltet. --84.135.140.238 07:48, 3. Mär. 2017 (CET)Beantworten
Städte des heutigen Deutschland waren im Mittelalter bis auf wenige Ausnahmen so klein, daß sie heute eher als Dorf gelten würden. Berlin hat erst im 17. Jahrhundert 5000 Einwohner erreicht. --M@rcela 10:35, 3. Mär. 2017 (CET)Beantworten
Sry Ralf, auch das ist Quark, einfach in der Wikipedia nachzuvollziehen. Bereits um 1400 hatte Berlin 7000 Einwohner, 1576 (also im 16.Jahrhundert) wurden 11.000 erreicht. Damit lag man in Ostdeutschland vor Leipzig und Rostock, gleichauf mit Dresden, nur Magdeburg hatte da mehr als 20.000 EW (Grenze Kleinstadt zu Mittelstadt). In den anschließenden 65 Jahren mit dem 30 Jährigen Krieg ging diese Zahl wie in weiten Teilen Deutschlands auf 6000 zurück. Aber kurz vor dem Krieg 1613 wurden 12.000 erreicht. Magdeburg wurde fast ausgelöscht, 1639 gerade noch 450 EW, keine 2 Prozent.Oliver S.Y. (Diskussion) 11:35, 3. Mär. 2017 (CET)Beantworten
Meines Wissens gibt es aber in Deutschland keine Stadt, die in der Gründerzeit so »explodiert« ist wie Berlin. Wobei ganze Stadtteile quasi auf die grüne Wiese gesetzt wurden und dann erst zusammenwuchsen. Auf manchen Zille-Fotos kann man das erahnen. Darin könnte die seltsame Abwesenheit eines echten Zentrums begründet sein. Paris dagegen wuchs wesentlich langsamer und organischer durch Erweiterungen der Stadtmauern und schließlich Bebauung jenseits der geschleiften Mauern. Was wiederum dazu führte, dass die Infrastruktur schließlich radikal erneuert und erweitert werden musste. Rainer Z ... 13:14, 3. Mär. 2017 (CET)Beantworten

Siliconallergie?

Ich hänge an Sauerstoff, die Zufuhrschläuche samt "Nasenbrille" sind aus Silicon. An den Berührungspunkten über den Ohren und im Nasenraum machen sie mich wund. Was gibt es für Alternativen? Das Sanitätshaus (Lieferant) erzählt mir nur blabla. Auf den Sauerstoff bin ich angewiesen. Gruß -- Dr.cueppers - Disk. 20:36, 2. Mär. 2017 (CET)Beantworten

Könnte es etwas bringen, wenn man den Brillen- und Zuleitungssatz eines anderen Herstellers probiert? Oder an den Kontaktstellen mit einem anderen (Kunst)Stoff umwirken. Ev. ein Schrumpfschlauch, die gibt es auch aus Polyolefinen oder PTFA, wobei letzteres natürlich arg hart sein und mechanisch reizen kann. --Elrond (Diskussion) 20:56, 2. Mär. 2017 (CET)Beantworten
Babypuder könnte helfen. Jedenfalls kann es nicht schaden. --M@rcela 21:42, 2. Mär. 2017 (CET)Beantworten
Es gibt außer aus Silikon auch Brillen aus PVC und auch welche aus Kraton. Kraton ist sehr weich und soll gut verträglich sein. Es beugt allergischen Reaktionen und Druckstellen vor. Sollten Sie auch eine Sehbrille tragen, ist die O2-Brille (Kickinger-Dirmeier-Brille) vielleicht einen Blick wert. Die Sauerstoffversorgung ist dabei in die normale Brille integriert. --83.135.224.233 21:46, 2. Mär. 2017 (CET)Beantworten
Danke für die Antworten: Sehbrille habe ich, brauche sie aber nur außerhalb der Wohnung - und die verlasse ich kaum noch. Nach PVC und Kraton werde ich fragen.Gruß -- Dr.cueppers - Disk. 22:17, 2. Mär. 2017 (CET)Beantworten
Es gibt auch welche aus Vinyl. Je nach dem kann es aber auch ein Hilfsstoffe sein, auf den du allergisch reagiert wie z.b. Desinfektionsmittel.--Bobo11 (Diskussion) 22:34, 2. Mär. 2017 (CET)Beantworten
PVC ist Vinyl. --Rôtkæppchen₆₈ 22:45, 2. Mär. 2017 (CET)Beantworten
Und PVC ist nicht besonders hautfreundlich, Polyvinylchlorid, siehe Polyvinylchlorid#Gesundheitliche_Gefahren. --M@rcela 22:58, 2. Mär. 2017 (CET)Beantworten
Mit Verlaub, aber wenn es solche Bestecke aus PVC geben sollte, werden sie keine Phthalate enthalten sondern Adipate oder Sebacate. Die sind deutlich bedenkenärmer. Freies Vinylchlorid schließe ich mal aus. --Elrond (Diskussion) 23:27, 2. Mär. 2017 (CET)Beantworten
Da hilft die Schläuche und die empfindlichen Stellen der Haut mit Bepanten einzufetten.--2003:75:AF10:D00:D152:2BCA:EA18:39D9 23:11, 2. Mär. 2017 (CET)Beantworten
Kraton ist eine Art Styrol-Butadien-Kautschuk. --Rôtkæppchen₆₈ 23:21, 2. Mär. 2017 (CET)Beantworten
@Dr.cueppers: Bist Du Dir sicher, dass das eine allergische Reaktion ist und nicht etwa nur eine mechanische Druckstelle? --Rôtkæppchen₆₈ 23:27, 2. Mär. 2017 (CET)Beantworten
Darauf hätte ich auch getippt. Die "Verspannung" so justieren, dass es gerade genug Auflagedruck gibt. (Das gilt auch für Apnoe-Masken...) GEEZER … nil nisi bene 07:33, 3. Mär. 2017 (CET)Beantworten
Andere Maske probieren, Wattepads unterlegen... hier gibt es Tipps von Nutzern solcher Masken, die damit Probleme hatten. --2003:76:E12:952A:EC00:6C66:CA30:F78F 01:03, 3. Mär. 2017 (CET)Beantworten
@Rotkaeppchen68: Wenn ich mir über Allergie oder was Anderes sicher gewesen wäre, hätte ich die Überschrift nicht mit Fragezeichen versehen! Ich weiß ja eben nicht, wo das herkommt! Beim Lieferanten hatte ich heute einen etwas kompetenteren Gesprächspartner, der mir Nasenbrillen aus anderem Material zum Ausprobieren schicken will. Gruß -- Dr.cueppers - Disk. 20:00, 3. Mär. 2017 (CET)Beantworten

Tote Hose auf Mittelwelle im Autoradio

Früher war die Mittelwelle im Grundigradio voll mit Sendern. Jetzt, neues Auto, hightech Radio, nix mit MW. Warum? Und warum unterstützen die noch MW und LW, wenns doch nix bringt? --93.212.207.119 23:51, 2. Mär. 2017 (CET)Beantworten

In Deutschland wurden die Mittelwellensender ersatzlos abgeschaltet. Dank Internetlivestream ist niemand mehr auf den Mittelwellenfernempfang angewiesen. Der Mittelwellenortsempfang wurde in Deutschland bereits ab 1949 mit dem ersten UKW-Sender ganz allmählich getötet. Als Konsequenz des Zweiten Weltkriegs verlor Deutschland sehr viele Mittelwellenfrequenzen. Deswegen wurde ab 1949 das UKW-Sendernetz mehrfach flächendeckend aufgebaut. Dadurch, dass überall drei bis vier UKW-Sender zu empfangen waren, musste niemand mehr Mittelwelle im Nahbereich hören. Lediglich der Mittelwellenfernempfang hatte noch seine Berechtigung. Mit dem Siegeszug des Internets in den 1990er-Jahren verlor der Mittelwellenfernempfang mehr und mehr an Bedeutung, weswegen die Mittelwellensenderbetreiber ihre Sender nach und nach abschalteten. Aus demselben Grund erlangte der AM-Nachfolger DRM nie Bedeutung. Außerhalb Deutschlands ist die Situation anders. In den USA wurden sogar Frequenzen oberhalb 1600 kHz neu dem Mittelwellenrundfunk zugewiesen. Die Langwelle hat nie wirklich Bedeutung erlangt, weil die zur Verfügung stehende Bandbreite nicht im Verhältnis zur Reichweite der Sender steht. --Rôtkæppchen₆₈ 00:06, 3. Mär. 2017 (CET)Beantworten
Kleiner Tipp: Das Lesen des Artikels Mittelwelle hätte die Frage auch beantwortet. --2003:76:E12:952A:EC00:6C66:CA30:F78F 00:51, 3. Mär. 2017 (CET)Beantworten
Dann haben die Maschinen bereits gewonnen. Wie soll der Widerstand dann kommunizieren?--Wikiseidank (Diskussion) 12:17, 3. Mär. 2017 (CET)Beantworten
Wie andere schon gesagt haben: Öffentlicher Hörfunk wird in Deutschland mittlerweile praktisch gar nicht mehr über AM gesendet (also LW/MW/KW). Das sieht in anderen Ländern anders aus, deshalb werden Radios auch weiterhin damit ausgestattet. Kann dein Hightech-Radio zufällig DAB (das es auch schon seit etwa 20 Jahren gibt)? --Kreuzschnabel 15:32, 3. Mär. 2017 (CET)Beantworten
DAB gibt es zwar schon seit 20 Jahren, konnte sich aber wegen schlechter Tonqualität, schweineteurer Enpfänger und schlechter Senderabdeckung nicht durchsetzen. Der Durchbruch kam erst mit der flächendeckenden Einführung von DAB+. Mittlerweile sind so gut wie alle DAB-Classic-Programme abgeschaltet. --Rôtkæppchen₆₈ 17:54, 3. Mär. 2017 (CET)Beantworten

Vielen Dank für die Antworten. Ich meine mich aber zu erinnern, daß ich vor etwa 40 Jahren im Autoradio auf MW noch RTL und nebendran Radio Vatikan, z.B., gut empfangen konnte. Und mit dem hightech Radio jetzt krieg ich keinen Sender gut rein. Die alten Dinger hatten eine Ferritantenne. --84.181.108.114 19:39, 3. Mär. 2017 (CET)Beantworten

3. März 2017

Ressourcenverknappung: Wie lange reichen unsere Metalle?

Alle reden von globaler Erwärmung und Peak Phosphorus. Aber ich habe bisher noch keine halbwegs verlässliche Schätzung gesehen, wie lange unsere Metalle wie Kupfer noch reichen, wenn man im derzeitigen Tempo Mobiltelefone und andere Produkte produziert, ohne die Rohstoffe nach Gebrauch im Sinne einer Kreislaufwirtschaft weiterzunutzen. Kennt Ihr einen idealerweise aktuellen Artikel aus einer verlässlichen wissenschaftlichen Fachzeitschrift, der für verschiedene Materialien eine Schätzung abgibt, wie lange wir die Stoffe, etwa Kupfer, noch nutzen können? 130.226.41.18 08:54, 3. Mär. 2017 (CET)Beantworten

Beim Kupfer scheint die Lage nicht so dramatisch zu sein: [20]. --Snevern 09:42, 3. Mär. 2017 (CET)Beantworten
Gerade Kupfer gibts genug, da droht keine Knappheit. --M@rcela 09:43, 3. Mär. 2017 (CET)Beantworten
Die Anzahl der nutz- und förderbaren Metalle reicht für die gegenwärtige Erdbevölkerung auf unbegrenzte Zeit. Was begrenzt ist, ist die Zugänglichkeit. Aber wenn Du einen Bergmann des Mittelalters erzählt hättest, das wir heute komplette Bergspitzen abtragen und bereits bis 4000 Meter tiefe Bergwerke nutzen, hätte er es auch nicht geglaubt. Die Menschheitsgeschichte hat gezeigt, daß die unvorhersehbaren Märkte jede vermeintlich seriöse Prognose ad absurdum führen. Es ist nur eine Frage des Preises, ob der Mensch überhaupt ein Metall nutzen will. Bestes Beispiel sind die seltenen Erden, wo man sich lieber von den Tagebauerzen in China abhängig macht, als eigene, europäische Vorkommen zu erschließen.Oliver S.Y. (Diskussion) 10:06, 3. Mär. 2017 (CET)Beantworten
Das klingt ja alles optimistisch. Sieht das der wissenschaftliche Konsens auch so? 130.226.41.18 11:30, 3. Mär. 2017 (CET)Beantworten
Dazu braucht man keine Wissenschaft. 7 Mrd. Menschen auf der Erde haben 3 Mrd. Tonnen Kupfer. Pro Mensch also 500 Kilo. Selbst wenn so ein Telefon nahezu komplett aus Kupfer wäre, hätte also jeder Mensch 5000 Telefone. Da der Kupferanteil jedoch sehr viel geringer ist.... --M@rcela 11:51, 3. Mär. 2017 (CET)Beantworten
Naja, wie wäre es, mal in der Wikipedia nachzulesen? Kupfer/Tabellen und Grafiken#Förderung, Reserven und Vorratsbasis. Blei gleichbleibender Fördermenge reichen die aktuellen Reserven, die "wirtschaftlich" gefördert werden können noch 34 Jahre. Die "Förderfähigen" Vorkommen reichen 63 Jahre. Das betrifft aber nichtmal die gesamten 29% der Erdoberfläche an Land. Weite Gebiete wie die Sahara, Amazonasbecken, Grönland und die Antarktis sind noch nichtmal erkundet. Und es ist nicht "unwissenschaftlich", die 71% von Wasser bedeckten Gebiete für ähnlich zu halten. Du wirst keine "verlässliche" Fachzeitschrift finden, die da genau sagen kann, ob es letztendlich 3,5 oder 10 Mrd. Tonnen sind. Schon der Unterschied zwischen "reserves" und der "reserve base" ist wesentlich, wird häufig durch Journalisten aber gleichgesetzt.Oliver S.Y. (Diskussion) 11:47, 3. Mär. 2017 (CET)Beantworten

Eine weitere, in Zukunft wohl immer wichtiger werdende Quelle ist unser Abfall. Deponien, und Schrott und andere Abfälle enthalten einiges an Metallen, bislang sind sie aber in vielen Fällen nicht wirtschaftlich zurückzugewinnen. Bei einigen Sachen aber schon, so ist bei Kupfer mindestens die Hälfte des eingesetzten Kupfers mittlerweile Altmetall https://www.kupferinstitut.de/de/werkstoffe/system/recycling-kupfer.html. Bei anderen Metallen sieht es ähnlich aus und diese Quote dürfte in Zukunft steigen. Limitierend ist meist die Effektivität bzw. die Energiemenge zur Herstellung. --Elrond (Diskussion) 12:30, 3. Mär. 2017 (CET)Beantworten

Dazu noch der Hinweis, daß in Sachsen (Helmholtzgesellschaft) und Polen bereits fortgeschrittene Experimente mit Mikroben stattfinden, welche eine Gewinnung von Kupfer und anderen Metallen aus vermeintlich minderwertigem Gestein und sogar aus mittelalterlichen Abraumhalden ermöglichen. Die kacken schlicht unverdauliches Kupfer aus :), das muss man nur noch zusammenfegen. Oliver S.Y. (Diskussion) 12:37, 3. Mär. 2017 (CET)Beantworten
Klassischer Überlegungsfehler. Metalle werden in der Regel nicht verbraucht, sondern könnten Wiederverwertet werden. Kurzum in Zukunft wird man vermehrt Metalle aus Mülldeponien usw. gewinnen müssen. Wenn das mit dem erschließen von Sekundär-Rohstoffquellen gemacht würde, gibt es nicht so schnell einen Versorgungsengpass. Man wird nicht bei den klassischen Metallen ein Problem hab, die fast rein verwendet werden (Kupfer, Eisen, Aluminium). Sonden bei dennen, die nur zu einem kleinen Prozentsatz im Endprodukt stecken.--Bobo11 (Diskussion) 12:44, 3. Mär. 2017 (CET)Beantworten
Prognosen sind schwierig, besonders, wenn sie die Zukunt betreffen.
Ich habe keine Ahnung, von welchen Voraussetzungen unsere Futorologen hier ausgehen. Wir haben auf der einen Seite einen zunehmenden Bedarf an Kupfer für Elektromotoren - zumindest, wenn die Prognosen der Regierungen zur Elektromobilität zutreffen. Um den dafür benötigten Strom zu erzeugen, werden wir auch noch etwas zusätzliches Kupfer für Generatoren benötigen. Dann bleibt die Frage, wie sich die Menschheit weiter vermehrt. Wenn ich einfach eine Gerade von 1800 bis heute lege, dann wächst die Menschheit unbeschränkt weiter, bis die Leute schliesslich auf den Schultern von anderen Leuten stehen oder nasse Füsse bekommen, weil an Land kein Platz mehr ist. Realistisch sollten wir aber vermuten, dass das karnickelartige Vermehren sich irgendwann verlangsamt (nebenbei bemerkt eine Katastrophe für unsere heutige Wirtschaft). Natürlich wird so eine kuschelige Nähe völlig andere Verhaltensweisen erfordern, beispielsweise müssen wir vermutlich auf den Köpfen Tabletts tragen, auf denen wir Grünzeug zur Ernährung anbauen, weil der Boden ja mit Menschen bedeckt ist.
Kurz: Du wirst hier keine seriöse Prognose bekommen. Aber jedem Menge von der anderen Sorte. Yotwen (Diskussion) 13:21, 3. Mär. 2017 (CET)Beantworten
Die Menschheitsvermehrung muss schon allein deshalb gestoppt werden, weil der technische Fortschritt die Menschen beschäftigungslos macht. Hierzulande kriegen die Menschen weniger Kinder und beschäftigen sich mit "schöngeistigen" Dingen. Woanders spielt die Jugend stattdessen Krieg. ("Spiel", weil total ineffektiv.) Ich wage mal eine Prognose, deren Eintreten ich vermutlich noch erlebe: Entwicklungshilfe, die ihren Namen tatsächlich verdient, wird in Zukunft an Bevölkerungsreduktion gekoppelt. Ob das in Afrika dann über Ein-Kind-Prämien oder über "Abschussprämien" realisiert wird, werden wir den afrikanischen Staaten sicherlich mal wieder selbst überlassen. Weil: Keine Einmischung. Bei den Leuten, die im Mittelmeer ersaufen, ist es uns ja heute auch schon egal. Es sei denn, irgendein Reporter hält auf einen in der Brandung rollenden Kinderleichnam drauf. Dann ist halt eine Woche Aufregung und danach wieder Normalbetrieb. -- Janka (Diskussion) 15:16, 3. Mär. 2017 (CET)Beantworten
Ein Wörtchen dürfte auch die natürliche Auslese mitspielen. Die Biologie reagiert , wenn auch sehr träge, auf veränderte Lebensbedingungen. --M@rcela 15:35, 3. Mär. 2017 (CET)Beantworten
Die en hat Peak copper und sonst haben wir bald Meeresbodenbergbau oder besser Asteroidenbergbau.--Asteroidenbergbauer (Diskussion) 16:32, 3. Mär. 2017 (CET)Beantworten
Nun ja, bevor man zum Asteroiden Bergbau schreitet, werden sicher zuerst mal die bislang unrentabel Erzlagerstätte abgebaut. So finden sich im Skandinavien (und im übrigen Europa) einigen Kupfer-Minen die stillgelegt wurden bevor sie leer geräumt waren. Auch in den Abraumhalden ehemaliger Kupfermienen wird es noch Kupfer haben. Wenn die grossen Minen aus denen heute das meiste Kupfer stammt erschöpft sind, wird sich schlicht weg der Rohstoffpreis erhöhen, und plötzlich rentieren sich Lagerstätten die man heute gar nicht auf dem Radar hat. Und es rentiert dann plötzlich auch Kupfer besser aus dem Abfall herauszulösen. Aber generell wird es keine Frage danach sein ob wir noch Kupfer haben, sondern eher ein Frage ob wir es uns für gewisse Anwendungen noch leisten können Kupfer zu "verschwenden". Übrigens könnte man schon heute bei etliche Elektroanwendungen auch mit Aluminium arbeiten, ist halt nicht ganz ein so guter Leiter und bisschen alterungs-anfälliger als Kupfer. Aber schon bei der SBB Ae 4/6 hat man wegen Kupfermangel im Zweiten Weltkrieg der Haupttransformator und die Fahrmotoren mit Aluminiumwicklungen ausgeführt. Denn grundsätzlich ist Alu statt Kupfer, bei Elektro-Anwendungen möglich. Das Kupfer besser geeignet ist streit ich nicht ab, aber es gibt Alternativen zu Kupfer. Denn Silber wären ja noch besser als Kupfer geeignet (ist der bessere Leiter), nur will das in der Regel niemand bezahlen.--Bobo11 (Diskussion) 19:21, 3. Mär. 2017 (CET)Beantworten

Im Buch B. Ilschner, R. F. Singer: Werkstoffwissenschaften und Fertigungstechnik Eigenschaften, Vorgänge, Technologien 5., neu bearbeitete Auflage, gibt es zu genau diesem Thema ein paar Seiten. "Wie aus Abb. 1.2 hervorgeht, müssen wir damit rechnen, dass viele wichtige Rohstoffreserven in den nächsten 20 bis 50 Jahren zur Neige gehen. Bei einigen wenigen Stoffen ist die Situation sogar noch wesentlich dramatischer. Dazu gehört beispielsweise Indium, für das nur noch Reichweiten von wenigen Jahren vorhergesagt werden"

Aus der erwähntn Abb. 1.2 entnehme ich mal die "Reservenreichweite" in Jahren. Zusätzlich ist noch eine höhere "Reserven- und Ressourcenreichweite" angegeben. ("Bei genauerer Betrachtung stellt sich die Situation für unsere meisten Rohstoffe allerdings weit weniger kritisch dar. [...] Reserven sind nachgewiesene, mit gegenwärtiger Technik zu gegenwärtigen Preisen wirtschaftlich gewinnbare Vorräte. Ressourcen sind dagegen geologisch nachgewiesene, aber gegenwärtig nicht wirtschaftlich abbaubare Vorräte.")

  • Uran - 46
  • Eisen - 95
  • Aluminium - 141
  • Titan - 122
  • Nickel - 41
  • Kupfer 31
  • Zink 22
  • Blei 20

Und zum Vergleich

  • Braunkohle - 293
  • Steinkohle - 137

Die Zahlen ergeben sich also nur mathematisch aus den derzeitigen Reserven dividiert durch den Jahresverbrauch. Technologischer Wandel, wie bspw. von Yotwen angesprochen und auch wirtschaftlicher Wandel sind da nicht berücksichtigt. --DWI (Diskussion) 19:27, 3. Mär. 2017 (CET)Beantworten

Ja, mir machen auch die "seltenen" und "unedlen" Metalle mehr Sorgen, als der "Klassiker" Kupfer, was die Versorgungssicherheit anbelangt. Denn bei denen Erdalkalimetallen Erdmetallen und den Seltenerdmetalle funktioniert das Recycling noch nicht so gut. Schlichtweg auch aus dem Grund das unedle Metalle zu gern "unpraktische" Verbindungen eingehen. Und wie oben schon angesprochen, je weniger von etwas in der endgültigen Werkstoff ist, desto schwerer und aufwendiger ist es als halbwegs reinen Wertstoff wieder heraus zu kriegen.--Bobo11 (Diskussion) 19:53, 3. Mär. 2017 (CET)Beantworten

Prozess gegen Anton Schlecker, Ausschluß der Öffentlichkeit?

Kann Anton Schlecker beantragen dass die Öffentlichkeit ausgeschlossen wird zum Prozess gegen ihn? Ich meine er wollte nie in der Öffentlichkeit stehen und wollte nicht das Bilder von ihm gemacht werden, darf er dann das auch vor Gericht geltend machen? --Fragesteller 2018 (Diskussion) 09:23, 3. Mär. 2017 (CET)Beantworten

Grundsätzlich: Nein. Ich sehe auf den ersten Blick auch keinen einschlägigen Ausnahmetatbestand, auf den er sich berufen könnte.
Angeklagte, die keine Bilder von sich haben wollen, verstecken sich regelmäßig hinter Aktendeckeln. Müsste er wohl auch tun. --Snevern 09:36, 3. Mär. 2017 (CET)Beantworten
Aufs Recht am eigenen Bild kann er sich gem. §23 KUG als absolute Person der Zeitgeschichte nicht berufen. --M@rcela 09:42, 3. Mär. 2017 (CET)Beantworten
Wenn man aber die Unschuldsvermutung beachtet, sollte man doch hier klarstellen, dass keine Bilder gemacht werden dürften bzw. veröffentlich bis ein Urteil steht?--Fragesteller 2018 (Diskussion) 10:35, 3. Mär. 2017 (CET)Beantworten
Bezogen aufs Recht am eigenen Bild ist das irrelevant. Egal ob schuldig oder unschuldig ist er bekannt genug. --M@rcela 10:37, 3. Mär. 2017 (CET)Beantworten
Es besteht gar kein Anlass, die Öffentlichkeit auszuschließen. Weder ist Anton Schlecker minderjährig, noch werden in diesem Prozess Staatsgeheimnisse berührt. --Rôtkæppchen₆₈ 11:39, 3. Mär. 2017 (CET)Beantworten
Ich wüsste auch nicht, wieso die Öffentlichkeit ausgeschlossen werden können sollte: Schon ein Unbekannter muss mit der Berichterstattung über seinen Prozess leben - Gerichtsverfahren sind und sollen nunmal öffentlich sein. Dass ein Angeklagter in der Öffentlichkeit steht, er also grundsätzlich schon mehr Berichterstattung über sich hinnehmen muss, kann nicht auf einmal dazu führen, dass weniger berichtet werden dürfte. Eher das Gegenteil. --83.135.224.214 15:24, 3. Mär. 2017 (CET)Beantworten
Prozessöffentlichkeit und journalistische Bilder sind zwei unterschiedliche Dinge. Bilder kann er als Person der Zeitgeschichte (Ralf verweist oben zurecht auf § 23 Kunstzrhg) bei einem öffentlichen Informationsinteresse (23.000 Arbeitsplätze vernichtet und ...es ist nichts mehr da...)“ nicht verhindern. Für Prozesse gilt der Öffentlichkeitsgrundsatz mit seinen Ausnahmeregeln. --80.130.231.152 19:17, 3. Mär. 2017 (CET)Beantworten

Woher kommen die alternativen Pluralformen im Bairischen

"enk" und "es"? In den übrigen deutschen Dialekten und in den heutigen germanischen Sprachen gibt es das meines Wissens nur selten. Es wird immer behauptet, dass es einen Zusammenhang mit dem Gotischen gebe. Stimmt das wirklich? Und wieso existieren eigentlich solche Formen?

--217.238.133.50 10:39, 3. Mär. 2017 (CET)Beantworten

Das sind alte Dualformen. Die Frage ist also weniger, warum sie existieren, sondern warum sie im Hochdeutschen nicht mehr existieren (wo doch eigentlich jede vollwertige Sprache über einen Dual verfügt – letzteres eine persönliche Anmerkung ;) --j.budissin+/- 10:45, 3. Mär. 2017 (CET)Beantworten
Wenn ich mir das ihm hiesigen Artikel anschaue, steht dort "Im Bairischen wird die ursprüngliche Dualform als allgemeine Pluralform verwendet. Einen eigenen Dual gibt es heute nicht mehr" --Elrond (Diskussion) 12:19, 3. Mär. 2017 (CET)Beantworten
@Elrond: Ja. Und? --j.budissin+/- 12:25, 3. Mär. 2017 (CET)Beantworten
Im Zusammenhang vielleicht noch interessant sind die isländischen Personalpronomen: Der alte Plural (entspr. dt. ihr) ist eine spezielle Höflichkeitsform geworden, während der alte Dual (wie im Bayerischen) die normale Pluralform übernommen hat. --L47 (Diskussion) 10:51, 3. Mär. 2017 (CET)Beantworten
Siehe Bairische Kennwörter#és und enk. --Komischn (Diskussion) 11:18, 3. Mär. 2017 (CET)Beantworten
Der Zusammenhang mit dem Gotischen ist dort leider nicht behandelt. Nach dem Beleg in bar:Bairische Kennwörter#ees und enk reichen die bairischen Schriftbelege in das 15. Jh. zurück. Es gehen voraus althochdeutsch *ez (ihr beide), althochdeutsch *enker (euch beiden) und althochdeutsch *enker (euer), vergleichbar mit gotisch *it, igqis und igqar [21]. --Pp.paul.4 (Diskussion) 17:23, 3. Mär. 2017 (CET)Beantworten

TÜV wegen Undichtigkeit - morgen nass

Morgen habe ich einen Nachtermin beim TÜV wegen Öl am Motor. Voraussichtlich wird es aber regnen. Muss ich dann nochmal kommen, wenn man wegen der Nässe wenig sieht, und womöglich die Nachgebühr ein zweites Mal zahlen?--82.113.121.63 14:50, 3. Mär. 2017 (CET)Beantworten

Ruf an und frag. Oder mach gleich einen Ersatztermin aus. --Rôtkæppchen₆₈ 15:16, 3. Mär. 2017 (CET)Beantworten
Dieser Abschnitt kann archiviert werden. Hier falsch. --Rôtkæppchen₆₈ 15:16, 3. Mär. 2017 (CET)
Hab' ich ein so besonderes Auto, dass der Motor auch bei Regen trocken bleibt? --Snevern 17:55, 3. Mär. 2017 (CET)Beantworten
Der Prüfer kann Wasser und Öl schon unterscheiden. Hat er beides schonmal gesehen. --2003:76:E32:57FC:9C7C:DCB4:94AC:734F 19:38, 3. Mär. 2017 (CET)Beantworten

Rücktritt und Schadensersatz bei Kaufverträgen

Warum gibt es eigentlich 2 unterschiedliche Varianten einen misslungenen Kauf abzuwickeln, und diese eigentlich auf das selbe hinauslaufen? Warum nicht nur eine? --91.66.137.253 18:35, 3. Mär. 2017 (CET)Beantworten

Es gibt sogar noch mehr Varianten, siehe § 437 BGB. Wo genau diese Varianten herkommen, ist eine gute Frage. Vieles, was wir heute haben, stammt ursprünglich aus dem römischen Recht. Wo diese Varianten deiner Meinung nach auf dasselbe hinauslaufen, müsstest du noch etwas weiter erläutern, damit wir den entsprechenden Unterschied darstellen können; es gibt z.B. mehrere Arten des Schadenersatzes. --83.135.224.214 19:20, 3. Mär. 2017 (CET)Beantworten

© 2022 Created with Royal Elementor Addons